Output
Output
2010
Mathematics
STEP 9465/9470/9475
October 2010
The Cambridge Assessment Group is Europe's largest assessment agency
and plays a leading role in researching, developing and delivering
assessment across the globe. Our qualifications are delivered in over 150
countries through our three major exam boards.
All Examiners are instructed that alternative correct answers and unexpected
approaches in candidates’ scripts must be given marks that fairly reflect the
relevant knowledge and skills demonstrated.
© UCLES 2010
Report Page
STEP Mathematics I 4
STEP Mathematics II 44
STEP Mathematics III 54
Question 1
Given that
5x2 + 2y 2 −6xy + 4x − 4y
≡ a(x − y + 2)2 + b(cx + y)2 + d
≡ a(x2 − 2xy + y 2 + 4x − 4y + 4) + b(c2 x2 + 2cxy + y 2 ) + d
≡ (a + bc2 )x2 + (2bc − 2a)xy + (a + b)y 2 + 4ax − 4ay + 4a + d,
so we require
a + bc2 = 5
2bc − 2a = −6
a+b=2
4a = 4
−4a = −4
4a + d = 0.
The fourth and fifth equations both give a = 1 immediately, giving b = 1 from the third
equation. Then the second equation gives c = −2 and the final equation gives d = −4.
We must also check that this solution is consistent with the first equation. We have
a + bc2 = 1 + 1 × (−2)2 = 5, as required. (Why is this necessary? Well, if the second
equation had begun with 7x2 + · · · , then our method would still have given us a = 1, etc.,
but the coefficients for the x2 term would not have matched, so we would not have been
able to write the second equation in the same way as the first.)
We thus deduce that
Spurred on by our success in the first part, we will rewrite the first equation in the
suggested form:
(x − y + 2)2 + (y − 2x)2 − 4 = 9. (3)
We are led to wonder whether the same trick will work for the second equation, so let’s
try writing:
a + bc2 = 6
2bc − 2a = −8
a+b=3
4a = 8
−4a = −8
4a + d = 0.
(We can write these down as the right hand side is the same as before.)
This time, a = 2 from both the fourth and fifth equations, so we get b = 1 from the
third equation. The second equation gives us c = −2. Finally, the sixth equation gives
us d = −8.
We must now check that our solution is consistent with the first equation, which we have
not yet used. The left hand side is a + bc2 = 2 + 1 × (−2)2 = 6, which works, so we can
write the second equation as
(If we had not checked for consistency, we might have wrongly concluded that 183x2 +
3y 2 − 8xy + 8x − 8y can also be written in the same way.)
These two equations now look remarkably similar! In fact, let’s move the constants to the
right hand side and write them together:
(x − y + 2)2 + (y − 2x)2 = 13
2(x − y + 2)2 + (y − 2x)2 = 22.
We now have two simultaneous equations which look almost linear. In fact, if we write
u = (x − y + 2)2 and v = (y − 2x)2 , we get
u + v = 13
2u + v = 22
(x − y + 2)2 = 9 (4)
(y − 2x)2 = 4. (5)
We can take square roots, so that (4) gives x − y + 2 = ±3 and (5) gives y − 2x = ±2.
Thus we now have four possibilities (two from equation (4), and for each of these, two from
equation (5)), and we solve each one, checking our results back in the original equations.
Therefore we see that the four solutions are (x, y) = (1, 0), (7, 12), (−3, −4) and (3, 8).
An alternative is to observe that equation (2) looks almost double equation (1), so we
consider 2 × (1) − (2):
4x2 + y 2 − 4xy = 4.
But the left hand side is simply (2x − y)2 , so we get 2x − y = ±2.
Substituting this into equation (3) gives us
(x − y + 2)2 + 4 − 4 = 9,
so that x − y + 2 = ±3.
Thus we have the four possibilities we found in the first approach, and we continue as
above.
x2 + y 2 − 2xy + 4x − 4y = 5,
so that
(x − y)2 + 4(x − y) = 5.
Writing z = x − y, we get the quadratic z 2 + 4z − 5 = 0, which we can then factorise to
give (z + 5)(z − 1) = 0, so either z = 1 or z = −5, which gives x − y = 1 or x − y = −5.
Substituting x − y = 1 into (3) now gives
(1 + 2)2 + (y − 2x)2 − 4 = 9,
x − a
The curve y = ex , where a and b are constants, has two stationary points.
x−b
Show that
a−b<0 or a − b > 4.
We begin by differentiating using first the product rule and then the quotient rule:
d x − a x (x − a) x
dy
= e + e
dx dx x − b (x − b)
(x − b).1 − (x − a).1 x (x − a) x
= e + e
(x − b)2 (x − b)
(a − b) x (x − a)(x − b) x
= e + e
(x − b)2 (x − b)2
x2 − (a + b)x + (ab + a − b) x
= e .
(x − b)2
dy
Now solving dx = 0 gives x2 −(a+b)x+(ab+a−b) = 0. Since the curve has two stationary
points, this quadratic must have two distinct real roots. Therefore the discriminant must
be positive, that is
(a + b)2 − 4(ab + a − b) > 0,
and expanding gives a2 − 2ab + b2 − 4a + 4b > 0, so (a − b)2 − 4(a − b) > 0. Factorising
this last expression gives
(a − b)(a − b − 4) > 0,
so (sketching a graph to help, possibly also replacing a − b with a variable like x), we see
that we must either have a − b < 0 or a − b > 4.
(i) Show that, in the case a = 0 and b = 12 , there is one stationary point on either
side of the curve’s vertical asymptote, and sketch the curve.
x
x
We are studying the curve y = 1 e .
x− 2
We have a − b = − 12 < 0, so the curve has two stationary points by the first part of the
question. The x-coordinates of the stationary points are found by solving the quadratic
x2 − (a + b)x + (ab + a − b) = 0,
as above.
Substituting in our values for a and b, we get x2 − 21 x − 12 = 0, so 2x2 − x − 1 = 0,
which factorises to (x − 1)(2x + 1) = 0. Thus there are stationary points at (1, 2e) and
(− 21 , 21 e−1/2 ).
The vertical asymptote is at x = b, that is at x = 12 .
Therefore, since the two stationary points are at x = 1 and x = − 21 , there is one stationary
point on either side of the curve’s vertical asymptote.
We note that the only time the curve crosses the x-axis is when x = a, so this is when
x = 0, and this is also the y-intercept in this case.
As x → ±∞, y ∼ ex (meaning y is approximately equal to ex ; formally, we say that y is
asymptotically equal to ex ), as the fraction (x − a)/(x − b) tends to 1.
We can also note where the curve is positive and negative: since ex is always positive,
y > 0 whenever both x − a > 0 and x − b > 0, or when both x − a < 0 and x − b < 0, so
y < 0 when x lies between a and b and is positive or zero otherwise.
Using all of this, we can now sketch the graph of the function. The nature of the stationary
points will become clear from the graphs. In the graph, the dotted lines are the asymptotes
(x = 12 and y = ex ) and the red line is the graph we want, with the stationary points
indicated.
y
(1, 2e)
(− 12 , 12 e−1/2 )
0 1 x
2
9
(ii) Sketch the curve in the case a = 2
and b = 0.
x − 9
2
This time, we are studying the curve y = ex .
x
Proceeding as in (i), we have a − b = 92 > 4, so again, the curve has two stationary points.
The x-coordinates of the stationary points are given by solving the quadratic
x2 − (a + b)x + (ab + a − b) = 0,
as above.
Substituting our values, we get x2 − 29 x + 29 , so 2x2 − 9x + 9 = 0. Again, this factorises
nicely to (x − 3)(2x − 3) = 0, giving stationary points at ( 23 , −2e3/2 ) and (3, − 12 e3 ).
The vertical asymptote is at x = b, that is at x = 0. This time, therefore, the stationary
points are both to the right of the vertical asymptote.
The x-intercept is at x = a, that is, at ( 29 , 0). There is no y-intercept as x = 0 is an
asymptote.
Again, as x → ±∞, y ∼ ex .
As in (i), y < 0 when x lies between a and b and is positive or zero otherwise.
Using all of this, we can now sketch the graph of this function. Note that the asymptote
y = ex is much greater than y until x is greater than 20 or so, as even then (x−a)/(x−b) ≈
15/20, and only slowly approaches 1. We don’t even attempt to sketch the function for
such large values of x!
0 9 x
2
( 32 , −2e3/2 )
(3, − 12 e3 )
Question 3
Show that
sin(x + y) − sin(x − y) = 2 cos x sin y
and deduce that
sin A − sin B = 2 cos 12 (A + B) sin 12 (A − B).
We use the compound angle formulæ (also called the addition formulæ) to expand the
left hand side, getting:
sin(x + y) − sin(x − y) = (sin x cos y + cos x sin y) − (sin x cos y − cos x sin y)
= 2 cos x sin y,
as required.
For the deduction, we want A = x + y and B = x − y, so x = 21 (A + B) and y = 12 (A − B),
solving these two equations simultaneously to find x and y. Then we simply substitute
these values of x and y into our previous identity, and we reach the desired conclusion:
Likewise, we have
cos(x + y) − cos(x − y) = (cos x cos y − sin x sin y) − (cos x cos y + sin x sin y)
= −2 sin x sin y,
Remark: The points P , Q, R and S all lie on an ellipse, which can be thought of as a
stretched circle, as their coordinates all have xa = cos θ and yb = sin θ, so they satisfy the
2 2
equation xa + yb = 1.
The lines P Q and SR are parallel if and only if their gradients are equal (and neither are
vertical, so their gradients are well-defined), thus
b sin q − b sin p b sin s − b sin r
P Q k RS ⇐⇒ =
a cos q − a cos p a cos s − a cos r
sin q − sin p sin s − sin r
⇐⇒ =
cos q − cos p cos s − cos r
2 cos 2 (q + p) sin 21 (q − p)
1
2 cos 12 (s + r) sin 21 (s − r)
⇐⇒ =
−2 sin 12 (q + p) sin 21 (q − p) −2 sin 12 (s + r) sin 21 (s − r)
cos 12 (q + p) cos 12 (s + r)
⇐⇒ =
− sin 21 (q + p) − sin 12 (s + r)
⇐⇒ cot 12 (q + p) = cot 21 (s + r)
⇐⇒ 12 (q + p) = 21 (s + r) + kπ for some k ∈ Z
⇐⇒ q + p = s + r + 2kπ for some k ∈ Z
⇐⇒ r + s − p − q = 2nπ for some n ∈ Z.
The last four lines could have also been replaced by the following:
P Q k RS ⇐⇒ · · ·
cos 21 (q + p) cos 12 (s + r)
⇐⇒ =
− sin 21 (q + p) − sin 12 (s + r)
⇐⇒ cos 21 (q + p) sin 12 (s + r) = cos 21 (s + r) sin 12 (q + p)
⇐⇒ sin 21 (s + r) cos 12 (q + p) − cos 21 (s + r) sin 21 (q + p) = 0
⇐⇒ sin 21 ((q + p) − (s + r)) = 0
1
⇐⇒ 2
(q + p − s − r) = π for some k ∈ Z
⇐⇒ r + s − p − q = 2nπ for some n ∈ Z.
We are almost there; we now only need to show n = 1 in the final line. We know that
0 6 p < q < r < s < 2π, so r +s < 4π and 0 < p+q < r +s, so that 0 < r +s−p−q < 4π,
which means that n must equal 1 if P Q and RS are parallel.
Thus P Q and RS are parallel if and only if r + s − p − q = 2π.
Question 4
1
Use the substitution x = , where t > 1, to show that, for x > 0,
t2 −1
Z
1 √ √
p dx = 2 ln x + x + 1 + c.
x(x + 1)
1 1 t−a
Z
[Note: You may use without proof the result 2 2
dt = ln + constant. ]
t −a 2a t+a
Using the given substitution, we first use the chain rule to calculate
dx 2t
= −(t2 − 1)−2 · 2t = − 2 .
dt (t − 1)2
(We could alternatively have used the quotient rule to reach the same conclusion.)
We can now perform the requested substitution, simplifying the algebra as we go:
1 1 dx
Z Z
p dx = q · dt
x(x + 1) 1
· t2 dt
t2 −1 t2 −1
1 −2t
Z
= t
· 2 2
dt
2 (t − 1)
Z t −1
−2
= 2
dt
t −1
1 t−1
= −2 × ln +c using the given result
2 t+1
t+1
= ln + c.
t−1
R 9/16
To find the volume of revolution, we need to calculate the definite integral 1/8
πy 2 dx:
Z 9/16
πy 2 dx
1/8
9/16 2
1 1
Z
=π √ −√ dx
1/8 x x+1
9/16
1 1 1
Z
=π − 2p + dx
1/8 x x(x + 1) x + 1
h √ √ i9/16
= π ln x − 4 ln x + x + 1 + ln(x + 1) using the above result
1/8
p p p p
9 9 25 25 1 1 9 9
= π ln 16 − 4 ln 16
+ 16
+ ln 16
− π ln 8
− 4 ln 8
+ 8
+ ln 8
3 3 5 5 1 1 3 3
= π 2 ln 4 − 4 ln( 4 + 4 ) + 2 ln 4 − π 2 ln 2 2 − 4 ln 2 2 + 2 2 + 2 ln 2 2
√ √ √ √
= π (2 ln 3 − 2 ln 4) − 4 ln 2 + (2 ln 5 − 2 ln 4) −
√ √ √
π −2 ln(2 2) − 4 ln 2 + (2 ln 3 − 2 ln(2 2)
= π(2 ln 3 − 4 ln 2 − 4 ln 2 + 2 ln 5 − 4 ln 2)−
π(−3 ln 2 − 2 ln 2 + 2 ln 3 − 3 ln 2)
= π(−4 ln 2 + 2 ln 5)
= 2π(−2 ln 2 + ln 5)
= 2π ln 45 .
Question 5
We take the advice and begin by writing out the expansion of (1 + x)n :
n n 0 n 1 n 2 n n
(1 + x) = x + x + x + ··· + x , (∗)
0 1 2 n
where we have pedantically written in x0 and x1 in the first two terms, as this may well
help us to understand what we are looking at.
Now comparing this expansion to the expression we are interested in, we see that the only
difference is the presence of the xs. If we substitute x = 1, we will get exactly what we
want:
n n n n n
(1 + 1) = + + + ··· + ,
0 1 2 n
as all powers of 1 are just 1.
n n n n
(ii) +2 +3 + ··· + n = n2n−1 ;
1 2 3 n
For the rest of the question, there are two very distinct approaches, one via calculus and
one via properties of binomial coefficients.
This one looks a little more challenging, and we must observe carefully nthat
r there is no
n
0
term.
Comparing to the binomial expansion, we see that the term r x has turned
n
into r .r. Now, setting x = 1 will again remove the x, but where are we to get the r
from? Calculus gives us the answer: if we differentiate with respect to x, then xr becomes
rxr−1 , and then setting x = 1 will complete the job. Now differentiating (∗) gives
n−1 n 0 n 1 n 2 n
n(1 + x) = .1x + .2x + .3x + · · · + .nxn−1 ,
1 2 3 n
so by setting x = 1, we get the desired result.
We know that
n n!
=
r (n − r)!r!
so we can manipulate this formula to pull out an r, using r! = r.(r − 1)! and similar
expressions. We get
n n!
=
r (n − r)!r!
1 n!
= .
r (n − r)!(r − 1)!
n (n − 1)!
= .
r (n − r)!(r − 1)!
n n−1
=
r r−1
so that r nr = n n−1
r−1
. This is true as long as r > 1 and n > 1, so we get
n−1 n−1 n−1
n n n
+2 + ··· + n =n +n + ··· + n
1 2 n 0 1 n−1
= n.2n−1
where we have used the result from part (i) with n − 1 in place of n to do the last step.
n 1 n 1 n 1 n 1
2n+1 − 1 ;
(iii) + + + ··· + =
0 2 1 3 2 n+1 n n+1
Spurred on by our previous success, we see that now the n2 x2 term gives us n2 . 13 , so we
We can try rewriting our identity so that the 1r stays with the r − 1 term; this gives us
1 n−1
1 n
= .
n r r r−1
n n−1
Unfortunately, though, our expressions involve r−1 terms rather than r−1
terms, but
we can fix this by replacing n by n + 1 to get
1 n+1 1 n
= .
n+1 r r r−1
n 2 n 2 n 2 n
(iv) +2 +3 + ··· + n = n(n + 1)2n−2 .
1 2 3 n
This looks similar to (ii), in that we have increasing multiples. So we try differentiating (∗)
twice, giving us:
n−2 n n 0 n 1 n
n(n − 1)(1 + x) = .1.0 + .2.1x + .3.2x + · · · + .n(n − 1)xn−2 .
1 2 3 n
want. But no matter: we can just add r and we will be done, as r2 = r(r − 1) + r, and
we know from (ii) what terms like n2 .2 sum to give us. So we have, putting x = 1 in our
above expression:
n−2 n n n n
n(n − 1)(1 + 1) = .1.0 + .2.1 + .3.2 + · · · + .n(n − 1).
1 2 3 n
The left side simplifies to n(n + 1)2n−2 , and thus we are done.
An alternative (calculus-based) method is as follows. The first derivative of (∗), as we
have seen, is
n−1 n 0 n 1 n 2 n
n(1 + x) = .1x + .2x + .3x + · · · + .nxn−1 .
1 2 3 n
Now were we to differentiate again, we would end up with terms like n(n − 1)xn−2 , rather
than the desired n2 xk (for some k). We can remedy this problem by multiplying the whole
identity by x before we differentiate, so that we are differentiating
n−1 n 1 n 2 n 3 n
nx(1 + x) = .1x + .2x + .3x + · · · + .nxn .
1 2 3 n
Differentiating this now gives (using the product rule for the left hand side):
n−1 n−2 n 2 0 n 2 1 n 2 2 n
n(1+x) +n(n−1)x(1+x) = .1 x + .2 x + .3 x +· · ·+ .n2 xn−1 .
1 2 3 n
Substituting x = 1 into this gives our desired conclusion (after a small amount of algebra
on the left hand side).
As this looks
similar to
the result of part (ii), we can start with what we worked out there,
namely r nr = n n−1
r−1
, giving us
This looks verysimilar to the problem of part (ii) with n replaced by n − 1, but now the
multiplier of nr is r + 1 rather than r, and there is also an n
0
term. We can get over
n n n
the first problem by splitting up (r + 1) r as r r + r , so this expression becomes
The first line is just part (ii) with n replaced by n − 1, so that it sums to (n − 1).2n−2 , and
the second line is just 2n−1 = 2.2n−2 by part (i). So the answer to the original question
(remembering the factor of n we took out earlier) is
d2 y dy
(x − 1) 2 − x + y = 0. (∗)
dx dx
dy d2 y
If y = ex , then = ex and = ex . Substituting these into the left hand side of (∗)
dx dx2
gives
d2 y dy
(x − 1) 2 − x + y = (x − 1)ex − xex + ex = 0,
dx dx
so y = ex satisfies (∗).
In order to find other solutions of this differential equation, now let y = uex , where u
is a function of x. By substituting this into (∗), show that
d2 u du
(x − 1) 2
+ (x − 2) = 0. (∗∗)
dx dx
d2 u du
(x − 1) 2
+ (2x − 2 − x) + (x − 1 − x + 1)u = 0,
dx dx
which gives (∗∗) on simplifying the brackets.
du
By setting = v in (∗∗) and solving the resulting first order differential equation
dx
for v, find u in terms of x. Hence show that y = Ax + Bex satisfies (∗), where A and B
are any constants.
du d2 u dv
As instructed, we set = v, so that 2
= , which gives us
dx dx dx
dv
(x − 1) + (x − 2)v = 0.
dx
This is a standard separable first-order linear differential equation, so we separate the
variables to get
1 dv x−2
=−
v dx x−1
and then integrate with respect to x to get
1 x−2
Z Z
dv = − dx.
v x−1
Performing the integrations now gives us
1
Z
ln |v| = − 1 − dx
x−1
= −x + ln |x − 1| + c,
which we exponentiate to get
|v| = |k|e−x |x − 1|,
v = ke−x (x − 1).
We now recall that v = du/dx, so we need to integrate this last expression once more to
find u. We use integration by parts to do this, integrating the e−x part and differentiating
(x − 1), to give us
Z
u = ke−x (x − 1) dx
Z
= k(−e )(x − 1) − k(−e−x ).1 dx
−x
Relative to a fixed origin O, the points A and B have position vectors a and b, respec-
tively. (The points O, A and B are not collinear.) The point C has position vector c
given by
c = αa + βb,
where α and β are positive constants with α + β < 1. The lines OA and BC meet
at the point P with position vector p, and the lines OB and AC meet at the point Q
with position vector q. Show that
αa
p= ,
1−β
and write down q in terms of α, β and b.
The condition c = αa + βb with α + β < 1 and α and β both positive constants means
that C lies strictly inside the triangle OAB. Can you see why?
We start by sketching the setup so that we have something visual to help us with our
thinking.
Q
R
C
A
O P
The line OA has points with position vectors given by r1 = λa, and the line BC has
points with position vectors given by
−−→ −−→
r2 = OB + µBC = b + µ(c − b) = (1 − µ)b + µc.
p = λa = (1 − µ)b + µc
= (1 − µ)b + µ(αa + βb)
= (1 − µ + βµ)b + αµa.
Since a and b are not parallel, we must have 1 − µ + βµ = 0 and αµ = λ. The first
equation gives (1 − β)µ = 1, so µ = 1/(1 − β). This gives λ = α/(1 − β), so that
αa
p= .
1−β
Now swapping the roles of a and b (and hence also of α and β) will give us the position
vector of Q:
βb
q= .
1−α
We could approach this question in two ways, either by finding the point of intersection
of OC and AB or by showing that the given point lies on both given lines. We give both
approaches.
We require r to lie on OC, so r = λc, and r to lie on AB, so r = (1 − µ)a + µb, as before.
Substituting for c and equating coefficients gives
so that
αλ = 1 − µ
βλ = µ.
c αa + βb
r= = .
α+β α+β
OQ OS
The lines OB and P R intersect at the point S. Prove that = .
BQ BS
S lies on both OB and P R, so we need to find its position vector, s. Once again, we
require s = λb = (1 − µ)p + µr, so we substitute for p and r and compare coefficients:
s = λb = (1 − µ)p + µr
αa αa + βb
= (1 − µ) +µ
1−β α+β
(1 − µ)α(α + β)a + µ(1 − β)(αa + βb)
=
(1 − β)(α + β)
((1 − µ)α(α + β) + µα(1 − β))a + µ(1 − β)βb
=
(1 − β)(α + β)
α(α + β − µα − 2µβ + µ)a + µ(1 − β)βb
=
(1 − β)(α + β)
−→ −−→
Now, since OQ and BQ are both multiples of the vector q, we can compare the lengths OQ
and BQ in terms of their multiples of q. This might come out to be negative, depending
on the relative directions, but at the end, we can just consider the magnitudes.
βb
We thus have, since q = ,
1−α
OQ β . β
= −1
BQ 1−α 1−α
β . β − 1 + α
=
1−α 1−α
β
= ,
β−1+α
while
OS β . β
= −1
BS α + 2β − 1 α + 2β − 1
β . −α − β + 1
=
α + 2β − 1 α + 2β − 1
β
= .
−α − β + 1
Thus these two ratios of lengths are equal, as the magnitude of both of these is
β
.
1 − (α + β)
Question 8
(i) Suppose that a, b and c are integers that satisfy the equation
a3 + 3b3 = 9c3 .
Explain why a must be divisible by 3, and show further that both b and c must
also be divisible by 3. Hence show that the only integer solution is a = b = c = 0.
d3 + 3e3 = 9f 3 .
Note that this is the same equation that we started with, so if a, b, c are integers which
satisfy the equation, then so are d = a/3, e = b/3 and f = c/3. We can repeat this process
indefinitely, so that a/3n , b/3n and c/3n are also integers which satisfy the equation. But
if a/3n is an integer for all n > 0, we must have a = 0, and similarly for b and c.
Therefore the only integer solution is a = b = c = 0.
[In fact, we can say even more. If a, b and c are all rational, say a = d/r, b = e/s, c = f /t
(where d, e, f are integers and r, s, t are non-zero integers), then we have
d 3 e 3 f 3
+3 =9 .
r s t
Now multiplying both sides by (rst)3 gives
with dst, ert and f rs all integers, and so they must all be zero, and hence d = e = f = 0.
Therefore, the only rational solution is also a = b = c = 0.]
(ii) Suppose that p, q and r are integers that satisfy the equation
p4 + 2q 4 = 5r4 .
By considering the possible final digit of each term, or otherwise, show that p and q
are divisible by 5. Hence show that the only integer solution is p = q = r = 0.
So the last digits of fourth powers are all either 0, 5, 1 or 6, and of twice fourth powers
are all either 0 or 2.
Also, 5r4 is a multiple of 5, so it must end in a 0 or a 5.
Therefore if 2q 4 ends in 0 (that is, when q is a multiple of 5), the possibilities for the final
digit of p4 + 2q 4 are
(0 or 1 or 5 or 6) + 0 = 0 or 1 or 5 or 6,
so it can equal 5r4 (which ends in 0 or 5) only if p4 ends in 0 or 5, which is exactly when
p is a multiple of 5.
Similarly, if 2q 4 ends in 2 (so q is not a multiple of 5), the possibilities for the final digit
of p4 + 2q 4 are
(0 or 1 or 5 or 6) + 2 = 2 or 3 or 7 or 8,
so it can not be equal to 5r4 (which ends in 0 or 5).
Therefore, if p4 + 2q 4 = 5r4 , we must have p and q both being multiples of 5.
Now as in part (i), we write p = 5a and q = 5b, where a and b are both integers, to get
53 a4 + 2.53 b4 = (5c)4 ,
which yields
a4 + 2b4 = 5c4
on dividing by 53 .
So once again, if p, q, r give an integer solution to the equation, so do a = p/5, b = q/5
and c = r/5. Repeating this, so are p/5n , q/5n , r/5n , and as before, this shows that the
only integer solution is p = q = r = 0.
[Again, the same argument as before shows that this is also the only rational solution.]
This is an example of the use of Fermat’s Method of Descent, which he used to prove one
special case of his famous Last Theorem: he showed that x4 + y 4 = z 4 has no positive
integer solutions. In fact, he proved an even stronger result, namely that x4 + y 4 = z 2
has no positive integer solutions.
Another approach to solving the first step of part (ii) of this problem is to use modular
arithmetic, where we only consider remainders when dividing by a certain fixed number.
In this case, we would consider arithmetic modulo 5, so the only numbers to consider are
0, 1, 2, 3 and 4, and we want to solve p4 + 2q 4 ≡ 0 (mod 5), where ≡ means “leaves the
same remainder”. Now a quick calculation shows that p4 ≡ 1 unless p ≡ 0, while 2q 4 ≡ 2
unless q ≡ 0, so that
p4 + 2q 4 ≡ (0 or 1) + (0 or 2) ≡ 0, 1, 2 or 3 (mod 5)
2a 2b
α
The diagram shows a uniform rectangular lamina with sides of lengths 2a and 2b leaning
against a rough vertical wall, with one corner resting on a rough horizontal plane. The
plane of the lamina is vertical and perpendicular to the wall, and one edge makes an
angle of α with the horizontal plane. Show that the centre of mass of the lamina is a
distance a cos α + b sin α from the wall.
We start by redrawing the sketch, labelling the corners and indicating the centre of mass
as G, as well as showing various useful lengths.
A G
a C
b
2a 2b
α
B
It is now clear that the distance of G from the wall is a cos α (horizontal distance from
wall to midpoint of AB) plus b sin α (horizontal distance from midpoint of AB to G), so
a total of a cos α + b sin α.
Also, in case it is useful later, we note that the vertical distance above the horizontal
plane is, by a similar argument from the same sketch, a sin α + b cos α.
The coefficients of friction at the two points of contact are each µ and the friction is
limiting at both contacts. Show that
a cos(2λ + α) = b sin α,
where tan λ = µ.
There are two approaches to this. One is to indicate the reaction and friction forces
separately, while the other is to use the Three Forces Theorem. We show both of these.
Approach 1: All forces separately
We start by sketching the lamina again, this time showing the forces on the lamina,
separating the normal reactions from the frictional forces.
F1
A R1 G
C
R2
W
α
F2 B
Since friction is limiting at both points of contact, we have F1 = µR1 and F2 = µR2 .
Substituting these gives:
R(↑) µR1 + R2 − W = 0 (1)
R(→) R1 − µR2 = 0 (2)
y
M (A) W (a cos α + b sin α) − 2aR2 cos α + 2aµR2 sin α = 0 (3)
Equation (2) gives R1 = µR2 , so we can substitute this into (1) to get W = (1 + µ2 )R2 .
Substituting this into (3) now leads to
We can clearly divide both sides by R2 , and we are given that tan λ = µ, so we substitute
this in as well, to get
We spot 1 + tan2 λ = sec2 λ, and so multiply the whole equation through by cos2 λ, as the
form we are looking for does not involve sec λ:
Since the form we are going for is b sin α = a cos(2λ + α), we make use of double angle
formulæ, after rearranging:
The ‘Three Forces Theorem’ states that if three (non-zero) forces act on a large body in
equilibrium, and they are not all parallel, then they must pass through a single point.
(Why is this true? Let’s say two of the forces pass through point X. Taking moments
about X, the total moment must be zero, so the moment of the third force about X must
be zero. Therefore, the force itself is either zero or it passes through X. Since the forces
are non-zero, the third force must pass through X.)
In our case, we have a normal reaction and a friction force at each point of contact. We
can combine these into a single reaction force as shown in the sketch. Here we have
written N for the normal force, F for the friction and R for the resultant, which is at an
angle of θ to the normal.
R
F
θ
N
We see from this sketch that tan θ = F/N = µN/N = µ. In our case, since tan λ = µ we
must have θ = λ.
We can now redraw our original diagram with the three (combined) forces shown:
R2
R1
D
X
λ
A G C
λ
2a
α
O P B
W
We can now use the Three Forces Theorem is as follows. Looking at the diagram, we
know that the distance OB = 2a cos α = OP + P B. Now we know OP = a cos α + b sin α,
so we need only calculate P B.
But P B = P X tan λ (using the triangle P BX), and
P X = OA + height of X above A
= 2a sin α + (a cos α + b sin α) tan λ.
Putting these together gives
OB = 2a cos α = OP + P B
= a cos α + b sin α + (2a sin α + (a cos α + b sin α) tan λ) tan λ
= a cos α(1 + tan2 λ) + b sin α(1 + tan2 λ) + 2a sin α tan λ
= a cos α sec2 λ + b sin α sec2 λ + 2a sin α tan λ.
An alternative argument using the Three Forces Theorem proceeds by considering the
distance XP . Using the left half of the diagram, we have
XP = OA + OP tan λ
= 2a sin α + (a cos α + b sin α) tan λ.
From the right half of the diagram, we also have XP = P B/ tan λ, and P B = 2a cos α −
OP = a cos α − b sin α, so that
as we wanted.
π
Show also that if the lamina is square, then λ = 4
− α.
A particle P moves so that, at time t, its displacement r from a fixed origin is given by
r = et cos t i + et sin t j.
Show that the velocity of the particle always makes an angle of π4 with the particle’s
displacement, and that the acceleration of the particle is always perpendicular to its
displacement.
To find the velocity, v, and acceleration, a, we differentiate with respect to t (using the
product rule).
We have
r = et cos t i + et sin t j
From these, we can easily find the magnitudes of the displacement, velocity and acceler-
ation:
p
|r| = et (cos t)2 + (sin t)2 = et
p
|v| = et (cos t − sin t)2 + (sin t + cos t)2
p
t
= e 2 cos2 t + 2 sin2 t
√
= et 2
p
|a| = 2et (− sin t)2 + (cos t)2 = 2et .
We can now find the angles between these using a.b = 2|a||b| cos θ; firstly, for displace-
ment and velocity we have
Geometric-trigonometric approach
There is another way to find the angles involved which does not use the scalar (dot)
product.
Recall that the velocity is v = et (cos t − sin t)i + (sin t + cos t)j . We can use the
π π
Likewise, a makes an angle of 4
with v, and so an angle of 2
with r.
One way of thinking about the path of the particle is that its displacement at time t is
t
et from the origin and at an
given by r = e (cos t)i + (sin t)j , so that it is at distance
angle of t (in radians) to the x-axis (as (cos t)i+(sin t)j is a unit vector in this direction).
Thus its distance at time t = 0 is e0 = 1, and when it has gone a half circle, its distance
is eπ , which is approximately e3 ≈ 20. So the particle moves away from the origin very
quickly!
Another thing to bear in mind is that its velocity is always at an angle of π4 to its
displacement. Since it is moving away from the origin, its velocity is directed away from
the origin, so initially it is moving at an angle of π4 above the positive x-axis.
As we sketch the path, we also indicate the directions of the velocities at the times t = 0,
t = π2 and t = π.
eπ/2
−eπ 1 x
A second particle Q moves on the same path, passing through each point on the path
a fixed time T after P does. Show that the distance between P and Q is proportional
to et .
We write rP = r for the position vector of P and rQ for the position vector of Q. We
therefore have
rP = et cos t i + et sin t j
so that p
|rP − rQ | = et 1 − 2e−T cos T + e−2T ,
which is clearly proportional to et , as required, since T is a constant.
Question 11
Two particles of masses m and M , with M > m, lie in a smooth circular groove
on a horizontal plane. The coefficient of restitution between the particles is e. The
particles are initially projected round the groove with the same speed u but in opposite
directions. Find the speeds of the particles after they collide for the first time and show
that they will both change direction if 2em > M − m.
u1 = u u2 = −u
Before M m
v1 v2
After M m
M u1 + mu2 = M v1 + mv2
v2 − v1 = e(u1 − u2 ).
Then solving these equations (by (1) − m × (2) and (1) + M × (2)) gives
(M − m − 2em)u
v1 = (3)
M +m
(M − m + 2eM )u
v2 = . (4)
M +m
The speeds are then (technically) the absolute values of these, but we will stick with these
formulæ as they are what are needed later.
Now, the particles both change directions if v1 and v2 have the opposite signs from u1
and u2 , respectively, so v1 < 0 and v2 > 0. Thus we need
But (6) is always true, as M > m, so we only need M − m < 2em from (5).
After a further 2n collisions, the speed of the particle of mass m is v and the speed of
the particle of mass M is V . Given that at each collision both particles change their
directions of motion, explain why
mv − M V = u(M − m),
The fact that the particles both change their directions of motion at each collision means
that if they have velocities v1 and v2 after some collision, they will have velocities −v1 and
−v2 before the next collision. This is because they are moving around a circular track,
and therefore next meet on the opposite site, and hence are each moving in the opposite
direction from the one they were moving in. (We do not concern ourselves with precisely
where on the track they meet, and we are thinking of our velocities as one-dimensional
directed speeds.)
Therefore, mvm + M vM is constant in value after each collision, where vm is the velocity
of the particle of mass m, and vM that of the particle of mass M , but it reverses in
sign before the next collision. So after the first collision, it it M u − mu to the right (in
our above sketch), and hence after an even number of further collisions, it will still be
M vM + mvm = M u − mu to the right. But after an even number of further collisions,
the particle of mass M is moving to the left, so vM = −V , vm = v. Thus
mv − M V = (M − m)u.
Also, since there are a total of 2n+1 collisions, we have, by 2n+1 applications of Newton’s
Law of Restitution,
V + v = e2n+1 (u + u).
Solving these two equations simultaneously as before then yields
(2me2n+1 − M + m)u
V =
M +m
2n+1
(2M e + M − m)u
v= .
M +m
Question 12
A discrete random variable X takes only positive integer values. Define E(X) for this
case, and show that
X∞
E(X) = P (X > n).
n=1
For the definition of E(X), we simply plug the allowable values of X into the definition
of E(X) for discrete random variables, to get
∞
X
E(X) = n P(X = n).
n=1
Now, we can think of n, P(X = n) as the sum of n copies of P(X = n), so that we get
∞
X
E(X) = n P(X = n)
n=1
= 1.P(X = 1) + 2.P(X = 2) + 3.P(X = 3) + 4.P(X = 4) + · · ·
= P(X = 1) +
P(X = 2) + P(X = 2) +
P(X = 3) + P(X = 3) + P(X = 3) +
P(X = 4) + P(X = 4) + P(X = 4) + P(X = 4) + · · ·
Adding each column now gives us something interesting: the first column is P(X =
1) + P(X = 2) + P(X = 3) + · · · = P(X > 1), the second column is P(X = 2) + P(X =
3) + · · · = P(X > 2), the third column is P(X = 3) + P(X = 4) + · · · = P(X > 3), and
so on. So we get
as we wanted.
An alternative, more formal, way of writing this proof is as follows, using what is some-
times called “summation algebra”:
∞
X
E(X) = nP (X = n)
n=1
∞ X
X n
= P(X = n) summing n copies of a constant
n=1 m=1
X
= P(X = n) writing it as one big sum
16m6n<∞
X∞ X∞
= P(X = n) see below
m=1 n=m
X∞
= P(X > m),
m=1
which is the sum we wanted. For the penultimate step, note the we are originally summing
all pairs of values (m, n) where n is any positive integer and m lies between 1 and n, so
we have 1 6 m 6 n < ∞, as written on the third line. This can also be thought of as
summing over all pairs of values (m, n) where m is any positive integer (i.e., 1 6 m < ∞),
and n is chosen so that m 6 n < ∞, that is, we are summing on n from m to ∞.
[One final technical note: we are allowed to reorder the terms of this infinite sum because
all of the summands (the things we are adding) are non-negative. If some were positive
and others were negative, we might get all sorts of weird things happening if we reordered
the terms. An undergraduate course in Analysis will usually explore such questions.]
I am collecting toy penguins from cereal boxes. Each box contains either one daddy
penguin or one mummy penguin. The probability that a given box contains a daddy
penguin is p and the probability that a given box contains a mummy penguin is q,
where p 6= 0, q 6= 0 and p + q = 1.
Let X be the number of boxes that I need to open to get at least one of each kind of
penguin. Show that P(X > 4) = p3 + q 3 , and that
1
E(X) = − 1.
pq
We ask ourselves: what needs to happen to have X > 4? This means that we need to
open at least 4 boxes to get both a daddy and a mummy penguin. In other words, we
can’t have had both a daddy and a mummy among the first three boxes, so they must
have all had daddies or all had mummies. Therefore P(X > 4) = p3 + q 3 .
This immediately generalises to give P(X > n) = pn−1 + q n−1 , at least for n > 3. For
n = 1, P(X > 1) = 1, and for n = 2, P(X > 2) = 1 = p1 + q 1 , as we argued above.
Therefore, we have
∞
X
E(X) = P(X > n)
n=1
= 1 + (p1 + q 1 ) + (p2 + q 2 ) + (p3 + q 3 ) + · · ·
= (1 + p + p2 + p3 + · · · ) + (1 + q + q 2 + q 3 + · · · ) − 1
1 1
= + −1 adding the geometric series
1−p 1−q
1 1
= + −1 as p + q = 1
q p
p+q
= −1
qp
1
= −1 again using p + q = 1.
pq
1
To show that E(X) > 3, we simply need to show that pq
> 4. But this is the same as
showing that pq 6 41 , by taking reciprocals.
Now, recall that q = 1 − p, so we need to show that p(1 − p) 6 14 . To do this, we rewrite
the quadratic in p by completing the square:
p(1 − p) = p − p2 = 1
4
− (p − 12 )2 .
Since (p − 21 )2 > 0 for all p (even outside the range 0 < p < 1), we have p(1 − p) 6 41 , as
required, with equality only when p = q = 12 .
This can also be proved using calculus, or using the AM–GM inequality, or by writing
1/pq = (p + q)2 /pq and then rearranging to get (p − q)2 > 0.
Question 13
The number of texts that George receives on his mobile phone can be modelled by
a Poisson random variable with mean λ texts per hour. Given that the probability
George waits between 1 and 2 hours in the morning before he receives his first text is p,
show that
pe2λ − eλ + 1 = 0.
Given that 4p < 1, show that there are two positive values of λ that satisfy this
equation.
Let X be the number of texts George receives in the first hour of the morning and Y be
the number he receives in the second hour.
Then X ∼ Po(λ) and Y ∼ Po(λ), with X and Y independent random variables.
We thus have
P(George waits between 1 and 2 hours for first text) = P(X = 0 and Y > 0)
= P(X = 0).P(Y > 0)
= e−λ .(1 − e−λ )
= p,
so that e−λ − e−2λ = p.
Multiplying this last equation by e2λ gives eλ − 1 = pe2λ ; a straightforward rearrangement
yields our desired equation.
(This equation can also be deduced by considering the waiting time until the first text;
this is generally not studied until university, though.)
The solutions of the quadratic equation in eλ are given by
√
λ 1 ± 1 − 4p
e = .
2p
But we are given that 4p < 1, so that 1 − 4p > 0 and there are real solutions. We need
to show, though, that the two values of eλ that we get are both greater than 1, so that
the resulting values of λ itself are both greater than 0.
We have
√
1± 1 − 4p p
> 1 ⇐⇒ 1 ± 1 − 4p > 2p
2p
p
⇐⇒ ± 1 − 4p > 2p − 1.
Now, since 4p < 1, we have 2p√< 12 , so 2p − 1 < 0, from which it follows that for the
positive√sign in the inequality, 1 − 4p > 0 > 2p − 1. It therefore only remains to show
that − 1 − 4p > 2p − 1. But
p p
− 1 − 4p > 2p − 1 ⇐⇒ 1 − 4p < −(2p − 1)
⇐⇒ 1 − 4p < (2p − 1)2
⇐⇒ 1 − 4p < 4p2 − 4p + 1,
which is clearly true as 4p2 > 0. (We were allowed to square between the first and second
lines as both sides are positive.)
Thus the two solutions to our quadratic in eλ are both greater than 1, so there are two
positive values of λ which satisfy the equation.
The number of texts that Mildred receives on each of her two mobile phones can be
modelled by independent Poisson random variables but with different means λ1 and λ2
texts per hour. Given that, for each phone, the probability that Mildred waits between
1 and 2 hours in the morning before she receives her first text is also p, find an expression
for λ1 + λ2 in terms of p.
Each phone behaves in the same way as George’s phone above, so the two possible values
of λ are those found above. That is, the values of eλ1 and eλ2 are the two roots of
pe2λ − eλ + 1 = 0.
We know that the product of the roots of the equation ax2 + bx + c = 0 is c/a,1 so in our
case, eλ1 eλ2 = 1/p, so that eλ1 +λ2 = 1/p, giving
λ1 + λ2 = ln(1/p) = − ln p.
Find the probability, in terms of p, that she waits between 1 and 2 hours in the morning
to receive her first text.
Let X1 be the number of texts she receives on the first phone during the first hour and
Y1 be the number of texts that she receives on the first phone during the second hour.
Then X1 and Y1 are both distributed as Po(λ1 ), so
P(X1 = 0) = e−λ1
P(Y1 = 0) = e−λ1 .
Now let X2 and Y2 be the corresponding random variables for the second phone, so we
have
P(X2 = 0) = e−λ2
P(Y2 = 0) = e−λ2 .
We must now consider the possible situations in which she receives her first text between
1 and 2 hours in the morning. She must receive no texts on either phone in the first hour,
and at least one text on one of the phones in the second hour. We use the above result
that λ1 + λ2 = − ln p, so that e−λ1 −λ2 = p.
1
Why is this? If the roots of ax2 + bx + c = 0 are α and β, then we can write the quadratic as
a(x − α)(x − β) = a(x2 − (α + β)x + αβ), so that c = aαβ, or αβ = c/a. Likewise, b = −a(α + β) so that
α + β = −b/a.
Thus
Alternative approach
This approach uses a result which you may not have come across yet: the sum X + Y of
two independent Poisson random variables X ∼ Po(λ) and Y ∼ Po(µ) is itself a Poisson
variable with X + Y ∼ Po(λ + µ).
Since the number of texts received on the two phones together is the sum of the number
of texts received on each one, the total can be modelled by a Poisson random variable
with mean Λ = λ1 + λ2 texts per hour.
Then the probability of waiting between 1 and 2 hours in the morning for the first text is
given by q, where
qe2Λ − eΛ + 1 = 0,
using the result from the very beginning of the question, replacing p with q and λ with Λ.
Since Λ = λ1 + λ2 = ln(1/p) from above, eΛ = 1/p.
Therefore
eΛ − 1
q=
e2Λ
1/p − 1
=
(1/p)2
= p2 (1/p − 1)
= p(1 − p).
Hints & Solutions for STEP II 2010
1 When two curves meet they share common coordinates; when they “touch” they also share a
common gradient. In the case of the osculating circle, they also have a common curvature at the
dy d2 y
point of contact. Since curvature (a further maths topic) is a function of both and , the
dx dx 2
question merely states that C and its osculating circle at P have equal rates of change of gradient.
It makes sense then to differentiate twice both the equation for C and that for a circle, with
equation of the form (x – a)2 + (y – b)2 = r2, and then equate them when x = 14 . The three
resulting equations in the three unknowns a, b and r then simply need to be solved simultaneously.
dy d2 y
For y = 1 – x + tan x , = – 1 + sec2x and 2
= 2 sec2x tan x .
dx dx
2
dy d2 y dy
2 2 2
For (x – a) + (y – b) = r , 2(x – a) + 2(y – b) = 0 and 2 + 2(y – b) + 2 = 0 .
dx
2
dx dx
When x = 14 , y = 2 14 and so 14 a 2 14 b r 2 ;
2 2
dy ( x a)
= 1 then gives a relationship between a and b;
dx ( y b)
d2 y 4
and =4= gives the value of b.
dx 2
2( y b)
Working back then gives a and r.
2 The single-maths approach to the very first part is to use the standard trig. “Addition” formulae for
sine and cosine, and then to use these results, twice, in (i); firstly, to rewrite sin3x in terms of
sin3x so that direct integration can be undertaken; then to express cos3x in terms of cos3x in
order to get the required “polynomial” in cosx. Using the given “misunderstanding” in (ii) then
leads to a second such polynomial which, when equated to the first, gives an equation for which a
couple of roots have already been flagged. Unfortunately, the several versions of the question that
were tried, in order to help candidates, ultimately led to the inadvertent disappearance of the
interval 0 to in which answers had originally been intended. This meant that there was a little bit
more work to be done at the end than was initially planned.
cos3x = cos(2x + x) = cos2x cos x – sin2x sin x = (2c2 – 1)c – 2sc.s = (2c2 – 1)c – 2c(1 – c2)
= 4c3 – 3c .
sin3x = sin(2x + x) = sin2x cos x + cos2x sin x = 2sc.c + (1 – 2s2)s = 2s(1 – s2) + s(1 – 2s2)
= 3s – 4s3
(i) I () = 7 sin x 8 sin x dx = sin x 2 sin 3x dx = cos x cos 3x
3 2
3 0
0 0
3 You don’t have to have too wide an experience of mathematics to be able to recognise the
Fibonacci Numbers in a modest disguise here. (However, this is of little help here, as you should
be looking to follow the guidance of the question.) In (i), you are clearly intended to begin by
substituting n = 0, 1, 2 and 3, in turn, into the given formula for F n , using the four given terms of
the sequence. You now have four equations in four unknowns, and the given result in (i) is
intended to help you make progress; with (ii) having you check the formula in a further case. In
the final part, you should split the summation into two parts, each of which is an infinite geometric
progression.
a a 1
n n
Fn 1 1 1
(iii) n 1 = = using the
n0 2 2 n0 2 2 n0 2 2 5 1 14 1 5 2 5 1 14
1 5
S formula for the two GPs;
1 4 1 4
= .
2 5 3 5 2 5 3 5
2 3 5 2 3 5 2 2 5
Rationalising denominators then yields = = 1.
5 9 5 5 9 5 5 4
4 Hopefully, the obvious choice is y = a – x for the initial substitution and, as with any given
result, you should make every effort to be clear in your working to establish it. Thereafter, the two
integrals that follow in (i) use this result with differing functions and for different choices of the
upper limit a. Since this may be thought an obvious way to proceed, it is (again) important that
your working is clear in identifying the roles of f(x) and f(a – x) in each case. In part (ii), however,
it is not the first result that is to be used, but rather the process that yielded it. The required
substitution should, again, be obvious, and then you should be trying to mimic the first process in
this second situation.
f ( x) f (a x)
a a
Then 2 I = dx = 1. dx = x a = a I = 1
a.
f ( x) f (a x) 0 2
0 0
For f(x) = ln(1 + x) , ln(2 + x – x2) = ln[(1 + x)(2 – x)] = ln(1 + x) + ln(2 – x)
1
f ( x)
and ln(2 – x) = ln(1 + [1 – x]) = f(a – x) with a = 1 so that f ( x) f (1 x) dx =
0
1
2 .
/2 /2 /2
sin x sin x sin x
0
sin x 14
dx =
0
sin x. 12 cos x. 1
dx = 2
0
sin x sin 2 x
1
dx = 14 2 .
2
(ii) For u =
1 1
, du = 2 dx and 12 , 2 2, 12 .
x x
2
1 sin x
2
1 x sin x
0.5
.sin u1
1
Then . 0.5 x 2 . sin x sin 1x dx = 2 sin u1 sin u .– du
u
dx =
0.5
x sin x sin 1x
2
1 sin u1 2
1 sin 1x
= . du or x sin x sin 1x dx
.
0.5
u sin u sin u1 0.5
2
x dx = ln x0.5 = 2 ln 2
1
Adding then gives 2 I = 2
I = ln 2 .
0.5
5 The opener here is a standard bit of A-level maths using the scalar product, and the following
parts use this method, but with a bit of additional imagination needed. In 3-dimensions, there are
infinitely lines inclined at a given angle to another, specified line, and this is the key idea of the
final part of the question. Leading up to that, in (i), you need only realise that a line equally
inclined to two specified (non-skew) lines must lie in the plane that bisects them (and is
perpendicular to the plane that contains, in this case, the points O, A and B). One might argue that
the vector treatment of “planes” is further maths work, but these ideas are simple geometric ones.
(1, 1, 1) (5, 1, 1) 1
cos2 =
3. 27 3
mn p
For l 1 to be the angle bisector, we also require (e.g.) = cos , where
m2 n2 p2 . 3
cos2 = 2 cos2 – 1 = 1
3 cos = 2
3
, so that m + n + p = m2 n2 p 2 . 2 .
(ii) If you used the above method then you already have this relationship; namely,
2uv + 2vw + 2wu = u2 + v2 + w2 .
Thus, 2xy + 2yz + 2zx = x2 + y2 + z2 gives all lines inclined at an angle cos – 1 2
3
to OA and
hence describes the surface which is a double-cone, vertex at O, having central axis OA .
6 Although it seems that 3-dimensional problems are not popular, this is actually a very, very easy
question indeed and requires little more than identifying an appropriate right-angled triangle and
using some basic trig. and/or Pythagoras. There are thus so many ways in which one can approach
the three parts to this question that it is difficult to put forward just the one.
D
(i) Taking the midpoint of AB as the origin, O,
with the x-axis along AB and the y-axis along
OC, we have a cartesian coordinate system to
help us organise our thoughts. C
Then A = 12 , 0, 0 , B = 12 , 0, 0 ,
C = 0, 2
3
, 0 by trig. or Pythagoras, and A
P
P = 0, 6
3
, 0 . The standard distance formula
O
3 6 2
then gives PA (or PB) = 3 and PD = 3 or 3 .
B
1 3
(ii) The angle between adjacent faces is (e.g.) DOC = cos – 1 6 in right-angled triangle
1 3
2
–11
DOP, which gives the required answer, cos 3 .
(iii) D The centre, S, of the inscribed sphere must, by symmetry,
2
3
6
3
lie on PD, equidistant from each vertex.
3
6
r By Pythagoras, x2 = 1
12 + 6
9 2 3
6
x x2 x= 4
6
.
r
Then r = x sin(90o – (ii)) = 1
3 x= 6
12 .
3
S 6
Alternatively, if you know that the sphere’s centre is at
r x The centre of mass of the tetrahedron, the point (S)
with position vector 14 (a b c d) , then the answer
3
P 6 A 1 6
is just 4 DP = 12 .
7 The first two parts of the question begin, helpfully, by saying exactly what to consider in order to
proceed, and the material should certainly appear to be routine enough to make these parts very
accessible. Where things are going in (iii) may not immediately be obvious but, presumably, there
is a purpose to (i) and (ii) which should become clear in (iii).
dy
(i) y = x3 – 3qx – q(1 + q) = 3(x2 – q) = 0 for x q .
dx
When x = q , y = q q 1 < 0 since q > 0
2
q , y = q q 1 < 0
2
When x = since q > 0 and q 1
Since both TPs below x-axis, the curve crosses the x-axis once only (possibly with sketch)
q q2 q3
(ii) x = u + x3 = u 3 3uq 3 3
u u u
q2 q3 q2
0 = x3 – 3qx – q(1+ q) = u 3 3uq 3 3 – 3qu – 3 q q 2
u u u
3
q 2
u3 + 3 q(1 q) = 0 or u 3 q(1 q) u 3 q 3 0
u
u =
q(1 q) q 2 (1 q) 2 4q 3
3
2
=
q
2
1 q 1 2 q q 2 4q
=
q
2
2 q
2
1 q 1 q = 1 q (1 q ) = q or q2
1 2 1 2
giving u = q 3 or q 3 and x = q 3 + q 3
y = e–x
O 2 3 4
y = – e–x
–1
e
sin x dx attempted by parts = e x . cos x e x . cos x dx or e x . sin x e x . sin x dx
x
(depending on your choice of ‘1st’ and ‘2nd’ part) = e x . cos x e x . sin x e x . sin x dx .
x
Then I = e (cos x sin x) – I (by “looping”) = 12 e (cos x sin x)
x
xn 1 xn 1
= 12 e
12 ( 4 n 1)
0 1 2 12 e ( 4 n 3) 0 1 2 =
1
2 1
2 e
12 ( 4 n 1)
1 e 2
Note that A 1 = 12 e
52
e 2
1 and A n + 1 = e 2 A n so that
A
n 1
n
= A1 1 e 2 e 2 ...
2
e 2
= 12 e
52
e 2
1 1
1 e 2
= 1 2
2 e
5
e 2
1 2
e 1
1
(using the S of a GP formula) = 12 e 2
9 Once you have written down all relevant possible equations of motion, this question is really quite
simple; the two results you are asked to prove arise from considering either times or distances to
the point of collision. There is, however, one crucial realisation to make in the process, without
which further progress is almost impossible; once noted, it seems terribly obvious, yet it probably
doesn’t usually fall within the remit of standard A-level examination questions.
For P 1 , x1 0 , x1 u cos , x1 ut cos , y1 g , y1 u sin gt , y1 ut sin 12 gt 2
For P 2 , x 2 0 , x 2 v cos , x 2 vt cos , y 2 g , y 2 v sin gt , y 2 vt sin 12 gt 2
u sin u 2 sin 2
Now P 1 is at its greatest height when y 2 0 t = y1 = h = and it follows
g 2g
that u sin = 2 gh
Note that if the two particles are at the same height at any two distinct times (one of which is t = 0
here), then their vertical speeds are the same throughout their motions. Thus u sin = v sin .
2v sin
y 2 = 0, t 0 t = . This is the time when P 2 would land. Also, the collision occurs
g
b
when x 2 = b t = is the time of the collision.
v cos
Then t P2 1
2 range < t(collision) < t P2 range (or by distances)
v sin b 2v sin v 2 sin cos 2v 2 sin cos
< < <b<
g v cos g g g
(v sin ) 2 2(v sin ) 2
cot < b < cot . Using u sin = v sin = 2 gh then gives
g g
2 gh 4 gh
cot < b < cot 2h cot < b < 4h cot .
g g
One could repeat all this work for P 1 , but this is not necessary. Since the particles are at their
maximum heights simultaneously (see the above reasoning) and would achieve their “ranges”
simultaneously also, we have 2h cot < a < 4h cot .
10 I always feel that collisions questions are very simple, since (as a rule) there are only the two main
principles – Conservation of Linear Momentum and Newton’s Experimental Law of Restitution –
to be applied. Such is the case here. Part (ii) is only rendered more difficult by the introduction of
a number of repetitions, and then the question concludes with some pure mathematical work using
logarithms.
2 2 2
2
11 A few years ago, a standard “three-force” problem such as this would have elicited responses
using Lami’s Theorem; since this tidy little result seems to have lapsed from the collective A-level
consciousness, I shall run with the more popular, alternative Statics-question approach of
resolving twice and taking moments. In order to get started, however, it is important to have a
good, clear diagram suitably marked with correct angles. The later parts of the question consist
mostly of trignometric work.
R T
Res. T sin( ) + R sin( + ) = W
B
l Res. T cos( ) = R cos( + )
C
l A W.2l cos = T.3l sin
2l
A
W
T cos( ) 3T sin
Substituting to eliminate T ’s (e.g.) T sin( ) + sin( + ) =
cos( ) 2 cos
2 cos cos . cos sin . sin sin . cos cos . sin
+ 2 cos cos . cos sin . sin sin . cos cos . sin
= 3 sin cos . cos sin . sin
Dividing by cos cos cos
2(cos tan . sin )(tan . cos sin ) 2(cos tan . sin )(tan . cos sin )
= 3 tan (1 tan . tan )
Multiplying out, cancelling and collecting up terms, and then dividing by tan tan then gives
the required answer 2 cot + 3 tan = cot .
1
= 30o, = 45o cot = 2.1 3. = 2 3 ,
3
o
and tan15 = tan(60 – 45 ) =
3 1
o
o
3 1
2
2 3
1
.
1 3 3 1 2 3
12 In some ways, the pdf f(x) couldn’t be much simpler, consisting of just two horizontal straight-
line segments (in the non-zero part). Part (i) is then relatively routine – use “total prob. = 1” to
find the value of k, before proceeding to find E(X); and the trickiest aspect of (ii) is in the
inequalities work. You also need to realise that the median could fall in either of the two non-zero
regions. For (iii), it is necessary only to follow through each possible value of M relative to E, the
expectation.
Since the pdf is only non-zero between 0 & 1, and the area under its graph = 1, if a, b are both <
(>) 1 then the total area will be < (>) 1. Since we are given that a > b, it must be the case that
a > 1 and b < 1.
1 b
1 k 1
(i) 1 = f ( x) dx = a dx + b dx = ax + bx = ak + b – bk k =
k 1
.
0 0 k 0 k
a b
1 k 1
ax 2 k bx 2 1 ak 2 b bk 2
E(X) = 0 xf ( x ) dx = 0 ax dx + k bx dx = 2
+
0 2 k
=
2
2 2
b ( a b) 1 b ba b 2 1 2b b 2 1 2b ab
2
= = = .
2 2 a b 2( a b ) 2(a b)
a ab
(ii) If ak 1
(i.e. M (0, k)) then 1
2a – 2ab a – b a + b 2ab
ab
2 2
1
1
and aM = 2 or M = .
2a
If ak (i.e. M (k, 1)), and noting that this is equivalent to a + b 2ab ,
1
2
1
then ak + (M – k)b = 12 or (1 – M)b = 12 M = 1 –
2b
1 2b ab 1 a 2ab a 2 b a b b(1 a ) 2
(iii) If a + b 2ab , then – M = = = >0
2(a b) 2a 2a ( a b) 2a ( a b)
and the required result follows.
1 2b ab 1 b 2b 2 ab 2 2ab 2b 2 a b
If a + b 2ab , then – M = 1 =
2(a b) 2b 2b(a b)
a (1 b) 2
= > 0 as required.
2b(a b)
13 This question is really little more than examining the various cases that arise for each outcome and
then doing a little bit of work algebraically. The result of part (i) is somewhat counter-intuitive, in
that Rosalind should choose to play the more difficult opponent twice, while one intutively feels
she should be playing the easier opponent. The real issue, however, is that she needs to beat both
opponents (and not just win one game): examining the probabilities algebraically makes this very
obvious. Part (ii) is a nice adaptation, where there is a cut-off point separating the cases when one
strategy is always best from another situation when either strategy 1 or 2 can be best. Here, it is
most important to demonstrate that the various conditions hold, and not simply state a couple of
probabilities and hope they do the job. [It is perfectly possible to do (iii) by “trial-and-error”, but I
have attempted to reproduce below an approach which incorporates a method for deciding the
matter.]
P(W 1 ) – P(W 3 ) = pq(q – p) 3 [ p q ] > 0 since q > p and p + q < 2 < 3 so that SI is
always better than S3
P(W 1 ) – P(W 2 ) = pq p 2 p 1 pq 2q = pq (2 p )(q p ) (1 p )
1 p 1
> 0 whenever q – p > 1 .
2 p 2 p
1
Now p + 12 < q < 1 0 < p < 12 13 < 1 < 12 , so that SI always better than SII
2 p
when q – p > 12 .
1 p
P(W 1 ) – P(W 2 ) > < 0 q – p > < .
2 p
1 p 3
Take p = 1
,q= 1
q–p= 1
< 1
and 7 > 14 so SII is better than SI.
4 2 4 2
2 p
1 p 3
Take p = 14 , q = 34 q – p = 12 < 12 and so choosing
2 p 7
1 p 3
< 73 12 141 (say 161 ) will give p = 14 , q = 16
11
and q – p = 167 > so that
2 p 7
SI is better than SII.
[I believe that q – p > k has k = 12 as the least positive k which always gives SI better than SII,
but it is a long time ago that the problem was originally devised and I may be wrong.]
STEP Mathematics III 2010: Solutions
1. The first two parts are obtained by separating off the final term of the summation and
expanding the brackets respectively giving , and
1
1
coth
2 cosh 2
and in the second
1
1 1 1
tan tan
1
2 sinh
2 sinh 2 2
or alternatively
4 cosh
2
1, 1 1 1 so 1,
1 1 1 so 1
1 1 1 so 1
1 1 1 1 1 1 so 1
1⇒ 1 0⇒ 1 ⋯ 1
1 is the only non-primitive root as no power of any other root less than the pth equals unity,
because p is prime, so ⋯ 1
and
so satisfies 0.
On the other hand if there is a common root, then it is found at the start of the question and as it
satisfies 0, the required result is found.
If 0 and , then and so the two
equations are one and trivially have a common root. Alternatively, if there is a common root and
, then the initial subtraction yields , and so the result is trivially true.
8. Substituting for , the desired integral is seen to be the reverse of the quotient rule, i.e.
arbitrary constant.
(ii) Rearranging the equation to be solved as , the
integrating factor is
As a result, the RHS we require to integrate is
Repeating similar working to part (i), except with 1 cos 2 sin and
sin cos , gives three linearly dependent equations,
5 2 , 3 2 ,4
Choosing e.g. 4, 5, 0, the solution is 4 5 sin 1 cos 2 sin
Section B: Mechanics
The initial velocity of the bullet relative to the block is – and the final velocity of the bullet
relative to the block is 0. If the time between the bullet entering the block and stopping moving
through the block is T, then using” “, 0
For the block, the initial velocity is 0 , the final velocity is , and again using ,
and so
as required.
If the distance moved by the block whilst the bullet is moving through the block is ,
using” 2 “, 2 andso
Once the bullet stops moving through the block, the next initial velocity of block/bullet is , the
final velocity is 0, the acceleration is – , so the distance moved using
“ 2 ” is given by 0 2 i.e.
Thus the total distance moved is
If , then the block does not move, and the bullet penetrates to a depth .
13. , 0
1 1 0
1 1
1 1
As 0 and 1,
,
, ,
1 1
0 is given.
1 implies 1
, implies as seen before.
, implies 1
and hence , , 1
for 1,2,3 as ,
, and
, , as a linear transformation does not affect correlation.
STEP Solutions
2011
Mathematics
STEP 9465/9470/9475
October 2011
The Cambridge Assessment Group is Europe's largest assessment agency
and plays a leading role in researching, developing and delivering
assessment across the globe. Our qualifications are delivered in over 150
countries through our three major exam boards.
All Examiners are instructed that alternative correct answers and unexpected
approaches in candidates’ scripts must be given marks that fairly reflect the
relevant knowledge and skills demonstrated.
© UCLES 2010
Report Page
STEP Mathematics I 4
STEP Mathematics II 46
STEP Mathematics III 56
STEP I 2011 Solutions
Question 1
a b ay 2
(i) Show that the gradient of the curve + = 1, where b 6= 0, is − 2 .
x y bx
We begin by differentiating the equation of the curve (ax−1 + by −1 = 1) implicitly with respect
to x, to get
dy
−ax−2 − by −2 = 0,
dx
so that
b dy a
− 2 = 2,
y dx x
giving our desired result
dy ay 2
= − 2.
dx bx
An alternative, but more complicated method, is to rearrange the equation first to get y in terms
of x before differentiating. We have, on multiplying by xy,
ay + bx = xy, (1)
The challenge is now to rewrite this in the form required. We can rearrange equation (1) to get
(x − a)y = bx, so that (x − a) = bx/y. Substituting this into our expression for the derivative
then gives
dy ab aby 2 ay 2
=− = − = −
dx (bx/y)2 b2 x2 bx2
as required.
STEP I 2011 Question 1 continued
a b
The point (p, q) lies on both the straight line ax + by = 1 and the curve + = 1,
x y
where ab 6= 0. Given that, at this point, the line and the curve have the same gradient,
show that p = ±q .
aq 2 a
− 2
=− ,
bp b
Since (p, q) lies on both the straight line and the curve, it must satisfy both equations, so
a b
ap + bq = 1 and + = 1.
p q
Now if p = q, then the first equation gives (a + b)p = 1 and the second gives (a + b)/p = 1, and
multiplying these gives (a + b)2 = 1.
Alternatively, if p = −q, then the first equation gives (a − b)p = 1 and the second equation gives
(a − b)/p = 1, and multiplying these now gives (a − b)2 = 1.
(ii) Show that if the straight line ax + by = 1, where ab 6= 0, is a normal to the curve
a b
− = 1, then a2 − b2 = 12 .
x y
We can find the derivative of this curve as above. A slick alternative is to notice that this is
identical to the above curve, but with b replaced by −b, so that
dy ay 2
= 2.
dx bx
The gradient of the straight line is −a/b as before, so as this line is normal to the curve at the
point (p, q), say, we have
aq 2 a
− = −1
bp2 b
as perpendicular gradients multiply to −1; thus a2 q 2 /b2 p2 = 1, or a2 q 2 = b2 p2 .
a
We therefore deduce that aq = ±bp, which we can divide by pq 6= 0 to get p = ± qb .
Now since (p, q) lies on both the straight line and the curve, we have, as before,
a b
ap + bq = 1 and − = 1.
p q
STEP I 2011 Question 1 continued
Now if ap = qb , the second equation would become 0 = 1, which is impossible. So we must have
a b
p = − q , giving
a a
+ = 1,
p p
a
so that p = − qb = 21 , giving p = 2a and q = −2b.
Substituting this into the equation of the straight line yields
a.2a + b.(−2b) = 1,
1
so that a2 − b2 = 2 as required.
Question 2
1
ex
Z
The number E is defined by E = dx .
0 1+x
Show that
1
xex
Z
dx = e − 1 − E,
0 1+x
1
x 2 ex
Z
and evaluate dx in terms of e and E.
0 1+x
as required.
We can play the same trick with the second integral, as
x2 1
=x−1+ ,
1+x 1+x
so that
1 Z 1
x 2 ex 1 x
Z
dx = x−1+ e dx
0 1+x 0 1+x
Z 1 Z 1 1
ex
Z
= xex dx − ex dx + dx.
0 0 0 1+x
Now we can use integration by parts for the first integral to get
Z 1 Z 1
1
xex dx = xex 0 − ex dx
0 0
= e − (e − 1)
= 1.
Therefore
1
x2 e x
Z
dx = 1 − (e − 1) + E = 2 − e + E.
0 1+x
STEP I 2011 Question 2 continued
Approach 2: Substitution
This is essentially identical to the first approach. The second integral follows in the same way.
The difficulty is now integrating the remaining integral. We again use parts, this time taking
dv 1
u = ex and =
dx (1 + x)2
so that
du 1
= ex and v=− .
dx 1+x
This gives
1
ex 1
Z 1
ex ex
Z
dx = − − − dx
0 (1 + x)2 1+x 0 0 1+x
= − 21 e + 1 + E.
For the second integral, we use a similar procedure, this time taking
x2 dv
u= and = ex
1+x dx
so that
du 2x + x2
= and v = ex .
dx (1 + x)2
We then get
1 2 x 1 Z 1
x2 e x x e (2x + x2 )ex
Z
dx = − dx
0 1+x 1+x 0 0 (1 + x)2
Z 1
(2x + x2 )ex
= 21 e − dx.
0 (1 + x)2
The integral in the last step can be handled in several ways; the easiest is to write
2x + x2 x2 + 2x + 1 − 1 1
2
= =1−
(1 + x) (1 + x)2 (1 + x)2
1 1
x2 e x ex
Z Z
x
dx == 12 e − e − dx
0 1+x 0 (1 + x)2
= 12 e − [ex ]10 + (− 12 e + 1 + E)
= −e + 1 + 1 + E
= 2 − e + E.
STEP I 2011 Question 2 continued
1−x
1
e 1+x
Z
(i) dx ;
0 1+x
This integral looks to be of a vaguely similar form, but with a more complicated exponential
part. We therefore try the substitution u = 1−x
1+x and see what we get.
1−x
If u = , then
1+x
du −(1 + x) − (1 − x) −2
= = ,
dx (1 + x)2 (1 + x)2
dx
so that du = − 12 (1 + x)2 . Also, when x = 0, u = 1, and when x = 1, u = 0.
1−x
We can also rearrange u = to get
1+x
(1 + x)u = 1 − x
so ux + x = 1 − u
1−u
giving x= .
1+u
Thus
1−x
1 0
e 1+x eu
Z Z
− 12 (1 + x)2 du
dx =
0 1+x 1 1+x
Z 1
1 u
= 2 e (1 + x) du reversing the limits
0
1
1−u
Z
1 u
= 2e 1+ du
0 1+u
1
2
Z
1 u
= 2e du
0 1+u
1
eu
Z
= du
0 1+u
= E.
STEP I 2011 Question 2 continued
√
Again we have a different exponent, so we try substituting
√ u = x2 , so that x = u, while
du dx 1
dx = 2x and so du = 2x . Also the limits x = 1 and x = 2 become u = 1 and u = 2, giving us
√
2 2 2
ex eu 1
Z Z
dx = du
1 x 1 x 2x
2
eu
Z
= du.
1 2u
This is very similar to what we are looking for, except that it has the wrong limits and a
denominator of 2u rather than u + 1 or perhaps 2(u + 1). So we make a further substitution:
u = v + 1, so that v = u − 1 and du/dv = 1, giving us
√
2 2 2
ex eu
Z Z
dx = du
1 x 1 2u
Z 1
ev+1
= dv
0 2(v + 1)
e 1 ev
Z
= dv
2 0 v+1
eE
= ,
2
where on the penultimate line we have written ev+1 = e.ev and so taken out a factor of e/2.
It is also possible to evaluate this integral more directly by substituting u = x2 − 1, so that
x2 = u + 1. The details are left to the reader.
Question 3
(These can be derived by expanding the right hand sides using the addition formulæ, and then
collecting like terms.)
Then initially taking A = 13 π − θ and B = 31 π + θ and using the first of the factor formulæ gives
= 2 sin θ(cos 2θ + 12 )
= 2 sin θ cos 2θ + sin θ.
We now use the second factor formula with A = θ and B = 2θ to simplify this last expression to
sin 3θ + sin(−θ) + sin θ = sin 3θ,
as required.
An alternative approach is to expand the second and third terms on the left hand side using the
addition formulæ, giving:
while
sin 3θ = sin(2θ + θ)
= sin 2θ cos θ + cos 2θ sin θ
= 2 sin θ cos2 θ + (cos2 θ − sin2 θ) sin θ
= 3 sin θ cos2 θ − sin3 θ.
We can differentiate a product of several terms using the product rule repeatedly. In general,
we have
d du dv dw
(uvwt . . .) = vwt . . . + u wt . . . + uv t... + ···
dx dx dx dx
In our case, we are differentiating a product of three terms, and we get
Now we are aiming to get an expressing involving cot, so we divide this result by (∗) to get
(ii) By setting θ = 61 π −φ in (∗), or otherwise, obtain a similar identity for cos 3θ and deduce
that
cot θ cot( 13 π − θ) cot( 13 π + θ) = cot 3θ .
To get cosines from this expression, we will need to use the identity sin( 12 π − x) = cos x. So we
rewrite this as
which is a similar identity for cos 3φ. Replacing φ by θ and reordering the terms in the product
gives
4 cos θ cos( 13 π − θ) cos( 13 π + θ) = cos 3θ.
Now dividing this identity by (∗) gives our desired identity for cot:
Show that √
cosec 19 π − cosec 95 π + cosec 79 π = 2 3 .
As before, we differentiate the expression (†) which we have just derived to get
When we negate this identity and then divide it by (†), we will have lots of cancellation and we
will be left with terms of the form cosec2 x/ cot x. Now
cosec2 x 1 sin x 1 2
= 2 . = = = 2 cosec 2x,
cot x sin x cos x sin x cos x sin 2x
so that the division gives us
To get the requested equality, we halve this identity and set 2θ = 91 π so that
√
cosec 19 π − cosec 59 π + cosec 79 π = 3 cosec 13 π = 3. √23 = 2 3
as required.
Question 4
The distinct points P and Q, with coordinates (ap2 , 2ap) and (aq 2 , 2aq) respectively, lie on
the curve y 2 = 4ax. The tangents
to the curve at P and Q meet at the point T . Show that
T has coordinates apq, a(p + q) . You may assume that p 6= 0 and q 6= 0.
We begin by sketching the graph (though this may be helpful, it is not required):
y
P
T φ
F x
The equation of the curve is y 2 = 4ax, so we can find the gradient of the curve by implicit
differentiation:
dy
2y = 4a,
dx
and thus
dy 2a
= ,
dx y
as long as y 6= 0. (Alternatively, we could write x = y 2 /4a and then work out dx/dy = 2y/4a;
taking reciprocals then gives us the same result.)
Therefore the tangent at the point P with coordinates (ap2 , 2ap) has equation
2a
y − 2ap = (x − ap2 ),
2ap
which can easily be rearranged to give
x − py + ap2 = 0.
Since y = 0 would require p = 0, we can ignore this case, as we are assuming that p 6= 0. [In
y
fact, if y = p = 0, we can look at the reciprocal of the gradient, dx
dy = 2a , and this is zero, so the
line is vertical. In this case, our equation gives x = 0, which is, indeed, a vertical line, so our
equation works even when p = 0.]
Thus the tangent through P has equation x − py + ap2 = 0 and the tangent through Q has
equation x − qy + aq 2 = 0 likewise.
We solve these equations simultaneously to find the coordinates of T . Subtracting them gives
(p − q)y − a(p2 − q 2 ) = 0.
STEP I 2011 Question 4 continued
y − a(p + q) = 0,
The point F has coordinates (a, 0) and φ is the angle T F P . Show that
pq + 1
cos φ = p
(p2 + 1)(q 2 + 1)
so that
TF2 + PF2 − TP2
cos φ = .
2.T F.P F
Now using Pythagoras to find the distance between two points given their coordinates, we obtain
2 2
T F 2 = a(pq − 1) + a(p + q)
= a2 (p2 q 2 − 2pq + 1 + p2 + 2pq + q 2 )
= a2 (p2 q 2 + p2 + q 2 + 1)
= a2 (p2 + 1)(q 2 + 1)
2
F P 2 = a(p2 − 1) + (2ap)2
= a2 (p4 − 2p2 + 1 + 4p2 )
= a2 (p4 + 2p2 + 1)
= a2 (p2 + 1)2
2 2
T P 2 = a(pq − p2 ) + a(p + q − 2p)
2 2
= ap(q − p) + a(q − p)
= a2 (p2 + 1)(q − p)2
Thus
so that
2a2 (1 + p2 )(1 + pq)
cos φ = p
2a2 (p2 + 1)(q 2 + 1)(p2 + 1)2
1 + pq
=p
(p + 1)(q 2 + 1)
2
as we wanted.
STEP I 2011 Question 4 continued
An alternative approach is to use vectors and dot products to find cos φ. We have
−−→ −→
F P .F T = F P.F T. cos φ
(where the dot on the left hand side is the dot product, but on the right is ordinary multiplica-
tion), so we need only find the lengths F P , F T as above and the dot product. The dot product
is
2
−−→ −→
ap − a apq − a
F P .F T = . = (ap2 − a)(apq − a) + 2ap.a(p + q)
2ap − 0 a(p + q) − 0
= a2 (p2 − 1)(pq − 1) + 2a2 (p2 + pq)
= a2 (p3 q − p2 − pq + 1 + 2p2 + 2pq)
= a2 (p3 q + p2 + pq + 1)
= a2 (p2 (pq + 1) + pq + 1)
= a2 (p2 + 1)(pq + 1)
Therefore we deduce
−−→ −→
F P .F T
cos φ =
F P.F T
a2 (p2 + 1)(pq + 1)
= p
a(p2 + 1).a (p2 + 1)(q 2 + 1)
pq + 1
=p
(p + 1)(q 2 + 1)
2
as required.
Now to show that the line F T bisects the angle P F Q, it suffices to show that φ is equal to the
angle T F Q (see the sketch above).
Now we can find cos(∠T F Q) by using the above formula and swapping every p and q in it, as
this will swap the roles of P and Q.
But swapping every p and q does not change the formula, so cos(∠T F Q) = cos(∠T F P ), and
so ∠T F Q = ∠T F P as both angles are strictly less than 180◦ and cosine is one-to-one in this
domain.
Thus the line F T bisects the angle P F Q, as required.
Question 5
Given that 0 < k < 1, show with the help of a sketch that the equation
sin x = kx (∗)
We sketch the graph of y = sin x in the range 0 6 x 6 π along with the line y = kx.
d
Now since dx (sin x) = cos x, the gradient of y = sin x at x = 0 is 1, so the tangent at x = 0 is
y = x. We therefore also sketch the line y = x.
y y=x
y = kx
0 α π x
It clear that there is at most one intersection of y = kx with y = sin x in the interval 0 < x < π,
and since 0 < k < 1, there is exactly one, as the gradient is positive and less that that of
y = sin x at the origin. (If k 6 0, there would be no intersections in this range as kx would be
negative or zero; if k > 1, the only intersection would be at x = 0.)
Let Z π
I= sin x − kx dx .
0
Show that
π 2 sin α
I= − 2 cos α − α sin α ,
2α
where α is the unique solution of (∗).
It is a pain to work with absolute values (the “modulus function”), so we split the integral
into two integrals: in the interval 0 6 x 6 α, sin x − kx > 0, and in the interval α 6 x 6 π,
sin x − kx 6 0. So
Z π
I= sin x − kx dx
0
Z α Z π
= sin x − kx dx + sin x − kx dx
Z0 α Z πα
= sin x − kx dx + − sin x + kx dx
0 α
α π
= − cos x − 12 kx2 0 + cos x + 21 kx2 α
π 2 sin α
= −2 cos α − α sin α +
2α
where the last line follows using kα = sin α so that k = (sin α)/α, and we have reached the
desired result.
Show that I, regarded as a function of α, has a unique stationary value and that this stationary
value is a minimum. Deduce that the smallest value of I is
π
−2 cos √ .
2
π 2 α cos α − sin α
dI
= + 2 sin α − sin α − α cos α
dα 2 α2
π2
= (sin α − α cos α) − 2 (sin α − α cos α)
2α
π2
= 1 − 2 (sin α − α cos α)
2α
dI
√
so dα = 0 if and only if 2α2 = π 2 or sin α = α cos α. The former condition gives α = ±π/ 2,
while the latter condition gives tan α = α.
A quick sketch of the tan graph (see below) shows that tan α = α has no solutions in the range
d
0 < α < π (though α = 0 is a solution); the sketch uses the result that dx (tan x) = sec2 x, so
the tangent to y = tan x at x = 0 is y = x.
y y=x
π
2 π
x
√ √
Thus the only solution in the required range is α = π/ 2 (and note that π/ 2 < π).
To ascertain whether it is a maximum, a minimum or a point of inflection, we could either
look at the values of I or dI/dα at this point and either side or we could consider the second
derivative.
√
Either way, we will eventually have to work out the value of I when α = π/ 2, so we will do so
now:
STEP I 2011 Question 5 continued
√
π 2 sin(π/ 2) π π π
I= √ − 2 cos √ − √ sin √
2π/ 2 2 2 2
π π π π π
= √ sin √ − 2 cos √ − √ sin √
2 2 2 2 2
π
= −2 cos √ .
2
We have
d2 I π2 π2
= 3 (sin α − α cos α) + 1 − 2 (cos α − cos α + α sin α)
dα2 α 2α
2 π2
π
= 3 (sin α − α cos α) + 1 − 2 α sin α
α 2α
√
Now when α = π/ 2, so that π 2 /2α2 = 1, we have
d2 I π2
= (sin α − α cos α).
dα2 α3
√ d2 I
Since α = π2 2 > π2 , we have sin α > 0 and cos α < 0, so dα2
> 0 and I has a local minimum at
this value of α.
Question 6
Use the binomial expansion to show that the coefficient of xr in the expansion of (1 − x)−3 is
1
2 (r + 1)(r + 2) .
Using the formula in the formula book for the binomial expansion, we find that the xr term is
−3 (−3)(−4)(−5) . . . (−3 − r + 1)
(−x)r = (−1)r xr
r r!
3.4.5. · · · .(r + 2) r
= x
r!
3.4.5. · · · .(r + 2) r
= x
1.2.3.4.5. · · · .r
(r + 1)(r + 2) r
= x
1.2
so the coefficient of xr is 12 (r + 1)(r + 2). But the argument as we’ve written it assumes that
r > 2 (as we’ve left ourselves with “1.2” in the denominator), so we need to check that this
this also holds for r = 0 and r = 1. But this is easy, as −3 0 1
0 (−1) = 1 = 2 × 1 × 2 and
−3 1 1
1 (−1) = 3 = 2 × 2 × 3.
Alternatively, we could have argued
3.4.5. · · · .(r + 2) r 1.2.3.4.5. · · · .(r + 2) r (r + 1)(r + 2) r
x = x = x
r! 1.2.r! 1.2
and this would have dealt with the cases r = 0 and r = 1 automatically, as we are not implicitly
assuming that r > 2.
1 − x + 2x2
(1 − x)3
We have
1 − x + 2x2
= (1 − x + 2x2 )(a0 + a1 x + a2 x2 + · · · + ar xr + . . . )
(1 − x)3
where ar = 21 (r + 1)(r + 2). Thus
1 − x + 2x2
= a0 + a1 x + a2 x2 + · · · + ar xr + · · ·
(1 − x)3
− a0 x − a1 x2 − · · · − ar−1 xr − · · ·
+ 2a0 x2 + · · · + 2ar−2 xr + · · ·
= a0 + (a1 − a0 )x + (a2 − a1 + 2a0 )x2 + · · ·
+ (ar − ar−1 + 2ar−2 )xr + · · ·
STEP I 2011 Question 6 continued
The denominators look like powers of 2, so we will rewrite the terms using powers of 2:
9 25 9 1 4 9 16 25 36 49
1+2+ +2+ + + ··· = + + + + + + + ···
4 16 8 1 2 4 8 16 32 64
and it is clear that the general term is r2 /2r−1 , starting with the term where r = 1.
We can rewrite this in terms of the series found in part (i) by writing
r2 r2 r2 + 1 1
r−1
= 2 · r
= 2 · r
−2· r,
2 2 2 2
so our series becomes
2 5 10 17 1 1 1 1
2 + + + + ··· − 2 + + + + ···
2 4 8 16 2 4 8 16
2 5 10 17 1 1 1 1
=2 1+ + + + + ··· − 2 1 + + + + + ···
2 4 8 16 2 4 8 16
=2·8−2·2
= 12,
where on the second line, we have introduced the term corresponding to r = 0, and on the
penultimate line, we have used the result from (i) and the sum of the infinite geometric series
1 + 21 + 41 + 18 + · · · = 1/(1 − 12 ) = 2.
STEP I 2011 Question 6 continued
An alternative approach is to begin with the result of part (i) and to argue as follows.
We have
∞
1 − x + 2x2 X 2
= (r + 1)xr
(1 − x)3
r=0
∞
X ∞
X
= r 2 xr + xr
r=0 r=0
∞
X ∞
X
=x r2 xr−1 + xr .
r=0 r=0
P∞ 2 ( 1 )r−1 , 1
But our required sum is r=0 r 2 so we put x = 2 into this result and get
∞ ∞
1 − 21 + 2( 12 )2 X X
= 1
2 r2 ( 21 )r−1 + ( 12 )r .
(1 − 12 )3 r=0 r=0
The last term on the right hand side is our geometric series, summing to 2. The left hand side
evaluates to 8, and so we get
∞
X
8= 2 1
r2 ( 21 )r−1 + 2.
r=0
In this question, you may assume that ln(1 + x) ≈ x − 12 x2 when |x| is small.
The height of the water in a tank at time t is h. The initial height of the water is H and
water flows into the tank at a constant rate. The cross-sectional area of the tank is constant.
(i) Suppose that water leaks out at a rate proportional to the height of the water in the
tank, and that when the height reaches α2 H, where α is a constant greater than 1, the
height remains constant. Show that
dh
= k(α2 H − h),
dt
for some positive constant k. Deduce that the time T taken for the water to reach height
αH is given by
1
kT = ln 1 +
α
and that kT ≈ α−1 for large values of α.
Since the tank has constant cross-sectional area, the volume of water within the tank is propor-
tional to the height of the water.
Therefore we have the height increasing at a rate a − bh, where a is the rate of water flowing in
divided by the cross-sectional area, and b is a constant of proportionality representing the rate
of water leaking out. In other words, we have
dh
= a − bh.
dt
dh
Now, when h = α2 H, dt = 0, so a − bα2 H = 0, or a = bα2 H, giving
dh
= bα2 H − bh = b(α2 H − h).
dt
Hence if we write k = b, we have our desired equation.
We can now solve this by separating variables to get
1
Z Z
dh = k dt
α2 H − h
so that
− ln(α2 H − h) = kt + c.
At t = 0, h = H, so
− ln(α2 H − H) = c,
which finally gives us
kt = ln(α2 H − H) − ln(α2 H − h).
STEP I 2011 Question 7 continued
√
(ii) Suppose that the rate at which water leaks out of the tank is proportional to h (instead
of h), and that when the height reaches α2 H, where α is a constant greater than 1, the
height remains constant. Show that the time T 0 taken for the water to reach height αH
is given by
√ √
0 1
cT = 2 H 1 − α + α ln 1 + √
α
√
for some positive constant c and that cT 0 ≈ H for large values of α.
where c0 is a constant.
√ √
An√alternative way of doing
√ this step is to use the√substitution v = α H − h, so that h =
(α H − v)2 = α2 H − 2αv H + v 2 and dh/dv = − 2α H + 2v. This gives us
Z
1 √
ct = (−2α H + 2v) = ct
v
√
−2α H
Z
= + 2 dv
v
√
= −2α H ln v + 2v + c0
√ √ √ √ √
= −2α H ln α H − h + 2 α H − h + c0
At t = 0, h = H, so √ √ √ √
c0 = 2 H + 2α H ln α H − H .
(a) Show that m > n. Show also that m < n + 1 if and only if 2n2 + 3n > 0 . Deduce
that n < m < n + 1 unless − 32 6 n 6 0 .
As n2 > 0, we have
m3 = n3 + n2 + 1
> n3 + 1
> n3
so m > n as the function f(x) = x3 is strictly increasing.
Now
m < n + 1 ⇐⇒ m3 < (n + 1)3
⇐⇒ n3 + n2 + 1 < n3 + 3n2 + 3n + 1
⇐⇒ 0 < 2n2 + 3n
so m < n + 1 if and only if 2n2 + 3n > 0.
Combining these two conditions, n < m always, and m < n + 1 if and only if 2n2 + 3n > 0, so
n < m < n + 1 unless 2n2 + 3n 6 0.
Now 2n2 + 3n = 2n(n + 32 ) 6 0 if and only if − 32 6 n 6 0, so n < m < n + 1 unless − 32 6 n 6 0.
(b) Hence show that the only solutions of (∗) for which both m and n are integers are
(m, n) = (1, 0) and (m, n) = (1, −1).
If solution to (∗) has both m and n integer, we cannot have n < m < n + 1, as there is no
integer strictly between two consecutive integers. We therefore require − 32 6 n 6 0, so n = −1
or n = 0.
If n = −1, then m3 = 1, so m = 1.
If n = 0, then m3 = 1, so m = 1.
Thus the only integer solutions are (m, n) = (1, 0) and (m, n) = (1, −1).
p3 = q 3 + 2q 2 − 1 .
A particle is projected at an angle θ above the horizontal from a point on a horizontal plane.
The particle just passes over two walls that are at horizontal distances d1 and d2 from the
point of projection and are of heights d2 and d1 , respectively. Show that
d21 + d1 d2 + d22
tan θ = .
d1 d2
B
d2
v d1
θ
d1 d2 x
We let the speed of projection be v and the time from launch be t. We resolve the components
of velocity to find the position (x, y) at time t:
R(→) x = (v cos θ)t (1)
R(↑) y = (v sin θ)t − 12 gt2 (2)
(v cos θ)t = d1
(v sin θ)t − 12 gt2 = d2
so that
d1
t=
v cos θ
giving
1
v sin θ gd2
d1 − 22 12 = d2 ,
v cos θ v cos θ
so that
gd21
d2 = d1 tan θ − .
2v cos2 θ
2
gx2
y = x tan θ − (4)
2v 2 cos2 θ
STEP I 2011 Question 9 continued
gd22
= d2 tan θ − d1 . (5)
2v 2 cos2 θ
Multiplying (3) by d22 gives the same left hand side as when we multiply (5) by d21 , so that
d21 + d1 d2 + d22
tan θ = .
d1 d2
Find (and simplify) an expression in terms of d1 and d2 only for the range of the particle.
The range can be found by determining where y = 0, so (v sin θ)t − 21 gt2 = 0. This has solutions
t = 0 (the point of projection) and t = (2v/g) sin θ. At this point,
2v 2 sin θ cos θ
x = (v cos θ)t =
g
2v cos2 θ
2
= tan θ.
g
We have written sin θ = cos θ tan θ because equation (3) gives us a formula for the fraction part
of this expression: we get
d21
x= tan θ.
d1 tan θ − d2
g d1 tan θ − d2
= ,
2v 2 cos2 θ d21
we deduce that
d21 tan θ
x=
d1 tan θ − d2
STEP I 2011 Question 9 continued
as before.
We can now simply substitute in our formula for tan θ, simplify a little, and we will be done:
2
d1 + d1 d2 + d22
2
d1
d1 d2
x= 2
d1 + d1 d2 + d22
d1 − d2
d1 d2
d1 (d21 + d1 d2 + d22 )
=
(d21 + d1 d2 + d22 ) − d22
d1 (d21 + d1 d2 + d22 )
=
d21 + d1 d2
d21 + d1 d2 + d22
=
d1 + d2
Alternative approach
An entirely different approach to the whole question is as follows. We know that the path of
the projectile is a parabola. Taking axes as in the above sketch, the path passes through the
three points (0, 0), (d1 , d2 ) and (d2 , d1 ). If the equation of the curve is y = ax2 + bx + c, then
this gives three simultaneous equations:
0 = 0a + 0b + c
d2 = d21 a + d1 b + c
d1 = d22 a + d2 b + c.
The first gives c = 0, and we can then solve the other two equations to get a and b. This gives
d22 − d21 d1 + d2
a= 2 =−
d1 d2 − d22 d1 d1 d2
d31 − d23 d1 + d1 d2 + d22
2
b= =
d21 d2 − d22 d1 d1 d2
Then the gradient is given by dy/dx = 2ax + b, so at x = 0, the gradient dy/dx = b, which gives
us tan θ (as the gradient is the tangent of angle made with the x-axis). The range is given by
solving y = 0, so x(ax + b) = 0, giving x = −b/a = (d21 + d1 d2 + d22 )/(d1 + d2 ) as before.
Question 10
Assume they collide at height H < h. The perfectly elastic bounce means that there was no loss
of energy, so A has the same total energy at height H on its upwards journey as it did when
travelling downwards. We can work out the speeds at the point of collision, calling them vA and
vB for A and B respectively. We write M for the mass of A and m for the mass of B (as in the
next part of the question). We have, by conservation of energy
2
M gH + 21 M vA = M gh
2
mgH + 12 mvB = mgh
2 = 2(gh − gH) and v 2 = 2(gh − gH), so |v | = |v | and the speeds of A and B are
so that vA B A B
the same.
The masses of A and B are M and m, respectively, where M > 3m, and the speed of the
particles immediately before the first collision is u. Show that both particles move upwards
after their first collision and that the maximum height of B above the plane after the first
collision and before the second collision is
4M (M − m)u2
h+ .
(M + m)2 g
This begins as a standard collision of particles question, and so I will repeat the advice from
the 2010 mark scheme: ALWAYS draw a diagram for collisions questions; you will do yourself
(and the marker) no favours if you try to keep all of the directions in your head, and you are
very likely to make a mistake. My recommendation is to always have all of the velocity arrows
pointing in the same direction. In this way, there is no possibility of getting the signs wrong
v1 − v2
in the Law of Restitution: it always reads v1 − v2 = e(u2 − u1 ) or = e, and you only
u2 − u1
have to be careful with the signs of the given velocities. The algebra will then keep track of the
directions of the unknown velocities for you.
uB = −u vB
B m B m
Before uA = u After vA
A M A M
STEP I 2011 Question 10 continued
M uA + muB = M vA + mvB
so
M u − mu = M vA + mvB ,
vB − vA = 1(uA − uB )
Then solving these equations (by (1) − m × (2) and (1) + M × (2)) gives
(M − 3m)u
vA = (3)
M +m
(3M − m)u
vB = . (4)
M +m
To show that both particles move upwards after their first collision, we need to show that vA > 0
and vB > 0. From equation (3) and M > 3m (given in the question), we see that vA > 0; from
equation (4) and 3M − m > 9m − m > 0 (as M > 3m), we see that vB > 0. Thus both particles
move upwards after their first collision.
To find the maximum height of B between the two collisions, we begin by finding the maximum
height that would be achieved by B following the first collision assuming that there is no second
collision. We then explain why the second collision occurs during B’s subsequent downward
motion and deduce that it reaches that maximum height between the collisions.
The kinetic energy (KE) of B before the first collision is 21 mu2 and after the first collision is
3M − m 2 2
1 2 1
2 mvB = 2 m M +m
u ,
so that B has a gain in KE of 12 mvB 2 − 1 mu2 . When B is again at height h above the plane,
2
which is where it was dropped from, it now has this gain as its KE. (This is because the KE
just before the first collision has come from the loss of GPE; when the particle is once again at
height h, this original KE ( 12 mu2 ) has been converted back into GPE.)
The particle B can therefore rise by a further height of H, where
2 !
−
2 3M m
mgH = 12 mvB − 21 mu2 = 12 mu2 −1 ,
M +m
so
u2 9M 2 − 6M m + m2 M 2 + 2M m + m2
H= −
2g (M + m)2 (M + m)2
u2 8M 2 − 8M m
=
2g (M + m)2
4u2 M (M − m)
= .
g (M + m)2
STEP I 2011 Question 10 continued
Thus the maximum height reached by B after the first collision, assuming that the second
collision occurs after B has started falling is
4M (M − m)u2
h+H =h+ .
(M + m)2 g
Finally, we have to explain why A does not catch up with B before B begins to fall. But this
is easy: B initially has a greater upward velocity than A (as vB − vA = 2u > 0), so the height
of B is always greater than the height of A. Therefore they can only collide again after A has
bounced on the ground and is in its ascent while B is in its descent.
An alternative approach is to use the formulæ for constant acceleration (“suvat”), as follows.
Just before collision, B has speed u, so the height H of B at this point is given by the “suvat”
equation v 2 = u2 + 2as, taking positive to be downwards:
u2 = 02 + 2g(h − H),
giving H = h − u2 /2g.
Immediately after the collision, B has velocity upwards given by equation (4) above. At the
maximum height, hmax , the speed of B is zero, so we can determine the maximum height using
v 2 = u2 + 2as again; this time, we take positive to be upwards, so a = −g:
2
(3M − m)u
2
0 = − 2g(hmax − H).
M +m
(3M − m)u 2
1
hmax = H +
2g M +m
u2 u2 (3M − m) 2
=h− +
2g 2g M +m
2 (3M − m)2 − (M + m)2
u
=h+
2g (M + m)2
u2 8M 2 − 8M m
=h+
2g (M + m)2
4M (M − m)u2
=h+
(M + m)2 g
as required.
Question 11
A thin non-uniform bar AB of length 7d has centre of mass at a point G, where AG = 3d.
A light inextensible string has one end attached to A and the other end attached to B. The
string is hung over a smooth peg P and the bar hangs freely in equilibrium with B lower
than A. Show that
3 sin α = 4 sin β ,
where α and β are the angles P AB and P BA, respectively.
We begin by drawing a diagram of the situation, showing the forces involved (the tension in the
string, which is the same at A and B since the peg is smooth, and the weight of the bar acting
through G). Clearly BG = 4d, which we have shown as well.
We have indicated the angles α, β and φ as defined in the question, and have also introduced
the angle θ as angle AGP . The point M is the foot of the perpendicular from P to AB, which
is used in some of the methods of solution.
P π P
π−θ−α 2 − (α − φ)
θ−β π
2 −φ−β
T T
T T
A α A α
M φ
θ π−θ
3d 3d
G G φ
4d β 4d β
W B W B
Diagram 1: Using θ for angle Diagram 2: Using φ for angle
between rod and vertical between rod and horizontal
Note that we have drawn the sketch with the weight passing through P . This must be the case:
both tensions pass through P and the system is in equilibrium. So taking moments around P
shows that W times the distance of the line of force of W from P must be zero, so that W acts
through P .
The simplest way of showing that 3 sin α = 4 sin β is to take moments about G:
y
M (G) T.3d sin α − T.4d sin β = 0
Given that cos β = 54 and that α is acute, find in terms of d the length of the string and show
that the angle of inclination of the bar to the horizontal is arctan 17 .
π
Approach 1: Show that ∠AP B = 2
sin(π − α − β) = sin(α + β)
= sin α cos β + cos α sin β
4 4 3 3
= 5 × 5 + 5 × 5
= 1,
π
so ∠AP B = 2 and the triangle AP B is right-angled at P .
Alternatively, as sin α = cos β, we must have α + β = π2 , so ∠AP B = π2 .
Thus AP = AB cos α = 7d. 53 = 21
5 d and BP = AB sin α = 7d. 45 = 28
5 d, so the string has length
( 21 28 49
5 + 5 )d = 5 d.
We apply the cosine rule to the triangle AP B, and we write x = AP and y = BP for simplicity.
This gives us
x2 = AB 2 + y 2 − 2AB.y cos β or
y 2 = AB 2 + x2 − 2AB.x cos α.
There are different ways of continuing from here. The most straightforward is probably to begin
by showing that BP = 43 AP or y = 43 x as in Approach 2. This then simplifies the two equations
STEP I 2011 Question 11 continued
to give
5y 2 − 128dy + 560d2 = 0 or
5x2 + 54dx − 315d2 = 0.
(y − 20d)(5y − 28d) = 0 or
(x + 15d)(5x − 21d) = 0.
28
The first equation gives two possibilities: y = 20d or y = 5 d, whereas the second only gives
one: x = 21
5 d.
For the first equation, y = 20d would imply x = 43 y = 15d, but then we would have
We use sin(π − φ) = sin φ and the addition (compound angle) formula to write
sin(π − α − β) = sin(α + β)
= sin α cos β + cos α sin β
4 4 3 3
= 5 × 5 + 5 × 5
= 1,
Therefore we get
sin(θ + α) = sin(θ − β).
We now use the addition formula for sine to expand these, and then substitute in our values for
sin α, etc., giving:
sin θ cos α + cos θ sin α = sin θ cos β − cos θ sin β
so
3
5 sin θ + 45 cos θ = 4
5 sin θ − 35 cos θ
giving
7 1
5 cos θ = 5 sin θ.
Dividing by cos θ now gives cot θ = 71 , hence the angle made with the horizontal is given by
tan φ = 71 , yielding φ = arctan 17 as required.
cos(α − φ) − cos(φ + β) = 0.
T sin( π2 − (α − φ) − T sin( π2 − φ − β) = 0.
Since both of the angles involved here are acute (as the triangle BGP has an obtuse angle at G),
they must be equal, giving α − φ = φ + β, so that 2φ = α − β.
Hence we have
I am selling raffle tickets for £1 per ticket. In the queue for tickets, there are m people each
with a single £1 coin and n people each with a single £2 coin. Each person in the queue
wants to buy a single raffle ticket and each arrangement of people in the queue is equally
likely to occur. Initially, I have no coins and a large supply of tickets. I stop selling tickets if
I cannot give the required change.
(i) In the case n = 1 and m > 1, find the probability that I am able to sell one ticket to
each person in the queue.
I can sell one ticket to each person as long as I have a £1 coin when the single person with a £2
coin arrives, which will be the case as long as they are not the first person in the queue. Thus
the probability is
1 m
1− = .
m+1 m+1
(ii) By considering the first three people in the queue, show that the probability that I am
m−1
able to sell one ticket to each person in the queue in the case n = 2 and m > 2 is .
m+1
This time, I can sell to all the people as long as I have one £1 coin when the first £2 coin is
given to me and I have received at least two £1 coins (in total) by the time the second £2 coin
is offered.
So we consider the first three people in the queue and the coin they bring; in the table below,
“any” means that either coin could be offered at this point. (This called also be represented as
a tree diagram, of course.) The probabilities in black are those of success, the ones in red are
for the cases of failure. Only one or the other of these needs to be calculated.
To determine the probabilities in the table, there are two approaches. The first is to find the
probability that the kth person brings the specified coin given the previous coins which have
been brought; this is the most obvious method when this is drawn as a tree diagram. The
second approach is to count the number of possible ways of arranging the remaining coins
and to
divide it by the total number of possible arrangements of the m + 2 coins, which is
m+2 1
2 = 2 (m + 2)(m + 1).
STEP I 2011 Question 12 continued
Alternatively, we could calculate the probability of failure (adding up the probabilities in red)
and subtract from 1 to get
2 2 2 1 + (m + 1) 2 m−1
1− − =1− . =1− = .
(m + 2)(m + 1) m + 2 m+2 m+2 m+2 m+2
(iii) Show that the probability that I am able to sell one ticket to each person in the queue
m−2
in the case n = 3 and m > 3 is .
m+1
This time, it turns out that we need to consider the first five people in the queue to distinguish
the two cases which begin with £1, £1, £2, £2; the rest of the method is essentially the same
as in part (ii).
m m−1 3 m−2
£1 £1 £2 £1 any yes × × ×
m+3 m+2 m+1 m
m−1
m+3
=
2 3
m m−1 3 2 m−2
£1 £1 £2 £2 £1 yes × × × ×
m+3 m+2 m+1 m m−1
m−2
m+3
=
1 3
m m−1 3 2 1
£1 £1 £2 £2 £2 no × × × ×
m+3 m+2 m+1 m m−1
m+3
=1
3
m 3 m−1 m−2
£1 £2 £1 £1 any yes × × ×
m+3 m+2 m+1 m
m−1
m+3
=
2 3
m 3 m−1 2 m−2
£1 £2 £1 £2 £1 yes × × × ×
m+3 m+2 m+1 m m−1
m−2
m+3
=
1 3
m 3 m−1 2 1
£1 £2 £1 £2 £2 no × × × ×
m+3 m+2 m+1 m m−1
STEP I 2011 Question 12 continued
m+3
=1
3
m 3 2 m m+3
£1 £2 £2 any any no × × =
m+3 m+2 m+1 1 3
3 m+2 m+3
£2 any any any any no =
m+3 2 3
(Alternatively, the four cases beginning £1, £2 can be regarded as £1, £2 followed by 2 people
with £2 coins and m − 1 people with £1 coins, bringing us back into the case of part (ii). So the
m 3
probability of success in these cases is m+3 × m+2 × m−2
m , where the final fraction comes from
the result of part (ii).)
Therefore the probability of success is
1
m(m − 1)(m − 2) + 3(m − 1)(m − 2)+
(m + 3)(m + 2)(m + 1)
6(m − 2) + 3(m − 1)(m − 2) + 6(m − 2)
m−2
= m(m − 1) + 6(m − 1) + 12
(m + 3)(m + 2)(m + 1)
m−2
= (m2 + 5m + 6)
(m + 3)(m + 2)(m + 1)
m−2
= .
m+1
Similarly, the probability of success can be calculated by considering the probability of failure:
the probability of success is therefore
6 6 6m 3
1− − − −
(m + 3)(m + 2)(m + 1) (m + 3)(m + 2)(m + 1) (m + 3)(m + 2)(m + 1) m + 3
12 + 6m + 3(m + 1)(m + 2)
=1−
(m + 3)(m + 2)(m + 1)
3m2 + 15m + 18
=1−
(m + 3)(m + 2)(m + 1)
3(m + 2)(m + 3)
=1−
(m + 3)(m + 2)(m + 1)
3
=1−
m+1
m−2
= .
m+1
There seems to be a pattern in these results, and one might conjecture that the probability of
m+1−n
being able to sell one ticket to each person in the general case m > n is . This turns
m+1
out to be correct, though the proof uses significantly different ideas from those used above.
Question 13
In this question, you may use without proof the following result:
Z p p
4 − x2 dx = 2 arcsin( 12 x) + 12 x 4 − x2 + c .
R∞
We know that −∞ f(x) dx = 1, so we begin by performing this integration to determine k.
We have
Z 0 Z 2 p 0 p 2
4 − x2 dx = 2kx −a + k 2 arcsin x2 + 12 x 4 − x2 0
2k dx + k
−a 0
= 2ak + k (2 arcsin 1 + 0) − (2 arcsin 0 + 0)
= 2ak + kπ
= k(2a + π)
= 1,
so k = 1/(2a + π).
We can now work out the mean of X; we work √ in terms of k until the very end to avoid ugly
1
calculations. We can integrate the expression x 4 − x2 = kx(4 − x2 ) 2 either using inspection
(as we do in the following) or the substitution u = 4 − x2 , giving du/dx = −2x, so that the
R0 1 3 0
integral becomes k 4 − 12 u 2 du = k − 13 u 2 4 = 83 k.
Z ∞
E(X) = xf(x) dx
−∞
Z 0 Z 2 p
= 2kx dx + kx 4 − x2 dx
−a 0
0 3 2
kx2 −a + − k3 (4 − x2 ) 2
= 0
3
2
= (0 − ka ) + (0 − (− k3 × 4 ))
2
= −ka2 + 83 k
8
− a2
3
=
2a + π
8 − 3a2
= .
3(2a + π)
STEP I 2011 Question 13 continued
1
(ii) Let d be the value of X such that P(X > d) = 10 . Show that d < 0 if 2a > 9π and find
an expression for d in terms of a in this case.
1
We have d < 0 if and only if P(X > 0) < P(X > d) = 10 , so we consider P(X > 0). Using the
above integration (or noting that X is uniform for x < 0), we have
π 1
< ;
2a + π 10
that is 10π < 2a + π, or 2a > 9π. Putting these together gives d < 0 if and only if 2a > 9π.
In this case, as d < 0, we have
9
d= −a
20k
9(2a + π)
= −a
20
9π − 2a
= .
20
Note that, since 2a > 9π, this gives us d < 0 as we expect.
√
(iii) Given that d = 2, find a.
Q1 (i) There are several routine features of a graph that one should look to consider on any curve-
sketching question: key points, such as where the curve meets or cuts either of the coordinate axes,
symmetries (and periodicities for trig. functions), asymptotes, and turning-points are the usual
suspects. In this case, the given function involves square-roots as well, so the question of the
domain of the function also comes into question. Considering all such things for
y 1 x 3 x should help you realise the following:
* the RHS is only defined for –3 x 1 (so the endpoints are at (–3, 2) and (1, 2));
* the graph is symmetric in the line x = –1, with its maximum at 1, 2 2 ; NB it must be a
maximum since 2 2 2 so there is no need to resort to calculus to establish this;
* the curve is thus –shaped, and the gradient at the endpoints is infinite. This last point wasn’t
of great concern for the purposes of the question, so its mention was neither rewarded nor its
lack penalised: however, this is easily determined by realising that each term in the RHS is of
1
1
the form X 2 , so their derivatives will be of the form X 2 which, when evaluated at an endpoint
1
will give one of them of the form symptomatic of an asymptote.
0
A quick sketch of y = x + 1 shows that there is only the one solution at x = 1.
(ii) Each side of this second equation represents an easily sketchable curve. Indeed, the RHS is
essentially the same curve as in (i), but defined on the interval [–3, 3]. The LHS is merely a
“horizontal” parabola, though only its upper half since the radix
sign denotes the positive
square-root. These curves again intersect only the once, when x < 0. Resorting to algebra …
squaring, rearranging suitably and squaring again then yields a quadratic equation in x having one
positive and one negative root.
Q2 The required list of perfect cubes is 1, 8, 27, 64, 125, 216, 343, 512, 729, 1000, though there were
no marks for noting them.
(i) In this question, it is clearly important to be able to factorise the sum of two cubes. So, in this
first instance x + y = k, (x + y)(x2 – xy + y2) = kz3 x 2 (k x) x (k x) 2 z 3 0 , which
gives the required result upon rearrangement. One could either treat this as a quadratic in x and
4z 3 k 2
deal with its discriminant or go ahead directly to show that ( y x) 2 0 which
3
4z k
3 2
immediately gives that is a perfect square and also that z 3 14 k 2 ; and the other half of
3
the required inequality comes either from z 3 k 2 3xy k 2 (since x, y > 0) or from noting that
the smaller root of the quadratic in x is positive. Substituting k = 20 into the given inequality then
4z 3 k 2
yields 100 z 3 400 z = 5, 6, 7 ; and the only value of z in this list for which is a
3
perfect square is z = 7, which then yields the solution (x, y, z ) = (1, 19, 7). Although not a part of
the question, we can now express 20 as a sum of two rational cubes in the following way:
3 3
1 19
20 = .
7 7
(ii) Although this second part of the question can be done in other ways, the intention is clearly
that a similar methodology to (i)’s can be employed. Starting from
x + y = z2 , (x + y)(x2 – xy + y2) = kz.z2 x 2 z 2 x x z 2 x kz 0
2
4kz z 4
we find that is a perfect square, and also that k z 3 4k . With k = 19, 19 z 3 76
3
z = 3 or 4. This time, each of these values of z gives
z 76 z 3
a perfect square, yielding the
3
two solutions (x, y, z ) = (1, 8, 3) and (6, 10, 4). Thus we have two ways to represent 19 as a sum
3 3 3 3
1 8 3 5
of two rational cubes: 19 = and .
3 3 2 2
Purely as an aside, interested students may like to explore other possibilities for x 3 y 3 kz 3 .
One that never made it into the question was
x + y = kz, ( x y ) x 2 xy y 2 kz.z 2 x 2 (kz x) x (kz x) 2 z 2 0
3 x 3kzx z (k 1) 0 . Then x
2 2 2 3kz 9k 2 z 2 12 z 2 (k 2 1) 1
6
6 z 3k 12 3k 2 ,
requiring 12 – 3k2 0 i.e. k2 4 k = 1 or 2.
When k = 1: x 3 y 3 z 3 has NO solutions by Fermat’s Last Theorem;
Q3 This question is all about increasing functions and what can be deduced from them. It involves
inequalities, which are never popular creatures even amongst STEP candidates. Fortunately, you
are led fairly gently by the hand into what to do, at least to begin with.
(i) (b) A key observation here is that the “1” is simply a disguise for
d
x , so you are actually
dx
being given that
d
arcsin x d x in the given interval; in other words, that f(x) = arcsin x – x
dx dx
is an increasing function. Since f(0) = 0 and f increasing, f(x) = arcsin x – x 0 for 0 x < 1, and
the required result follows.
x sin x x cos x
(i) (c) Writing g(x) = g ( x) = 2
> 0 for 0 < x < 12 using (a)’s result.
sin x sin x
Now, it may help to write u = arcsin x, just so that it looks simpler to deal with here. Then u x by
(b)’s result g(u) g(x) since g(x) 0 and the required result again follows.
(ii) There is a bit more work to be done here, but essentially the idea is the same as that in part (i),
only the direction of the inequality seems to be reversed, so care must be taken. An added
difficulty also arises in that we find that we must show that f 0 by showing that it is increasing
tan x x sec 2 x tan x 2 x sin 2 x
from zero. So g(x) = , g (x) = . Examining f(x) = 2x – sin2x
x x2 2 x 2 cos 2 x
(since the denominator is clearly positive in the required interval): f(0) = 0 and f (x) = 2 – 2cos2x
0 for 0 < x < 12 f 0 g (x) 0 g increasing. Mimicking the conclusion of (i) (c),
the reader should now be able to complete the solution.
Q4 (i) Using sin A = cos(90o – A) gives = 360n (90o – 4 ) – Note that you certainly should be
aware of the periodicities of the basic trig. functions 5 = 360n + 90o or 3 = 360n + 90o .
These give either = 72n + 18o = 18o , 90o , 162o or = 120n + 30o = 30o , 150o .
Now using the double-angle formulae for sine (twice) and cosine, we have c = 2.2sc.(1 – 2s2). We
can discount c = 0 for = 18o, so that 1 = 4s(1 – 2s2) which gives the cubic equation in s = sin,
8s3 – 4s + 1 = 0 (2s – 1)(4s2 +2s – 1) = 0. Again, we can discount c = 12 for = 18o) which
leaves us with sin18o the positive root (as 18o is acute) from the two possible solutions of this
5 1
quadratic; namely, sin18o = .
4
(ii) Using the double-angle formula for sine, we have 4s2 + 1 = 16s2(1 – s2) 0 = 16s4 – 12s2 + 1
12 80 3 5
s2 = . At first, this may look like a problem, but bear in mind that we want
32 8
2
6 2 5 5 1
2
it to be a perfect square. Proceeding with this in mind, s = so that we have
16 4
5 1
the four answers, sin x = .
4
(iii) To make the connection between this part and the previous one requires nothing more than
division by 4 to get sin 2 x 14 sin 2 2 x , and the solution x = 3 = 18o, 5 = 30o = 6o
immediately presents itself from part (ii). However, in order to deduce a second solution (noting
that = 45o is easily seen to satisfy the given equation), it is important to be prepared to be a bit
flexible and use your imagination. The other possible angles that are “related” to 18o and might
satisfy (ii)’s equation, can be looked-for, provided that sin5 = 12 (and there are many
possibilities here also). A little searching and/or thought reveals
5 1
sin x = 3 = 180o + 18o = 198o also works, since 5 = 330o has sin5 = 12 ,
4
and the second acute answer is = 66o.
Q5 The simplest way to do this is to realise that OA is the bisector of BOC, so that A is on the
diagonal OA of parallelogram OBAC (in fact, since OB = OC, it is a rhombus) b + c = a
for some (giving the first part of the result). Also, as BC is perpendicular to OA, (b – c) . a = 0
ab
(2b – a) . a = 0 = 2 .
aa
bc
Similarly (replacing a by b and b by c in the above), we have d = kb – c where k = 2
bb
b a b b ab ab
= 2 2 2 d = 2 2 b a b = b a where
bb bb bb
ab a b2
2
bb
1 or 4 a ab b 1 .
a b 2 ab
2
Finally, = 1
1
4 1 4 1 , and using the scalar product formula
a ab b
2 2 ab
ab 3
cos gives cos . [Note that a . b has the same sign as .]
ab 8
To begin with, it is essential to realise that the integrand of I f ( x) f ( x) dx must have its
2 n
Q6
two components split up suitably so that integration by parts can be employed. Thus
n
I = f ( x) f ( x) f ( x) dx = f ( x)
1
n 1
f ( x)n 1 f ( x) 1 f ( x)n 1 dx.
n 1
Now (and not earlier) is the opportune moment to use the given relationship f ( x) kf ( x) f ( x) ,
so that I = f ( x)
1
f ( x)n 1 kf ( x) 1 f ( x)n 2 dx, which is now directly integrable
n 1 n 1
as f ( x)
1
f ( x)n 1 1
k f ( x)
n3
(+ C).
n 1 (n 1)(n 3)
(i) For f(x) = tan x, f (x) = sec2x and f (x) = 2 sec2x tan x = kf ( x) f ( x) with k = 2.
sec 2 x tan n 1 x 2 tan n 3 x
Also, differentiating I = gives
n 1 (n 1)(n 3)
dI
dx n 1
1
sec 2 x.(n 1) tan n x. sec 2 x 2 sec x. sec x tan x. tan n 1 x
–
1
(n 1)(n 3)
2(n 3) tan n 2 x. sec 2 x = sec4x tannx = f ( x)2 f ( x)n as required,
although this could be verified in reverse using integration. Using this result directly in the first
given integral is now relatively straightforward:
sin 4 x sec 2 x tan 5 x 2 tan 7 x
cos 8 x C .
4 4
d x = sec x tan x dx =
5 35
(ii) Hopefully, all this differentiating of sec and tan functions may have helped you identify the
right sort of area to be searching for ideas with the second of the given integrals.
If f(x) = sec x + tan x, f (x) = sec x tan x + sec2x = sec x(sec x + tan x)
and f (x) = sec2x(sec x + tan x) + sec x tan x(sec x + tan x)
= sec x (sec x + tan x)2 = kf ( x) f ( x) with k = 1.
n 1 1 n 1 1 1 n 1
=
1
1
= 1 n 1 1 , since – 1 = 2 and – 1 = 2
2
1 n
n 1 1 n 1 1 1
= a n 1 2 and, similarly, a r = = bn 1 .
2 r 0 2 2 2
(ii) There is no need to be frightened by the appearance of the nested sums here as the ‘inner sum’
has already been computed: all that is left is to work with the remaining ‘outer sum’ and deal
2n
m 2n 1 1 2n 1
carefully with the limits: a r bm 1 bm (since b0 = 0)
m0r 0 m0 2 2 m0
=
1 1
2 2
a 2n 2 2 = 2 n 2 2 n 2 2 =
1
2
1 n 1 2
2
2 n 1
n 1 2
since = –1
and n + 1 is even when n is odd = bn 1 when n is odd. However, when n is even, n + 1 is odd
1 2
2
2n
m
1
and a r = bn 1 2 or a n 1 .
2 1 2
m0r 0 2 2
2
n
(iii) We already have the result a r = bn 1 , so the only new thing is
1 2
r 0 2
2 n 2 1 2 n 2 1
GPs, merely with different common ratios, having sum .
2 1 2 1
Now
2 1 3 2 2 1 2 1 2 2 and
2 1 3 2 2 1 2 1 2 2 ,
2 n 2 2 = bn 1 when n is odd, and bn 1 2 when n is even.
n
1 2n 2 1 1
so a 2 r 1 = 2 2
r 0 2 2 2
2
n n
Thus a r – a 2r 1 = 0 when n is odd / = 2 when n is even.
r 0 r 0
dx
sin t t cos t sin t t cos t by the Product Rule; = 0 when t = 0, (x, y) = (1, 0) or
dt
t = 12 , (x, y) = 12 , 1 . This is xmax so t0 = 12 .
The required area under the curve and above the x-axis is
1
2
1
2
using the double-angle formulae for sine and cosine. As the integration here may get very messy,
it is almost certainly best to evaluate this area as the sum of three separate integrals:
1 1 1 1 1 3
2 t sin 2t dt = 4
t cos 2t 1
1 4 cos 2t dt = 4
t cos 2t
8
sin 2t
1
=
8
;
1
2
2 2
2
1 1 7 3
2t
2
dt = t 3 =
1
6 1 48
2 2
1 1 3 3
t cos 2t dt = t 2 sin 2t 2 t sin 2t dt = 0 –
1 2
and 2 = using a previous answer.
1
4 1 1 8 8
2 2 2
7 3 3
Thus A = .
48 4
For the total area swept out by the string during this process (called Involution), we still need to
3
add in the area swept out between t = 0 and t = 12 , which is
(there is, of course, no
48 4
need to repeat the integration process), and then subtract the area inside the semi-circle. Thus the
7 3 3 3 3
total area swept out by the string is + (area inside semi-circle) = .
48 4 48 4 2 6
Q9 Collisions questions are always popular, as there are only two or three principles which are to be
applied. It is, nonetheless, good practice to say what you are attempting to do. Also, a diagram,
though not an essential requirement, is almost always a good idea, if only since it enables you to
specify a direction which you are going to take to be the positive one, especially since velocity
and momentum are vector quantities. Once these preliminaries have been set up, the rest is fairly
easy. By CLM, 3mu = 2mVA + mVB and NEL/NLR gives e.3u = VB – VA . Solving these
simultaneously for VA and VB yields VA = u(1 – e) and VB = u(1 + 2e).
Next, after its collision with the wall, B has speed | f VB | away from the wall.
For the second collision of A and B, by CLM (away from wall), fmVB – 2mVA = 2mWA – mWB,
and NEL/NLR gives WA + WB = e(VA + f VB). Substg. for VA & VB from before in both of these
equations 2WA – WB = u f (1 2e) 2(1 e) and WA + WB = eu(1 e) f (1 2e). Solving
these simultaneously for WA (not wanted) and WB then gives WB = 13 u 2 1 e 2 f 1 4e 2 , as
required.
Noting that 1 4e 2 can be negative, zero, or positive, it may be best (though not essential) to
consider the possible cases separately:
if e = 12 , WB = 13 u2 34 f 0 12 u 0 ;
if 1
2
< e < 1, WB = 13 u 2 1 e 2 f 4e 2 1 > 0 for all e, f since each term in the bracket is > 0;
if 0 < e < 1
2 , 1 – e2 > 3
4 and WB > 13 u32 f 1 4e 2 13 u32 1 1 0 .
u sin
Q10 The maximum height of a projectile is when y u sin gt 0 t . Substituting this
g
u 2 sin 2
into y ut sin gt H = 1
2
2
(although some people learn it to quote).
2g
u 2 sin 2
When the string goes taut, its length l is given by l = 1
2 H . But l is also given from the
4g
y-component of P’s displacement as l = ut sin 12 gt 2 , which gives the quadratic equation
2u sin 4u 2 sin 2 4 gH
gt 2 (2u sin )t H 0 in t. Solving by the quadratic formula, t =
2g
=
2 2 gH 8 gH 4 gH
2g
=
1
g
2 gH gH =
H
g
2 1 , where we take the smaller of the
two roots since we want the first time when an unimpeded P is at this height.
When the string goes slack, we must consider the projectile motion of R, which has initial velocity
u sin
components u cos and . [Note that both P and R move in this way, so P no longer
2 2
u sin g
interferes with R’s motion.] R’s vertical displacement is zero when y R t t 2 = 0 (t 0)
2 2 2
u sin
t (and this is the extra time after the string has gone slack). The total distance
g 2
O A O B O C A
k k k Tsin30o
T U V j
30o 60o
O iO
P P P
Vsin60o
U cos
B C
2
It might also be wise to note the sines and cosines of the given angles: tan = 2 sin
3
1 2 1 2 2
and cos , and tan = sin and cos . Having noted these carefully, it
3 4 3 3
is now reasonably straightforward to state that the vector in the direction of PB is
(iii) Having set the system up in vector form, the fundamental Statics principle involved is that
TA + TB + TC + W = 0 .
Comparing components in this vector equation gives
1 6
( i ) 0 U V 0 U V 6
3 3
1 2 3 5 3
(j) T U V 0 (using U V 6 ) T V
2 3 6 3
3 6 1 5 3
(k) T U V W (using U V 6 and T V)
2 3 2 3
W 3 W 6 W
T , U , V .
3 5 5
Q12 It is important in these sorts of contrived games to read the rules properly: in this case, you must
ensure that you are clear what is meant by ‘match’, ‘game’ and ‘point’. Then, a careful listing of
cases is all that is required.
Next, w 1
2 1 p 2 (2 p ) p (1 2 p )
, and since 2 – p > 0, w 12 has the same sign as
2
2( 2 p ) 2( 2 p )
1 – 2p and hence as 12 p . Hence, w > 12 if p < 12 and w < 12 if p > 12 .
To be fair at this point, the final demand of part (i) ended up being rather less demanding than was
originally intended, as the answer is either “Yes” or “No” … though you would of course, be
expected to support your decision; no marks are given for being a lucky guesser! The following
calculus approach is thus slightly unnecessary, as one can simply provide an example to show that
w can decrease with p. The following, more detailed analysis had been intended.
dw (2 p)(2 p) 1 p 2 (1)
=
1
p2 4 p 1
1
[2 p]2 3 .
dp (2 p ) 2
(2 p) 2
(2 p ) 2
dw dw
Then > 0 for 0 < p < 2 3 and < 0 for 2 3 < p 1.
dp dp
For a fair game, Y’s expectation should be 0. Thus, using E(gain) = g i P( g i ) , where gi is the
“gain function” for Y, with w = 5
12 when p = 2
3 , we have 0 = (k ) 125 (1) 127 k = 1.4 .
When p = 0, the results run YXY ... re-match ... YXY ... re-match ... and the match never ends.
(i) For the next part, you should understand how the “expectation” function behaves.
E X = E X 3 3X 2 3 2 X 3 = E X 3 3E X 2 3 2 E X 3
3
= E X 3
3 2
2 3 2 . 3 using E[X] = and E[X2] = 2 + 2
= E X 3
3 2
3 , as required.
For a given distribution, this next bit of work is very routine indeed.
2 1
1 1
1 1
E X 2 x 2 dx = x 3 = 23 ; E X 2 2 x 3 dx = x 4 = 1
2 2 181 ; and
0 3 0 0 2 0
3. 23 . 181 278
2 1
1 2
2 2
E X 3 2 x 4 dx = x 5 = 2
5 ; all of which then lead to 5
= when
0 5 0 18 18
1
5
substituted into the given (previously deduced) formula.
x
(ii) Here, F(x) = 2 x dx = x 2 (0 x 1) F – 1(x) = x (0 x 1)
0
F 1 109 2 F 1 12 F 1 101 3 2 5 1 4 2 5
3 2 1
D 10 2 10
= 2 5.
F 1
F 101
9
10
1 3
10
1
10
3 1 2
M
M is given by 2 x dx = 1
2 M2 = 1
2 M= 1
2
(OR by M = F – 1( 12 ) = 1
2
).
0
3 23 1
And P = 1
2
6 2 9.
3 2
In order to establish the given inequality “chain”, we must show that D > P and P > (there is no
point in proving that D > ). One could reason this through by considering approximants to 2
and 5 , but care must be taken not to introduce fallacious “roundings” which don’t support the
direction of the inequality under consideration. The alternative is to establish a set of equivalent
numerical statements; for example, to show that D > P …
2 5 6 2 9 11 5 6 2
121 5 22 5 72 (after squaring, since both sides are positive)
54 22 5 or 27 11 5 729 605 (again, squaring positive terms)
and this final result clearly IS true, so the desired inequality is established.
STEP Mathematics III 2011: Solutions
1. (i) The differential equation can be solved either by separating variables or using
an integrating factor. In either case, , or the negative of it is required, and this can
be found either by re-writing as 1 or using the substitution, 1.
Thus the solution is 1 .
(i) To show that the nth root of 2 is irrational, consider 2, and evaluate
1 and 2 , then apply the stem of the question.
(ii) Considering the turning points of 1 , there can only be one real
root. Evaluating 2 and 1 and applying the stem gives the result.
4. (i) is the area between the curve , the x axis, and the line
is the area between the curve , the y axis, and the line .
The sum of these areas is greater than or equal to the area of the rectangle, with equality
holding if .
5.
tan
(ii) √ 1 √ √ 1 √ 1
1 1 1
1 1 as required.
1 as required.
and
11. On the one hand the distance between the point on the disc vertically below , 0,0
and P is sin as the string length b makes an angle with the vertical. On the other, it is
2 sin , the third side of an isosceles triangle with two radii a at an angle , and hence the
required result.
The horizontal component of the tension in each string is sin and it acts at a
perpendicular distance cos from the axis of symmetry. Thus the couple is
sin cos . Resolving vertically, cos . Substituting for T in the
expression for the couple and then using bsin 2 sin to eliminate , gives the
required result.
The initial potential energy relative to the position where the strings are vertical is
1 cos . This is converted into kinetic energy . Equating these
expressions and once again using bsin 2 sin to eliminate , gives the required
result.
Section C: Probability and Statistics
12. As , ′ ′ ′ , and as 1 1, ′ 1
, ′ 1 , and ′ 1 , the first result follows.
Similarly, ′′ ′′ ′ ′ ′′ , and
1 1 1
′′ 1 ′ 1 ′ 1 ′′ 1
as required.
A fair coin tossed until a head appears is distributed so . The PGF for
the number of heads when a fair coin is tossed once is . Thus .
13. (i) , 1
1
and so . The most probable value of X is the minimum value of r such
that 1 , because this expression decreases as r increases. All the factors are
positive so it is simple to rearrange the algebra to obtain 1 so .
The answer is not unique when there is a value of r such that 1 , in which case,
, which will only happen if divides 1 .
1 , and so .
Again, the most probable value of X is the minimum value of r such that
1, giving , and this is not unique if 2 divides
1 1 .
STEP Solutions
2012
Mathematics
STEP 9465/9470/9475
November 2012
The Cambridge Assessment Group is Europe's largest assessment agency
and plays a leading role in researching, developing and delivering
assessment across the globe. Our qualifications are delivered in over 150
countries through our three major exam boards.
All Examiners are instructed that alternative correct answers and unexpected
approaches in candidates’ scripts must be given marks that fairly reflect the
relevant knowledge and skills demonstrated.
© UCLES 2010
Report Page
STEP Mathematics I 4
STEP Mathematics II 8
STEP Mathematics III 18
STEP I ‐ Hints and Solutions
Question 1
The relationships for and can be obtained by substituting the coordinates of the three known
points into the equation of the line, or by using the formula for calculating the gradient from a pair
of points.
The formula for the sum of the distances is then easy to find and differentiation with respect to
will allow the minimum value to be found.
Similarly, the distance for the second part can be written as and again differentiation can
be used to find the value of for which the minimum occurs. Since the minimum value of
occurs at the same value of as the minimum value of , the differentiation
can be simplified by just differentiating with respect to . It is then a simple matter to
write this answer in a form similar to that of the first part.
Question 2
To sketch the graph it is important to know where the stationary points are. Either by considering
the graph of or by differentiating it can be seen that there are two minima and one
maximum.
The equation in the second part can be rearranged to show that the solutions correspond to the
intersections of the graph of and a straight line. The case requiring care is the two
solution case as this must include the straight line which touches the two minima.
For the next part of the question, differentiation of the equation twice leads quickly to the two
possible values of . Both cases then need to be considered, but it should be clear that one graph is
a reflection of the other in the ‐axis, so the sets of values for will be the same for both cases.
The final graph should clearly have a minimum for some negative value of . will still be 0 at
, so there will be two points of inflexion.
Question 3
The sketch of the graph, including a chord and tangent should not cause much difficulty. Adding the
line should show that the area under the graph lies between two triangles, both with a base
of length and with heights and . Integrating the function between the two limits and then
rearranging will give the correct relationship.
For the second part of the question a different diagram is needed, this time showing the area under
a curve contained within a trapezium and with a trapezium contained within it. The differentiation
and integration of will produce the expressions required in the final answer and so the
vertical lines and can be used to define the regions.
Question 4
The equations of the tangents at and should be easy to find and then the solution of
simultaneous equations will give the required coordinates for . Similarly, the equations for the
normal should be easy to find, but it is more difficult to find simplified expressions for the solution to
the simultaneous equations (which are useful for the final part of the question). The factorisation of
the difference between two cubes, will be useful for avoiding a lot
of algebraic manipulation. If this expression is simplified correctly then the final result becomes
straightforward.
Question 5
The most obvious approaches to the first integration are to integrate by parts or to use the identity
and then make a substitution. The second integration can also be evaluated by
integrating by parts, but the identity for is not as useful.
For the third integration it is necessary to rewrite the integral in a form from which the previous
results can be applied. The first point to note is that the expression within the logarithm is not a
simple cosine function and so the first step to making the expression similar to those used previously
Question 6
By writing down expressions for the height of the pole using the tangent of each of the angles of
elevation, the problem is quickly reduced to a two dimensional problem about lines within a circle.
The simplest way to tackle this is to observe that the triangles in the diagram are similar to each
other, but approaches working with various right angled triangles also lead to the correct solution.
The proof of the identity should be straight forward for those familiar with the commonly used
trigonometric relationships and the inequality is then easily found by considering the consequence
of the constraint placed on p and q. Given that cot is a decreasing function in the required range of
values, the final result follows easily.
Question 7
By substituting for all of the terms in the recurrence relation the result for the first part should
follow easily. For the second part, the two values and can be substituted in both orders into
the relation, giving two equations in and . Solving these two equations then leads to the correct
set of possibilities.
Following the same principle, the substitution of , and into the relation can be done in three
different ways, leading to three simultaneous equations. Solving these equations gives an equation
in and , which can be solved to give the two different cases. The case where is easy
to check, and for the case where it should be noticed that this
can only occur if .
Question 8
The new differential equation follows quite easily once the substitution given has been followed. It is
an example of a differential equation that can be solved by separating the variables and so by
evaluating the two integrals that are reached the required form can be obtained.
The same substitution will also reduce the second differential equation to a simpler form and it is
again an example that can be solved by separating the variables. It should be clear that partial
fractions are an appropriate method for evaluating the integral that is required.
Question 9
The first part of the question is a straightforward calculation involving the equations for motion
under uniform acceleration. It is important to explain the reason for choosing the positive square
root however. The final result follows from correct use of an identity for .
It is also easy to find an expression for the range once the time has been calculated and further
application of an identity for will give an expression for this in terms of . Differentiation
with respect to will then give the result that the maximum value occurs when . The final
part of the question can be solved by substituting the appropriate values of into the formula for
the range.
Question 10
Although the equation looks complicated, calculations for the time that it takes the stone to drop
and the time for the sound to return allow the first relationship to be deduced quite easily. The
second relationship can be shown by simplifying the expression for and showing that it is equal to
. This can then be rearranged to give . The final part of the question is then a
substitution of the values given into the formula to obtain the estimate.
Question 11
While the diagram may look a little more complicated than the standard questions on this topic, the
first section of this question requires the usual steps to establish a pair of simultaneous equations.
The difference in the second part of the question is that the acceleration cannot be assumed to be
constant (as the pulley in the middle of the diagram is able to move) and the important extra
relationship that is needed is the relationship between the accelerations at the three points (the two
particles and the pulley).
Question 12
The probabilities for a failure in each year long period need to be calculated by evaluating the
integral and from these it is possible to construct a tree diagram from which the probability can be
calculated. The final part of the question is simply the calculation of a conditional probability. As
always with conditional probability the important step is to deduce which two probabilities need to
be calculated.
Question 13
There are a number of ways to approach this problem. The most obvious is to work out how many
possibilities there are for each number of digits. A clear method for categorising these is needed to
work out the number of possibilities in each case. For example, if there are 4 different digits then
there are five choices for the digits to be used and four choices for the digit to be repeated. There
are then ten choices for the positions of the repeated digits and 3! choices for the order of the
remaining digits. This gives 1200 altogether.
Hints & Solutions for Paper 9470 (STEP II) June 2012
Question 1
To be honest, the binomial expansions of (1 x)n, in the cases n = 1, 2, are used so frequently within AS-
and A-levels that they should be familiar to all candidates taking STEPs. Replacing x by xk is no great
further leap.
2
The general term in 1 x 6 is easily seen to be (n + 1) x6n and the x24 term in 1 x 6
1 x 3 comes
2
1
from 1.x24 + 2x6.x18 + 3x12.x12 + 4x18.x6 + 5x24.1, so that the coefficient of x24 is 1 + 2 + 3 + 4 + 5 = 15,
arising from a sum of triangular numbers. Thus, the coefficient of xn is
0 if n 6k {1, 2, 4, 5}
1
2 (k 1)(k 2) if n 6k 3
1 (k 1)(k 2) if n 6k
2
which is most easily described without using n directly, as here.
In (ii), f(x) = 1 2 x 6 3 x 12 4 x 18 5 x 24 ... 1 x 3 x 6 x 9 ... 1 x x 2 x 3 ... and the x24 term
comes from
1.1.5x24 + 1.x6.4x18 + 1.x12.3x12 + 1.x18.2x6 + 1.x24.1
+ x3.x3.4x18 + x3.x9.3x12 + x3.x15.2x6 + x3.x21.1
+ x6.1.4x18 + x6.x6.3x12 + x6.x12.2x6 + x6.x18.1
+ x9.x3.3x12 + x9.x9.2x6 + x9.x15.1
+ x12.1.3x12 + x12.x6.2x6 + x12.x12.1
+ x15.x3.2x6 + x15.x9.1
+ x18.1.2x6 + x18.x6.1
+ x21.x3.1
+ x24.1.1
giving the coefficient of x24 as 15 + 2 (10 + 6 + 3 + 1) = 55.
However, there are lots of ways to go about doing this. For instance ...
Note that, because every non-multiple-of-3 power in bracket 3 is redundant, the x24 term comes from
considering f(x) = 1 x 6 1 x
2 3 2
= 1 2 x 6 3x 12 4 x 18 5 x 24 ... 1 2 x 3 3 x 6 4 x 9 ... .
Again, every non-multiple-of-6 power in this 2nd bracket is also redundant, so one might consider only
f(x) = 1 3x 6 5 x 12 7 x 18 9 x 24 ... 1 2 x 6 3 x 12 4 x 18 5 x 24 ...
from which the coefficient of x24 is simply calculated as 1 5 + 3 4 + 5 3 + 7 2 + 9 1 = 55. This
result, in some form or another, gives the way of working out the coefficient of x6n for any non-negative
n
integer n. It is immediately obvious that it is (n 1 r )(2r 1) which turns out to be the same as
r 0
n 1
r
r 1
2
16 (n 1)(n 2)(2n 3) .The proof of this result could be by induction or direct manipulation of the
The coefft. of x25 is 55, the same as for x24, since the extra x only arises from replacing 1 by x, x3 by x4,
etc., in the first bracket’s term (at each step of the working) and the coefficients are equal in each case.
(ii) Deg[RHS] = 4 while Deg[LHS] max n 2 , 2n, n , so it follows that n = 2 and p(x) is quadratic.
Setting p(x) = ax2 + bx + c, we have
2p(p(x)) = 2a(ax2 + bx + c)2 + 2b(ax2 + bx + c) + 2c
= 2a a 2 x 4 2abx 3 2acx 2 b 2 x 2 2bcx c 2 + 2b(ax2 + bx + c) + 2c
3 p( x) = 3 a 2 x 4 2abx 3 2ac b 2 x 2 2bcx c 2 and –4p(x) = – 4ax2 – 4bx – 4c.
2
Question 3
1
It helps greatly to begin with, to note that if t x 2 1 x , then x 2 1 x . These then give the
t
dx 1 1 2
result x 12 t 12 t 1 , from which we find t and (changing the limits) x : (0, ) t : (1, ),
dt 2 2
f
1 1 1
so that x 1 x dx = f t 2 1 2 dt = 2 f x 1 2 dx , as required.
2 1
0 1 t 1 x
1 1
x
For the first integral, I1 = 2
dx , we are using f(x) = in the result established initially.
0
2
1 x x2
1
x
1 1 1
Then I1 = 1
2 1 1 x 2 . x 2 dx = 1
2
2
x 4 dx = 1
2
x 3x 3 =
1
2
0 1 13 23 .
1 1
In the case of the second integral, the substitution x = tan dx = sec2 d . Also 1 x 2 sec and
the required change of limits yields 0, 12 0, . We then have
1
1
sec
3
2 2
1
I2 = d = 0 sec tan d [Note the importance of changing to sec and tan]
0 1 sin
3
1
2
sec x2 1
0 sec tan 3 .sec d =
x
2
= 3
dx .
0
2
1 x
1
1
t
t t 1 , so that
2 2
We now note, matching this up with the initial result, that we are using f(t) =
t3 2t 4
t 2 1 t 2 1 1
t 1 2
I2 = 12 2 . 4 dt = 1
4
2
2t 4 t 6 dt = 1
4
t 3t 3 5t 5 =
1
4
0 1 23 15 157 .
1
t 2t 1 1
Question 4
(i) This first result is easily established: For n, k > 1, nk + 1 > nk and k + 1 > k so (k 1) n k 1 k n k
1 1
k 1
k (since all terms are positive).
(k 1)n kn
1 1 1 1 1 1 1
Then ln1 2 3 4 5 ... (a result which is valid since 0 < 1 )
n n 2n 3n 4n 5n n
1 1 1 1 1 1
2 3 4 5 ... since each bracketed term is positive, using
n 2n 3n 4n 5n n
A1
1 n
1 1
the previous result. Exponentiating then gives 1 e n 1 e .
n n
(ii) A bit of preliminary log. work enables us to use the ln(1 + x) result on
2 y 1 1 1 1 1 1 1 1
ln = ln1 ln1 = ...
2y 1
2 3 4 5
2y 2 y 2 y 2( 2 y ) 3(2 y ) 4( 2 y ) 5(2 y )
1 1 1 1 1
– 2
3
4
5
...
2 y 2( 2 y ) 3(2 y ) 4( 2 y ) 5(2 y )
1 1 1 1
= 2 3
5
... (since all terms after the first are positive).
2 y 3(2 y ) 5(2 y ) y
1
Again, note that we should justify that the series is valid for 0 < 1 i.e. y > 1
2 in order to justify the
2y
y
2 y 1
use of the given series. It then follows that ln > 1, and setting y = n + 1
(the crucial final step)
2y 1
2
n 1 n 1
2n 2 2 1 2
gives ln 1 1 e.
2n n
(iii) This final part only required a fairly informal argument, but the details still required a little bit of care
in order to avoid being too vague.
n 1 n 1 n n
1 2 1 1 2 1 1
As n , 1 1 1 1 1 1 from above and e is squeezed
n n n n n
into the same limit from both above and below.
Question 5
With any curve-sketching question of this kind, it is important to grasp those features that are important
and ignore those that aren’t. For instance, throughout this question, the position of the y-axis is entirely
immaterial: it could be drawn through any branch of the curves in question or, indeed, appear as an
1
asymptote. So the usually key detail of the y-intercept, at 0 , 2 in part (i), does not help decide
a 1
what the function is up to. The asymptotes, turning points (clearly important in part (ii) since they are
specifically requested), and any symmetries are important. The other key features to decide upon are the
“short-term” (when x is small) and the “long-term” (as x ) behaviours.
In (i), there are vertical asymptotes at x = a – 1 and x = a + 1; while the x-axis is a horizontal asymptote.
There is symmetry in the line x = a (a consequence of which is the maximum TP in the “middle” branch)
1
and the “long-term” behaviour of the curve is that it ultimately resembles the graph of y = 2 .
x
In the first case, where b > a + 2 (i.e. a + 1 < b – 1), there are five branches of the curve, with 4 vertical
asymptotes: x = a 1 and x = b 1. As the function changes sign as it “crosses” each asymptote, and the
1
“long-term” behaviour is still to resemble y = 2 , these branches alternate above and below the x-axis,
x
with symmetry in x 2 (a b) .
1
In the second case, where b = a + 2 (i.e. a + 1 = b – 1), the very middle section has collapsed, leaving
only the four branches, but the curve is otherwise essentially unchanged from the previous case.
Question 6
A A quick diagram helps here, leading to the important observation, from the
GCSE geometry result “opposite angles of a cyclic quad. add to 180o”, that
b BCD = 180o – . Then, using the Cosine Rule twice (and noting that
a cos(180o – ) = – cos ):
D in BAD: BD2 = a2 + d 2 – 2ad cos
B in BCD: BD2 = b2 + c2 + 2bc cos
a2 b2 c2 d 2
b c Equating for BD2 and re-arranging gives cos =
2(ad bc)
C
Next, the well-known formula for triangle area, 12 ab sin C , twice, gives Q = 12 ad sin 12 bc sin ,
2Q 4Q
since sin( – ) = sin . Rearranging then gives sin = or .
ad bc 2(ad bc)
Then, 16Q 2 = 2ad 2bc a 2 b 2 c 2 d 2 2ad 2bc a 2 b 2 c 2 d 2 by the difference-of-two-
squares factorisation
= [b c] 2 [a d ] 2 [a d ] 2 [b c] 2
= [b c] [a d ][b c] [a d ][a d ] [b c][a d ] [b c]
using the difference-of-two-squares factorisation in each large bracket
= b c d a a b c d a c d b a b d c .
Splitting the 16 into four 2’s (one per bracket) and using 2s = a + b + c + d
Q2 =
2s 2a 2s 2b 2s 2c 2s 2d ( s a)(s b)(s c)(s d ) .
2 2 2 2
Finally, for a triangle (guaranteed cyclic), letting d 0 (Or s – d s Or let D A), we get the result
known as Heron’s Formula: = s ( s a )( s b)( s c) .
Question 7
Many of you will know that this point G, used here, is the centroid of the triangle, and has position vector
g = 13 x1 x 2 x 3 .
Then GX 1 x1 g 2x1 x 2 x 3 and so GY1 13 1 2x1 x 2 x 3 , where 1 > 0.
1
3
Also OY1 OG GY1 13 x1 x 2 x 3 13 1 2x1 x 2 x 3 13 [1 2 ]x1 [1 1 ](x 2 x 3 ) , the first
printed result.
The really critical observation here is that the circle centre O, radius 1 has equation | x |2 = 1 or x . x = 1,
where x can be the p.v. of any point on the circle.
Thus, since OY1 OY1 1 , we have
1 1
9
(1 2 ) 1
2
2(1 1 ) 2 2(1 21 )(1 1 )x1 (x 2 x 3 ) 2(1 1 ) 2 x 2 x 3
9 1 41 41 2 41 21 2(1 21 )(1 1 )x1 (x 2 x 3 ) 2(1 1 ) 2 x 2 x 3
2 2
3
1 , using x 2 x 3 , x 3 x1 and x1 x 2 .
3 2 2
3 3
Similarly, 2 and 3 .
3 2 2 3 2 2
GX i 1 GX 1 GX 2 GX 3 1 1 1 9 ( ) 4( )
Using (i = 1, 2, 3),
GYi i GY1 GY2 GY3 1 2 3 3
9 3( )
= = 3.
3
n
GX i
[Interestingly, this result generalises to n points on a circle: n .]
i 1 GYi
Question 8
2 2 q2
q (> 0) 2 2 2 q 2 2 2 q 2 2 2 the opening
2 2 q2
result, 20.
un q 2
2
un 1
equating for q2 u n u n u n 2 u n 1 u n 1 u n 1 .
un un 2 un 1 un 1 u un 1
Now this gives which n 1 is constant (independent of n). Calling
un 1 un un
this constant p gives u n 1 pu n u n 1 0 , as required. In order to determine p, we only need to use the
un 1 un 1
fact that p = for all n, so we choose the first few terms to work with.
un
2 q2
2 q2 u u2 2 2 q2
u2 = and p 0 = .
u1
2 q2 2 2 q2
Alternatively, u2 = = = p – p
2
2 q2
2 q 2
q q
=
2 2 2 2 2
and u3 = p p
2 q2
2
q2 q =
2 2
2 q2 2
2 2
q
= 2 2
since 2 q 2 0 as u2 non-zero (given). Since p is consistent for any chosen , , the proof follows
inductively on any two consecutive terms of the sequence.
Question 9
In the standard way, we use the constant-acceleration formulae to get
x = ut cos and y = 2h – ut sin – 12 gt2 .
a ga 2
When x = a, t . Substituting this into the equation for y y 2h a tan 2 sec 2 .
u cos 2u
As y > h at this point (the ball, assuming it to be “a particle”, is above the net), we get
ga 2 1 2(h a tan )
h a tan 2 sec 2 2 , as required.
2u u ga 2 sec 2
There are several ways to proceed from here, but this is (perhaps) the most straightforward.
b 2 g 2 sec 2
Squaring u 2 sin 2 4 gh 2
2bg tan u 2 sin 2
u
b 2 g sec 2
Cancelling u 2 sin 2 both sides & dividing by g 4h 2b tan
u2
1 2(2h b tan ) 1
Re-arranging for 2
u 2
b 2 g sec 2 u
1 2(h a tan ) 2(2h b tan ) 2(h a tan )
Using the first result, 2 , in here
u a g sec
2 2
b 2 g sec 2 a 2 g sec 2
Re-arranging for tan ab(b a) tan hb 2 2a 2 , which leads to the required final answer
hb 2 2a 2
tan . However, it is necessary (since we might otherwise be dividing by a quantity that
ab(b a )
could be negative) to explain that b > a (we are now on the other side of the net to the projection point)
else the direction of the inequality would reverse.
Question 10
As with many statics problems, a good diagram is essential to successful progress. Then there are
relatively few mechanical principles to be applied ... resolving (twice), taking moments, and the standard
“Friction Law”. It is, of course, also important to get the angles right.
a M
A
F1 W (or mg)
For the second part, it seems likely that we will have to resolve twice (not having yet used this particular
set of tools), though we could take moments about some other point in place of one resolution. There is
also the question of which directions to resolve in – here, it should be clear very quickly that “horizontally
and vertically” will only yield some very messy results.
a r2
cos gives tan 2
r a 2 1 2
.
a2 2 a
2
r
1 2 2
r r
R R2
Finally, tan 1 tan , from the first result, < tan < .
R1 R2
Question 11
Again, a diagram is really useful for helping put ones thoughts in order; also, we are going to have to
consider what is going on generally (and not just “pattern-spot” our way up the line).
u Vi – 1 Vi
i
Pi Bi – 1 Bi
u 2nu u
The last collision occurs when Vn , i.e.
n 1 N ( N 1) n (n 1) n 1
2n (n 1) N ( N 1) n (n 1) n (n 1) N ( N 1) there are N collisions.
2
N
u N
1
Now, the total KE of all the Pi ’s is
i 1
1
2 (i m) =
i
1
2 mu 2 .
i 1 i
2
u 2 N
The final KE of the block is N ( N 1)mV N = N ( N 1)m 12 mu .
1 2 1
N 1 N 1
2 2
N
1 N
Therefore, the loss in KE is the difference: 12 mu 2 – 12 mu 2 .
i 1 i N 1
N 1
N 1 1 1 1 1 1 1
1 , the loss in KE is 12 mu 2 1 ... 1 = 2 mu .
2
Since
N 1 N 1 2 3 N N 1 i2 i
Question 12
This can be broken down into more (four) separate cases, but there is no need to:
P(light on) = p 34 12 + (1 – p) 14 12 = 18 (1 2 p ) , and then the conditional probability
1
(1 p ) (1 p )
P(Hall | on) = 18 = .
8 (1 2 p ) (1 2 p )
To make progress with this next part of the question, it is important to recognise the underlying binomial
distribution, and that each day represents one such (Bernouilli) trial. We are thus dealing with B(7, p1),
(1 p )
where p1 = is the previously given answer.
(1 2 p )
For the modal value to be 3, we must have P(2) < P(3) < P(4); that is,
7 7 7 7
( p1 ) 2 (1 p1 ) 5 ( p1 ) 3 (1 p1 ) 4 and ( p1 ) 4 (1 p1 ) 3 ( p1 ) 3 (1 p1 ) 4 .
2 3 4 3
(1 p )
Using p1 = gives
(1 2 p )
2 5 3 4
1 p 3p 1 p 3p
21 35 33 p 51 p p 145
1 2 p 1 2 p 1 2 p 1 2 p
and
4 3 3 4
1 p 3p 1 p 3p
35 35 1 p 3 p p 14 .
1 2 p 1 2 p 1 2 p 1 2 p
Question 13
Working with the distribution Po( = ky2), P(no supermarkets) = e ky and P(Y < y) = 1 – e ky .
2 2
Differentiating w.r.t. y to find the pdf of Y f(y) = 2kπ y e ky , as given. Then
2
2kπ y 2 e ky dy . Using Integration by Parts and writing 2kπ y 2 e ky as y 2kπ y e ky
2 2 2
E(Y) =
0
gives E(Y) = y e ky e ky dy = 0 + e ky dy. It is useful (but not essential) to use the
2 2 2
0
0 0
1 12 x 2 1 1
simplifying substitution x = y 2k at this stage to get e dx = (by the
2
2k 0 2 k 2 k
given result, relating to the standard normal distribution’s pdf, at the very beginning of the question).
2kπ y 3 e ky dy , and using Integration by Parts and, in a similar way to earlier,
2
Next, E(Y 2) =
0
writing 2kπ y 3 e ky as y 2 2kπ y e ky , E(Y 2) =
2 2 y 2 e ky 2 2 y e ky 2 dy
0 0
1 ky 2 1 ky 2 1
=0+ 2k ye = e (using a previous result, or by substitution) =
k 0 k 0 k
1 1 4
Var(Y) = , the given answer, as required.
k 4k 4k
STEP 3 2012 Hints and Solutions
1. The stem integrates to give and for part (i) using 1 , the stem gives
which can solved for z using the initial conditions, and the integrated stem is a first
order differential equation for y, which when solved, again with the initial conditions, produces the
required result. Part (ii) follows the same pattern, with 2 instead, which has solution
.
3. The two parabolas, with vertices oriented in the direction of the positive axes, touch in the
third quadrant in case (a), and in the first quadrant in the other three cases. In case (b), there are
intersections in the second and third quadrants, in (c) in the third and fourth, and in (d), the trickiest
case, they are in the third quadrant and in the first, between the touching point and the vertex of
the parabola on the x axis. The first result of part (ii)is obtained by eliminating y between the two
equations, the second by differentiating and equating gradients, and the third, by eliminating ,
( , from the first result using the second.
√
The cases that arise are 1, 21 , 2 (d), , √13 , 2 (d),
√
and , √13 , 0 (a).
4. Writing as and then cancelling the first fraction, give exponential series.
! ! !
Similarly, 1 can be written as 1 3 1, and 2 1 as 8 1 2
12 1 2 1 , the latter giving the result 21 1 . Using partial fractions, can
be written as 1 , the first term giving a GP, and the other two, log series. The result for
part (ii) is thus 12 16 ln 2 .
5. For non‐integer rational points, it makes sense to use values of cos and sin based on
Pythagorean triples such as 3, 4, 5 or 5, 12, 13, The technique for (i) (b) can be used for (ii) (a),
merely by changing the value of m, whereas for (ii) (b), a slightly more involved expression is needed
such as cos √ sin , sin √ cos . For (ii) (c), there are two alternatives
that work sensibly, cosh √ sinh , sinh √ cosh
or sec √ tan , tan √ sec .
A completely different approach for the last part of the question is to write
7 and to choose √2 with nearly any choices of rational
a and b possible. Then, as , and numbers of the form √2 are a field over the
operations and , has to be of the correct form, and then solving for x and y, they
likewise have to be of the required form.
Some possible solutions are
(i)(a) 1,0 and , , (b) 1,1 , 1 , and , (using 1 , cos )
(ii)(a) 1, √2 and √2, √2 (using 2 , cos ) , (b) √2, √2 (using
3, 1, cos ), (c) √2 , √2 (using 2, 3 and cosh ,
sinh or sec , tan )
6. Substituting for z in the quadratic equation and equating real and imaginary parts
yields the first two results, the imaginary gives two situations, one as required and the other
substituted into the real gives the second. The first of these two results substituted into the real
gives a circle radius 1, centre the origin, whilst the second gives the real axis without the origin. The
second quadratic equation succumbs to the same approach giving the real axis without the origin
(again), and a circle centre 1,0 radius 1 also omitting the origin. The same approach in the third
√
case yields the real axis with and considering the discriminant, 0 and 2.
8. 1, 2, 3,
5 so both expressions in part (i) equal ‐1. Considering either
or ,
and applying the recurrence relation, both can be found to be zero. \so the given expression is
shown to be 1 if n is odd and ‐1 if n is even. The tan compound angle formula enables the proof in
part (iii) to be completed once the initial recurrence relation and the result from part (ii) have been
applied to the expression obtained following algebraic simplification. Rearranging the result and
substituting into the required sum gives, by the method of differences, ∑ tan
9. Eliminating , , , and between the five equations ,
, , , and yields the required
result for . The only difference in the second part is that the second equation becomes
, and so the same elimination yields ,
which can be seen to be smaller than that in part (i). The first four equations in this second part give
, and so as 0 , the required result follows.
10. After motion commences, the next at rest position has the string at to the vertical.
Conserving energy between the two at rest positions gives 3 . Conserving energy for the
general position and resolving radially, bearing in mind that the angle of the radius to the vertical is
twice the angle of the string to the vertical, and using a double angle formula gives the required
result. The discriminant being negative or completing the square demonstrates that the reaction
force is always positive.
11. Various approaches can be used to find the energy terms. If potential energy zero level is
taken to be at P, then the initial potential energy is 2 . When the particle has fallen a
distance x, the kinetic energy of the particle is , the potential energy of the particle is ,
the potential energy of the part of the stationary piece of string of length x is 2 , the
and the kinetic energy of the shorter moving piece is 2 . This yields the first result and
differentiation of it yields the second. As 0 2 , , as 0 and the
12. The sketched region contained by AB, and the two line segments connecting A and B to the
centre of the triangle. A simple approach for the pdf is via ∝ , finding the
constant of proportionality and then differentiating to give the required result. .
Using , and so
| | | | | |
STEP Solutions 2013
Mathematics
STEP 9465/9470/9475
October 2013
The Admissions Testing Service is a department of Cambridge English
Language Assessment, which is part of Cambridge Assessment, a not-for-
profit department of the University of Cambridge.
All Examiners are instructed that alternative correct answers and unexpected
approaches in candidates’ scripts must be given marks that fairly reflect the
relevant knowledge and skills demonstrated.
The Admissions Testing Service will not enter into any discussion or
correspondence in connection with this mark scheme.
© UCLES 2013
Report Page
STEP Mathematics I 4
STEP Mathematics II 13
STEP Mathematics III 19
STEP 1 2013 Hints & Solutions
Q1 This question is all about using substitutions to simplify the working required to solve
various increasingly complicated looking equations. To begin with, you are led gently by the hand
in (i), where the initial substitution has been given directly to you. You are also reminded that y
must be non-negative, since x denotes the positive square-root of x (positive unless x is 0, of
course). The result is obviously a quadratic equation, y 2 3 y 12 0 , and is solvable by (for
3 11
instance) use of the quadratic formula. However, only one of the apparent solutions, y ,
2
2
11 3
is positive, so the other is rejected and we proceed to find that x = 5 32 11 .
2
In (ii) (a), the approach used in (i) should lead you to consider the substitution y = x2,
which gives the quadratic equation y 2 10 y 24 0 . This, in turn, yields y = x 2 = –12 or 2.
Again, this must be non-negative, so we find that x 2 2 and x = 2.
In (ii) (b), it should not now be too great a leap of faith to set y 2 x 2 8 x 3 which, with
a bit of modest tinkering, yields up y 2 2 y 15 0 y = 2 x 2 8 x 3 = –5, 3. Again, y 0, so
2 x 2 8 x 3 = 3 (with cancelling) x 2 4 x 6 0 x 2 10 x 2 14 4 10 . The
final step here is to check that both apparent solutions work in the original equation, since the
squaring process usually creates invalid solutions. [Note that it is actually quite easy to see that both
solutions are indeed valid, but this still needs to be shown, or otherwise explained. Longer methods
involving much squaring usually generated four solutions, two of which were not valid.]
Q2 The “leading actor” throughout this question is the integer-part function, x , often referred
to as the floor-function. Purely as an aside, future STEP candidates may find it beneficial to play
around with such strange, possibly artificial, kinds of functions as part of their preparation because,
although they clearly go beyond the scope of standard syllabuses at this level, they are within reach
and require little more than a willingness to be challenged. [Note that some care needs to be taken
when exploring such things. In the case of this floor-function, at least a couple of function plotting
software programs that recognise the “INT” function do so incorrectly for x < 0: for instance,
interpreting INT(–2.7) as –2 rather than –3.]
The key elements of the sketch in (i) are as follows. The jump in the value of x whenever
x hits an integer value means that the graph is composed of lots of “unit” segments, the LH end of
which is included but not the RH end. The usual convention for signalling these properties is that
the LH endpoint has a filled-in dot while the RH endpoint has an open dot. Then, in-between
n
integer values of x, each segment of the curve is of the form and thus appears to be a portion of a
x
reciprocal curve.
The purpose of parts (ii) and (iii) is to see if you can use your graph to decide how to solve
some otherwise fairly simple equations: the key is to have a clear idea as to where the various
portions of the graph exist. The analysis looks complicated, but candidates were actually only
required to pick the appropriate n’s and write down the relevant answers (so the working only
needed to reflect what was going on “inside one’s head”). Note that, for n x n 1 , x n so
n n
f(x) = . Also, f ( x ) 1 for x > 0, and f(x) 1 for x < 0, so f(x) = 127 only in [1, 2), yielding
x n 1
1 7
the equation in (ii) x = 127 .
x 12
n 17
Similarly, 24n > 17n + 17 n > 2 73 , i.e. n 3; so f(x) = 17 24 only in [1, 2)
n 1 24
1 17 2 17
and [2, 3). In [1, 2), f ( x) x = 17 24
and in [2, 3), f ( x) x = 17 48
. Next, for
x 24 x 24
n n 4
x < 0, 1 f ( x) , and –4n – 4 > –3n n < –4; so f(x) = 43 only in [–4, –3),
n 1 n 1 3
[–3, –2), [–2, –1) and [–1, 0). However, since f(–3) = 1 there is no solution in [–4, –3). Otherwise,
3 4 2 4
in [–3, –2), f ( x) x = 94 ; in [–2, –1), f ( x) x = 32 ; and in [–1, 0),
x 3 x 3
1 4
f ( x) x = 34 .
x 3
n 9 8 9
For (iii), for n > 9 so f(xmax) = 109 in [8, 9) and f ( x) x = 809 .
n 1 10 x 10
Only the very last part required any great depth of insight, and the ability to hold one’s
nerve. The equation f(x) = c has exactly n roots when the horizontal line y = c cuts the curve that
number of times. That is …
n n 1 n 1 n
… when x > 0 : c ; … when x < 0: c , n 2; … and c 2 for n = 1.
n 1 n2 n n 1
Q3 This vector question is tied up with the geometric understanding that, for distinct points with
position vectors x and y, the point with p.v. x + (1 – )y cuts XY in the ratio (1 – ): (though it is
important to realise that this point is only between X and Y if 0 < < 1). Part (i) tests (algebraically)
the property of commutativity (whether the composition yields different results if the order of the
application of the operation is changed):
XY = YX x + (1 – )y = y + (1 – )x (2 – 1)(x – y) = 0 (since x y) = 12 .
Part (ii) then explores the property of associativity (whether the outcome is changed when
the order of the elements involved in two successive operations remains the same but the pairings
within those successive operations is different). Here we have
(XY)Z = (x + (1 – )y) + (1 – )z = 2x + (1 – )y + (1 – )z
and X(YZ) = x + (1 – )[y + (1 – )z] = x + (1 – )y + (1 – )2z
Thus, (XY)Z – X(YZ) = (1 – )(x – z) and the two are distinct provided 0, 1 or X Z.
Part (iii) now explores a version of the property of distributivity (although usually referring
to two distinct operations): (XY)Z = 2 x (1 )y (1 )z , and
(XZ)(YZ) = [x + (1 – )z][y + (1 – )z] = 2 x (1 )z + (1 )y (1 ) 2 z
= 2 x (1 )y (1 )z , and the two are always equal.
Next, X(YZ) = x (1 )y (1 ) 2 z , and
(XY)(XZ) = [x + (1 – )y][x + (1 – )z]
= 2 x (1 )y + (1 )x (1 ) 2 z
= 2 x (1 )y (1 )z .
Hence X(YZ) = (XY)(XZ) also.
In (iv), you will notice that the condition 0 < < 1 comes into play, so that P1 cuts XY
internally in the ratio (1 – ):. Following this process through a couple more steps shows us that
Pn cuts XY in the ratio (1 – n):n , which is easily established inductively.
Q4 The first part to this question involved two integrals which can readily be integrated by
“recognition”, upon spotting that
d
f (tan x) f (tan x) sec 2 x and d f (sec x) f (sec x) sec x tan x .
dx dx
(They can, of course be integrated using suitable substitutions, etc.) Thus, we have
1 n 1 1
tan x. sec x dx = n 1 tan x = n 1
n 2
n n 1 1
and sec x. tan x dx = sec x. sec x tan x dx = sec n x =
2 1
.
n
n n
The two integrals in part (ii) can be approached in many different ways – the examiners
worked out more than 25 slightly different approaches, depending upon how, and when, one used
the identity sec2x = 1 + tan2x, how one split the “parts” in the process of “integrating by parts”, and
even whether one approached the various secondary integrals that arose as a function of sec x or
tan x. Only one of these approaches appears below for each of these two integrals..
/4
sec 4 x / 4 / 4 sec 4 x 1
/4
0 – J , where J = sec
4 4
x sec x tan x dx = x . dx (by parts) = x dx.
4 0 0
4 4 4 0
/4 /4
1
Then, J = sec
2
x dx + sec
2
x tan 2 x dx = tan x 13 tan 3 x =
4
3
, and our integral is .
4 3
0 0
Next, x 2 sec 2 x. tan x dx = x 2 . 12 tan 2 x – 2 x . 12 tan 2 x dx (by parts) =
2
32
x sec 2 x 1
/4
2
K x dx, where K = x sec
2
= x dx.
32 0
Then, K = x . tan x – tan x dx = x tan x – ln(sec x) =
1
ln 2 , so that this last integral is
4 2
2 1
2
2 1
– ln 2 + or ln 2 .
32 4 2 32 16 4 2
Q5 This question simply explores the different possibilities that arise when considering curves
of a particular quadratic form. In (i), with a zero product term, we have equal amounts of x2 and y2,
and this is symptomatic of a circle’s equation: x 2 3 x y 2 y 0 x 32 y 12
2 2
1
2
2
10 ,
which is a circle with centre 32 , 12 and radius 12 10 . This circle passes through the points (0, 0),
(0, –1) & (–3, 0).
In (ii), with k = 10
3 , we have 3 x y x 3 y 3 0 . This factorisation tells us that we have
the line-pair y = –3x & x + 3y = –3; the first line passing through the origin with negative gradient,
while the second cuts the coordinate axes at (0, –1) and (–3, 0).
Part (iii) is the genuinely tough part of the question, but help is given to point you in the
right direction. When k = 2, we have x y 3 x y 0 , and using = 45o in the given
2
X Y X Y
substitution gives x y X 2 and y x Y 2 x and y . Then
2 2
3 X 3Y X Y
x y 2 3 x y 0 becomes 2X 2 0 2 X 2 2 2 X Y 2 or
2 2
2
2 X 1 1 Y 2 . This is now in what should be a familiar form for a parabola, with axis of
1 x y 1
symmetry X (substituting back)
i.e. x y 1 . For the sketch, we must
2 2 2
rotate the standard parabola through 45o anticlockwise about O in order to get the original parabola
x 2 2 xy y 2 3 x y 0 .
For those who have encountered such things, all three curves here are examples of conic
sections.
Q6 It should be pretty clear that this question is all about binomial coefficients. The opening
result – the well-known Pascal Triangle formula for generating one row’s entries from those of the
previous row – is reasonably standard and can be established in any one of several ways. The one
n 1
intended here was as follows: the coefficient of xr in (1 + x)n + 1 is , and this is obtained
r
from (1 + x)(1 + x)n, where the coefficient of xr comes from
n r 1 n r
1 x ..... x x .....
r 1 r
and the required result follows.
In the next stage, for n even, write n = 2m so that
2m 2m 1 2m 2 m 1 m
B2m + B2m + 1 = .....
0 1 2 m 1 m
2m 1 2m 2m 1 2m 2 m 1
+ ..... .
0 1 2 3 m
and, pairing these terms suitably, this is
2m 1 2m 2m 2m 1 2m 1 m 1 m 1 m
..... .
0 0 1 1 2 m 1 m m
2m 1 2m 2 m m 1
Now, using the opening result, and the fact that 1 , we have
0 0 m m 1
2m 2 2m 1 2m m 2 m 1
..... ,
0 1 2 m m 1
m 1 2( m 1) j
and this is just = B2m + 2, as required.
j0 j
In the case n odd, write n = 2m + 1, so that
2m 1 2m 2m 1 m 2 m 1
B2m + 1 + B2m + 2 = .....
0 1 2 m 1 m
2m 2 2m 1 2m 2m 1 m 2 m 1
+ .....
0 1 2 3 m m 1
which gives, upon pairing terms suitably,
2m 2 2m 1 2m 1 2m 2m m 2 m 2 m 1 m 1
.....
0 0 1 1 2 m 1 m m m 1
2m 3 2m 2 2m 1 m 3 m 2
= .....
0 1 2 m m 1
2m 2 2m 3
using the opening result and the fact that 1
0 0
m 1 2( m 1) 1 j
= = B2m + 3.
j0 j
To complete an inductive proof, we must also check that the starting terms match up properly, but
this is fairly straightforward. We conclude that, since B0 = F1 , B1 = F2 and Bn & Fn satisfy the
same recurrence relation, we must have Bn = Fn + 1 for all n.
Q7 In (i), there is a generous tip given to help you on your way with this question. Starting from
dy du du 1
y = ux we have ux , so that the given differential equation becomes u x u or
dx dx dx u
1
u du x dx upon separation of variables. You are now in much more familiar territory and may
y2
proceed in the standard way: 12 u 2 2 ln x C y 2 x 2 2 ln x 2C . Using the given
2x
conditions x = 1, y = 2 to determine C = 2 then gives the required answer y x 2 ln x 4 .
However, there is one small detail still required, namely to justify the taking of the positive square-
root, which follows from the fact that y > 0 when x = 1. (Note that you were given x > e – 2 , for the
validity of the square-rooting to stand, so it is not necessary to justify this. However, it should serve
as a hint that a similar justification may be required in the later parts of the question.)
In (ii), either of the substitutions y = ux or y = ux2 could be used to solve this second
differential equation. In each case, the method then follows that of part (i)’s solution very closely
indeed; separating variables, integrating, eliminating u and substituting in the condition x = 1, y = 2
to evaluate the arbitrary constant. The final steps require a justifying of the taking of the positive
square-root and a statement of the appropriate condition on x in order to render the square-rooting a
valid thing to do. The answer is y x 5 x 2 1 for x > 1
5
.
In (iii), only the substitution y = ux2 can be used to get a variable-separable differential
1 1
equation, which boils down to u du 2 dx 12 u 2 D . Using x = 1, y = 2 (u = 2) to
x x
evaluate the constant D leads to the answer y x 6 x 2 2 x for x > 1
3 .
Q8 This question is all about composition of functions and their associated domains and ranges.
Whilst being essentially a very simple question, there is a lot of scope for minor oversights. One
particular pitfall is to think that x 2 x , when it is actually | x |. Also, when considering the
composite function fg, it is essential that the domain of g (the function that “acts” first on x) is
chosen so that the output values from it are suitable input values for f. You should check that this is
so for the four composites required in part (i).
In (ii), the functions fg and gf look the same (both are | x |) but their domains and ranges are
different: fg has domain ℝ and range y 0, while the second has domain | x | 1 and range y 1.
In (iii), the essentials of the graph of h are: it starts from (1, 1) and increases. Since x 2 1
is just (x – a tiny bit) after a while, the graph of h approaches y = 2x from below. Using similar
reasoning, the graph of k for x ≥ 1, also starts at (1, 1) but decreases asymptotically to zero. (It is
well worth noting that x x 2 1 is the reciprocal of x x 2 1 , since their product is 1.)
However, this second graph has a second branch for x –1, which is easily seen to be a rotation
about O (through 180o) of the single branch of h, this time approaching y = 2x from above. Finally,
note that the domain of kh is x 1, and since the range of h is y 1, the range of kh is 0 < y 1.
Q9 This question incorporates the topics of collisions and projectiles, each of which consists of
several well-known and oft quoted results. However, much of the algebraic processing can be
shortcut by a few insightful observations. To begin with, if the two particles start together at ground
level and also meet at their highest points, then they must have the same vertical components of
velocity. Thus u sin v sin . Then, if they both return to their respective points of projection, the
collision must have ensured that they both left the collision with the same horizontal velocity as
when they arrived; giving, by Conservation of Linear Momentum, that mucos = Mvcos. Dividing
these two results gives the required answer, m cot = M cot.
u sin v sin
The collision occurs when t (from the standard constant-acceleration
g g
u 2 sin cos
formulae) and at the point when A has travelled a distance b = and B has travelled a
g
v 2 sin cos 1
distance (v sin )(v cos ) , the sum of these two distances being denoted d.
g g
Md
Substituting for the brackets using the two initial results then gives b , as required.
mM
u 2 sin 2
Moreover, the height of the two particles at the collision is given by y = , so that
2g
1 u 2 sin cos sin 1
h= = b tan .
2 g cos 2
Q10 This question makes more obvious use of the standard results for collisions, but also ties
them up with Newton’s 2nd Law (N2L) of motion and, implicitly, the Friction Law and resolution
of forces (in the simplest possible form). Thus, if R is the normal contact reaction force of floor on
puck, F the frictional resistance between floor and puck, we have (in very quick order) the results
R = mg, F = R = mg and, by N2L, mg = –ma, where a is the puck’s acceleration. The constant-
acceleration formula v2 = u2 – 2as then gives wi 1 vi 2gd (where vi is the speed of the puck
2 2
when leaving the i-th barrier, for i = 0, 1, 2, …, and wi is its speed when arriving at the i-th barrier,
for i = 1, 2, 3, …). Also, Newton’s (Experimental) Law of Restitution (NEL or NLR) gives
vi 1 r.wi 1 , from which it follows that vi 1 r 2 vi 2r 2 gd , as required.
2 2
Iterating with this result, starting with v1 r 2 v 2 2r 2 gd , then leads to the general result
2
v n r 2 n v 2 2r 2 gd 1 r 2 r 4 ... r 2 n 2 . The large bracket is the sum-to-n-terms of a GP,
2
1 r 2n
namely , and you simply need to set vn = 0 and tidy up in order to obtain the result
1 r 2
v2
r 2n
1 r 2n r 2 .
2gd 1 r2
v2 r2
by k and re-writing the expression for r2n 2 , we simply have to
Replacing
2 gd
r k 1 r
2
r2
take logs and solve for n to get n =
ln 2
r k 1 r2 . Setting r = e – 1 in this result, and tidying
2 ln r
up, then gives n = 12 ln 1 k e 1 .
2
v2
When r = 1, the distance travelled is just nd, and we have v2 = 2 gnd and n k.
2gd
T cos( )
resultant force on the block is TA + TB +W = , which is in the
T sin( ) T tan 2 ( )
1
sin( ) tan 12 ( )
direction tan 1 (relative to i). Since + = 2 , this is the direction
cos( )
sin 12 ( )
2 sin 12 ( ) cos 12 ( )
1 sin 2 ( )2 cos 2 ( ) 1
1 cos 12 ( ) 1 2 1
tan tan
cos( ) cos( ) . cos 2 ( )
1
tan tan 2 ( ) = 2 ( ) , noting that 2 cos 2 12 ( ) 1 cos( ) .
1 1 1
Q12 As with all such probability questions, there are many ways to approach the problem. The
one shown here for part (i) is one that generalises well to later parts of the question. Suppose the
3 3 3 3
container has 3R, 3B, 3G tablets. Then the probability is for one specified order (e.g.
9 8 7 56
9
RBG). We then multiply by 3! = 6 for the number of permutations of the 3 colours to get . The
28
final part of (i) is really a test of whether you realiuse that this is the same situation viewed “in
reverse”, so the answer is the same.
Using the method above, with a suitable notation, in part (ii) we have
n n n 2n 2 n3
P3(n) = 3! = or .
3n 3n 1 3n 2 3n 13n 2 3n
3
Then in (iii), P(correct tablet on each of the n days) = P2(n)P2(n – 1)P2(n – 2) … P2(2)P2(1)
n 2 . 2 ! (n 1) 2 . 2 ! (n 2) 2 . 2 ! 2 2 . 2 ! 12 . 2 !
=
...
2n(2n 1) (2n 2)(2n 3) (2n 4)(2n 5) 4 . 3 2 .1
n 2n
( n !) 2 2 n 2n n 2n
= or .Then, using n! 2n and (2n)! 4 n , we have
( 2n) ! 2n e e
n
n 2n
2 n 2n
e 2n 2n 2 n n
Prob. = = n .
2 n2n 2n
2 n 2 2n
2
4 n 2n
e
Q13 First, note that x {0, 1, 2, 3}, so there are four probabilities to work out (actually, three,
and the fourth follows by subtraction from 1). The easier ones to calculate are X = 0 and X = 3, so
let us find these first.
For P(X = 0): the 7 pairs from which a singleton can be chosen can be done in 26C7 ways;
7
26
C7 27
then, we can choose one from each pair in 2 ways; so that P(X = 0) = 52
.
C7
26.25.24.23.22.21.20 52 52.51.50.49.48.47.46 3520
Now 26C7 = & C7 = P(X = 0) = .
7 .6 .5 .4 .3 .2 .1 7 .6 .5 .4 .3 .2 .1 5593
P(X = 3): the 3 pairs from 26 can be chosen in 26C3 ways; then the one singleton from the
remaining 23 pairs can be chosen in 23C1 = 23 ways, and the one from that pair in 21 ways ; so that
26
C 3 23 C1 21 5
P(X = 3) = 52
= (similarly for calculation).
C7 5593
For P(X = 1): the 1 pair can be chosen in 26C1 = 26 ways; the 5 pairs from which a singleton
is chosen can be done in 25C5 ways; and the singletons from those pairs in 25 ways; so that
26 25 C 5 2 5 1848
P(X = 1) = 52
= .
C7 5593
For P(X = 2): the 2 pairs can be chosen in 26C2 ways; then the 3 pairs from which a
singleton is chosen can be done in 24C3 ways; and the singletons from those pairs can be chosen in
26
C 2 24 C 3 2 3 220
23 ways; so that P(X = 2) = 52
= .
C7 5593
Then E(X) = x . p( x) =
1
0 3520 1 1848 2 220 3 5
5593
2303 7 7 47 7
= .
5593 7 17 47 17
STEP 2 2013 Hints and Solutions
Question 1.
The gradient of a line from a general point on the curve to the origin can be calculated easily and the
gradient of the curve at a general point can be found by differentiation. Setting these two things to
be equal will then lead to the correct value of . A similar consideration of gradients to the origin
will establish the second result and if the line intersects the curve twice then a sketch will illustrate
that there must be one intersection on each side of the point of contact found in the first case. A
similar process will establish the result for part (ii).
For part (iii) the gradient of the line must be smaller than the gradient of the line through the origin
which touches the curve, so the intersection with the y-axis must be at a positive value. This means
that the conditions of part (ii) are met, which allows for the comparison between and to be
made.
The condition given in part (iv) is equivalent to stating that the line is parallel to the one found at the
very beginning of the question. This implies that the intersection with the y-axis is at a negative
value and so an adjustment to the steps taken in part (ii) will establish the required result.
Question 2.
The obvious substitution in the first part leads easily to the required result. It should then be easy to
establish the second result by making the integral into the sum of two integrals and noting that
taking out a common factor leaves ( ) to be simplified. Integration by parts will lead to the
next result after which taking out one of the factors of ( ) will allow the integral to be split into
a difference of two integrals.
The result in part (ii) is most easily proved by induction. It is necessary to fill in the gap in the
factorial on the denominator by multiplying both the numerator and denominator by the missing
even number. In alternative approaches, it needs to be remembered that the product of the even
numbers up to and including can be written as
The final part is a straightforward substitution, although care needs to be taken with the signs. The
final result can be obtained using the relationship established in part (i) as none of the reasoning
requires to be an integer.
Question 3.
For it to be possible for the cubic to have three real roots it must have two stationary points. Since
the coefficient of is positive it must have a specific shape. A sketch will show that only the two
cases given will result in an intercept with the y-axis at a negative value.
In order for the cubic in part (ii) to have three positive roots, both of the turning points must be at
positive values of . Differentiation will allow most of the results to be established. The condition
that is needed to ensure that the leftmost root is also positive.
The condition implies that there must be a turning point at a positive value of . The shape
of the graph is as in part (i), but this time the intersection with the y-axis is at a positive value. This is
sufficient to deduce the signs of the roots.
For part (iv) it is easiest to note that changing the value of does not (as long as remains negative)
change whether or not the conditions of (*) are met. As this represents a vertical translation of the
graph any example of a case satisfying (*) can be used to create an answer for this part by
translating the graph sufficiently far downwards.
Question 4.
The equations of the line and circle are easily found and so the second point of intersection (and so
the coordinates of M) can be easily found. The two parts of this question then involve regarding the
coordinates of M as parametric equations.
In part (i) is the parameter and is restricted so that the point that the line passes through is inside
the circle. This gives a straight line between the points which result from the cases and
. The length of this line can be determined easily from the coordinates of its endpoints.
In part (ii) it is again quite easy to eliminate the parameter from the pair of equations and the shapes
of the loci should be easily recognised. In part (b) however, the restriction on the values of need to
be considered as the locus is not the whole shape that would be identified from the equation.
Question5.
Simple applications of the chain rule lead to relationships that will allow the three cases of zero
gradients to be identified in part (i).
In part (ii) the relationships follow easily from substitution and therefore the three stationary points
identified in part (i) must all exist. By considering the denominator there are clearly two vertical
asymptotes and the numerator is clearly always positive. Additionally, the numerator is much larger
than the denominator for large values of . Given this information there is only one possible shape
for the graph.
In part (iii) the solutions of the first equation will already have been discovered when the
coordinates of the stationary points in part (ii) were calculated. The range of values satisfying the
first inequality should therefore be straightforward. One of the solutions of the second equation
should be easy to spot, and consideration of the graph shows that there must be a total of six roots.
Applying the two relationships about the values of will allow these other roots to be found. The
solution set for the inequality then follows easily from consideration of the graph.
Question 6.
The definition of the sequence can be used to find a relationship between and and
therefore also a relationship between and . Taking the difference of these then leads to the
required result.
It is clear from the definition of the sequence that, if one term is between 1 and 2, then the next
term will also be between 1 and 2. This is then easy to present in the form of a proof by induction for
part (ii).
The result of part (i) shows that the sequence in part (iii) is increasing and the result proved in part
(ii) shows that it is bounded above. The theorem provided at the start of the question therefore
shows that the sequence converges. Similarly the second sequence is bounded below and
decreasing (and therefore if the terms are all multiplied by -1 a sequence will be generated which is
bounded above and increasing). Therefore the second sequence also converges to a limit.
The relationship between and established in part (i) can then be used to find the value of
this limit and, as it is the same for both the odd terms and the even terms, the sequence must tend
to the same limit as well.
Finally, starting the sequence at 3 will still lead to the same conclusion as the next term will be
between 1 and 2 and all further terms will also be within that range, so all of the arguments will still
hold for this new sequence.
Question 7.
A solution of the equation should be easy to spot and a simple substitution will establish the new
solution that can be generated from an existing one. This therefore allows two further solutions to
be found easily by repeated application of this result.
In part (ii) write and and then substitute into (*). With some simplification the
required relationship will be established.
Since is a prime number there is only two ways in which it can be split into a product of two
numbers ( and ). The right hand side of the equation is clearly a difference of two
squares and therefore a pair of simultaneous equations can be solved to give expressions for and
. Finally, the expression for is similar to the relationship established in part (ii), so solutions to
the original equation can be used to generate values of , and which satisfy this equation.
Question 8.
Begin by calculating the largest area of a rectangle with a given width and then maximize this
function as the width of the rectangle is varied. The definition of can be reached by setting the
derivative of the area function to 0.
The definition of involves the differentiation of an integral of which uses the variable as the
upper limit. The derivative of ( ) is therefore ( ). The next statement relates the area bounded
by the curve and the line ( ) with the area of the largest rectangle with edges parallel to the
axes that can fit into that space, so the first area must be greater and since that integral is equal to
( ) ( ) the result that follows is easily deduced.
The final part of the question involves finding expressions for ( ) and ( ) and then simplifying
the relationship established at the end of part (ii).
Question 9.
Resolving the forces vertically will establish the first result. For the second part of the question it can
be established that all of the frictional forces are equal in magnitude by taking moments about the
centre of one of the discs. Resolving forces vertically and horizontally for the discs individually will
then lead to simultaneous equations that can be solved for the magnitudes of the reaction and
frictional forces.
Since the discs cannot overlap there is a minimum value that can take and the value of is
increasing as increases. This allows the smallest possible value of the frictional force between the
discs to be calculated and therefore it can be deduced that no equilibrium is possible if the
coefficient of friction is below this minimum value.
Question 10.
Following the usual methods of considering horizontal and vertical parts of the motion will lead to
the first result (some additional variables will need to be used, but they will cancel out to reach the
final result.
If and are the same point then the result in part (i) can be applied for this point which will give
an equation which is easily solved to give once the double angle formula has been applied.
For the final part it is possible to find the times at which the particle reaches each of the two points.
The two equations reached can then be used to find an expression for the difference between the
time at which the particle reaches each of the two points and then it can easily be deduced whether
this is positive or negative, which will show which point is reached first.
Question 11.
The standard methods of conservation of momentum and the law of restitution will allow the
speeds after the second collision to be deduced. A third collision would have to be between the first
and second particles and this will only happen if the velocity of the first particle is greater than that
of the second one.
Providing a good notation is chosen to avoid too much confusion, it is possible to find the velocities
after the third collision and then consider the velocities of the second and third particles to
determine whether or not there is a fourth collision.
Question 12.
The formula for the expectation of a random variable should be well known and both of the
expectations can easily be written in terms of and .
Similarly, the formula for variance should be well known and so it is a matter of rearranging the
sums in such a way as to reach the forms given in the question. Note that the definitions of and
are such that .
Since the ( ) ( ) the equation in the final part of the question can be rewritten in
terms of the variables defined at the start of the question. It can then be shown that this is not
possible for any non-zero value of .
Question 13.
An alternating run of length 1 must be two results showing the same side of the coin. It is then easy
to show that the probability is as given. Similarly a straight run of length 1 must be two different
results (in either order) and so the probability can again be calculated easily. The terms involved are
those in the expansion of ( ) and so starting with the statement that ( ) then
relationship between the two probabilities can be established.
An alternating run of length 2 must be one result followed by the other one twice, while a straight
run of length 2 must be two identical results followed by the other one. They will therefore be
calculated by the same sums (with the products in a different order each time) so the probabilities
must be equal. By considering the ways in which runs of length 3 can be obtained it is clear that
these two probabilities must also be equal.
An alternating run of length must be of each of the two possibilities followed by a repeat of
whichever came last. A straight run of length must be of one of the possibilities followed by 1
of the other. Taking the difference between these two probabilities gives an expression which can be
seen to always have the same sign, which will determine which probability is greater. A similar
method will also work for the final case.
STEP 3 2013 Hints and Solutions
1. The first two results, whilst not necessarily included in current A2 specifications, are
standard work. Applying them, 2 , which can then be
obtain the required result, tan tan tan must be simplified using the relevant
√ √
compound angle formula.
2. It is elegant to multiply by the denominator, then differentiate implicitly, and finally multiply
by the same factor again to achieve the desired first result. The general result can be proved by then
using induction, or by Leibnitz, if known. The general result can be used alongside the expression for
, and the first derived result with the substitution 0 to show that the general term of the
!
Maclaurin series for even powers of x is zero, and for odd powers of x is . Thus, as
!
√
⋯ the required infinite sum is with , that is .
! !
3. The scalar product of with ∑ , which is of course zero, can be expanded giving
. 1 and three products . which are equal by symmetry, giving the required result.
Expanding the expression suggested in (i), gives ∑ . 2 . . , which, bearing in mind
that . 1, . 1 , and that . ∑ 0 , gives the correct result. Considering that
. , . 1 , and that a is positive, enables the given values to be found. Similarly
√ √
. , . , and . 1 yields , , , . In (iii), using the
√
√ √ √ √ √
∑ 16 4
√ √
16 4 which is sufficient.
4. The initial result is obtained by expanding the brackets and expressing the exponentials in
trigonometric form. The (2n)th roots of ‐1 are , 1 , which lead to the
factors of 1 and these paired using the initial result give the required result. Part (i) follows
directly from substituting in the previous result, and as n is even, 1 2 . Using the
given factorisation in part (ii), the general result can be simplified by the factor
6. The opening result is the triangle inequality applied to OW, OZ, and WZ where OW and OZ
are represented by the complex numbers w and z.
∗ ∗ ∗ ∗
Part (i) relies on using | | , , | | | | | | , and
∗ ∗
substituting 2| | . Having obtained the desired equation , the reality of E is
apparent from the reality of the other terms and its non‐negativity is obtained from the opening
result of the question. Part (ii) relies on the same principles as part (i).
The inequality can be most easily obtained by squaring it, and substituting for both numerator and
denominator on the left hand side using parts (i) and (ii), and algebraic rearrangement leads to
1 | | 1 | | 0 which is certainly true. The argument is fully reversible as | | 1 , and
| | 1,| ∗ | 1, and so 1 ∗
0 so the division is permissible, and the square rooting of
the inequality causes no problem as the quantities are positive. The working follows identically if
| | 1 , and | | 1 .
initial conditions. The deduction follows from the non‐negativity of . In part (ii), it can be
8. The sum is evaluated by recognising that it is a geometric progression with common ratio
⁄ ⁄
which may be summed using the standard formula and as 1 0 , the denominator
is non‐zero so the sum is zero. By simple trigonometry, cos . As , .
Thus where 1 ⁄ . Simplifying, . The
summation of the reciprocals of this expression is simply found using a double angle formula and
then by expressing the trigonometric terms as the real part of the sum at the start of the question.
10. The initial result can be obtained in a number of different ways, but probably use of the
parallel axes rule is the simplest. Conserving angular momentum about P,
11. As the distance from the vertex to the centre of the equilateral triangle is , the extended
length of each spring is giving the tension in each as which simplifies to the
given result. Resolving vertically 3 sin 3 , and using the result for , substituting ,
and rationalising the denominator gives the required value for . Conserving energy, when ,
!
12. 1 , so . There are arrangements of the As and Bs, and the
! !
!
number of arrangements with a B in the 1 th place and an A in the th place is ,
! !
thus ∑ 1 and so
13. integrating 0 between limits of 0 and gives the result of (a) (i), and
integrating the left hand side by parts yields part (ii). As is a probability density function,
1 , which can be evaluated using the result of (a) (ii) with 2 and so
2. 2 2 2 and using (a) (ii) ,
2 1 , as
, integration by parts gives
. Part (iii) is derived from part (ii) by rearranging
Mathematics
STEP 9465/9470/9475
October 2014
The Admissions Testing Service is a department of Cambridge English
Language Assessment, which is part of Cambridge Assessment, a not-for-
profit department of the University of Cambridge.
All Examiners are instructed that alternative correct answers and unexpected
approaches in candidates’ scripts must be given marks that fairly reflect the
relevant knowledge and skills demonstrated.
The Admissions Testing Service will not enter into any discussion or
correspondence in connection with this mark scheme.
© UCLES 2014
Report Page
STEP Mathematics I 4
STEP Mathematics II 11
STEP Mathematics III 18
STEP 1 2014 Hints and Solutions
Q1 This question is the traditional starter, involving ideas that should certainly be familiar to all
candidates. That doesn’t necessarily mean it is easy, just that everyone should be able to make a
start with it, and make some good progress thereafter. In fact, parts (i), (ii) and (iii) are each
especially amenable to all; and (i) is intended to help get you started along this particular road by
having you first write out some numerical examples of differences of two squares, before asking
you to move on to the algebra.
To begin with, (ii) requires only the observation that each odd number is the difference of
consecutive squares, 2k – 1 k2 – (k – 1)2; and then (iii) relies on noting that multiples of 4 arise
from the difference of squares of numbers that are two apart, namely 4k (k + 1)2 – (k – 1)2. Part
(iv) develops these ideas further, although it is important now to deconstruct the problem into its
building blocks; in this case, that means examining the four possible cases for a2 – b2 when a and b
are either odd or even, none of which cases yield an even answer that is not automatically a multiple
of 4. Alternatively, this is very easily addressed by examining squares modulo 4.
Part (v) draws on ideas of factorisation of (positive) integers into factor-pairs. Here, you are
given the product pq where both p and q are odd. Using the difference of two squares factorisation,
if pq = a2 – b2 = (a – b)(a + b), then either a – b = 1 and a + b = pq OR a – b = p and a + b = q
(taking p < q w.l.o.g.), giving the (exactly) two required factorisations. However, if p = 2, then pq
is a multiple of 2, but not 4, and we have already shown this case to be impossible.
The final part of the question pulls all these ideas together in a numerical example, and we
need only prime-factorise 675 = 3352 and note that this yields (3 + 1)(2 + 1) = 12 factors and hence
six factor-pairs.
Q2 The main ideas behind this question are relatively straightforward, but there are many
difficulties in the execution of them. In (i), an integral of the form ln(x) dx would usually have to
be approached by writing it as the product ln( x) 1 dx and integrating by parts. On this occasion,
however, with a given result, it is perfectly possible to verify by differentiation. Curve-sketching
(rather than plotting) is a key skill and one that needs practice. The things to look for are crossing-
points on the two coordinate axes (which arise here when ln(1) appears), asymptotes (from when
we get ln(0)), symmetries and – if further detail is required – turning points. It should be obvious
here, for instance, that the curve is an even function, so there is a turning point when x = 0. In (iii),
you are given the area to be found so that you can check you are doing everything correctly before
you go on to answer the rather tougher part (iv). You should first realise (from your sketch-graph)
that it is required to integrate ln(4 – x2) between 3 and 3 , though you should always be on the
lookout for opportunities to make the working easier – here, you could integrate between 0 and 3
and then double the answer. Next, there is a very strong hint supplied by part (i) to split the log.
term up as ln(2 – x) + ln(2 + x), the first term of which has already been done and the second can be
deduced from it with a bit of care.
Part (iv) actually asks nothing new. Curve B is just curve A with all portions drawn above
the x-axis and it is only required that you deal with the extra bit(s) of area, and the awkwardness of
finding an area up to the asymptote is covered for you by the footnote that follows (iv).
Q3 This question was actually devised to address what happens when students misunderstand or
mis-apply a “rule” of mathematics and it turns out to give the right answer. Part (i) starts you off
gently: integrating both terms, squaring the RHS and solving very quickly gives b = 43 . Part (ii)
develops in much the same way, but with a non-zero lower limit to the integrals, and we
immediately see that the algebra gets much more involved. Importantly, it should be very clear that
whatever expression materialises must have (b – 1) as a factor (since setting b = a would definitely
give a zero area, thus trivially satisfying the given integral statement). This leads to the required
cubic equation.
The final part of (ii) requires a mixture of different ideas (and can be done in a number of
different ways). The most basic approach to demonstrating that a cubic curve has only one zero is to
illustrate that both of its TPs lie on the same side of the x-axis (or to show there are no TPs). The
popular Change-of-Sign Rule for continuous functions can be used to identify the position of this
zero.
Having got you started with some simple lower limits, part (iii) develops matters more
generally, and derives the (perhaps) surprising result that the exploration of this initial “stupid idea”
requires b and a to be “not too far apart” to an extent that is easily identifiable.
Q4 This is quite a sweet little question, and deals with the movements of the hands of a clock.
dx ab sin
Using the Cosine Rule and differentiating gives . However, it is
d b 2 a 2 2ab cos
dx d2x dx d2x d
important to note that we can only work with and , rather than and , since is
d d 2
dt dt 2
dt
constant (so that extra terms in the use of the Chain Rule simply cancel). Then, differentiating again
with respect to , equating to zero and solving leads to the quadratic equation in C = cos :
abC2 – b 2 a 2 C + ab = 0
which can be factorised (or solved otherwise). Only one of these solutions gives | C | < 1, and
a
substituting cos = gives the required result. [In the normal course of events, one should justify
b
that this is a maximum value rather than a minimum – but it is rather clear that it must be so in this
d 11
case.] Finally, 6 is the constant rate of change of , the angle between the two hands, and
dt
solving cos116 12 gives a time of 112 hours 11 minutes, as required.
Q5 This is another question that asks you to deploy your graph-sketching skills – in association
with the supporting algebra, of course – and finding the coordinates of the TPs of the family of
cubics given here, at (a, 0) and (–5a, 108a3) makes it clear that the requested result holds.
However, the case a = 0 (despite its relative triviality) still needs to be addressed separately.
Using (i)’s result with a = y then immediately gives 27xy2 (x + 2y)3 33 so that xy2 1.
Equality holds (from part (i)) when x = a = y and from when x + 2y = 3 x = y = 1.
Part (iii) requires a little more care and perseverance, but setting x = p and 2a = q + r gets
you off to a good start. There is now a little bit of “working to one side” needed if you are to
qr
2
convince yourself that qr . Those students who have previously encountered the
2
Arithmetic Mean – Geometric Mean Inequality will have spotted this straightaway; otherwise, this
result becomes obvious upon rearrangement, as it is true (q – r)2 ≥ 0, which is clearly true.
Having to write it out in this way does have the advantage of highlighting that equality holds if and
only if q = r here (leading to p = q = r for the main result).
1
Q8 Part (i) directs you to finding the equation of La, which is y 1 ( x a ) , and it follows
a
1
immediately that Lb has equation y 1 ( x b) similarly. Solving simultaneously yields their
b
point of intersection at ab, (1 a)(1 b) . As b a, this point of intersection a 2 , (1 a ) 2 , and
it is clear that a = x , so that 0 <
x < 1 and 0 < x < 1, and that y 1 x . 2
with equation y 1 c 2
1
1
x c . This rearranges into the form y 1
1
x c ,
c c
which is simply L c
where 0 < c < 1, as required.
Q9 This question provides rather a nice twist to the standard sort of projectiles question. Firstly,
there is a non-zero horizontal component of the acceleration. However, since this doesn’t affect the
U sin 2U sin U cos
vertical motion, it is still the case that TH = and TL = . Next, the time T =
g g kg
is introduced, which is quickly identified as the time when the horizontal component of the velocity
U sin 1
is zero. Writing it in the form T = at an early stage is really helpful, partly because
g k tan
U sin
it pulls out the common factor of , but mostly as it identifies the factor of k tan which is
g
the major determining feature of the question. Having done this, you should realise that the three
cases do little more than ask you to sketch what happens depending upon when the wind (for surely
that is what is supplying this opposing force on the projectile) causes the particle to “trun round”
horizontally, relative to when the maximum height and greatest range are achieved. In the first case,
we get a “shortened” parabola compared to the usual shape of a non-wind-resisted projectile (so that
T isn’t reached before landing); in the second, the particle turns round horizontally before landing;
and in the third, the wind is so strong that the particle begins to be blown backwards before
reaching its greatest height. In the case of the “afterthought”, the particle is constrained to move up
and down in a straight line, returning to the point of projection.
Q10 In this question, we examine the dropping of two balls together, smaller on top of larger,
which leads to the surprising outcome where the smaller ball bounces so much higher than one
would expect it to. To begin with, we examine the bounce of the larger ball when it falls on its own.
Since it falls a distance H, it hits the ground with speed u given by u2 = 2gH, either by energy
considerations or by using the (“suvat”) constant acceleration formulae. It then leaves the ground
with speed v = eu according to Newton’s (Experimental) Law of Restitution, attaining a height H1
given by v2 = 2g(H1 – R). Substituting for v and then u gives the printed answer.
In the extended situation, we again use u 2 gH (for both balls) and v = eu (for the larger
ball after it bounces on the ground), though these do not need to be substituted immediately into the
“collision” statements that are gained when using the principle of Conservation of Linear
Momentum and N(E)LR for the subsequent collision between the balls. Solving simultaneously, the
Q12 As with all such questions, one can make this much easier to deal with by being systematic,
and by presenting one’s working in a clear and coherent manner. To begin with, for instance, set out
a table of the six possible outcomes, along with their associated probabilities. Of course, the reason
why you are then asked for E(X 2) rather than E(X) is clearly because squaring negates any concerns
about the sign of whatever is inside a pair of “modulus” lines, so that they can simply be replaced
by ordinary brackets. You are then told that the (given) answer is a positive integer, which restricts
(k – 1) to being a multiple of 6 … namely, 1 or 7. Checking each of these in the possible E(X)’s that
arise enables you to eliminate k = 1 and confirm that k = 7.
Now that this information is known, it is possible to draw up the probability distribution for
21 4
X (again, a table works very well), work out the probabilities P(X > 25) = 144 and P(X = 25) = 144 ,
and then evaluate the expected payout
E(W) = w P( w) = w 144
21
1 1444 0
for the gambler (where W represents “winnings”). For this to be in the casino’s favour, this
expression must be negative, giving w < 7 and so the largest integer value of w is 6.
Q13 Although you are not asked for a sketch, a quick diagram might well help prevent stupid
mistakes. Since the area under the triangle’s two sloping sides is equal to 1, it follows that the
2
height of the triangle is h , and so the gradient of the first line is this divided by (c – a) and
ba
the equation – that is, g(x) – follows immediately. Without further working, h(x) can be written
down immediately by substituting b for a appropriately (i.e. not in the bit of the expression for h;
and remembering that c – b is negative).
In (i), it is definitely not enough to think “Aha! I recognise that expression” and simply
throw the word “centroid” at the problem and hope to get all the points. You won’t. The question
makes it clear that it is intended for you to do the work to show that this expression is the mean.
(For a start, the values a, b and c lie on a line and aren’t “masses” placed at the vertices of the
triangle.) The value of E(X) must be found by integrating over the two regions, and then sorted out
algebraically.
For the final part of the question, it is important first to spend a moment making sure that
you can identify the different possible cases that arise. These depend upon whether c is < , = , or >
than 12 ( a b) . The equality case is the easiest, and the easiest to overlook, since then m = c (by
symmetry). In the first case, the median lies under the first of the sloping line segments, and will be
gained by setting the area of a triangle equal to 12 . In a similar way to earlier, the third case can be
dealt with by “working down from b” instead of “working up from a). These give, respectively,
ma 1
2 (b a )( c a ) and m b 1
2 (b a )(b c ) .
STEP II 2014
Question 1:
Drawing a diagram and considering the horizontal and vertical distances will establish the
relationships for 𝑥 cos 𝜃 and 𝑥 sin 𝜃 easily. The quadratic equation will then follow from use of the
identity cos2 𝜃 + sin2 𝜃 ≡ 1. The same reasoning applied to a diagram showing the case where P
and Q lie on AC produced and BC produced will show that the same equation is satisfied.
1
(*) will be linear if the coefficient of 𝑥 2 is 0, so therefore cos(𝛼 + 𝛽) will need to equal − 2, which
gives a relationship between 𝛼 and 𝛽. For (*) to have distinct roots the discriminant must be
positive. Using some trigonometric identities it can be shown that the discriminant is equal to
4(1 − (sin 𝛼 − sin 𝛽)2 ) and it should be easy to explain why this must be greater than 0.
The first case in part (iii) leads to 𝑥 = √2 ± 1 and so there are two diagrams to be drawn. In each
case the line joining P to Q will be horizontal.
√3
The second case in part (iii) is an example where (*) is linear. This leads to 𝑥 = 3
. Therefore Q is at
the same point as C and so the point P is the midpoint of AC.
Question 2:
𝜋 𝜋 𝜋 𝑛2 𝜋
By rewriting in terms of cos 2𝑛𝑥 it can be shown that ∫0 sin2 𝑛𝑥 𝑑𝑥 = 2 and ∫0 𝑛2 cos2 𝑛𝑥 𝑑𝑥 = 2
.
Therefore (*) must be satisfied as 𝑛 is a positive integer. The function 𝑓(𝑥) = 𝑥 does not satisfy (*)
and 𝑓(0) = 0 but 𝑓(𝜋) ≠ 0. The function 𝑔(𝑥) = 𝑓(𝜋 − 𝑥) will therefore provide a
counterexample where g(𝜋) = 0, but 𝑔(0) ≠ 0.
In part (ii), 𝑓(𝑥) = 𝑥 2 − 𝜋𝑥 will need to be selected to be able to use the assumption that (*) is
satisfied. The two sides of (*) can then be evaluated:
𝜋
𝜋5
∫ 𝑥 4 − 2𝜋𝑥 3 + 𝜋 2 𝑥 2 𝑑𝑥 =
0 30
𝜋
𝜋3
∫ 4𝑥 2 − 4𝜋𝑥 + 𝜋 2 𝑑𝑥 =
0 3
To satisfy the conditions on 𝑓(𝑥) for the second type of function, the values of 𝑝, 𝑞 and 𝑟 must
satisfy 𝑞 + 𝑟 = 0 and 𝑝 + 𝑟 = 0. Evaluating the integrals then leads to 𝜋 ≤ 22
7
.
2
Since (22
7
) < 10, 𝜋 ≤ 22
7
leads to a better estimate for 𝜋 2 .
Question 3:
By drawing a diagram and marking the shortest distance a pair of similar triangles can be used to
𝑐⁄ 𝑑
show that 𝑚
= 𝑐 , which simplifies to 𝑑 = 𝑐⁄ 2 .
𝑐√𝑚2 +1⁄ √𝑚 + 1
𝑚
For the second part, the tangent to the curve at the general point (𝑥, 𝑦) will have a gradient of 𝑦′
and so the 𝑦-intercept will be at the point (0, 𝑦 − 𝑥𝑦′). Therefore the result from part (i) can be
(𝑦 − 𝑥𝑦′)
applied using 𝑚 = 𝑦′ and 𝑐 = 𝑦 − 𝑥𝑦′ to give 𝑎 = ⁄ , which rearranges to give
√(𝑦′)2 + 1
the required result.
𝑎2 𝑦 ′ + 𝑥(𝑦 − 𝑥𝑦 ′ ) = 0.
If 𝑦 ′′ = 0 then the equation will be of a straight line and the 𝑦-intercept can be deduced in terms of
𝑚.
If 𝑎2 𝑦 ′ + 𝑥(𝑦 − 𝑥𝑦 ′ ) = 0, then the differential equation can be solved to give the equation of a
circle.
Part (iii) then requires combining the two possible cases from part (ii) to construct a curve which
satisfies the conditions given. This must be an arc of a circle with no vertical tangents, with straight
lines at either end of the arc in the direction of the tangents to the circle at that point.
Question 4:
In part (i), if the required integral is called 𝐼 then the given substitution leads to an integral which can
be shown to be equal to −𝐼. This means that 2𝐼 = 0 and so 𝐼 = 0.
1
𝑏 arctan 𝑢
In part (ii), once the substitution has been completed, the integral will simplify to ∫1/𝑏 𝑑𝑢.
𝑢
1 𝜋 1 𝑏 𝜋
Since arctan 𝑥 + arctan (𝑥) = 2 the integral can be shown to be equal to 2 ∫1/𝑏 2𝑥 𝑑𝑥, which then
simplifies to the required result.
In part (iii), making with the substitution in terms of 𝑘 and simplifying will show that the integral is
equivalent to
∞
𝑘𝑢2
∫ 2 2 2 2
𝑑𝑢
0 (𝑎 𝑢 + 𝑘 )
1 ∞ 𝑢2 1 ∞ 1 1 ∞ 𝑎2
∫ 𝑑𝑢 = ∫ 𝑑𝑢 − ∫ 𝑑𝑢
𝑎2 0 (𝑎2 + 𝑢2 )2 𝑎2 0 𝑎2 + 𝑢2 𝑎2 0 (𝑎2 + 𝑢2 )2
∞ 1
The result then follows by using the given value for ∫0 𝑎 2 +𝑥 2
𝑑𝑥.
Question 5:
𝑎 − 2𝑏 − 4 = 0
2𝑎 + 𝑏 − 3 = 0
𝑑𝑌 𝑋−2𝑌
Solving these and substituting the values into the differential equation gives 𝑑𝑋 = 2𝑋+𝑌, and so
𝑑𝑋 2𝑋 + 𝑌
=
𝑑𝑌 𝑋 − 2𝑌
This is the same differential equation as in part (i), with 𝑥 = 𝑌 and 𝑦 = 𝑋. Most of the solution in
part (i) can therefore be applied, but the point on the curve is different, so the constant in the final
solution will need to be calculated for this case.
Question 6:
In part (i), the definition can be rewritten as 𝑆2 (𝑥) = sin 𝑥 + 12 sin 2𝑥. The stationary points can then
be evaluated by differentiating the function. The sketch is then easy to complete.
For part (ii), differentiating the function gives 𝑆𝑛′ (𝑥) = cos 𝑥 + cos 2𝑥 + ⋯ + cos 𝑛𝑥. Applying the
result from the start of the question, this can be written as
sin(𝑛+12𝑥) − sin 12𝑥
𝑆𝑛′ (𝑥) =
2 sin 12𝑥
Since sin 12𝑥 ≠ 0 in the given range, the stationary points are where sin (𝑛+12)𝑥 − sin 12𝑥 = 0. This can
1
then be simplified to the required form by splitting sin (𝑛+12)𝑥 into functions of 𝑛𝑥 and 2 𝑥 and noting
that sin 12𝑥 ≠ 0 and cos 12𝑥 ≠ 0 in the given range, so both can be divided by. By noting that the
1
difference between 𝑆𝑛−1 (𝑥) and 𝑆𝑛 (𝑥) is 𝑛 sin 𝑛𝑥 the result just shown can be used to show the
final result of part (ii). Part (iii) then follows by induction.
Question 7:
𝑎 + 𝑏 − 2𝑥 𝑥≤𝑎
𝑓(𝑥) = 𝑏−𝑎 𝑎<𝑥<𝑏
2𝑥 − 𝑎 − 𝑏 𝑥≥𝑏
Therefore the graph of 𝑦 = 𝑓(𝑥) will be made up of two sloping sections (with gradients 2 and -2
and a horizontal section). The graph of 𝑦 = 𝑔(𝑥) will have the same definition in the regions 𝑥 ≤ 𝑎
and 𝑥 ≥ 𝑏, with the sloping edges extending to a point of intersection on the 𝑥-axis. The
quadrilateral with therefore have sides of equal length and right angles at each vertex, so it is a
square.
In part (ii), sketches of the cases where 𝑐 = 𝑎 and 𝑐 = 𝑏 show that these cases give just one
solution. If 𝑎 < 𝑐 < 𝑏 there will be no solutions and in the other regions there will be two solutions.
In part (iii) the graphs for the two sides of the equation can be related to graphs of the form of 𝑔(𝑥)
(apart from the section which is replaced by a horizontal line) in the first part of the question. Since
𝑑 − 𝑐 < 𝑏 − 𝑎, the horizontal sections of the two graphs must be at different heights so the number
of solutions can be seen to be the same as the number of intersections of the graphs of the form of
𝑔(𝑥).
Question 8:
The coefficients from the binomial expansion should be easily written down. It can then be shown
that
𝑐𝑟+1 𝑏(𝑛 − 𝑟)
=
𝑐𝑟 𝑎(𝑟 + 1)
This will be greater than 1 (indicating that the value of 𝑐𝑟 is increasing) while 𝑏(𝑛 − 𝑟) > 𝑎(𝑟 + 1),
𝑛𝑏−𝑎 𝑐𝑟+1 𝑛𝑏−𝑎 𝑐 𝑛𝑏−𝑎
which simplifies to 𝑟 < . Similarly, = 1 if 𝑟 = and 𝑟+1 < 1 if 𝑟 > . Therefore the
𝑎+𝑏 𝑐𝑟 𝑎+𝑏 𝑐𝑟 𝑎+𝑏
𝑛𝑏−𝑎
maximum value of 𝑐𝑟 will be the first integer after 𝑎+𝑏 (and there will be two maximum values for
𝑛𝑏−𝑎
𝑐𝑟 if 𝑎+𝑏 is an integer. The required inequality summarises this information.
In parts (i) and (ii) the values need to be substituted into the inequality. Where there are two
possible values, it needs to be checked that they are equal before taking the higher if this has not
been justified in the first case.
In part (iii) the greatest value will be achieved when the denominator takes the smallest possible
value, therefore 𝑎 = 1, and then in part (iv) the greatest value will be achieved by maximising the
numerator. Since the maximum possible value of 𝐺(𝑛, 𝑎, 𝑏) is 𝑛, 𝑏 ≥ 𝑛 will achieve this maximum.
Question 9:
Once a diagram has been drawn the usual steps will lead to the required result:
Resolving vertically:
𝐹 + 𝑇 cos 𝜃 = 𝑚𝑔
Resolving horizontally:
𝑇 sin 𝜃 = 𝑅
Taking moments about A:
𝑚𝑔(𝑎 cos 𝜑 + 𝑏 sin 𝜑) = 𝑇𝑑 sin(𝜃 + 𝜑)
Limiting equilibrium, so 𝐹 = 𝜇𝑅:
𝜇𝑇 sin 𝜃 + 𝑇 cos 𝜃 = 𝑚𝑔
Therefore:
𝑇𝑑 sin(𝜃 + 𝜑) = 𝑇(𝜇 sin 𝜃 + cos 𝜃)(𝑎 cos 𝜑 + 𝑏 sin 𝜑)
And so:
𝑑 sin(𝜃 + 𝜑) = (𝜇 sin 𝜃 + cos 𝜃)(𝑎 cos 𝜑 + 𝑏 sin 𝜑)
If the frictional force were acting in the opposite direction, then the only change to the original
equations would be the sign of 𝐹 in the first equation. Therefore the final relationship will change to
𝑑 sin(𝜃 + 𝜑) = (−𝜇 sin 𝜃 + cos 𝜃)(𝑎 cos 𝜑 + 𝑏 sin 𝜑)
For the final part, the first and third of the equations above can be used to show that
𝑇𝑑 sin(𝜃 + 𝜑)
𝐹= − 𝑇 cos 𝜃
𝑎 cos 𝜑 + 𝑏 sin 𝜑
𝑎+𝑏 tan 𝜑
Since 𝐹 > 0 if the frictional force is upwards, this then leads to the condition 𝑑 > tan 𝜃+tan 𝜑. Since
the string must be attached to the side 𝐴𝐵, 𝑑 cannot be bigger than 2𝑏, which leads to the final
result of the question.
Question 10:
Consideration of the motion horizontally and vertically and eliminating the time variable leads to a
Cartesian equation for the trajectory:
𝑔𝑥 2
𝑦 = 𝜆𝑥 − (1 + 𝜆2 )
2𝑢2
The maximum value can be found either by differentiation or by completing the square. Completing
the square gives:
𝑔𝑥 2 𝑢2 𝑢2 𝑔𝑥 2
𝑦=− (𝜆 − ) + −
2𝑢2 𝑔𝑥 2𝑔 2𝑢2
𝑢2 𝑔𝑥 2
Which shows that 𝑌 = 2𝑔 − 2𝑢2 . If this graph is sketched then the region bounded by the graph and
the axes will represent all the points that can be reached.
The maximum achievable distance must lie on the curve and the distance, 𝑑, of a point on the curve
2
𝑢2 𝑔𝑥 2
can be shown to satisty 𝑑2 = (2𝑔 + 2𝑢2 ) , which must be maximised when 𝑥 takes the maximum
value possible.
Question 11:
A similar method for the horizontal motion of 𝑃 and 𝑅 gives the two equations
𝑇 − 𝑇 sin 𝛼 = −𝑚𝑥̈
For part (ii), eliminating 𝑇 from the last two equations gives the required relationship. A sketch of
𝑥
the graph of 𝑦 = (1−𝑥)2 will then show that for any value of 𝑘 there is a possible value between 0
and 1 for sin 𝛼.
In part (iii), elimination of T from the two equations formed by considering the motion of 𝑃 gives the
required result.
Question 12:
In the case of small values of 𝛿𝑡, 𝐹(𝑡 + 𝛿𝑡) − 𝐹(𝑡) ≈ 𝑓(𝑡)𝛿𝑡, which leads to the correct probability.
1
In part (ii), differentiation gives 𝑓(𝑡) = 𝑎, and substituting into the definition of the hazard function
1
gives ℎ(𝑡) = 𝑎−𝑡. Both graphs are simple to sketch.
𝐹′(𝑡) 1
In part (iii), using the definition of the hazard function gives 1−𝐹(𝑡) = 𝑡 . Integrating gives
− ln|1 − 𝐹(𝑡)| = ln|𝑘𝑡|, and so the probability density function can be found by rearranging to find
𝐹(𝑡) and then differentiating.
A similar method in part (iv) shows that if ℎ(𝑡) is of the form stated then 𝑓(𝑡) will be of the given
form. Similarly, if 𝑓(𝑡) has the given form then ℎ(𝑡) can be shown to have the form stated.
In part (v), a differential equation can again be written using the definition of the hazard function
and this can again be solved by integrating both sides with respect to 𝑡.
Question 13:
Considering the sequence of events for 𝑋 = 4, the 1st, 2nd and 3rd numbers must all be different and
then the 4th must be the same as one of the first three. The probability is therefore
1 2 3
𝑃(𝑋 = 4) = (1 − ) (1 − )
𝑛 𝑛 𝑛
The result of part (i) is then found by observing that the probabilities of all possible outcomes add up
to 1.
For part (iii) observe that any case where 𝑋 ≥ 𝑘 will have the first 𝑘 − 1 numbers all different from
each other. Therefore
1 2 𝑘−2
𝑃(𝑋 ≥ 𝑘) = (1 − ) (1 − ) ⋯ (1 − )
𝑛 𝑛 𝑛
The first formula in part (iv) can be shown by considering 𝑘𝑃(𝑌 = 𝑘) to be equal to the sum of 𝑘
copies of 𝑃(𝑌 = 𝑘) and then regrouping the sum for 𝐸(𝑌). Finally this gives two different
expressions for 𝐸(𝑌), which must be equal to each other:
2 1 2 1 2 3 1 2 𝑛−1
+ 3 (1 − ) + 4 (1 − ) (1 − ) + ⋯ + (𝑛 + 1) (1 − ) (1 − ) ⋯ (1 − )
𝑛 𝑛 𝑛 𝑛 𝑛 𝑛 𝑛 𝑛 𝑛
1 1 2 1 2 𝑛−1
= 1 + 1 + (1 − ) + (1 − ) (1 − ) + (1 − ) (1 − ) ⋯ (1 − )
𝑛 𝑛 𝑛 𝑛 𝑛 𝑛
Rearranging and using the result from part (i) then gives the required result.
STEP 3 2014 Hints and Solutions
1. The stem results are obtained through algebraic expansion and equating coefficients. Using
the expression 1 1 1 for 1 , manipulating the logarithm
of the product, and the series expansions for expressions like ln 1 yields the
displayed result. In parts (ii), (iii), and (iv), it is simplest to find , , ,
, and by expanding the series for ln 1 , and
choosing a counter‐example, selecting a, b and c so that 0.
2. The first part is solved using the given method, the formula cosh 2 2 cosh 1 , and
then employing partial fractions or the standard form quoted in the formula book. The
second part requires the substitution, sinh , the formula cosh 2 1 2 sinh ,
√
and a standard form to give tan √2 . The third part can be approached by
making the substitution and division of the resulting fraction in the numerator and
denominator by to give half the difference of the integrals in the first two parts.
Alternatively, a similar style of working with the substitution results in a sum
instead of a difference.
3. (i) Given that the shortest distance between the line and the parabola will be zero if they
meet, investigating the solution of the equations simultaneously , and the discriminant of
the resulting quadratic equation, the first result of the question is the case that they do not
meet. The closest approach is the perpendicular distance of the point on the parabola
where the tangent is parallel to the line, so using the standard parametric form, it is the
perpendicular distance of , from , giving the required result with care
being taken over the sign of the numerator bearing in mind the inequalities.
(ii) The shortest distance of a point on the axis from the parabola, is either the distance from
the vertex to the point, or the distance along one of the normals (which are symmetrically
situated) which is not the axis. If the normal at ,2 passes through , 0 , then
2 . From this it can be simply shown that shortest distance is if 2 , and is
2 if 2.
Then for the circle, the results follow simply, that the shortest distance will be if
, and 0 otherwise if 2 , and 2 if 4 or 0 otherwise if
2.
4. Expanding the bracket in the integral , and employing sec 1 tan yields plus
the integral of a perfect differential which can be evaluated simply. For 0,
tan 0 over the interval which can be solved using an integrating factor and then
the condition 0, 1 enables the arbitrary constant to be evaluated giving the
required result. In part (ii), similar working can be undertaken with the integral which is to
be considered, given . The argument requires no discontinuity in the interval so
. The function cos can be shown to meet the requirement.
5. ABCD is a parallelogram if and only if which yields the required result. To be a
square as well, angle 90 , and | | | | , so . Treating the
two results as simultaneous equations to be solved for and in terms of and , the
second result of the stem can be shown with reversible logic. For part (i) the same logic can
be used for as just used for . From the stem, XYZT is a square if and only if
, and
and given the working for X in part (i), these can be shown to be true
treating Y, Z, and T similarly
7. Part (i), the intersecting chords theorem, is basic bookwork relying on angle properties in
circles to establish similar triangles and hence the result. Part (ii) can be obtained by
considering that lies on and so , that also lies on
producing a similar result and then equating these two expressions, finally rearranging to
give (*). Assuming that 0 and using (*) leads to
which, in view of the distinctness of the four points and the intersection of and
at , leads to the contradiction 0 . Re‐writing as
8. The initial result is obtained by extending the given inequality so that each term of the sum
is compared with and . Part (i) is obtained using the stem, the given
function, 2 , and summing the sums. The deduction relies on considering the lower limit
of the sum. The same approach applies to part (ii), with the new function given and
considering the upper limit which is obtained as a geometric progression. Counting the
number of elements of 1000 gives the method for obtaining using the same
function as part (i) except 0 if has one or more 2 s in its decimal representation
and with 10 , again with the sum of a geometric progression. The final result is
particularly attractive, demonstrating how few terms need to be removed from the non‐
convergent harmonic progression (of part (i)) in order to produce a convergent sequence.
.
obtains the first displayed result after re‐arrangement, as does tan the second.
.
tan tan can be shown to be sinh kT kT which leads to the two
final inequalities.
10. The first result is obtained by considering Newton’s second law applied to the mass X under
the tension in PX and the thrust of XY. is similarly obtained considering
Y. Subtracting the two equations gives a SHM second order differential equation for ,
and adding them gives similar for . Solving these using initial conditions give
cos and cos √3 . The final result is particularly elegant,
and possibly a little surprising that a conservative oscillating system does not return to its
starting position. Treating the previous two results as simultaneous equations for and ,
and solving , yields 1 cos √3 and 1 cos , so that √3 2 and
2 for non‐zero integers and , yielding the contradiction √3 .
11. Resolving vertically and horizontally, and solving the resulting simultaneous equations and
then tidying up the trigonometric expressions yields and
12. The first result, , is obtained merely by considering probabilities, and the given
pdf of Y can be obtained by standard techniques or by consideration of changing the variable
in the integral of the pdf of X. The mode result relies on differentiation of the pdf of Y
equated to zero to give a stationary value. The explanation in part (iii) is simply that the
required integral is merely that of the pdf of a Normal variable with mean . The
expectation of Y is obtained in the standard manner, using an integral and the pdf of Y, and
then a change of variable, in which exponential terms can be combined so as to use the
explained result having completed the square in the exponent. Using the three previous
parts gives , because X is symmetric, and, as stated,
13. The first result is a trivial application of the definition of a probability generating function,
and the second similarly. In order to obtain the first printed result in part (iii), it is necessary
to obtain a similar result to those in parts (i) and (ii) giving as the score is one higher
and then applying the conditionality of the probabilities of these three results which is done
by considering the probability of a score in the three cases to give the coefficient of .
Re‐arranging the formula for , either differentiation or the binomial theorem can be
used to find the required probability formula. Finding 1 ⁄ and the
knowledge that 1 enables the result of part (iii) to be rearranged to that of
part (iv).
STEP Solutions 2015
Mathematics
STEP 9465/9470/9475
October 2015
The Admissions Testing Service is a department of Cambridge English
Language Assessment, which is part of Cambridge Assessment, a not-for-
profit department of the University of Cambridge.
© UCLES 2015
Q1 This question is intended to be a relatively straightforward entrée into the paper, and thus its
demands are fairly routine in nature. That does not mean that it is easy, merely that the appropriate courses
of action should be readily accessible to all candidates of a suitable standard. To begin with, the demand for
a sketch (of any function) should lead you to consider things such as
key points (such as where the curve meets either of the coordinate axes);
asymptotes (note the information given in italics at the end of part (i) regarding what happens as
x –, which indicates that the negative x-axis is an asymptote in this case);
turning points of the curve, which are clearly flagged as being of significance when considering what
happens when y = k; i.e. when the curve meets a horizontal line;
long-term behaviour (you already have sorted for you the “x –” side of things, so there is only a
quick decision to be made about what happens as x +).
For the key points, first set x = 0 and then y = 0; the asymptote is effectively given; the TPs come from
setting the first derivative to zero and solving for x again (noting, of course, that ex is always positive); and
the curve clearly grows exponentially as x increases positively. The rest of (i) then simply requires a bit of
thought as to how many times a horizontal line will cut, or touch, the curve depending upon the value of k.
In part (ii), it is clear that the x in part (i) has now been replaced with an x2, and this second curve
must therefore have reflection symmetry in the y-axis, as all negative values of x are being squared to give
the positive counterpart. Previously, when x was equal to zero, we now havex2 = 0, and so each previous
crossing-point on the positive x–axis leads to two, one on each side of the y-axis (and at the square-root of
its former value). However, the previous y-intercept is unchanged, but must now appear at a TP of the curve
(otherwise the symmetry of the gradient would be compromised). Also, the previous TP with positive x-
value (the negative one has gone) occurs at the square-roots of the previous value, but again with unchanged
y-coordinate.
Q2 It is clear that (i) is an introductory part that requires the use of the cos(A – B) formula with suitably-
chosen values of A andB. Using the sin(A – B) formula then leads to the second result, although there are
alternative trig. identities that could be used in both cases, such as a double-angle formula. Repeated use of
these, or the double-angle formulae (or de Moivre’s Theorem for those from a further maths background)
lead to the (relatively) well-known ‘triple-angle formula’ cos3α 4cos3α – 3cosα, which gives x = cosα as a
root of the equation 4 x 3 3 x cos 3 0 . Although there are several possible methods here, a simple
division/factorisation leads to ( x c) 4 x 2 cx c 2 3 = 0, and the quadratic formula leads to the
remaining two roots which, for their simplest form, requires the use of the most elementary of trig.
identities, s 2 1 c 2 .
A bit of insight is needed in part (iii), where one should first realise that the constant term is intended
be a cosine value (of 3 some angle), and the most obvious candidate is 1
2 2 = cos 45o, so that is 15o.
1
From here, it is now clear that “x” is 2 y , and that the three roots are those from part (ii) with the exact
numerical forms of the sine and cosine of 15o from part (i) waiting to be deployed in order to find the surd
forms requested.
Q3 In this question, it is important to draw suitable diagrams in order to visualise what is going on, and
these are not difficult to manage, with the guard either at a corner of the yard (C) or at its middle (M).
However, this second case has two possible sub-cases to consider, depending upon whether the ‘far’ corners
of the yard are visible to him/her (which, in fact, turns out to be the b = 3a case which separates the two
cases that the question invites you to consider), and it is the extra length of the opposite wall that is visible
4b 2 b2
that makes for different working. These lengths are (from C), (from M, with the ‘far’ corners not
ba a
2b( 2b 3a )
visible) and (from M, with the ‘far’ corners visible). Once obtained, these should be compared
ba
in order to find that the guard should stand at C for b < 3a and at M for b > 3a (and at either when b = 3a).
Q4 A quick read-through of the question should make it clear that it is the lower end of the rod that is
being referred to (as do the subtractions within the given coordinates x and y). The fact that the rod is
tangent to the given parabola means that its direction is tan 12 x , which gives the coordinates of the rod’s
midpoint as (tan, tan2); a simple right-angled triangle and some accompanying basic trig. then leads to the
given answer. The second part of the question is equally straightforward once one realises that when xA = 0,
2 tan
2 tan b cos b yA= tan 2 . There are several ways to attack the area between two
cos
curves – e.g. as y
1 y 2 dx, or by translating the bit below the x-axis up by tan2 and calculating the
difference between area under the “new” curve and a triangle; the key is to eliminate the “b” and then the
given answer follows.
Q5 Once one realises that the x within the integral is not the variable, then both integrations are
2x
, while in (ii) g(x) = 1 x 1 x … remember to use
1
relatively straightforward. In (i), we get f(x) =
x x
the modulus function when taking square-roots (although one could, alternatively, work out a piece-wise
definition for g; that is, in bits). The sketch of f should prompt the solver to differentiate in order to identify
1
the turning point at , e ln 2 . Noting that y + as x 0 and that y + as x + gives all else that
ln 2
is needful to draw the graph in (i). In (ii), the piece-wise definition of g is certainly more useful now since its
graph is made up of two ‘reciprocal’ curve bits joined by a horizontal straight-line in the middle.
Q7 The crucial observation here is that a (continuous) function takes its maximum value on a finite
interval either at a maximum turning-point or at an endpoint. Differentiating (a ‘negative’ cubic – so we
know what its shape is) gives a MIN. TP at (0, 0) and a MAX. TP at ( 13 a, 19 a3), and evaluating at the
endpoints gives f(– 13 ) = 19 (3a + 2) and f(1) = 3a – 6.
Now, a comparison of these possible values for f then yields that 19 (3a + 2) 19 a3 a 0, a 2; and
that 19 a3 3a – 6 holds for all a 0; and also that 19 (3a + 2) 3a – 6 a 7
3 (which, actually, affects
19 (3a 2) 0 a 2
nothing, but the working should be done anyhow). Thus M(a) = 19 a 3 2a3 .
3a 6 a3
Q8 The standard “bookwork” approach to this opening part is to write the sum (S) both forwards and
backwards, add the terms in pairs (n pairs, each of value n + 1) and then to half this to get S = 12 n(n + 1). As
with any such invitation to establish a result, one should not simply seek to quote a result and thus merely
“write down” the given answer. When looking at part (ii)’s question, the binomial theorem should really be
screaming at you from the page, and all that is needed is to observe that the binomial expansion of (N – m)k
consists of k + 1 terms, the first k of which contain a factor of (at least one) N. The final term, since k is odd
must be –mk which then conveniently cancels with the + mk term to leave something that is clearly divisible
by N.
In the next part of the question, you are invited to explore the cases n odd and n even separately
(indeed the results that follow are slightly different). To begin with,
S = 1k + 2k + … + nk (an odd no. of terms) = 0k + 1k + 2k + … + nk (an even no. of terms)
So these terms can now be paired up:
n with 0, n – 1 with 1, …, ( 12 n 12 ) with ( 12 n 12 ) ,
so that all pairs are of the form (n – m)k+ mk , which was just established as being divisible by n. Next, in the
case when
S = 1k + 2k + … + nk (an even no. of terms) = 0k + 1k + 2k + … + nk (an odd no. of terms),
the pairs are now
n with 0, n – 1 with 1, …, ( 12 n 1) with ( 12 n 1) ,
but with an odd term, ( 12 n) k , left over. This gives us (from the same previous result as before) a sum
1
consisting of terms divisible by n and one that is divisible by 2 n, giving the second result.
Then, for n even, so that (n + 1) is odd, S + (n + 1)k is divisible by n + 1 (by the previous result) S
is divisible by n + 1; and for n odd, so that (n + 1) is even, S + (n + 1)k is divisible by 12 (n + 1). Thus, since
hcf(n, n + 1) = 1 hcf( 12 n, n + 1) = 1 for n even, and hcf(n, 1
2 (n + 1)) = 1 for n odd, it follows that S is
1
divisible by 2 n(n + 1) for all positive integers n.
2u sin
Q9 The standard time taken to land (at the level of the projection) of a projectile is t . Thus, a
g
2u
bullet fired at time t, 0 t , lands at time TL t sin t . Differentiating this w.r.t. t and
6 g 3
2u 2 sin cos
setting it equal to zero, gives k cos t . The horizontal range is then given by R and
3 g
this gives the required answer. Moreover, substituting the endpoints of the given time interval 0 t
6
1 3 dTL
into k cos t gives k . However, if k < 12 , then one sees that 0 throughout the
3 2 2 dt
gun’s firing, so that TL is a (strictly) decreasing function. Hence its maximum value occurs at t = 0, i.e.
2u 2 1 3 u2 3
, whence R .
3 g 2 2 2g
Q10 The difficulty in this question lies in ignoring unnecessary information (not given). Firstly, then, note
that the speed of the rain relative to the bus is vcos – u (or u – vcos if negative), and when u = 0, the area
of the bus getting wet, A, is such thatAhvcos + avsin . Now the given result follows from observing that
when vcos – u> 0, the rain hitting the top of the bus is the same, whilethe rain hits the back of the bus as
before, but with speed vcos – u instead of vcos ; and whenvcos – u< 0, the rain hitting the top of the bus
is the same, while the rain hits the front of the bus as before, but with u – vcos instead of vcos.
1 av sin h | v cos u |
Next, as the journey time , we need to minimise J . For vcos – u > 0
u u u
av sin hv cos
and w vcos , we minimise J h , and this decreases as u increases, and this is done
u u
av sin hv cos
by choosing u as large as possible; i.e. u = w. For u – vcos > 0, we minimise J h,
u u
and this decreases as u increases if a sin > h cos , so we again choose u as large as possible; i.e. u = w.
Next, if a sin < h cos , then J increases with u when u exceeds vcos , so we choose u = vcos in this
case. Finally, if a sin = hcos then J is independent of u, so we may as well take u = w.
av sin hv cos
For the return journey, simply replace by 180o – to give J h , which
u u
always decreases as u increases, so take u = w again.
Q12 Here, you are given the relevant Poisson result at the outset, and this is intended to guide your
thinking later on in the question. To begin with, though, part (i) is actually a Binomial situation … requiring
just a single general term. In part (ii), you were asked to prove algebraically a result that you might usually
be required to quote and use. This requires a good understanding of the use of the sigma-notation and a clear
grasp as to which of the various terms are constant relative to the summation, and then combining the
remaining terms together appropriately to give the requested Poisson answer. Most important of all, of
course, it is essential to have the first line of working correct; this is
nr
e 8 8 n
r
n! 1 3
P(S = r) =
nr n! r ! (n r ) ! 4 4
e 8 2 r 6 n r
and one follows this through to the point where the result
r ! n r (n r ) !
is obtained. At this stage
another simple trick is required – effectively a re-labelling of the starting-point, using m = n – r to re-write
e 8 2 r 6 m
this as
r ! m0 m!
. The required result follows immediately since the infinite sum is just e6.
Having established this, the final part of the question is relatively straightforward, requiring only the
use of the conditional probability formula applied to P(M = 8 |M + T = 12).
Q13 The first three parts of this question are very easy indeed, if looked at in the right way. In part (i) it is
not necessary at all that you recognise the Geometric Distribution (indeed, some of you may not have
encountered it at all), but the result asked for is simply “(n – 1) failures followed by 1 success”, and one can
write down immediately, and without explanation, the answer P(A) = 56 n 1 16 . In (ii), you have a situation
in which one can apply the principle of symmetry: either a 5 arises before a 6, or vice versa, so the required
probability is just P(B) = 12 . Part (iii) can be approached similarly, in that the first 4s, 5s, 6s can arise in the
orders 456, 465, 546, 564, 645, 654 P(BC) = 1
3 (i.e. also “by symmetry”, but with three pairings to
consider).
Parts (iv) and (v), however, each turn out to require the use of the result given at the end of the
question, as the outcomes (theoretically) stretch off to infinity. For (iv), it is best to consider only on which
throw the first 6 occurs (since we stop at that point). It cannot occur on the first throw, so we have the sum
of the situations:
a 5 occurs on the first throw, followed by a 6 on the second;
one 5 and a 1-4 occur, in either order, followed by the 6 on the third;
one 5 and two 1-4s occur, in any of three possible orders, followed by a 6 on the fourth;
etc.
16 16 16 64 16 16 64 2 16 ... , and this factorises as 361 1 2 23 3 23 2 ..., and the
2 3
Thus P(D) =
1 1
2
big bracket is just the given result with x = 3 and n = 2.
Before getting too deeply into part (v), a couple of simple results should be noted. Firstly, we use the
fact that P(E) = P(D) = 14 , the answer to (iv); and then that we will need to use the basic probability result
1
P(DE) = P(D) + P(E) – P(DE) = 2 – P(DE). Turning this around, since it is far easier to calculate the
probability, P(DE), that both one 4 and one 5 occur before the first 6. Again, looking at this from the
viewpoint of finishing after the first 6 is thrown, we see that
3 4 2
P(DE) = 62 16 16 63 62 16 16 63 62 16 16 ... = 108
1
1 3 12 6 12 2 ...
1 2
1 23
and the big bracket is the given result with x = 2 and n = 3 , leading to the answer P(DE) = 54 .
STEP 2 2015 Hints and Solutions
Question 1
For the first result, show that the gradient of the function is positive for all positive values of (by
differentiating) and also that 0 0. Once this result has been established sum a set of the terms,
using , note that ln 1 can be written as ln 1 ln and then the required result
follows.
For the second part, first show that ln 1 is negative for 0 1 and then use the
substitution , noting that ln 1 can be written as ln 1 2 ln ln 1 . Deal
with the sum starting with 2 and then add the initial 1 afterwards.
Question 2
As with all geometric questions a good diagram of the information given makes the solution to this
question much easier to reach. The first result in this question follows from an application of the sine
rule with applications of the relevant formulae for sin and the double angle formulae. From
a diagram of the triangle it should then be an easy application of trigonometry to show that .
There are a number of different methods for establishing that trisects the angle – one
method is to show that sin ∠ , following which it is relatively straightforward to work out
the sizes of angles and in terms of and show that they must satisfy the correct
relationship.
Question 3
For the first part note that can be interpreted as the triangles that can be made using the
rod of length 8 and two other, shorter rods. These can then be counted by noting that there are 6
possibilities if the length 7 rod is used, 4 possibilities if the length 6 (but not the length 7) rod is used
and 2 possibilities if the length 5 (but not 6 or 7) rod is used. It is clear that at least one rod longer
than length 4 must be used. To evaluate note that it is equal to and
then evaluate in a similar manner to . Similar reasoning easily gives formulae for
and .
For the induction, the rule for deduced in the previous part can be used to show the
inductive step, while the easiest way to show the base case is to list the possibilities. The easiest way
to establish the result for an odd number of rods is to use the formula for and the
formula for that was just proven.
Question 4
For the first part, note that the graph of arctan satisfies the requirement of being continuous, but
does not satisfy 0 . Since tan tan , a translation of the graph of arctan
vertically by a distance of gives the required graph.
It should be clear that the graph of has no vertical asymptotes, approaches the ‐axis as
→ ∞ and passes through the origin. Identifying the stationary points should be the next task
after which a graph should be easy to sketch. The graph of should then be easy to sketch
by considering the fact that is an increasing function and is obtained by composing the
two functions already sketched.
To sketch the graph of first note that there must be two vertical asymptotes. Once
stationary points have been checked for it should be straightforward to complete the sketch. In this
case, the asymptotes need to be considered to deduce the shape of the graph for as the
composition with will lead to discontinuities. Noting again that tan tan the
discontinuities can be resolved by translating sections of that graph vertically by a distance of .
Question 5
The initial proof by induction is a straightforward application of the tan formula. The final
part of section (i) requires recognition that there are many possible values of to give a particular
value of tan , but only one of them is the value that would be obtained by applying the arctan
function. The result can therefore be shown by establishing that the difference between consecutive
terms of the sequence is never more than .
For the second part of the question a diagram of the triangle and application of the tan 2 formula
shows that the value of must be of the form used in the first part of the question. All that
remains is then to show that the limit of the sum must give the required value.
Question 6
The first part of the question requires use of the cos formula. Following this the integral
should be easy to evaluate given that sec tan . In the second part, apply the
substitution and note that the limits of the integral are reversed, which is equivalent to multiplying
by ‐1. Following this a simple rearrangement (noting that the variable that the integration is taken
over can be changed from to ) should establish the required result. The integral at the end of this
part can then be evaluated simply by applying this result along with the integral evaluated in part (i).
In the final part of the question it is tempting to make repeated applications of the result proven in
part (ii). However, this is not valid as it would require the use of a function satisfying sin ,
which is not possible on the interval over which the integral is defined. Instead, application of a
similar substitution to part (ii) to sin will simplify to allow this integral to be evaluated
based on the integration of . An application of the result from part (ii) will also be required.
Question 7
For part (i) note that the lines joining the centres of the two circles and one of the points where the
bisection occurs form a right‐angled triangle, so the radius of the new circle can be calculated. To
show that no such circle can exist when note that the diametrically opposite points on must
be a distance of 2 apart, and no two points on a circle of radius can be that far apart. For the case
note that the new circle would be the same as (and so would have more than two
intersection points).
For part (ii) a similar method can be used to deduce the distances between the centre of the new
circle and each of and . From these distances equations can be formed relating the and
coordinates of the centre of the new circle. It is then an easy task to eliminate the ‐coordinate of
the centre of the circle from the equations to get the given value of the ‐coordinate.
The expression for can easily be found by substituting back into the equations obtained from the
distance between the centres of two of the circles. Once this is done, note that 0 to obtain the
final inequality.
Question 8
The first part of the question follows from consideration of similar triangles in the diagram if the line
through and the centres of the circles is added. For the second part, expressions can be written
down for the position vectors of and by noting that the same method as in part (i) will still apply.
The vectors and can then be compared to show that one is a multiple of the other.
For the final part of the question, note that will lie halfway between and if .
Question 9
A diagram to represent this situation will show the angles that will be required to calculate the
moments of each of the particles about in terms of . Following this, simple trigonometric
manipulation should lead to a relationship between sin and cos . From this, either a right‐angled
triangle or one of the basic trigonometric identities can be used to reach the required result.
For the second part of the question the amount of potential energy that needs to be gained by the
system should be easy to calculate and this must be equal to the initial kinetic energy of the system.
Question 10
The component of the velocity of the particle in the direction of the string at any moment must be
equal to , which leads to cosec as the speed of the particle along the floor. Alternatively,
introduce a variable to represent the length of string still in the room or the height of the room and
then differentiate , the distance of the particle from the point directly beneath the hole, with
respect to time. The length of the string (to the hole in the ceiling) is decreasing at a rate of ,
which then allows the introduced variable to be eliminated to reach an expression for the speed of
the particle.
Differentiation of the speed of the particle allows the acceleration to be calculated. Finally, note that
the particle will remain on the floor as long as the vertical component of the tension is less than the
weight of the particle and then the point at which the particle leaves the floor can be identified.
Question 11
For the first part, the coordinates of are found by applying simple trigonometric ratios and
differentiation with respect to time gives the velocity of . In the second part, the first equation
results from consideration of conservation of momentum and the second results from conservation
of energy (with a substitution based on the first equation made to eliminate one variable).
Since no energy is lost in any collisions the relationships from part (ii) must continue to hold and this
shows that cannot be 0 which means that the direction in which changes remains the same
unless there is a collision. Since the first collision occurs when 0, the second one must be when
.
For the final part, note that the equations in part (ii) must still hold, and if 0, the kinetic energy
of must be 0. Since the kinetic energies of and must be equal (by symmetry) it must be the
case that the kinetic energy of is and can also be calculated from the expression for the
velocity of shown in part (i). Since 0, this can then be used to find the values of . Finally,
note that given these values of , will only be 0 on the occasions when is positive.
Question 12
For the first part, note that can only win the game if the first two tosses result in heads, since once
there has been a tail, will win as soon as two consecutive heads have been tossed and cannot
win until there have been two consecutive heads and one further toss. In the second part, note that
this logic still applies to the game for and similar reasoning can be applied to the game for . For
the other two players switching heads and tails in any sequence that results in a win for will give a
sequence that results in a win for , and vice versa, so the probabilities must be equal. Since only
sequences which alternate between heads and tails forever (and the probabilities of such sequences
tend to zero as the lengths of the sequences increase) the probabilities must both also be ¼ .
For the final part, note that must win if the first two tosses are TT. Since only the previous two
tosses are important in determining what could happen on the next toss, each case can be analysed
by a tree diagram which shows the outcomes after one further toss.
This yields three equations in the three unknowns which allows all of the individual probabilities to
be calculated. Once this is done the overall probability can be calculated.
Question 13
To calculate the expected value of the total cost, note that there is a constant component of and
then the expected value of the given that must be added, which can be calculated
by integration of with respect to , between and ∞. Differentiating the expression
for with respect to allows the position of the stationary point to be found. If this is at a
negative value then should be chosen to be 0 and otherwise the value of for the stationary point
should be used.
A slightly more complicated integration is needed to establish the formula for and then
differentiation of this gives a value that is clearly negative for positive values of , which shows that
the variance is decreasing as increases.
STEP 3 2015 Hints and solutions
1. The first result can be obtained by simplifying the LHS and then writing it as
and integrating this by parts. To obtain the evaluation of , the first result
can be re‐arranged to make the subject, and then iterating the result to express it in terms of
which is a standard integral. The expression can be tidied by multiplying numerator and
denominator by 2 2 2 … 2 . The first result for (ii) is obtained by means of the
substitution , the second by adding the two versions of , and the third by the substitution
, being careful with limits of integration and employing symmetry. Part (iii) is solved by
expressing the integrand as and then employing first part (ii) then part (i) to obtain
!
, which is .
!
2. Part (ii) is the only false statement, and a simple counter‐example is 1 and 2 for
odd, and 2 and 1 for even. Part (i) 1000 is a suitable value, then 1000
and as is positive, the inequality can be multiplied by it giving the required result. Part (iii)
requires the use of the definition twice with values and say, and then using
max , . For part (iv), we can choose 4 , and an inductive argument such as
1 1 1 2 2 2 2 works.
3. The part (i) inequality for sec can be obtained by making the subject of the formula as
sec and invoking remembering that 0 is not permitted.
Then the points lie on a conchoid of Nicomedes with A being the pole (origin), d being b, and L being
the line sec (" " ). A sketch is
In part (ii), the extra feature is the loop as specified with end‐points at the pole corresponding to
sec . A sketch is
So in the given case, the area is given by 2 sec 2 which is √3 4 ln 2 √3 .
5. (i) Step 3 is straightforward on the basis of steps 1 and 2, noting that no lowest terms
restriction need be made in part 1. Step 5 requires that the given expression is a positive integer as
well as well as being integer when multiplied by root two. Step 6 requires justification that
√2 1 1 .
(ii) The rationality of 2 on the basis of 2 being rational is simply obtained by squaring
the latter, and the opposite implication can be made by squaring the former or dividing 2 by the
former. To construct the similar argument, let the set be the set of positive integers with the
following property: is in if and only if 2 and 2 are integers, and taking to be the
smallest positive integer in that set, consider 2 1 to produce the argument.
6. Treating the equations for and as simultaneous equations for and , one finds that
√ ∓√
and which demonstrates that if ∈ and 2 , i.e. ∈ ,
and are real. If and are real, then and are (trivially) and 2 0.
In (ii), the first two equations yield 3 1 , making it possible to write the third equation as
√
1 1 which has an obvious factor of 1 leading to 1 or
from the quadratic equation. If one of the solutions of the quadratic equation gives the same root
1 , then there are not three possible values, i.e. if 2 . From the first part of the question,
for and to be real, we would want to be real, to be real, and 2 , in
other words . So a counter‐example could be 1 giving 2 2 0 which has a
negative discriminant.
7. The opening result is simply achieved by following the given explanation for with
. Parts (i) and (ii) can both be shown usual the principle of mathematical induction with
initial statements
8. Transforming the differential equation in part (i) is made by substituting for and as
given, for using cos sin and a similar result for , and then simplifying the
algebra by multiplying out and collecting like terms bearing in mind that a factor can be cancelled
as 0 . The transformed equation can be solved by separating variables or using an integrating
factor, to give , the sketch of which is
The same techniques for part (ii) yields a differential equation 0 which is solved by
separating the variables and then employing partial fractions giving a variety of possible solution
sketches
9. Whilst the first part can be obtained otherwise, the simplest approach is by conserving
energy, when √ leads to the required answer simply. is
sin
10. The position vector of the upper particle is so differentiating with respect to
cos
time yields the correct velocity and acceleration which gives the second result when used in
Newton’s second law resolving horizontally and vertically. The corresponding equations are
θcos sin sin 0
for the other particle by merely swapping the
θsin cos cos 1
direction of the tension and the displacement from the midpoint. The deductions are obtained by
adding the two equations of motion, and in the case of θ , subtracting the two equations and then
θcos
eliminating between the equations for each component. Using and a
θsin
similar equation for the lower particle, initial values of and can be found and then the time for
the rod to rotate by can be obtained and substituted in the displacement equation under
uniform acceleration to obtain the final result.
11. (i) The first result is obtained, as the question prompts, by considering a component of force
on the rod due to P, and taking moments about the hinge to find that that component is zero with
the consequence that any force exerted on the rod by P must be parallel to the rod. Bearing in mind
the horizontal acceleration of P towards the centre of the circle it describes, resolving perpendicular
to the rod and writing the equation of motion for P leads directly to the given equation with the
stated substitution being made. The force exerted by the hinge on the rod is along the rod towards
P and resolving vertically for forces on P and rearranging gives sec .
(ii) Taking moments for the whole system about the hinge gives
13. (i)
lead to if 0 1, 1 2 if 1 2,
0 if 0 and 1 if 2.
(ii)
lead to
1
0 1
2
1
1 1
2
1
the probability density function .
1 0
went on to find the pdf of , where ln and finished by showing that is also
uniformly distributed on 0,1 . ]
STEP Solutions 2016
Mathematics
STEP 9465/9470/9475
November 2016
The Admissions Testing Service is a department of Cambridge English
Language Assessment, which is part of Cambridge Assessment,
a not-forprofit department of the University of Cambridge.
2
Contents
Solutions Page
STEP Mathematics I 5
STEP Mathematics II 19
STEP Mathematics III 24
3
4
STEPI2016Solutions
Question1
Asastartertothepaper,thisisastraightforwardquestionintermsofitsearlydemandsandinvolveslittle
morethantheneedtosortoutsomeclearlyͲsignpostedalgebra.Tobeginwith,itisclearthat,whenevern
isodd,theexpression(xn+1)has(x+1)asafactor(bytheFactorTheorem),sothat
qn(x)= x 2 n x 2 n 1 x 2 n 2 ... x 3 x 2 x 1 .
Examiningthepn(x)’sinturn,usingthebinomialtheorem(andPascal’sTriangleforthecoefficients),gives
p1(x)= x 2 2 x 1 3x(1) = x 2 x 1 ,
p2(x)= x 4 4 x 3 6 x 2 4 x 1 5 x( x 2 x 1) = x 4 x 3 x 2 x 1 ,and
p3(x)= x 6 6 x 5 15x 4 20 x 3 15x 2 6 x 1 –7x( x 2 x 1 )2.
Expanding( x 2 x 1 )2isrelativelystraightforward,anditisrelativelyeasytoobtaintherequiredresults.
There are several ways to demonstrate that two given expressions of the given kind are not identically
equal.Oneistoexpandthembothaspolynomialsandshowthattheyarenotthesame.Inthiscase,
p4(x)= x 8 x 7 x 6 2 x 5 7 x 4 2 x 3 x 2 x 1 whileq4(x)= x 8 x 7 x 6 x 5 x 4 x 3 x 2 x 1 .
Alternatively,oneneedonlyshowthatonecorrespondingpairofcoefficientsarenotthesame–here,the
coefficientsof(say)x5arenotequal.However,thesimplestthingistofindanyonevalueofxforwhichthe
twoexpressionsgivedifferentvalues.Itturnsout,infact,thatonlyx=0actuallydoesgiveequaloutputs,
soalmostanychosenvalueofxwouldsuffice,andthekeyisthentochooseoneforwhichtheworking
19 1
involvestheminimumofeffort,suchasp4(1)=28–9.1.33=13zq4(1)= 1 .
11
In(ii)(a),thegivennumericalexpressionisclearlythatforq1(x)withx=300.Sincepandqarethesame
thingwhenn=1,weinsteadexaminep1(300),anditbecomesclearthatifxis3timesaperfectsquare(in
thiscase3u102)thenwecanusethedifferenceͲofͲtwoͲsquaresfactorisationon(301)2–(3u10)2togetthe
answer271u331.
Part(b)hasasimilarthinggoingon,buthereweneedxtobe7timesaperfectsquare,andwefindthat
>
we have 77 1 @ 7 7 7 1 , which again requires the use of the differenceͲofͲtwoͲsquares
3 2 8 14 7 2
dy
ax 2 bx c u §¨1 12 1 x 2
1
> @ 1
2
.2 x ·¸ 2ax b ln x 1 x 2
dx x 1 x ©
2 ¹
©
1
¹
>
+ dx e §¨ 12 1 x 2 2 .2 x ·¸ d 1 x 2 @
anditiseasytobeputoff;itisespeciallyimportantnottoattempttoomuch“inyourhead”.Youshould
findthissimplifiesto
dy ax 2 bx c > 1 x x@+ 2ax b ln x
2
x dx e
dx >x 1 x 2
@ u
1 x 2
1 x 2 +
1 x 2
d 1 x2
1 1 x 2
andcollectinguptermssuitably,andnotingthat { leadstoanexpressionwhichcontains
1 x 2 1 x2
1
onlysimplemultiplesof and ln x 1 x 2 ;namely
1 x 2
dy ( a 2 d ) x 2 (b e ) x ( c d )
+ 2ax b ln x 1 x 2 .
dx 1 x 2
Alltheresultsoftheremainingpartsofthequestioncannowbededucedbychoosingsuitablevaluesfor
theconstantsatoe.
dy (0) x 2 (0) x (0)
In(i),choosinga=d=0,b=1,e=–1andc=0gives + 0 1 ln x 1 x 2 ,so
dx 1 x 2
³ 1 x 2 dx= 12 ln x 1 x 2 12 x 1 x 2 (+C).
Andin(iii),choosinga= 12 ,b=e=0,c= 14 andd= 14 gives
dy ( 12 12 ) x 2 (0) x ( 14 14 )
+ x 0 ln x 1 x 2
dx 1 x 2
andhence ³ x ln x 1 x 2 dx= 1
2 x2 1
4
ln x 1 x 2 14 x 1 x 2 (+C).
6
Question3
Ifyouhavenotseenthissortoffunctionbefore,thenitisworthwhileplayingaroundwithsuchthingsas
partofyourpreparationfortheSTEPs,whichfrequentlytestperfectlysimpleideasincontextsthatarenot
astandardpartofAͲlevel(orequivalent)courses.Beingabletothinkthingsthroughcalmlyandcarefully
underexaminationconditionsisanespeciallyhighͲlevelskill,butonethatcanbepractised.
In this case, the “integer part” function is a relatively simple one to deal with, as it only changes values
whenthefunctionitactsonhitsanintegervalue.Beforecommencingworkonthisquestion,notethatthe
“integerpart”ofanegativenumberistheonetotheleftofit(ifitliesbetweenintegers,ofcourse),and
manyfunctionͲplottingpackagesaresetto“goright”fornegativenumbers,whichisunfortunate.Thereis
also the small matter of how to illustrate the “y” values at those points when the “step” occurs … the
traditionistoemploya“filled”circle(භ)forinclusionandan“open”circle(ӑ)forexclusion.Withabitof
careyoushouldfindthatthefourgraphsrequiredherelookasfollows.
(i) (ii)
(iii) (iv)
7
Question4
AswithQ2,thisbeginswithasimpleinstructiontodifferentiate;again,theideaisthatyouwilltidyupthe
z
finalanswerforlaterreference.UsingtheQuotientandChainRuleson y gives
1 z2
1
dy 1 z 2 .1 z. 12 1 z 2 2
.2 z
= 2
dz 1 z2
1
whichsimplifiesto 3 .
1 z 2 2
The given expression in part (ii) initially appears to be quite awful, until you realise that writing, for
§ d2 y · dz
¨¨ 2 ¸¸
instance, z
dy
turns © dx ¹ N into dx N ,andthiscannowbeseentobeastandard
3 3
dx ° § dy · 2 ½° 2 1 z2 2
®1 ¨ ¸ ¾
°̄ © dx ¹ °¿
dz
“separable variables” firstͲorder differential equation: ³ 3 ³ N dx . Using (i)’s result then gives
1 z2 2
z
N ( x c) (wheretheusual“+c”hasbeenincorporatedintoaslightlymorehelpfulformhere).
1 z2
u
ReͲarranging this for z or z2 leads to z 2 N 2 ( x c) 2 ( z 2 1) z r
where (again) the more
1 u2
complicatedͲlookingtermhasbeengivenanewlabel,whichisasimplebuteffectivedevicetomakewhat
todonextmoreobvious:here,u=N(x+c).
dy d y d u du
Wenowsubstitutebackforz= . andusetheChainRule(e.g.)with N toobtainanother
d x d u dx dx
dy u u
“separablevariables”firstͲorderdifferentialequation, N r or ³ N dy r ³ du.Atthis
du 1 u2 1 u2
u
point,youshouldbeabletoseethat ³ ducanbeintegrated(by“recognition”,“reversechainrule”
1 u2
orasubstitution) to give 1 u 2 . Substituting for u then gives ky d # 1 N 2 ( x c ) 2 and
squaring both sides leads towards a circle equation (Ny d ) 2 1 N 2 ( x c) 2 or
2 2
§ d· §1· § d·
( x c) 2 ¨ y ¸ ¨ ¸ ,whichistheequationofacircle,withcentre ¨ c, ¸ andaradiuswhichis
© N ¹ ©N ¹ © N¹
thereciprocalofthecurvature .
8
Question5
Thissortofsituationisrelativelycommonin‘MathsChallenges’andtheusualapproachistojoinallthe
circles’ centres to the points of tangential contact and then form some rightͲangled triangles by
considering(here)thehorizontallinethroughB’scentre.BecauseofthewellͲknown(GCSE)CircleTheorem
“tangent perpendicularto radius” result, it is the case that each of AB, BC and CA is astraight line. This
enablesustousePythagoras’Theorem:
PR=PQ+QR ( a c ) 2 ( a c ) 2 = (b a ) 2 (b a ) 2 + (c b ) 2 (c b ) 2
which simplifies to 4ac = 4ab + 4bc and, upon division throughout by 4abc , gives the required
1 1 1
answer .
b c a
There are many ways to approach the next result, but it should be clear that each will, at some stage,
require the replacement of the b’s with a’s and c’s (or equivalent). The most direct route would be to
examinetheLHSandRHSof(**)separately,andthenshowthattheymatchup.Thiswouldlooklike:
§ 1 1 1 · 2 2 § 1 4 6 4 1 · 4 12 4 8 8
LHS= 2¨ 2 2 2 ¸ 2 2¨¨ 2 2 ¸¸ = 2 2 .
©a c ¹ 2
b a c ©a a ac ac c ac c ¹ a ac c a ac c ac
2
§ 1 1 1½ 1 ·
2 2 2
§ 1 1 1· 2 §2 2 2· §1 1 1·
RHS= ¨ ¸ ¨ ®
¨a ¾ ¸¸ ¨ ¸ 4¨ ¸
©a b c¹ © ¯a ac c ¿ c ¹ ©a ac c ¹ ©a ac c ¹
§ 1 3 1 2 2 ·
4¨ 2 2 ¸ ,andtheseareclearlythesame.
© a ac c a ac c ac ¹
Working in the other direction is trickier, but not much more so, and it is again helpful to reͲlabel the
variables to make things look simpler, especially if we can somehow remove the need for everything to
appearasafraction.So,followinganinitialobservationthat
2
§ 1 1 1 · §1 1 1· 1 1 1 2 2 2
2¨ 2 2 2 ¸ = ¨ ¸ 2 2 2
©a b c ¹ ©a b c¹ a b c ab bc ca
1 1 1
wecouldwrite x ,y ,z ,sothatwearenowtryingtoprovethat
a b c
x4 y4 z4 2x2 y 2 2 y 2 z 2 2z 2 x2 .
(Althoughitisnotessentialtodothisatthisstage,itisoftenthecasethatfolksforgettodoitattheendif
theydon’t;andthatistoconsiderthegivenconditionsb<c<a,whichtranslatetoy>z>x.)
2
Now, completing the square: x 2 z 2 y 2 4 x 2 z 2 x 2 z 2 y 2 r 2 xz z # x
2
y 2 , and there
arethefourcasestoconsider:y=x–z,y=z–x,y=x+zory=–x–z.Considerationoftheabove
1 1 1
conditionsonx,y,zthenshowsthatonlyy=x+zissuitable,andso ,asrequired.
b c a
9
Question6
This is a fairly straightforward vectors question that involves little beyond working with the vector
equationofaline.Tobeginwith,youarerequiredtoexplainacoupleofintroductoryresultsthatrelyon
thefactthattwo(nonͲzero)vectorsaremultiplesofeachotherifandonlyiftheyareparallel.Thus,OX||
OAx=ma,with0<m<1,sinceXisbetweenOandA;andBC||OAc–b=kaandsoc=ka+b,with
k<0sinceBCisintheoppositedirectiontoOA.
Then, lines OB and AC have vector equations r = E b and r = a + D (c – a) respectively, for some scalar
parametersDandE.Replacingcbyka+bandequatingthetwor’sforthepointofintersectionthengives
Eb=a+D(ka+b–a).Sinceaandbarenotparallel,wecanequatetermstofindthat1–D+Dk=0and
1 1
D=E.SolvingleadstoD=E ,sothatd= b.
1 k 1 k
§ 1 ·
Inanexactlysimilarway,wethenhaveY=XDBCma+D ¨ b ma ¸ =b+Eka.(Notethatthereis
©1 k ¹
no reason why we have to use differentsymbols for the scalar parameterseach time, as they are of no
D
actualsignificanceinrelationtotheproblem.)Equatingcoefficients m– Dm– Ek=0and 1 ,so
1 k
thaty=kma+b.
§ 1 ·
Next,Z=OYAB(1–D)a+Db=E kma b ,andso1–D–kmE=0andD=E ¨ ¸ ,givingz=
© 1 km ¹
§ km · § 1 · 1 § km 1 1 ·
¨ ¸ a+ ¨ ¸ b;andT=DZ OA Da= b+ E ¨ a b b ¸ ,whence D=
© 1 km ¹ © 1 km ¹ 1 k © 1 km 1 km 1 k ¹
Ekm 1 E E § m ·
and0= ,sothatt= ¨ ¸a .
1 km 1 k 1 km ©1 m ¹
Noticethatallthathasbeendonesofaristoworkoutthepositionvectorsofthepointscreatedasthe
intersectionsofvariouspairsoflines.Ifthisisdifficulttovisualize,thenadiagramshouldbedrawnfirst.
Allthatremainsistosetupthelengthsofthevariouslinesegmentsofinterest.IfwecallOA=a,thenit
§ m · § m2 ·
¸¸a , TA = §¨
1 ·
follows that OX = ma, OT = ¨ ¸ a , TX = ¨¨ ¸a and XA = (1 – m)a. (Note that the
© 1 m ¹ © 1 m ¹ © m¹
1
shrewd solver would simply take, w.l.o.g., the value of a to be 1, as it isan arbitrary length and cancels
throughoutanyoftheworkingthatfollowsinordertoobtainthetwogivenanswers,
1 1§ 1· 1 1 § m · 2 § 1 ·
¨1 ¸ andOT.OA= ¨ ¸a ( ma ) . ¨ ¸a =OX.TA.
OT a © m ¹ OA OX ©1 m ¹ ©1 m ¹
10
Question7
Firstly,ST=thesetofallpositiveoddnumbers;andST=I.
Next,wemustshowthattheproductoftwodifferentelementsofSisalsoanelementofS,andthendoa
similarthingfortwodifferentelementsofT.Thisinvolvesnotingthat
(4a+1)(4b+1)=4(4ab+a+b)+1,
whichisanelementofS;and
(4a+3)(4b+3)=4(4ab+3a+3b+2)+1,
whichis,infact,inSratherthanT.
Theresultofpart(iii)essentiallyrequiresproofbycontradiction,sowefirstsupposethattisanelementof
T,andthatalloft’sprimefactorsareinS.Notingthattherearenoevenfactors,wecanwrite
t=(4a+1)(4b+1)(4c+1)…(4n+1).
ButwehavealreadynotedthattheproductofanypairofelementsofSwillalwaysyieldanotherelement
ofS,andhence(inductively,Isuppose),t=4{………}+1isalwaysinS.Whichcontradictstheassumption
thattisanelementofT.
Forpart(iv)(a),wenotethatanelementofTiseitherTͲprimeorTͲcomposite.Ifitisthelatter,thenitcan
beexpressedasaproductofTͲprimes.However,wehavealreadyestablishedthateverypairoffactorsinT
multiply to give an element of S, as do every pair of elements of T. So, after every pairingͲup of this
element’s factors, there must be an odd one left over to multiply by in order to give an element of T.
Hence,altogether,thereisanoddnumberofthem.
Forthefinalpartofthequestion,wearerequiredtofindnonͲprimesinSthatareproductsofelementsof
T that can be “reͲgrouped” suitably. One such example involves the numbers 9, 21, 33 and 77, each of
whichisbothSͲprimeandaproductofelementsinT:
9=3u3,21=3u7,33=3u11and77=7u11;with3,7and11inT.
This leads to the example 9 u 77 = 21 u 33 (= 693). Of course, the main purpose of the question is to
demonstrate the existence of perfectly reasonable numberͲsets (in this case, S), having the standard
properties of multiplication, yet for which the “unique factorisation” principle no longer holds. This is a
very important principle relating to prime numbers within the set of positive integers, which you have
beentaught(quiterightly)toassume,butthatIimagineyouhaveneverhadreasontothinkthatitmight
notnecessarilyholdinthiscase,orinothersimilarsituations.
11
Question8
Thisisasimpleideainvolvingtermsofseries. Themostimportantthing here istobesurethatyoucan
justifytheformofthenthtermforanygivenseries.Tobeginwith,itishelpfultorealisethattheBinomial
Theoremappliedtothenegativeintegers,givesthefollowingresults:
(1 x) 1 1 x x 2 x 3 ... x n ... ,withthecoefficientofxnbeing1;
(1 x) 2 1 2 x 3x 2 4 x 3 ... (n 1) x n ... ,withthecoefficientofxnbeing(n+1);and
(1 x) 3 1 3x 6 x 2 10 x 3 ... 12 (n 1)(n 2) x n ... ,havingthetriangularnumbersascoefficients;
etc.
Havingestablishedtheseresults,itiseasytoshowthat
0 x 2 x 2 3x 3 ... nx n ... =x (1 x) 2
andthat
x(1 x) 3 x 1 3x 6 x 2 10 x 3 ... 12 n(n 1) x n 1 ... = 0 x 3x 2 6 x 3 ... 12 n(n 1) x n ...
hascoefficientofxnequaltoun= 12 n 2 12 n .
2x x
Usingthesefirsttworesults:2u(2nd)–(1st)gives withun=n2.
(1 x) (1 x)
3 2
Thereareseveralwaystoproceedwithpart(ii)(a);thesimplestbeingtonotethat
a akx ak 2 x 2 ak 3 x 3 ... ak n x n ... = a kx a akx ak 2 x 2 ak 3 x 3 ... ak n x n ...
=a+kxf(x)
§ 1 ·
andsof(x) a¨ ¸ .
© 1 kx ¹
Forpart(ii)(b),youshouldnotethatthegivensecondͲorderrecurrencerelation(i.e.twoprecedingterms
areinvolved)requirestwostartingtermsbeforethingsgetgoingsystematically,soitisbesttosplitoffthe
firsttwotermsoftheseriesbeforeattemptingtomakeuseofthisdefiningfeature.
f f f f
Writingf(x)= 0 x ¦ u n x n = 0 x ¦ u n 1 u n 2 x n = x x ¦ u n 1 x n 1 x 2 ¦ u n 2 x n 2
n 2 n 2 n 2 n 2
Note:wearetryingtoreͲcreatef(x)ontherightͲhandside
f f
= x x ¦ u n x n x 2 ¦ u n x n = x x^f ( x) 0` x 2 f ( x)
n 1 n 0
x
sothatf(x)= .
1 x x2
12
Question9
AswithallStaticsproblems,itissoimportanttohaveacleardiagram,suitablymarkedwithalltherelevant
forces(andpreferablyinthecorrectdirections).Thegivendiagramappearstogivethesituationwhenthe
horizontalrailismorethanhalfͲwayalongtherod;inthisconfiguration,therodwillslipsothattheendA
slidesdownthewall,andtotheleftrelativetothecontactatP(asshownbelow).
RP
B
FAGbPFP
Ta
A RA
W
d
Thenextessentialpartofanysolutionistowritedownallthekeystatementsbeforeattemptingtoprocess
them in some way. The guidance “resolve twice and take moments” is surely a stock part of every
mechanics teacher’s repertoire, and it is sound advice. It is customary to resolve in two perpendicular
directions (vertically and horizontally here) and to find some point about which to take moments that
minimisesthe“clutter”ofsubsequentalgebraͲandͲtrigonometry(Ahasbeenchosenhere).Theadditional
useoftheFrictionLawfor,onthisoccasion,thecaseoflimitingequilibriumatthetwopointsofcontactis
alsoneeded.Thisgivesusthefollowing“ingredients”foruseinasolution.
FrictionLaw FA=ORAandFP=PRP.
Res.n W=FA+RPsinT+FPcosT
Res.o RA=RPcosT–FPsinT
օA WasinT=RP(a+b)
EliminatingtheF’sfromthetworesolvingstatementsgives
W=ORA+RPsinT+PRPcosTandRA=RPcosT–PRPsinT
andintroducingdinthemomentsstatement(notingthatitneedstoappearintheanswer)gives
Wasin2T=RPd.
SincethislastequationinvolvesjusttheforcesWandRPitmakessensetoeliminateRAnext,toget
W RP O cosT OP sin T sin T P cos T .
Finally,dividingtheselasttwoleadstothegivenanswer,
dcosec2T a [O P ] cosT [1 OP ]sin T .
ForthecasewhenPislessthanhalfͲwayalongtherod,whichwillnowslideupthewallatA(etc.),wecan
simplywriteFA o–FA;FP o–FP;a+b oa–b,orjustswitchthesignsof Oand P,aseverythingelse
remainsunchanged.Thecorrespondingresultisthusshowntobe
dcosec2T a [O P ] cosT [1 OP ]sin T .
13
Question10
When it comes to it, collisions questions involve only the use of two (sometimes three, if energy
considerations are involved) principles. In this case, the principles of Conservation of Linear Momentum
(CLM)andNewton’s(Experimental)LawofRestitution(NELorNLR).Diagramsarealsoquiteimportantin
thesetypesofquestions,butlargelytoenablethesolvertobeclearaboutwhatdirectionsarebeingtaken
aspositive:rememberthatvelocityandmomentumarevectorialinnature.Ifyouareeverunsureabout
whichdirectionanyoftheobjects(particlesorspheres,etc.)willbemovinginafterthecollisionhastaken
place,thenjusthavethemallgoingthesameway;thismakesitmucheasiertointerpretnegativesignsin
anylateranswers.
Tobeginwith,therearetwoseparatecollisions,betweenA/BandC/D,asshownbelow.
ou o0 0m um
A B C D
ovA ovB mvC mvD
ForcollisionA/B CLM:m(Ou=OvA+vB)andNEL:eu=vB–vA
andforcollisionC/D CLM:m(u=vC+vD)andNEL:eu=vC–vD
Solvingeachpairofequations,separately,gives
O e O (1 e)
vA u , v B u , vC 12 (1 e)u and vD 12 (1 e)u ;
O 1 O 1
eventhoughsomeoftheseturnoutnottoberequired(thoughtheymaybelateron,ofcourse).
ThereisthenafurthercollisionbetweenB/C:
ovB vCm
B C
o0 owC
CLM:m(vB–vC)=mwCandNEL:e(vB+vC)=wC
Now,substitutingpreviousanswersintermsofeandu,andidentifyingfore,leadstotherequiredanswer
O 1
e .Tojustifythefollowingconditionone,notethat
3O 1
O 1 1 § 3O 3 · 1 § 3O 1 4 · 1 4
e ¨ ¸ ¨ ¸ 3
1
3O 1 3 © 3O 1 ¹ 3 © 3O 1 ¹ 3 3O 1 3
sincethetermbeingsubtractedispositive.(Itisnotsufficientsimplytoshowthateo 13 .)
14
(1 e)(O 1)
Finally,usingwC= u frompreviouswork,andequatingthistovD,gives
2(O 1)
(1 e)(O 1)
u = 12 (1 e)u ;
2(O 1)
andsubstitutingfore(e.g.)enablesustosolveforOandthenfinde: O 5 2 ,e= 5 2 .
15
Question11
Projectilesquestionsareoftenstraightforwardinprinciple,butcanoftenrequirebothcarefulthoughtand
sometrickytrigonometricmanipulation.Occasionally,thereistheadditionalrequirementtomaximiseor
minimisesomethingͲorͲotherusingcalculus.
Tobeginwith,itisusefultoknow(ortobeabletoderivequickly)theTrajectoryEquationoftheparabola:
gx 2 gx 2
y x tan D .Settingy=–handreͲarrangingthengives 2h cos 2 D 2 x sin D cosD ,andit
2u cos D
2 2
u 2
shouldbefairlyclearfromthegivenanswerthatalittlebitofworkusingthedoubleͲangleformulaewill
gx 2
leadtotherequiredresult h(1 cos 2D ) x sin 2D .
u2
d § gx 2 · § dx ·
Differentiatingw.r.t.D ¨¨ 2 ¸¸ h 2 sin 2D ¨ x.2 cos 2D sin 2D . ¸ .Noticethatthereisactually
dD © u ¹ © dD ¹
d
noneedtoreͲarrangefor ... sincewerequirebothderivativetermstodisappear.Thisimmediately
dD
gh 2 tan 2 2D
leads to x = h tan2D ; and, substituting back, we get h(1 cos 2D ) h tan 2D sin 2D . By
u2
cancellingoneoftheh’sand(e.g.)writingalltrig.termsinc=cos2Dthenyields
gh 1 c 2 1 c
2
1 c gh ghc 2 u 2 c 2 c 3 c c 3 .
u 2c 2 c
Asaquadraticinc: 0 > @
u 2 gh c 2 u 2 c gh = u 2 gh c gh (c 1) …notethatitisalwaysworthtrying
tofactorisebeforedeployingthemessierquadraticformula.
gh
Wenowhavecos2D= 2 ,andsubstitutingx=htan2Dandy=–hin '2 x 2 y 2 ,thengives
u gh
'2 h2sec22Di.e. ' h sec 2D
u 2 gh u2
sothat ' h. h ,asrequired.
gh g
16
Question12
Bizarrely,thisprobabilityquestionconcerningtossingfaircoinshasallofitsanswersequalto 12 .Asone
mightimagine,suchananswercanbeobtainedinverymanywaysindeedandsothekeyistomakeyour
workingreallyclearastohowtheanswerisarrivedat.Merelywritingdownawholeloadoffractionson
thepageisreallynotveryenlightening.Thereneedstobesome(visible)systematicapproachtocounting
cases,followedbythenumericalworkthatgoeswithit.Forinstance,inpart(i),wecouldbreakdownthe
answerintoeachpossiblevalueforAandthevaluesofBthatcouldthengowithit.
e.g. p(A=0).p(B=1,2or3)+p(A=1).p(B=2or3)+p(A=2).p(B=3)= 14 u 78 2 u 14 u 84 14 u 18 = 12 .
Forpart(ii),oneshouldbynowhavebeenableto“see”howtheresultsarise,andcanappealtoa“similar”
process;e.g. 18 u 461641 83 u 61641 83 u 4161 18 u 161 …noticetheappearanceofthebinomialcoefficients
1
forcountingtherelevantnumbersofB’spossiblevalues.Thisgives 128 15 3 u11 3 u 5 1 1
128 64 = 12 .
Inpart(iii),youshouldnotethat,wheneachofthemhastossedncoins,
p(BhasmoreHs)=p(AhasmoreHs)=p2
andthatp(AH=BH)=p1.Thusp1+2p2=1.
Next,consideringwhathappenswhenBtossestheextracoin,
p(BhasmoreHs)=p(BalreadyhasmoreHs)up(BgetsT)
+p(Balreadyhasmore,orequal,Hs)up(BgetsH)
= p2 u 2 ( p1 p2 ) u 12 = 12 ( p1 2 p2 ) 12 .
1
17
Question13
FortheiͲtheͲmail,thepdfis f i (t ) Oe Ot .Integratingthisgivesthecdf: Fi (t ) e Ot +C,andF(0)=0
C=1.
Then,forneͲmailssentsimultaneously,
F(t)=P(Tdt)=1–P(allntakelongerthant)= 1 e Ot ,
n
(usingtheproductofnindependentprobabilities)
= 1 Oe Ont .
DifferentiatingthisthengivestherequiredpdfofT,f(t)= nOe Ont .
f
Findinganexpectedvalueisastandardintegrationprocess:E(T)= ³ t u nOe Ont dt,andthisrequiresthe
0
f
ª e Ont º f 1
>
useofintegrationbyparts:E(T)= te Ont @ + ³ nOe O
f nt
dt=[0]+ « » = .
¬ O n ¼ 0 nO
0
0
Fortheveryfinalpart,onecouldgoagainthroughtherouteofpdfsandcdfs,butitshouldbeobviousthat
thewaitingͲtimeforthe2ndemailissimplythe1stfromtheremaining(n–1)…withexpectedarrivaltime
1 1 1 1 §1 1 ·
,givingatotalwaitingtimeof ¨¨ ¸ .
(n 1)O nO (n 1)O O © n (n 1) ¸¹
18
STEP//2016Solutions
Question1
Useݐଵ andݐଶ torepresentthevalueoftheparameterݐateachofthepointsPandQ.The
equationsofthetwotangentscanthereforebefoundintermsofݐଵ andݐଶ andthefactthat
POQisarightanglecanbeusedtofindarelationshipbetweenݐଵ andݐଶ .Thepointof
intersectionofthetwotangentscanthereforebefoundintermsofjustݐଵ andthisisapair
ofparametricequationsforthecurvethatthepointofintersectionmakes.
Substitutingtheparametricequationsforܥଵ intotheequationforܥଶ givesacubicequation
in ݐଶ whichcanbesolvedbyinspectiontoshowthattherearejusttwointersectionsandso
thetwocurvesjusttouch,butdonotcross.
Question2
Substituteܿ ൌ ܽ ܾintotheexpressiontoshowthatܽ ܾ െ ܿisafactor.Oncethisis
done,thesymmetryshowsthatܾ ܿ െ ܽandܿ ܽ െ ܾmustalsobefactorsandtherefore
thereisjustaconstantmultiplierthatneedstobededucedtoobtainthefullfactorisationof
(*).
Forpart(i),choicesofܽ,ܾandܿneedtobemadesothat
ܽ ܾ ܿ ൌ ݔ ͳ
ʹ ݔଶ ͷ
ܽଶ ܾ ଶ ܿ ଶ ൌ
ʹ
Ͷ ݔଷ ͳ͵
ܽଷ ܾଷ ܿ ଷ ൌ
Ͷ
Oncethesehavebeenidentifiedthesolutionstotheequationfollowfromthefactorisation
alreadydeduced.
Oncethesubstitution݀ ݁ ൌ ܿhasbeenmadeitisonlynecessarytoidentifythepartsof
theexpressionwhichdifferfrom(*)inthefirstpartofthequestion(whicharisefromtheܿ ଶ
andܿ ଷ terms).Thefactorisationandsolutionoftheequationthenfollowasimilarprocess
tothefirstpartofthequestion.
19
Question3
Thedifferentiationtoshowtheresultinpart(i)shouldnotpresentmuchdifficulty,although
itisimportanttoshowthatalloftheterms(andnoothers)arepresent.
Forpart(ii)observethateachindividualtermof݂ ሺݔሻhasapositivecoefficient,soforany
positivevalueofݔthevalueof݂ ሺݔሻmustbepositive.
Forpart(iii),usetheresultinpart(i)torewrite݂ Ԣሺݔሻintermsof݂ ሺݔሻandnotethat݂ ሺܽሻ
and݂ ሺܾሻmustbe0.Thismeansthatanypairofrootsmusthaveagradientofthesame
sign,whichleadstoanargumentthattheremustbeanotherrootbetweenthetwo.Asthis
wouldleadtoaninfinitenumberofrootstoapolynomial,therecannotbemorethanone
root.
Toestablishthenumberofrootsinthetwocasesconsiderthebehaviourofthegraphas
ݔ՜ λandas ݔ՜ െλ
Question4
Theequationgivencanberewrittenasaquadraticinݔ.Thediscriminantthenestablishes
therequiredresult.Toshowthesecondresult,showthatݕଶ ͳ ሺ ߠ ݕെ ߠሻଶ,
whichcanbeshownbywriting ߠ ݕെ ߠintheformܴ ሺߠ ߙሻandthenthisresult
isaquadraticinequalitythatleadsdirectlytothenextresult.
ସାξ
Inthecase ݕൌ ,carefulmanipulationofsurdsshowstherequiredresultandsothe
ଷ
valueofߠmustbethevalueofߙobtainedintheprevioussection.Finally,thevalueofݔ
canbeobtainedbyreturningtotheoriginalequationandsubstitutinginthevaluesthatare
known.
20
Question5
Thebinomialexpansionforሺͳ െ ݔሻିே shouldbeeasyenough,itisthenrequiredtowrite
theproductintermsoffactorialssothattheexpressioncanbewrittenintermsofቀݍቁ.
21
Question8
Theintegralrequiredatthestartofthequestionshouldbeastraightforwardoneto
evaluate.Whenmakingasketchtoillustratetheresultinthesecondpart,ensurethatthe
sumisindicatedbyaseriesofrectangles,withthegraphofthecurvepassingthroughthe
midpointsofthetops.
Inpart(i),theintegralthatwouldmatchthesumgivenresultsinananswerof2,sothisis
thefirstoftheestimates.Theremainingestimatesarisefromusingtheintegraltoestimate
mostofthesum,buttakingthefirstfewtermsastheexactvalues(soineachcasethe
integrationistakenfromadifferentlowerlimit).
Forpart(ii),evaluatetheintegralforoneparticulartermofthesumandnotethatitis
ଵ ଷଷ
approximately .Finally,usingthemostaccurateestimateforܧቀ ቁthesumfrom ݎൌ ͵
ସ ర ଶ
ଵ
onwardscanbecalculatedandthenthefirsttwovaluesof రcanbeaddedtoachievethe
desiredresult.
Question9
Theresultinpart(i)followsfromconsiderationofkineticenergylostandworkdone.
Inpart(ii)applyconservationofmomentumtothecombinedblockandbulletafterthe
bullethitstheblock.Bycomparingtothecaseinpart(i)themotionofthebulletuntilitisat
restrelativetotheblockcanbeanalysed.Oncealloftherelevantequationsofmotionhave
beenwrittendown,aseriesofsimultaneousequationswillhavebeenfoundfromwhichthe
valuesofܾandܿcanbefound.
Question10
Thefirstrequirementwillbetofindthecentreofmassofthetriangle.Oncethisisdonea
diagramwillbeveryusefulandnotationswillneedtobeaddedforvariousdistances,angles
andthefrictionalforce.Fromthisdiagramtheforcescanberesolvedintwoperpendicular
directionsandmomentscanbetaken.Thisleadstoaseriesofequationswhichcanbe
solvedtoworkoutthevaluethatthefrictionalforcewouldhavetotaketopreventslipping.
Fromthistherequiredresultcanbeestablished.
22
Question11
Theparticlesmustcollideiftheywouldbeinthesamepositionforoneparticularvalueofݐ.
Therefore,writingouttheequationsofmotionforthetwoparticlesandeliminatingthe
variablesthatarenotneededtherequiredresultcanbereached.
Forthesecondpart,thetimeofthecollisioncanbefoundbyconsideringtheheightsofthe
bulletandtargetattimeݐandnotingthatthesemustbeequal.Oncethevalueofݐhas
beenfound,thefactthatthismustbepositiveleadstotheinequalitythatisrequiredforthe
firstresult.
Forthefinalpart,notethatgravityaffectsboththebulletandtargetinthesameway,soifit
isignoredthenthetimeofcollision(ifthereisone)willbethesameandthisisasituationas
inpart(i).Clearly,inpart(i)thetwoobjectsmustbemovingtowardseachotherifthereis
tobeacollision.
Question12
Replaceܤwithሺܥ ܤሻintheresultthatyoumuststartwithandthenobservethat
ת ܣሺܥ ܤሻisthesameasሺܤ ת ܣሻ ሺܥ ת ܣሻ.Thecorrespondingresultforfourevents
shouldbeclear,butcaremustbetakentoincludeallofthepossiblepairs.
Theresultsforparts(i),(ii)and(iii)shouldbeclearfromconsiderationofarrangementsin
eachcaseandtheresultrequiredfollowsfromthegeneralisationoftheresultfromthestart
ofthequestion.
Theprobabilitythatthefirstcardisinthecorrectpositionandnoneoftheothersiscanbe
establishedandthereforetheprobabilitythatexactlyonecardisinthecorrectpositionwill
be݊timesthat.
Question13
Forpart(i)theapproximationofthebinomialdistributionbyanormaldistributionshould
beknownandtheareaunderthecurve(applyingacontinuitycorrection)canthenbe
approximatedbyarectangle.
Thesecondresultfollowsfromasimilarapproximationandtheuseoftheformulafora
probabilityfromthebinomialdistribution.
Part(iii)followsfromanapproximationofaPoissondistributionwithanormaldistribution
andagainapproximatingtherequiredareabyarectangle.
23
^dW///ϮϬϭϲ^ŽůƵƚŝŽŶƐ
1. Part(i)ismostsimplydealtwithbythesuggestedmethod,changeofvariable,anditis
worthcompletingthesquareinthedenominatortosimplifythealgebraleadingtoatrivialintegral.
Part(ii)caneitherbeattemptedimmediatelyusingintegrationbyparts,startingfromܫ and
ஶ ଶ௫ሺ௫ାሻ
obtainingିஶ ሺ௫ మ ݀ ݔandthenwritingthenumeratoras
ାଶ௫ାሻశభ
ʹ݊ሺ ݔଶ ʹܽ ݔ ܾሻ െ ݊ܽሺʹ ݔ ʹܽሻ െ ʹ݊ሺܾ െ ܽଶ ሻ.Alternatively,useofthesamesubstitutionin
ܫାଵ asinpart(i)leadstotheneedtointegrate ଶ ݑ,whichinturncanbewrittenas
ଶିଶ ݑሺͳ െ ଶ ݑሻ ൌ ଶିଶ ݑെ ଶିଶ ݑ ݑǤ ݑ,withthesecondtermbeingsusceptible
tointegrationbyparts.Part(iii)followsfromthepreviouspartsbyinductionusingpart(ii)to
achievetheinductivestepand(i)thebasecase.
2. Therearenumerouscorrectwaysthroughthisquestion.Workingparametricallywith
ݔൌ ܽ ݐ, ݕൌ ʹܽݐgivesanormalas ݔݐ ݕൌ ܽ ݐଷ ʹܽݐandimposingthatthispassesthrough
ଶ
3. Differentiating,multiplyingbydenominatorsanddividingbytheexponentialfunction,gives
ሾܳሺܲ ܲԢሻ െ ܲܳԢሿሺ ݔ ͳሻଶ ൌ ሺ ݔଷ െ ʹሻܳଶ which,invokingthefactortheorem,givesthefirst
requiredresult.DenotingthedegreeofPbypandthatofQbyqinthisexpressionyields
ݍ ʹ ൌ ʹ ݍ ͵andhencethedesiredresultin(i).Furthermore,inthegivencase,substitution
inthesameresultandpostulatingܲሺݔሻ ൌ ܽ ݔଶ ܾ ݔ ܿyieldsconsistentequationsforܽ,ܾandܿ
andthusܲሺݔሻ ൌ ݔଶ െ ʹݔ.
Forpart(ii),commencingasinpart(i)demonstratesagainthatQhasafactorሺ ݔ ͳሻas
ሾܳሺܲ ܲԢሻ െ ܲܳԢሿሺ ݔ ͳሻ ൌ ܳଶ .Supposingܳሺݔሻ ൌ ሺ ݔ ͳሻ ܵሺݔሻ,where݊ ʹand
ܵሺെͳሻ ് Ͳ,withܲሺെͳሻ ് Ͳandsubstitutingintheexpressionalreadyderivedleadstoa
contradiction.
ሺ௫ିଵሻ௫ ೝ
4. Theconsideredexpressionequatesto ሺଵା௫ ೝ ሻሺଵା௫ ೝశభ ሻandso,bythemethodofdifferences,
௫ೝ ଵ ଵ ଵ
σே
ୀଵ ൌ ሺ௫ିଵሻ ቂ െ ቃ,andlettingܰ ՜ λ,thedesiredresultisobtained.
ሺଵା௫ ೝ ሻሺଵା௫ ೝశభ ሻ ଵା௫ ଵା௫ ಿశభ
ଶ షೝ
Writing ሺݕݎሻas andsimilarly ൫ሺ ݎ ͳሻݕ൯,theresultofpart(i)with ݔൌ ݁ ିଶ௬ can
ଵା షమೝ
beusedtoobtaintheresult.Careneedstobetakentowriteσஶ
ୀିஶ ሺݕݎሻ ൫ሺ ݎ ͳሻݕ൯as
ʹൣσஶ
ୀଵ ሺݕݎሻ ൫ሺ ݎ ͳሻݕ൯ ݕ൧whichwiththepreviousdeductionof(ii)canbe
simplifiedtoʹ ݕ.
24
ܲଵǡାଵ ܲାଵǡଶାଵ ,combiningthegivenresultand(ii),thedesiredresultisobtained.Part(iv)is
obtainedbyuseofstronginductionwiththesupposition,ܲଵǡ ൏ Ͷ forall݉ ݇forsome
particular݇ ʹ,andconsideringthecaseskevenandoddseparatelyandmakinguseof(iii).
6. Usingܴ ሺ ݔ ߛሻ ൌ ܴሺ ߛ ݔ ߛ ݔሻ,ܴ ൌ ξܤଶ െ ܣଶ and
ߛ ൌ ିଵ if ܤ ܣ Ͳ.If ܤൌ ܣ,then ݔ ܣ ݔ ܤൌ ݁ܣ௫ .Ifെ ܣ൏ ܤ൏ ܣ,the
expressioncanbewrittenasሺ ݔ ߛሻwithܴ ൌ ξܣଶ െ ܤଶ andߛ ൌ ିଵ .If ܤൌ െܣ,
then ݔ ܣ ݔ ܤൌ െି ݁ܣ௫ ,andif ܤ൏ െܣ,theexpressioncanbewrittenas
ܴ ሺ ݔ ߛሻwithܴ ൌ െξܤଶ െ ܣଶ andߛ ൌ ିଵ .Forpart(i),solvingsimultaneouslygives
ܽ ݔ ܾ ݔൌ ͳ,whichgivesthedesiredsolutionsusingthefirstresultofthequestion.
ଵ
Similarlyforpart(ii)usingtheappropriateresult, ݔൌ ିଵ ቀ ቁ െ ିଵ .For(iii),we
ξమ ି మ
ଵ
requirethattheconditionsfor(i)givetwosolutions,i.e.thatܾ ܽandቀ ቁ ͳ,andso
ξ మ ିమ
ܽ ൏ ܾ ൏ ξܽଶ ͳ,andviceversa,ifthisappliesthereareindeedtwosolutions.For(iv),werequire
case(i)togivecoincidentsolutions,i.e.ܾ ൌ ξܽଶ ͳandhence ݔൌ െ ିଵ మ ,andso
ξ ାଵ
ଵ
ݕൌ .Thereverseargumentalsoapplies.
ξమ ାଵ
7. Consideringሺ߱ ሻ establishesbythefactortheoremthateachfactorontheLHSisafactor
oftheRHS,andcomparingcoefficientsof ݖ betweenthetwosidesestablishesthatnonumerical
factorisrequired.Forpart(i),representingܺ by߱ ,thentherearetwocasestoconsider,ܲwill
ഏ ഏ
ቀ ାగቁ
berepresentedeitherby ݁ݎ ,or ݁ݎ .Theproductofmoduliisthemodulioftheproductof
factors,andtheproductofthefactorscanbesimplifiedusingthestemandchoosingݖinturnasthe
representationsofܲtogivetherequiredresultinbothcases.Proceedingsimilarlyfornodd,the
firstcaseyieldsȁܱܲȁ ͳ,andthesecond,ȁܱܲȁ െ ͳ,ifȁܱܲȁ ͳ,andͳ െ ȁܱܲȁ ifȁܱܲȁ ൏ ͳ.
Usingthesamerepresentationsfortheܺ inpart(ii),andthesametechniquewiththemoduli,the
stemcanbedividedbyሺ ݖെ ͳሻtogiveሺ ݖെ ߱ሻሺ ݖെ ߱ଶ ሻ ǥ ሺ ݖെ ߱ିଵ ሻ ൌ ݖିଵ ݖିଶ ڮ ͳ
whichthengivesthedesiredresultwhen ݖൌ ͳ.
25
భ షభ
ା௫ ଷఒ ଵ ξଷ ξଷ ௫ ଷ௫ మ మ
ξయ
,leadstoെߣ െ ݔ ʹߣ ቀ ܽ ݔቁ ቀͳ ቁ ൌ ݉ݔሷ whichmakingan
మ ξଷ ଶ ଶ ସమ
ටమ ାቀ భ ା௫ቁ
ξయ
ଷఒ
approximationforsmallݔandthebinomialexpansionleadstoെ ൫Ͷܽ െ ξ͵݈൯ ݔൌ ݉ݔሷ ,andhence
ସ
thefinalresult.
10. Resolvingalongalineofgreatestslopeinitially,bearinginmindtheaccelerationdueto
circularmotion,givesanexpressionfortheinitialtensioninthestringwhichcanbesubstitutedin
theexpressionobtainedfornormalcontactforceobtainedbyresolvingperpendiculartotheslope.
Requiringapositivenormalcontactforcethengivesthedesiredresult.Tocompletecircles,there
mustbeatensioninthestringwhentheparticleisatthehighestpointitcanreachontheplane.
Conservingenergygives ݒଶ ൌ ݑଶ െ Ͷܽ݃ ߚ ߙandresolvingdowntheplaneyields
௩మ
ܶ' ߚ ݉݃ ߙ ൌ ݉ resultinginݑଶ ͷܽ݃ ߚ ߙ;thiscombinedwiththefirst
ୡ୭ୱ ఉ
resultwillgivethefinaldesiredresult.(Thefirstresultcanbefoundelegantlybyresolving
perpendicularlytothestring.)
11. Inpart(i),expressingtheresistanceas݇ݒ,thenthezeroaccelerationconditiongives
ௗ௩
݇ൌ .Writingtheequationofmotionusingܽ ൌ ݒ ,andsolvingthedifferentialequationby
ଵ మ ௗ௫
ସା௩ ଶ
separatingvariables,theintegrationgivesܺଵ ൌ ቂ ቀʹܷ ቀ ቁ െ ݒቁቃ whichevaluatedand
ସି௩
rearrangedistherequiredresult.Part(ii)followsasimilarroute,insteadexpressingtheresistance
as݇ ݒଶ ,with݇ ൌ .Thesametechniqueforthedifferentialequationgivesaslightlysimpler
ସ య
ସ
integrationtoyieldtheresult.ߣܺଵ െ ߣܺଶ canbemanipulatedtobe ʹͶ െ ʹ ͷ െ ͳwhichcan
ଷ
beshowntobepositiveusingtheappropriateboundsandsoansweringpart(iii)thatܺଵ isthe
larger.
26
27
.
The Admissions Testing Service is part of Cambridge English Language Assessment, a not-for-profit department of
the University of Cambridge. We offer a range of tests and tailored assessment services to support selection and
recruitment for educational institutions, professional organisations and governments around the world. Underpinned
by robust and rigorous research, our services include:
STEP 9465/9470/9475
November 2017
2
Contents
STEP Mathematics I 4
STEP Mathematics II 11
STEP Mathematics III 24
3
STEP I 2017
Hints and Solutions
Question 1
Part (i)
Using tan and rearranging will give an algebraic fraction where the denominator is
and the numerator is a multiple of , after which the integration can be completed
easily.
Part (ii)
For the first integration, differentiating the denominator of the fraction will show that an
appropriate substitution is sec tan , following which the integration follows
easily. The second integration requires some rearrangement of the algebraic fraction:
division of the numerator and denominator by cos puts it into a form where it should be
clear that the same substitution will work for this integration as well.
Question 2
Part (i)
Taking definite integrals of the two sides of the inequality with the lower limit as 1 and the
upper limit as leads directly to the required result in the case 1. To show the result in
the case 0 1, note that 1 holds for 0 1 and perform definite integrals with
the lower limit as and the upper limit as 1.
Part (ii)
Following the same process as for part (i) will lead to the required result.
Part (iii)
Integrating (*) and rearranging leads to one half of the inequality and integrating (**) and
rearranging leads to the other half of the inequality. In both cases take limits of 1 and for
the integrals and consider the cases 0 1 and 1 separately.
4
Question 3
Using implicit differentiation on the equation of the curve gives 2 4 , which then
leads easily to an expression for the gradient of the curve at the point and so the equation
of the tangent can be deduced easily from this point. The equation of the tangent at can
then be written down as it simply requires changing to throughout.
A diagram of the situation described then allows a strategy for the areas of each triangle to
be worked out: for , one side is vertical and so this length can be used as the base and
so the required measurements are easily deduced from the coordinates of the points. There
are many ways of deducing the area of , for example by adding horizontal lines through
and and then considering the triangle to be a trapezium with two triangles cut away
from it.
Question 4
Part (i)
In the case where 1 3 it can be seen from the graph that there is only one possible
value for as the horizontal line for this value of only intersects the graph once. Since is
defined in terms of , the value of must also be determined uniquely.
In the other case, can be rearranged to make the subject and then substituted
into the function for to deduce the required equation.
Part (ii)
Considering a graph of the function for will again determine the values of that
determine uniquely and a similar approach by substituting will lead to the quadratic
equation required for the last part of the question.
5
Question 5
A diagram is very helpful in understanding the description in the first paragraph. From such
a diagram expressions for the width and height can be identified which will then lead to the
formula for the area of the rectangle. Since and are aligned vertically, they must have
the same ‐coordinate and this can be used to give an expression for in terms of the other
variables.
When performing the differentiation required next, remember that is a function of and
differentiate the expression for to find . This can then be substituted into the expression
for .
The greatest possible area can be found by setting 0 and application of trigonometric
identities will allow the final result to be shown.
Question 6
Part (i)
Note that if f does not take any negative values then the value of the integral must be
positive (and similarly, if it does not take any positive values then the value of the integral
must be negative). The result then follows from this.
Part (ii)
The integral that needs to be considered can be shown to be equal to 0 and so the result
from part (i) implies that g must have both positive and negative values in the
interval. Since 0 it must also be the case that g has both positive and
negative values in the interval and so it must also take the value 0 somewhere (by the result
that can be assumed in this question).
Substituting the form of g into the three integrals given in (*) gives a set of simultaneous
equations that can be solved to find an example of such a function and it is then
straightforward to confirm that there is a 0 by showing that the endpoints of the interval
give different signs for the value of g .
Part (iii)
To be able to apply the result from part (ii), it will need to be the case that g h′ .
Once the three integrals have been shown to have values that are consistent with part (ii)
the result follows immediately.
6
Question 7
Part (i)
Applying the cosine rule to the triangle leads to the required result and the
corresponding result for is easy to deduce.
Part (ii)
Applying the cosine rule to triangle leads directly to the first result required. Since the
formula is symmetric in , and , it must be the case that all three sides of are equal
and so the triangle is equilateral. The final result of this part can be shown by expressing the
area of in terms of the length of one side.
Part (iii)
The equivalence of the conditions follows by applying the formula for cos and
rearranging.
The final deduction will follow by combining the results from parts (ii) and (iii).
Question 8
Following the standard procedure for proof by induction establishes the first result.
Part (i)
The required fact about the sequence can be proven by induction, and then division of
(*) by will show that as → ∞, will approach the root of a quadratic equation and this
equation can be solved to give the required result.
Part (ii)
Finally, calculating the first few terms of the sequences leads to the required
approximations.
7
Question 9
Part (i)
By considering the horizontal speed it is possible to find the amount of time before the
particle passes through . Considering the vertical speed then allows a relationship between
and to be deduced. Differentiating with respect to and rearranging then leads to the
first result.
The relation between and can then be deduced by considering the relationship between
the graphs of tan and cot .
Part (ii)
Considering the horizontal and vertical components of the velocity as the particle passes
through allows the tangent of the angle at which it is travelling to be deduced. Graphical
considerations can then be used to deduce the relationship between this angle and .
Question 10
Part (i)
Part (ii)
It is clear that each particle must be involved in at least one collision. To show that there are
no more than two collisions it is required to show that each particle will still be moving
faster than the one behind it following the second collision.
Part (iii)
When all of the particles will have the same velocity following the collision and it can
be seen that the kinetic energies will form a geometric progression and so the sum to
infinity can be calculated and then the fraction of kinetic energy lost deduced.
Part (iv)
In this case the particles can be seen to come to rest eventually and so all of the kinetic
energy is lost.
8
Question 11
Adding the forces to the diagram and then resolving parallel and perpendicular to the slope
gives two equations relating all of the forces and lengths. Taking moments about the centre
of the rod gives a further equation. Solving these equations and using the relationship
between frictional and reaction forces then allows an equation in terms of tangents of the
appropriate variables to be found. Rearranging then allows the formula for tan to
be obtained and therefore the required result can be shown to be the only solution to the
equation that matches the situation described.
Question 12
Part (i)
The probability that any one participant does not pick the winning number is 1 and
since the participants’ choices are independent, the probability that no participant picks the
winning number is this value raised to the power . The profit is then £ in the case where
no one picks the winning number and £ if someone does pick the winning number.
Applying the given approximations then allows the result to be shown.
Part (ii)
The first relationship that is needed follows from the fact that the probability that one of the
numbers will be chosen must be 1. The expected profit can then be calculated by following a
similar procedure as in part (i).
In the case described at the end of this part the information along with the relationship
between , and is enough to deduce the values of and and that the organiser
expects to make a profit if 2 .
9
Question 13
It should be clear that the first slice of the loaf of bread can never be the second of two
slices to make a sandwich and so the value of can be deduced. To explain the first of the
two equations note that to use the slice of bread for toast means that the previous slice
must have been either the second slice of a sandwich or used to make toast (the total
probability of this is ). This must then be multiplied by the probability of using
this next slice to make toast. Since it is not possible to follow this reasoning for the first slice
of toast, the equation can only be valid for values of 2 or greater. Similarly, it cannot be valid
for the final slice as in that case it must be used for toast.
The fact that the second equation begins 1 ⋯ suggests that a sensible approach is to
begin with identifying all of the cases that prevent the slice being used for toast.
To show the next result, first eliminate the bracketed expressions from the two equations to
get an equation relating and . Substituting this into the second equation then gives the
required result.
The formula for can then easily be proven by induction and the relationship between
and used to find the corresponding expression for .
Finding expressions for the final terms of the sequences can now be achieved by finding the
appropriate equations to relate the use of the final slice of bread to the previous one and
substituting the expressions for and into these.
10
STEP II 2017
Hints and Solutions
Question 1
The opening “Note” clearly flags a result which will prove important in this question; this is a
‘standard’ result, but one that can slip by unnoticed in single‐maths A‐levels; it is, therefore,
worth learning as an “extra”, if necessary.
In (i), it should be obvious that the process of integration by parts is to be used and that the
initial hint indicates that the “first part” must be the “arctan ” term, despite appearing as
the second term of the product to be integrated. This will lead directly to the given result,
1
x
following which the substitution of 0 leads to the integral 1 x
0
2
dx. This can be done
derivative of the denominator – the standard “log. integral” form; in the second instance,
the substitution 1 will also work, though it might take just a few lines more
working.
In (ii), one needs only to use the result given in (i), this time replacing by 2 to find an
expression for 3 ; and then adding to it the given expression for 1 . This
leads to a result in which two integrals must be added to get, when simplified,
1
xn 1 1 x2
0
1 x2
dx, which should need no further comment. Setting 0 and then 2 in
this result then yields a numerical answer for , since has already been calculated.
In (iii), no matter how demanding the process of mathematical induction appears to be, it is
very formulaic in some respects and there are always some marks to be had. To begin with,
there is always the “baseline” case of (usually) 1. In this case, one must set 1 in the
proposed formula and check it against the value of already known. This reveals the value
of to be 41 21 ln 2 . However, since it is constant, it remains fixed during all of the
remaining work and one can most easily progress through the rest of the inductive proof by
simply continuing to use the letter .
A clearly stated induction hypothesis is enormously helpful (usually replacing the by
another dummy index, say) rather than vague statements such as “assume the result is
true for ” or meaningless statements such as “assume ”. We thus proceed
2k
1
assuming (4k + 1) = – 12 ( 1) r .
r 1 r
The remainder of the proof relies more on carefulness than inspiration, especially as the
process relies on exactly the same techniques as were used in part (ii), using and 1
in turn in the given result.
11
Question 2
The opening part of the question is essentially identical to the process of composition of
functions. Moreover, if subscripts are likely to prove confusing, then begin with a statement
aX 1
a 1
aX 1 X b
such as “Let = X.” Thus = and = , which simply needs
X b aX 1
b
X b
( a 2 1) X ( a b)
tidying up. This yields = . The “Note” in (i) reminds the reader that
( a b) X (b 2 1)
the period of a periodic sequence is the length of the smallest cycle of repetition; thus, we
require = but not = . A moment’s thought should reveal it to be obvious that
a constant sequence should still yield part of the same algebra which gives = and it
is worth exploring this first:
if = then –1 0 – – 1.
One might be tempted to try to solve this (using the Quadratic formula, for instance), but
there is really nothing to be gained by so doing, since the constant sequence is of no interest
to us, only the conditions that give it (which we need to have in mind later on). Proceeding
to explore = gives us, upon collecting up, 0 – – 1.
Fortunately, the factor is readily apparent, but the quadratic factor should have
been anticipated also, for the reasons outlined above. Thus, 0 is a necessary
condition for a period 2 sequence. (There is no requirement to explore the issue of
sufficiency, which could be done by setting – in the initial expression for and
then following it through to see what happens.)
Finally, we are asked to see what happens when = , and this can be done the long
3 2 2
( a 2 a b ) X ( a ab b 1)
way round by finding = 2 2 3
and then
( a ab b 1) X ( a 2b b )
axn 3 1
= , but there is at least one very obvious shortcut to what is starting to look
xn 3 b
like some complicated algebra: namely, considering the “two‐step” result already found and
repeating that, going from to via . It also helps if one realises that whatever
algebraic expression appears, we know that it must have the previously found factors of
and – – 1 within it.
12
Question 3
Whilst this question might appear somewhat daunting on first reading, it involves little more
than an understanding of the sine curve and the key results that relate to “angles in all
quadrants”.
To begin with, “sin sin ” is the kind of response expected from those
candidates who have failed to understand that there are many functions (even the simple
ones covered at A‐level) that don’t map “one‐to‐one”. Though the general formula “sin =
sin –1 ” might be unfamiliar, it only says that if sin = sin (imagining
to be positive and acute) then could be any “first quadrant” equivalent of … an even
multiple of plus … or a “second quadrant” supplement of … an odd multiple of minus
. (Apart from that, ’s corresponding to non‐acute ’s arise from application of the same
principles, with “quadrants” taking care of themselves due to the symmetries of the sine
curve.)
In (i), it is then found that the general formula (or its equivalent in two bits) gives three
graphs that are of interest here: –1 – – , 0( ) and 1(
– ), all of which give straight‐line segments in the interval required.
In (ii), there is rather less thinking to be done – at least to begin with – since one is only
required to differentiate twice and do some tidying up. This calculus can be done implicitly
or directly after rearranging into an arcsine form. That is not to say that it is easy calculus,
since the Product, Quotient and Chain rules can all play a part in the processes that follow.
dy 1
In sketching the graph, one should start simple and work up. Initially, = 2 at (0, 0), with
dx
d2 y
the curve increasing to a maximum at ( 2 , 6 ), since 0 . This gives
dx 2
Thereafter, for whatever “other” bits there are to the curve, these follow from symmetries,
applied to the above portion: namely, reflection symmetry in 2
; rotational symmetry
about O; and reflection symmetry in
2
.
Part (iii)’s graph follows by applying the result cos sin ( 2 – ).
13
Question 4
This is an interesting question and very straightforward in some respects. To begin with, you
are told in part (i) that f( ) = 1. At this point, it would be wise to write down what the
Schwarz inequality gives in this case:
2 2
b b b b b
g ( x) dx 1 dx g ( x)2 dx ; i.e. g ( x) dx b a g ( x)2 dx .
a a a a a
A few moments of careful thought (inspecting the given answer) should make you realise
that 0, give the terms – – 0 and e – e when g( ) = e . Following
it through from there is relatively routine, provided one spots the difference‐of‐two‐squares
factorisation and that we can divide throughout by (e 1), which is guaranteed to be
positive (an important consideration when dealing with inequalities).
In (ii), it is (again) best to start by seeing how things appear when you have used the
given information that f( ) = and, by clear implication, 0 and 1:
2 2
1 1 1 1 1
x g ( x) dx x 2 dx g ( x)2 dx ; i.e. x g ( x) dx 13 g ( x)2 dx .
0 0 0 0 0
2
1 1 x2 1 x2
The e 4 in the given answer, along with the fact that e 4 = e 2 should point the way
1 x2
towards choosing g( ) = e 4 . Following this through carefully again yields the required
result.
The result in part (iii) clearly requires the use of the Schwarz inequality twice, once each for
the right‐ and left‐halves of the given result. Setting f 1, g( ) = sin x , 0 and
1
2
1
2
leads to
0
sin x dx
2
. However, the left‐hand half of the result does require a
bit more thought and, preferably, familiarity with the integration of trig. functions where
powers of sin (in this case) appear along with its derivative, cos . The real clue is that, for
1
2
the
0
sin x dx to appear on the other side of the inequality to the one found using the
obvious candidates that led to the right‐hand half of the result, sin x must now be the
result of the squaring process. It is then experience (or insight) that suggests setting f( ) =
cos , g( ) = 4
sin x , 0 and = 21 . You will then find that the LHS of the Schwarz
inequality requires the integration of a function of sin multiplied by its derivative, cos ;
and this (as in Q1) can be done by “recognition” or substitution. (You will also need to be
able to integrate cos , which calls upon the use of the standard double‐angle formula for
cosine.)
14
Question 5
dy
In many ways, much of this question is also relatively routine. Find for the gradient of
dx
the tangent; find its negative reciprocal for the gradient of the normal and then any one of a
number of formulae for the equation of a line. At some stage, you will need to replace by
for the normal at and then replace and in this equation by and 2 for another
point on the curve. Solving for in terms of – noting that the factor ( – ) must be
involved somewhere, since must be one solution to whatever equation arises as the
line is already known to meet the curve at – should then yield the given answer.
2
In (ii), employing the distance formula = ( P – N)2 + ( P – N)2 is clearly the way
2
forwards, as is replacing by p at some stage of the proceedings. The rest is just
p
careful algebra. Differentiating the given expression for with respect to is routine
enough, in principle, and it is then only required to justify that the (only) values of that
arise will give minimum points. One could use the first‐derivative test (looking for a change
of sign), the second‐derivative test (examining its sign) or argue from the shape of the curve
16 a 2 2
( ) ( p 1) 3 … which is symmetric in the ‐axis, asymptotic to the ‐axis for small
p4
values of , and can be arbitrarily large as | | ; thus, any turning‐points must be
minima.
For part (iii), one starts by noting that and are perpendicular (since = 90, by
2 2
“Angle in a semi‐circle”). Then, setting the products of their gradients, and ,
pq nq
2
equal to – 1, replacing by p once again, and using 2, takes you almost the
p
2q 2
whole way there: q 2 . From this point, we have 0 or = 2 . Finally, these
p p
final two cases should be eliminated by noting that they give , i.e. , which is not
the case as they are being taken to be distinct points.
15
Question 6
This question is about two different types of proof – induction and direct manipulation. Both
of which in isolation are generally well understood, but it is very easy to get lost in the
algebra, especially with the added complication of inequalities.
Part (i) explicitly required induction, so there is a standard procedure to follow – check it
works when 1, assume it works when and show that this leads to it being true
when 1. Only the last part causes any issue. There is some fairly subtle logic: using
the assumption it can be shown that
1
2√ 1
√ 1
The first part of part (ii) is also just about squaring up and showing that the statement is
equivalent to an “obviously” true statement (in this case that 16 24 9 1
16 24 9 ). No induction required! But why are we being asked to do this....?
In the final part we first need to come up with a conjecture. A reasonable place to start is to
try 1, so that
1 2.5
For this to work we need that 1.5 so 1.5 is the smallest value that works for .
However will this work for all subsequent too? It turns out that it does, but that requires
proving the conjecture
1
2√ 1.5
√
This requires induction, following a very similar argument to part (i). One line of the algebra
in the proof requires 4 1 √ 1 4 3 √ , which can be done using the initially
unimportant fact at the beginning of part (ii) – always look out for making links between the
different parts of questions!
16
Question 7
In this question the difficulty is being able to see that some result is “obviously” true but
then having great difficulty in justifying it from particular starting‐points: it is not enough to
make a true statement (especially when it is given in the question) … one must justify it fully
from given, or known, facts and careful deductive reasoning.
Here, in (i), it is known that, for 0 < < 1, is positive and ln is negative. Thus
0 ln ln can be deduced by multiplying the first inequality throughout by a
negative quantity (remembering that this reverses the direction of inequality signs). This is
just ln 1 ln ln and, since the logarithmic function is strictly increasing,
1 f . A more complete argument along similar lines shows that
g f . The final part requires no further justification; since for 1, ln 0
we now have f g .
For part (ii), it is customary to use logs first and then differentiate implicitly.
In (iii), only an informal understanding regarding the justification of limits is expected, but
one still should have a grasp as to how things should be set out. Here, something along the
lines of
_|Å f ( x) = _|Å e = _|Å e = 1
x0 x0
x ln x
x0
0
would be expected.
1
In (iv), the use of calculus is the most straightforward approach, differentiating = ln x
x
for 0 and showing that it has a unique minimum turning point at (1, 1). This is then fed
in to the derivative of g( ) – again using the logarithmic form and implicit differentiation –
along with a simple observation that squares are necessarily non‐negative and this all falls
nicely into place. Most of what is required in order to sketch , f and g( ) has already
been established and all that is left is to put it together in a sensibly‐sized diagram.
17
Question 8
Although vectors expressed in general terms are not handled well by the majority of STEP
candidates, such questions invariably involve little that is of any great difficulty. If one is
sufficiently confident in handling vectors, this question is perhaps the easiest on the paper.
The only things involved in this question are the equations of lines in the standard vector
form and the use of the scalar product for finding angles (in particular the result
that, for non‐zero vectors and , 0 and are perpendicular).
Thus, we have the line through perpendicular to is and the line through
perpendicular to is , which meet when
.
Since is perpendicular to , a b u (a c) = 0 which leads to a scalar expression
for and hence a vector expression for .
Next, , and
CP AB a c u b a a c b a u b a .
Now u (b c) 0 since is perpendicular to u b u c . Substituting this into
CP AB leads very quickly to the required zero for the perpendicularity result required.
a c b a u c a .
[Note: those readers familiar with the common geometric “centres” of triangles will, no
doubt, have spotted that this question is about nothing more than the orthocentre of a
triangle; that is, the point at which the three altitudes meet. In this question, you are given
two altitudes; find their point of intersection, and then show that the line from the third
vertex through this point meets the opposite side at right‐angles.]
18
Question 9
Before doing anything else in a question like this you need to draw a BIG diagram with all
relevant forces labelled.
With nearly all questions like this it is a matter of setting up equations by resolving forces
and taking moments. The main tactical decision is about in which direction to resolve forces
and about which point to take moments. Because the equation we are trying to find in part
(i) does not involve the weight or the normal reaction force of the cylinder and the ground
there is a strong indication that we will be resolving horizontally. Focussing on just one
cylinder we get:
cos sin 1
. .
Part (ii) does bring in the normal reaction with the floor, so resolving vertically will be a
useful tool. For the cylinder this gives:
– cos – sin
where is the weight of the cylinder. Unfortunately, we do not want in our expression,
but we do need to bring in a . The easiest way to do this is to resolve vertically for the
plank:
2 cos 2 sin
Combining these two expressions with the result from part (i) and the fact that cos
sin 1 leads to the required result.
If there is no slipping then . Substituting in the result we have just obtained turns
this into something that can be rearranged into something similar to the last result in part
(i):
19
It is always important when dealing with these algebraic expressions to try to make links.
We can use the final result in part (i) to show that (by multiplying by ):
2 sin 1 cos
But 1 cos must be less than 2 1 cos , therefore as long as the inequality
from (i) is true then inequality (2) will be satisfied.
Sometimes it is important to see the thrust of a question. The first two parts have been
steering you towards the idea that the important inequality is 2 sin 1 cos . Our task
is to now turn this into an inequality involving only sin . It is tempting to subtract 1 from
both sides and square, as that will lead to an inequality involving only sines. However that is
technically flawed: we cannot easily square expressions which are sometimes positive and
sometimes negative [i.e., it is not generally true that leads to ; for instance,
consider 2 1]. It is better to square up the original expression as it is, in the context of
the question, never negative. After a bit of manipulation, it becomes:
0 5 cos 3 cos 1
From this it can be deduced that cos , and in turn a graphical argument leads to the
required result.
This then needs to be related to the physical situation. By finding an appropriate right‐
angled triangle you can show that sin , which leads to the required result.
20
Question 10
With questions involving lots of “show that” work it is particularly important to not simply
write down expressions which are true, but could have come from “working backwards”.
d
The first critical idea is that the work done by the car is F dx where
0
is the force exerted
by the car.
The second critical idea is to use Newton’s 2nd Law:
To obtain the second integral a change of variable is required. This needed a clear
explanation of how to change both the limits and the d .
In part (i) the integral had to be split up to reflect the two parts of the journey. In theory
either the integral with respect to or with respect to could be used, but you might think
that the fact that you were led towards the integral with respect to just above suggests
that it would be the better choice, and this is indeed the case. It was also important to
explain why the condition was needed.
In part (ii) the given condition needs to be interpreted in terms of the speed at which the
force is zero. This needs to be compared to to check that it is achieved. Then some more
integrals similar to part (i) and some algebra leads to the required result.
21
Question 11
As with most mechanics questions, a large clear diagram is very useful. Although not
mentioned in the question, defining the angle of projection is a very good idea in projectile
questions.
Conserving energy provides a fairly standard start to this question. We then needed to
transfer to kinematics to introduce angles. An important decision needs to be made about
where to set the origin. It turns out that the top of the first wall makes a very sensible
choice. Standard kinematic equations can be used to write the vertical and horizontal
displacement when the particle passes over the second wall. Eliminating the time from
these equations and using the result from the first part leads to a familiar looking
trigonometric expression.
To obtain the distance of from the foot of the wall it is useful to find the angle of
projection. To do this it is useful to find something that doesn’t change to form into an
equation; in this case the horizontal component of the velocity. This leads to finding the
cosine of the angle of projection. Using a trigonometric identity can turn it into the sine of
the angle. You can then use a kinematics equation to describe the vertical displacement,
finding a quadratic equation for the time taken to get to the height of the wall. This time can
be used to find the horizontal displacement.
Part (ii) follows a similar pattern to part (i). Energy considerations can be used to find the
speed over the first wall. Then kinematics equations (or more directly the trajectory
equation) can used to form a quadratic equation in the tan of the angle passing over the
first wall if it just passes the second wall. Examining the discriminant (after a fair amount of
algebra) shows that this equation does not have a solution, so the particle cannot pass over
the second wall.
22
Question 12
Part (i) required thinking about the different ways in which the total number of fish caught
could be – for each value of , there is a corresponding value of . This leads to a sum.
Each probability can be written using the formula for the Poisson distribution. It is useful to
have an idea of what you are trying to get to (a Po distribution). Pulling out some
common factors leaves something very close to a binomial expansion of . Artificially
pulling out another factor of leaves exactly the required expansion.
!
Part (ii) starts by turning the situation described into a probability statement, then using the
formula for conditional probability. Substituting in the expressions from the Poisson
distributions and a little algebra leads to a standard binomial expression.
Part (iii) is all about linking with part (ii). When the first fish is caught the total number of
fish caught is one, and you want to know the probability that Adam caught it.
Part (iv) requires some quite subtle thinking. The expected waiting time can be split into the
expected waiting time with Adam first or with Eve first. Some careful thought is required to
realise that, for example, the waiting time with Adam first can be broken down into the time
for a fish to be caught followed by the time for Eve to catch a fish.
Question 13
The first step of part (i) is to find an expression for the probability of getting the correct key
on the th attempt. This can be done from a tree diagram or by using the geometric
distribution. From these probabilities an expression for the expectation can be found which
is strongly related to the binomial expansion suggested.
Part (ii) also needs to start with an expression for the probability of getting the correct key
on the th attempt. This can be found by telescoping expressions from a tree diagram or just
using the symmetry of the situation: each possible selection is equally likely to find the
correct key. An expression can again be found and simplified for the expectation.
Part (iii) : A tree diagram type approach forms a series of telescoping fractions, simplifying
to the given expression. Pulling out a factor of ( 1) from the expression for the
expectation leaves a series of partial fractions which can be written as the difference
between the infinite sum given and a finite sum. The difference between an infinite quantity
and a finite quantity must be infinite.
23
STEP III 2017
Hints and solutions
Question 1
The first result is simply obtained by expanding the bracketed expression on the right‐hand
side using the definition of the binomial coefficients, and then combining the fractions using
the lowest common denominator. ∑ is determined by employing the first result
of the question, finding that the terms telescope and then observing that → ∞ as
→ ∞, to give the answer . The deduced result is obtained by letting 2 in the
previous result and subtracting the first term of the sum in the general result. The first
inequality of (ii) can be obtained by expanding as , observing that and
! ! !
!
rearranging. Similarly, is which is negative as 3 and
so the denominator is positive, leading to the second inequality. The first numerical
inequality in the final result is obtained from the second inequality of part (ii) using the final
result of (i) for the first summed term, the penultimate result of (i) for the second summed
term (adjusting the index over which it is summed), and including the terms for 1 and
2. The second numerical inequality in the final line is obtained from the first inequality
of part (ii), again including the two extra terms and using the final result of (i).
Question 2
(i) is simply obtained by applying e to . Using the result of (i) twice for and for
a rotation about and equating, both sides can be multiplied by e . In the case
2 , 1 e 0, so cannot be found, and then is a translation by
1 e . If , and if 2 π , then
1 e 1 e and so either or if , 2 . If
2 , , 2 , or 2
24
Question 3
Writing down the sum of the three roots of the cubic gives an expression which must be in
the quartic and in the cubic. In the specific case, the cubic equation is
Question 4
Letting log f ,f e (using the result from the formula book) and so,
ln f ln ln log f , which substituted in the geometric mean definition
simplifies rapidly to the required result. Part (ii) can be obtained by substituting for h in
the expression for H and then manipulating using the logarithm of a product. Part (iii)
can be obtained using the result of part (i) with , and then checking separately that
the simple case 1 works. In part (iv), setting the defined expression for the geometric
mean of f equal to f , and taking the logarithm of both expressions yields, after
minor rearrangement, ln f d ln f . Differentiating this with respect to , and
25
Question 5
Converting polar coordinates to Cartesian and differentiating each of and with respect
to gives a fraction which simplifies to once the numerator and
denominator have been divided by cos . Setting the product of two such expressions for f
and g equal to negative 1 and simplifying leads to fg f g sec 0 and hence the
desired result. Substituting for g in the displayed result and solving the resulting first order
differential equation for f (by integrating factor or separation of variables) leads to
f which simplifies by eliminating cos in favour of sin , and hence
using the given point, f 2 1 sin .
Question 6
The appropriate substitution for part (i) was , and having changed variable the
resulting integral has limits and ∞ , which can be expressed as the difference of two
integrals with these as their upper limits and zero as their lower. To obtain in part (ii),
one method is to make (the constant) the subject of the formula and to differentiate with
respect to ; an alternative is to differentiate directly, to multiply numerator and
denominator by 1 , to expand the numerator and then to express it as 1
leading to the desired result. Applying this substitution to the defined T ,
results in an integral which can be expressed again as the difference of two integrals as in
part (i). Taking the result of (i) and rearranging to make T the subject, T can be
substituted for using the result just found and in turn T can be replaced using the result
of (i) with as . The final result of (ii) is achieved by letting , and in that
just found. Throughout, it is important that the conditions expressed as inequalities are
substantiated. To find T √3 in (iii), apply the final result of (ii) letting √3 ,
whereas to find T √2 1 , let √2 1 and 1 in the result
26
Question 7
Showing lies on the ellipse is merely a matter of substituting ’s coordinates into the left‐
hand side of the ellipse equation and simplifying to equal 1. The first result of (i) is
tantamount to finding the equation of the tangent at . Parametric differentiation leads to
giving as which , satisfies, thus
simplifying to the required result. The deduction can be made by requiring that the
discriminant of the quadratic in is positive and geometrically , is a point outside the
ellipse. Relaxing the restriction on the value of , , so the inequality implies 0
and thus there is a vertical tangent and another with one possible configuration as shown.
The first result of (ii) is obtained by considering and to be the roots of the quadratic in ,
and hence being able to write down their product. Similarly, . The final
result is obtained by finding expressions for and in terms of and respectively
(without loss of generality), imposing the condition on and to get an equation in
and , and then using the first two results from this part of the question to substitute for
the product and sum of and .
Question 8
The stem can be achieved by adding the two summations, expanding the brackets, and
observing that the resulting two summed terms telescope. (i) is simply a case of using the
given expression for , and letting 1 (or any constant) in the stem result,
simplifying the left‐hand side using the given note, and dividing through by sin . Part (ii)
can be obtained by letting and sin 1 sin (or similarly
cos ) in the stem which simplifies to give and 1 .
27
Question 9
The first result can be obtained by writing the equation of motion for each particle
separately and then adding the two equations to eliminate the unknown tension; two
integrations with respect to time complete the working once the constants of integration
(both zero) are evaluated. Using the result obtained, at the time (given) with the value of
as , works out to also be . As a consequence, conservation of energy has no elastic
energy term at that instant, merely kinetic energy for each particle and the lost potential
energy of . Combining this equation with that obtained after the first integration in the
initial result of the question gives simultaneous equations for the two speeds at that instant,
and substituting for the speed of gives a quadratic with the desired result as its repeated
root.
Question 10
The first result is obtained by conserving energy for the rod and particle together
(rotational kinetic energy and gravitational potential energy) and simplifying the algebra.
Differentiating that result with respect to time and then simplifying gives 2 3
3 2 cos . Alternatively, the same result can be obtained by taking moments about
an axis through . Resolving perpendicular to the rod for the particle and rearranging the
equation generated yields an expression for the normal reaction, cos ,
having used the previously obtained expression for . This is demonstrably positive under
the given conditions. Resolving along the rod towards (i.e. radially inwards) yields an
expression for the friction which is simplified using the first obtained result of the question,
and then applying the conditions for limiting friction yields the given result. In the case
2 , the particle loses contact immediately as the rod falls away quicker than the particle
accelerates downwards; this can be shown either by considering the equation of rotational
motion for the rod alone about and finding , or by observing from previous
working that the normal reaction of the rod on the particle would need to be negative for
the particle to stay in contact with the rod.
28
Question 11
Conserving linear momentum in part (i) leads to , and using this result leads
directly to the displayed kinetic energy result. Conserving momentum before and after the
gun is fired gives 1
which leads to the required result, and summing that result for 1 to gives, on the
right‐hand side, a sum of terms, each of which can be shown to be less than (or equal to in
one case) , and hence the result. For (iii), considering the energy of the truck and the
1 projectiles before and after the projectile is fired,
1 .
Simplifying this by collecting the terms leads to the
printed result via the difference of two squares factorisation and use of the result from (ii).
Once again, summing as before with telescoping terms leads to the second printed result,
and the final inequality follows using the inequality from (ii) via a little algebraic
simplification; the final step is quite a slack inequality.
Question 12
To obtain the first result of (i), sum in turn over and from 1 to to obtain the total
probability (1!) yields , and then sum over . For independence, it would be
necessary that P , P P and it is possible to simplify the
algebra of this equation having substituted for each probability to see that there are
numerous examples for which this does not hold. Proceeding as at the start of the question
to obtain E and summing over using the printed result of (i) to find
29
Question 13
30
31
Cambridge Assessment Admissions Testing offers a range of tests to support selection and
recruitment for higher education, professional organisations and governments around the world.
Underpinned by robust and rigorous research, our assessments include:
Cambridge Assessment
Admissions Testing
1 Hills Road
Cambridge
CB1 2EU
United Kingdom
©UCLES 2017
STEP MATHEMATICS 1
2018
Hints and Solutions
This document should be read in conjunction with the corresponding mark scheme in order to gain full
benefit from it. Since the complete solutions appear elsewhere, much of this Hints and Solutions document
will concentrate more on the “whys and wherefores” of the solution approach to each question and less on
the technical details.
The solutions that follow, presented either in outline or in full, are by no means the only ones, not even
necessarily the ‘best’ ones. They are simply intended to be the ones that, on the evidence of the marking
process, appear to be the ones which arose most frequently from the ideas produced by the candidates and
that worked for those who could force them through to a conclusion. If you “see” things in a different way, I
hope you can still both follow and appreciate what is given here.
The first question on the first STEP is always intended to have a more familiar feel to it and this is
one such question.
Equating the two expressions for y, for the two given entities to meet, then cancelling the obvious x,
gives a simple quadratic: and you should know of a few ways to tackle it to find the coordinates (don’t
forget the y-coordinate) of both P and Q. The first “curve” is a line through the origin and the second is a
cubic, also through O. The repeated factor in y = x(b – x)2 indicates tangency at x = b so the curve – a
“positive” cubic, in the sense of the sign of the dominant (x3) term – has the classical up-down-up shape,
passes through O and touches the x-axis at b.
Next, “equation of tangent” should suggest that you to differentiate, substitute in the values of x and
y at P, and build up the tangent’s equation in the form required. Remember to show clearly how the answer
is obtained; a lot of candidates lose marks by jumping to the given answer without justifying fully their
arrival. A given answer always requires more detailed working; even if you are perfectly capable of doing
all the working ‘in your head’, the working must be shown in writing.
The areas in the next part of the question can be found by integration but it is important to have a
decently drawn diagram to point you in the right direction; especially as one of the regions in question is a
triangle, which hardly requires the use of calculus (although, on this occasion, incorporating all into a single
integral is just as straightforward).
Finally, now that you have algebraic expressions for S and T, you are asked to establish an
inequality, S > 13 T. Inequalities can often be quite difficult to handle well. However, there is a sensible
“trick” that often works well, and that is to prove an equivalent inequality involving zero: in this case, either
S – 13 T > 0 or 3S – T > 0. Establishing the sign of a single expression (such as 3S – T) is almost always far
more easily accomplished than having to compare a variable LHS with a variable RHS.
In this question, the important thing is to be careful with the directions of the inequalities, and
particularly the sign of anything that you intend to multiply or divide by (variable items are always tricky as
they can be both positive and negative … at different “times”, of course). Remember to say why you are
doing what you are doing clearly; the biggest loss of marks in these sorts of questions invariably comes from
those who write down statements, often correct ones, but the marker is unable to spot why they have been
written down. Referring “invisibly” to a result a dozen lines up the page is not generally sufficient, and the
markers are not required to hunt around and identify the reasons why you might have made such-and-such a
conclusion and give you credit for their own understanding of the written work confronting them: spell it out
clearly for them. If necessary, give each key result a circled number reference so you can cite them later on:
see part (iii) below for an example.
In this question, to begin with, you are reminded of a result commonly referred to as the Change-Of-
Base-Formula. Establishing it is easy provided you realise that the statements x = logb c and b x c are
equivalent. Taking logs to base a for this second statement then gives the opening result.
In (i), all that is needed is to take logs to base 10 with both the given (useable) statement 2 < 10 and
then with the LHS of the given inequality (using the COBF noted above). This gives (essentially) just the
1
two lines of working: 2 < 10 log10 2 < 1 2 log10 < 1 log10 < 12 2
log10 π
1 1 log10 2 log10 5 log10 2 log10 5 log10 10 1
and by the COBF = = = >2.
log 2 π log 5 π log10 π log10 π log10 π log10 π log10 π
Part (ii) is a little more protracted, and this time the COBF needs to use logs to base e (i.e. ln’s).
π ln π ln e 1
log 2 15 log2 – log2 e > 15 5 by the COBF ln > 1 + 15 ln 2
e ln 2 ln 2
and e2 < 8 ln e2 < ln 23 2 ln e < 3 ln 2 ln 2 > 23
so that, putting the two together, ln > 1 + 15 ln 2 > 1 + 15 . 23 = 17
15 , as required.
Notice the way in which it is easiest to keep the direction of the inequalities consistent. This is a technique
we shall continue to adopt in (iii). Taking logs to base 10 (suggested by the presence of log10 2), we have
20 < e3 log10 20 < 3 log10 e log10 10 + log10 2 < 3 log10 e 1 + log10 2 < 3 log10 e
and 2 < 10 log10 < 12 from above
and 3
10 < log10 2
ln e 1
It follows that 13
10 < 1 + log10 2 < 3 log10 e using and so that 13
30 < log10 e = ln 10 < 30
13
ln10 ln10
ln π
and log10 < 1
2 () < 1
2 ln < 1
2 ln 10 < 1
2 . 13
30
= 15
13 .
ln10
This question involves a collection of fairly simple ideas, especially in part (i), but leaves the
candidate to explore the difference between “” and “” lines of reasoning with part (ii). Again, at the
outset, a good diagram can help in getting started:
P(x, y)
R(–a, 0) S(2a, 0)
y y y 2 tan 2 xy a
Noting first that tan = and tan = , if = 2 then tan 2 = = = .
xa 2a x 2a x 1 tan 2 1 y 2
2
( xa )
Since y 0, this all tidies up to the required y 2 3 x 2 a 2 .
2
In (ii), starting from y 3 x a2 2
leads backwards to tan = tan2. The difference now is that this
gives the more general result, when “un-trigging” (think about your quadrants work, or the symmetries and
periodicities of the tan function), that = 2 + n. All that is needed now is to consider which integer values
of n give a viable triangle setting for and (one of which must be the obvious = 2, of course.)
Most of the difficulty inherent in this question is of a technical nature. It should be clear that you
must differentiate and show that (1 – ln x) is a factor of both f(x) and f (x), but finding the correct derivative
will usually involve application of the product, quotient and chain rules.
In (i), to begin with, the limits are an irrelevance, since the integrand is the same in both portions of
the function F. Most of the time, in standard A-level papers, you are given the required substitution; not so
here, although the only really obvious simplifying choice of u = ln t turns out to ‘do the trick’ perfectly well.
The other thing that then turns out to be different from A-level is that, here, it is really important to integrate
t – 1 as ln | t |; however, there is a big push to consider this when you realise that, in the two explicitly stated
regions of 0 < x < 1 and x > 1, the ln function is first negative and then positive. Since all powers of ln t are
even, it turns out that
F(x) = ln| ln x | – (ln x)2 + 14 (ln x)4 + 34
in each of the two regions. Indeed, the even powers, and the modulus, ensure that F = F – 1.
The sketch of the curve is interesting but all the clues are there already. There is a vertical asymptote
of x = 1 (since ln(ln 1) = ln 0 is inadmissible; thereafter, the final result of part (i) actually tells you that the
curve exhibits a sort of reflection symmetry between the two regions. The final key observation is that each
region has a point of inflexion as F crosses the x-axis.
In many ways, as with a lot of STEP questions, the real difficulty here lies in having to put together,
without a great deal (if any) of notification, a number of different mathematical ideas from almost anything
you may have learnt over the years, but to do so in a suitably sophisticated way.
For a starter, you are asked for the most general quartic that leaves a remainder of 1 upon division by … .
The required polynomial is clearly k(x – 1)(x – 2)(x – 3)(x – 4) + 1 (the most common error being to think
the leading coefficient is always 1) and it is helpful to note that k cannot be zero. This leads to the suggested
P(x) = k(x – 1)(x – 2)(x – 3) … (x – N) + 1, k 0, in (ii) and substituting x = N + 1 leads to the required
conclusion that P(N + 1) 1 either by brute force or by a ‘contradiction’ argument that notes that, if this
were the case, then we would have a polynomial of degree N with (N + 1) distinct roots.
For the very last part of (ii), notice that you are only required to find one such r and, after a bit
algebra, it becomes clear that r = 2 does the job; verifying that it does so is rather easy.
The same sorts of ideas then come into play in part (iii), where we have a quartic polynomial
equation, S(x) – 1 = 0, with four distinct integer zeroes a, b, c and d. Setting x = e then gives
(e – a)(e – b)(e – c)(e – d) = 17
and, although 17 does have four distinct integer factors (1, 17), the two 17s cannot both be used. In (b) a
similar consideration of S(0) leads to abcd = 16 so that only the numbers 1, 2, 4, 8, 16 can be used.
There are several possible approaches (three of which appear in the published Mark Scheme) but the most
important things needed to be clear and systematic in your presentation.
This does look horrible (potentially, at least) and, in fact, it can indeed be solved in a very lengthy
way if one is not careful. The opening result is a simple application of the well-known “Addition Formulae”
for cos(A B). Expanding the LHS of the given result so that each term on the LHS is of the form
2sinPsinQ leads to a “collapsing” (or “telescoping”) series where almost all of the terms appear once
negatively and once positively, leaving only the first and last ones standing.
This result is then used in (i), once you have correctly split the area into n equal rectangular strips of
equal bases but heights given by the height of each strip’s midpoint: choosing the value of suitably and
π π
collapsing the series gives the required outcome, An cosec , which I have written in this form for later
n 2n
use.
The Trapezium Rule formula gives a similar result once you have factored in (to both sides, of
π
course) the sin , although a satisfactorily simplified answer requires the use of a final step involving
2n
π π
cos( – ) = –cos in order to end up with Bn cot .
n 2n
The two results of part (iii) now turn out to involve little more than bits of (unprompted) trig. identity
work on the expressions for An and Bn in the above forms.
What is especially pleasing about this question (from a teaching point of view) is that it principally
deals with the simple details of calculus – both differentiation and integration – unencumbered by the
specifics of individual functions (even though the “s” and “c” do look as if they have something to do with
sine and cosine). In this sense, there is something fundamentally “pure” about the application of the various
rules that are being effected.
In (i), one must simply appreciate that a function differentiating to zero is constant and, in which
case, whatever value one substitutes in, that constant will be the output. The required results in (ii) follow
from the use of the product and quotient rules and the identity obtained in (i).
The first result of (iii) follows directly from the given statement c (x) = –s(x)2, requiring only the
most basic grasp of integration as anti-differentiation. For the second result, candidates need to identify how
to split what appears to be the single term, needing to be integrated, into two useable “parts” and, following
a bit of algebra, recognising a “reverse chain rule” integration (which, of course, can always be done with a
suitable substitution).
In a similar vein, part (iv) tests your capacity to think in terms of the key elements of the substitution
–integration process; while (v) then requires you to deploy a mixture of all of the above skills.
A simple diagram can be remarkably effective in enabling you to put your thoughts in order, even if
it consists of little more than this:
S
x H
d
L
Now, in order to reach H from S, the GPE lost from S to L must exceed that gained from L to H (so that
there is a non-negative amount of energy left for a non-negative kinetic energy). Thus
mg x sin mg d sin and x sin d sin .
The rest of the background work for the question can be done using the so-called “constant acceleration
formulae”,
s = ut + 12 at2 , v = u + at , v2 = u2 + 2as and s = 12 (u + v)t ,
with suitably chosen values of the variables.
2x 2x
Using v2 = 2gx sin v = 2gxsin , and then gives t1 = = .
v g sin
Next, gives d = vt2 – 1
2 g sin t22, which is a quadratic equation in t2 (and it is the first, smaller, root
which is required). The bulk of the working for this main part of the question is then taken up in re-forming
T = t1 + t2 into the given answer,
1
g sin 2
T = (1 k ) x k x kd .
2
2
Once this has been obtained, we are required to do the usual sorts of calculus:
d 1 k k2 (1 k ) 2 k4
( RHS ) = 0 when 2
dx 2 x 2 k 2 x kd x k x kd
1 2k k 2 kx d k 3 x k 2k 2 k 3 k 3 x (1 k ) 2 d
(1 k )
2
and x d.
k (1 2k )
A lot of mechanics questions are best approached with a helpful, carefully marked, diagram:
The rest of the question involves the application of N2L (Newton’s Second Law) to various bits of this
system, or the whole thing. The trick is to let what is asked for guide your thoughts to the best “bit” to
choose to apply it to.
To begin with, apply N2L to the whole train: 2D – nR = (2M + nm)a
and then to the first engine (E1): D – T = Ma. Eliminating D gives the required result for (i).
The next result may call for a bit of investigation, but each application of N2L takes time, so it is best
to look at a representative carriage between E1 and E2 first, and then at one after E2 (if necessary). The first
of these gives Tr–1 – Tr = R + ma, for 1 r k, which is positive, so the tensions are decreasing. Then, the
same principle will also apply to the carriages after E2.
Next, use N2L for the final section of the train: U – (n – k)R = (n – k)ma.
Thereafter, we examine the quantity T – U, eliminate the D in the numerator of (i)’s expression for T, and
end up with
T – U = (k – 12 n)(R + ma)
which gives the required result in (ii) that T > U provided k > 1
2 n.
Having seen that all the tensions in the couplings between the components in the front part of the
train are positive, if the suggested outcome does occur, it must be in one of the carriages in the back part of
the train. Try N2L for E2: D + Tk – U = Ma Tk = U + Ma – D = U + Ma – (T + Ma) = U – T < 0 from
the result of (ii).
If the diagrams for the other mechanics questions were of questionable value, in questions such as
this they are an absolute necessity:
B A
R
T F
d T
Q P
mg 2
O mg
In (i), we can reason as follows: as P O, 2 and as P A, so 2.
In (ii), the standard process of resolution of forces comes into play. These give
Res. || plane for P: mg sin + Tcos = F
Res. . plane for P: R + Tsin = mg cos
r
Res. for Q: T = mg
Friction Law (eqlm.): F R
and it follows that R = mg cos – T sin = mg(cos – sin ) by and .
Since R 0 (for P in contact with plane) for all values of , cos sin as 2 ; that is,
cos > 2 sin cos 1 > 2 sin .
For (iii), using F R (for now) with and , mg(sin + cos ) tan .mg(cos – sin )
sin
sin + cos (cos – sin )
cos
sin cos sin cos sin( )
Solving for then leads to = , as required.
cos cos sin sin cos( )
The very final section requires a bit of careful reasoning to arrive at the “corresponding result for
2 ” of sin( ).
and the individual probs. are relatively easy to work out. Then the standard results E(N1) = x.p( x) and
Var(N1) = x .p( x) – (E(N1))
2 2
give p and 2p(1 – p) respectively, as required.
If it helps to put your thoughts in order, a tree diagram can help here – remember just to fill in the
branches that are relevant – and the probabilities for the various numbers of heads that can arise are
x p(x)
2 1
3 ( p1 p 2 p 2 p3 p3 p1 )
1 1
3
p1 (1 p 2 ) p1 (1 p 3 ) p 2 (1 p1 ) p 2 (1 p 3 ) p 3 (1 p1 ) p 3 (1 p 2 )
0 1
3 (1 p1 )(1 p2 ) (1 p2 )(1 p3 ) (1 p3 )(1 p1 )
The application of the same standard results then give
E(N2) = 2p and Var(N2) = 2p – 4p2 + 2
3 p1 p2 p2 p3 p3 p1 .
For (iv), we show that Var(N1) – Var(N2) 0. This boils down to
2p2 – 23 p1 p 2 p 2 p3 p3 p1 0
i.e. 2
9 ( p1 p2 p3 ) 2 – 2
3 p1 p2 p2 p3 p3 p1 0 or 2 ( p1 p2 p3 ) 2 – 6 p1 p 2 p 2 p3 p3 p1 0
which, upon squaring, gives 2 p1 2 p 2 2 p3 – 2 p1 p 2 p2 p3 p3 p1 0. Now this expression is simply
2 2 2
(p1 – p2)2 + (p2 – p3)2 + (p3 – p1)2 and, being the sum of three squares, is guaranteed to be non-negative.
Indeed, it is also immediately clear that equality only occurs when all three squared terms are zero; that is
p1 = p2 = p3.
For Candidate A, if k 2, she can score a maximum of 4 marks so cannot pass. If k = 3, the only way
to pass is by getting them all correct, with probability n13 . For k = 4, she can score 5 marks with 3 correct
answers and 8 marks with 4 correct answers, giving probability 4C3 n13 (1 1n ) + 14 = 4 n3 . Then, finally, if
n n4
k = 5, she can score 7 marks with 4 correct answers and 10 marks with 5 correct answers, so the probability
of passing is 5C4 n14 (1 1n ) + 15 = 5n4 . It now remains to demonstrate that P4 – P3 > 0 and that P4 – P5 > 0 to
n n5
justify that k = 4 is best.
For Candidate B’s strategy, we have a conditional probability:
4n 3
1
P(k 4 & pass) n
4 6
P(k = 4 | pass) = = ,
P(pass) 1 1 1 4n 3 1 5n 4
6 3 6 4 6 5
n n n
where the denominator consists of the terms P(k = 3 & pass), P(k = 4 & pass) and P(k = 5 & pass)
respectively.
In the case of Candidate C, the probability of passing is just
P(3H) P(pass | 3H) + P(4H) P(pass | 4H) + P(5H) P(pass | 5H)
3 4
= 5C3 ( nn1)5 n13 + 5C4 ( nn1)5 4n 3 + 5C n5 5n4 = 25n 9 .
5
n4 ( n 1)5 n5 5( n 1)
The first result can be shown by substituting into the quartic expression.
It then follows for part (i) that the only way to achieve one distinct root is for that root to be either 1
or ‐1. In either case the factorised form of the quartic can then be considered to find the values of
and .
Similarly, for three distinct roots, there must be one pair , along with either 1 or ‐1.
Substitution of 1 or ‐1 into the quartic then leads to the required relationships.
For part (iii) note that the case 2 2 corresponds to the case where there is a root of ‐1 and it
can be seen that it must be a repeated root. The other factor is therefore a quadratic, which can
then be solved.
Finally, the conditions for there to be three roots can be found by considering the discriminant of the
quadratic (and the corresponding one for the other case). It is also necessary to confirm that this
quadratic does not repeat the root of either 1 or ‐1 depending on which case is being considered.
The point with ‐coordinate 1 is a point within the range , (for 0 1), and
1 is the ‐coordinate of a point on the chord joining the points , and
, . Therefore, for any position between the two endpoints on the curve, the inequality is
comparing the point on the curve with the point on the chord joining the two points. The sketch
therefore needs to show the chord entirely below the curve. The final part of the introductory
section can be shown either with a proof by contradiction or by arguing that 0 means that
the gradient is always decreasing within the interval.
Part (i) requires choosing values for and so that the inequality can be applied and then
applying this process multiple times to reach the required result. In each case the choice of and
need to be made so that they lie within the range for which applying the inequality is valid.
Parts (ii) and (iii) both follow from the result of part (i), but it is important to check that the function
being used is concave in the relevant range which needs to be stated clearly. In part (ii) the result
follows immediately, whereas the final part requires some manipulation of logarithms to reach the
final form of the relationship.
The differentiation for the first part of the question can be achieved by applying the chain rule.
Consideration of the sine function within the interval then allows the range to be determined.
Consideration of the gradient function then allows the graph to be sketched ‐ it can be seen from the
symmetry of the sine function that , which means that the graph must have
rotational symmetry about the point where .
A sketch of a rotationally symmetric function is a helpful way of demonstrating the second part as it
allows the distances that must be equal to be identified clearly. It is important to show clearly that
the result works in both the if and only if directions. For the final question in this part, note that the
sections of the graph above the axis must exactly match those below the axis, so the area must be 0.
For part (iii), begin by showing that the equation from part (ii) holds for this function. Once the
rotational symmetry has been demonstrated it follows that the area of any interval with the centre
of rotation in the centre will be equal to the area of a rectangle over the same interval passing
through the centre of rotation.
For the first part, note that two of the terms in the left‐hand side of the equation have a coefficient
of 1 and two have a coefficient of 3. Applying the given identity to each of these pairs gives a
common factor of cos . The equation can therefore be factorised and then another application of
the given identity will allow the full set of roots to be found.
The identity given at the start of the question can be applied to the first two terms of the left‐hand
side of the equation in part (ii) and the double angle formula can be applied to the cos 2 . This then
leads to an equation that can easily be factorised to show the required result. The range of possible
values needs to be considered when considering the case where cos cos .
For the final part a similar process to part (ii) can be used to create a quadratic function of
cos . Completing the square or considering a discriminant then allows the solutions to be
found.
For the first part of the question note that ln 1 can be obtained by integrating 1 and
so the required expansion can be found by integrating the binomial expansion term by term. Note
also, that the integration produces a constant, which needs to be shown to be 0.
In part (ii), the series expansion of can be obtained by adjusting the series expansion of . To
evaluate the integral, substitute the series expansion for the , but leave the unchanged. The
integration can then be completed term by term.
For part (iii) note that a substitution of ln will transform the integral into one that can be
expressed in terms of the integral in part (ii), which then allows the result to follow.
For the first part, note that 5 will certainly be a factor of the left‐hand side in any case where 5.
It then remains to check the other cases one at a time.
For part (ii), first explain how the two theorems show that there will not be any solutions if 7, by
showing that 4 and so there must be a prime factor of the right‐hand side that cannot exist in
the product on the left‐hand side.
The remaining cases then need to be checked one at a time, noting that the individual numbers
within the product can be split into their prime factorisation and then combined differently to form
the right‐hand side.
It is very useful to draw a diagram to represent the situation described at the start of this question.
Defining the vectors and as scalar multiples of and (using two new unknowns) allows the
position vector of Q to be written in two different ways. Since the vectors and are not parallel,
the coefficients of these vectors can be equated and this then leads to the correct expression for .
A similar process then leads to an expression for the position vector of L in terms of and , but
since L lies on OB, the coefficient of must be 0.
Making the substitution given reduces the differential equation into one for which it is easy to
separate the variables. The two sides can then be integrated to find a general solution to the
equation and then the boundary condition can be applied to find the required solution.
The differential equation in part (ii) is similar to the one from part (i), so a similar substitution should
work (using a cube root rather than square root). The same process can then be followed as in part
(i) to solve this differential equation.
In part (iii), the information that can be used to simplify the equations being considered and
then it can be seen that the two curves will both approach an asymptote at 1. We also know
that the curves both pass through the origin and the differential equations show that the curves
should both have gradient 0 at the origin. All that remains is to deduce the relative positions of the
two curves by considering the behaviour of the exponential function.
Since the two particles are released from rest the distance between them will remain constant until
A reaches the ground. The height of B above the ground at this point can therefore be calculated.
Application of the uniform acceleration formulae will therefore give the speeds of A and B at the
moment A hits the ground and the coefficient of restitution can then be used to calculate the speed
with which A rebounds. Since they both continue to move under gravity the speed of B relative to A
will remain constant and therefore the time until they collide can be calculated. Once the time is
known one of the uniform acceleration formulae can then be used to determine the height at which
the collision happens and the two speeds at this time.
For the final part another application of the uniform acceleration formulae can be used to find the
velocity of A immediately after the collision. Conservation of momentum can then be used to find
the velocity of B, although care needs to be taken at this stage with the signs of the terms. Finally,
the velocities before and after the collision can be used to calculate the coefficient of restitution.
Finding an expression for the length of the string at time allows the speed of the point on the string
to be determined. The differential equation can then be set up by adding the speed of the ant to the
speed of the point on the string. The next result can then be verified by applying the quotient rule to
perform the differentiation.
Once the differential equation has been verified, integration leads to a relationship between and ,
which then leads to the required result.
For the journey back, the differential equation needs to be changed so that the speed of the ant is
subtracted rather than added. The differential equation can then be rewritten in a manner similar to
the first part of the question and solved.
A diagram representing the situation will help to ensure that the correct calculations are performed.
In particular it is important to note that the frictional force will be acting in the direction of motion
of the motorbike. Taking moments about the centre of mass as instructed and then setting the
reaction at the front wheel of the motorbike to 0 for the case when the front wheel loses contact
with the ground gives the maximum possible frictional force for this motion. Comparing this to
then gives the first inequality.
When the rear wheel is about to slip the frictional force will be taking its maximum value.
Substituting this into the equation found by taking moments and resolving forces vertically then
allows the value of this frictional force to be found. Newton’s second law then gives the
acceleration.
For the final part, first show that the maximum acceleration is at the moment when the front wheel
would be about to leave the ground. The value of the frictional force at this point can then be found
and the acceleration can then be deduced.
First note that the only winning sequence is heads in a row and the probability of this can be found
easily. The expected winnings can then be expressed as a function of ( ). By considering the value
of it can be shown that the expected winnings increases until , remain the same for the
next case and then decreases thereafter.
For the second part there are multiple sequences that lead to a win. Begin with a sequence of
heads and then consider adding tails to any of the positions before those heads (only 1 tail can be
placed in each position). The number of ways of winning with a total of tails in the sequence can
therefore be seen to be . The sum of these probabilities can then be seen to be a binomial
expansion and can therefore be simplified. An expression for the expected winnings can therefore
be found. The case where 2 leads to a function of the form of the previous part, so the point at
which the maximum value occurs can be written down immediately. Taking logarithms of this
maximum value allows it to be shown that the value is very close to 3 .
The probabilities in the first part of the question can most easily be deduced by using a tree diagram.
For the second part, note that the probabilities at B must be equal to the probabilities at D by the
symmetry of the problem. The sum of the four probabilities for any value of must be equal to 1.
Therefore, it is possible to deduce a recurrence relation for and see that this remains at a
constant value. With the values of and known recurrence relations for and can be
found. These recurrence relations can be related to geometric sequences in order to find the
formula for the general term.
Differentiating and setting equal to zero yields a cubic equation with a single real root, the
quadratic factor being demonstrated to be non‐zero either by completing the square or considering
the discriminant. The stationary point is thus 1, 1 and with the asymptote the sketch
results.
The same strategy for yields a cubic equation with three real roots, two being coincident, and
thus a maximum 2, and a point of inflection 1,3 .
(i) is obtained by differentiating the defined function using the product rule and a little tidying up
of and its differential. The inductive step in the proof of part (ii) can be established by
differentiating the assumed case and then removing the three differentials using the result of part
(i); the base case is established by obtaining 2 and 2 4 from the original
definition. The deduction in (ii) is most simply obtained by eliminating between the result for
just obtained and the similar one for . Part (iii) is obtained by using the deduction of part (ii) to
establish the desired induction, the base case being obtained using the same results used for the
base case in part (ii)’s induction.
ln if , and ln if .
Expressing cot θ as and employing De Moivre’s theorem gives the first result.
Expanding the left‐hand side of the first equation binomially and collecting like terms for the
odd powered terms and dividing by 2i then simplifies to the left‐hand side of the second
result, which then equals zero if 2n 1 θ mπ where m 1, 2, … , n and satisfies the
initial condition. Part (ii) is obtained by considering the sum of roots of the equation just
obtained in (i). The first result of (iii) can be derived from the given small angle inequality
taking a little care with positivity when reciprocating and squaring, and then using the
appropriate Pythagorean trigonometrical identity. Summing the inequalities for m 1 to n,
rearranging to give the required object of the summation and using the value of θ as
defined for part (i), gives bounds of π and π , both of which tend to
the desired limit as n → ∞ .
That sin β can be found by taking moments about O, A, or G (say, the centre of mass
of the combined disc and particle). Applying the cosine rule to triangle OAP and using the
first result obtained leads directly to the first displayed result. The constant expression is
obtained by conserving energy, the first term being the kinetic energy of the disc, the
second term being the kinetic energy of the particle and the last term being the potential
energy of the combined centre of mass relative to a zero energy level defined by the
equilibrium position of G. Differentiating the energy expression with respect to time,
substituting for m, I, sin β , and cos β , the derived equation simplifies to that of an
approximate simple harmonic motion, which with the small angle approximation leads to
SHM with the required period.
Integrating in the usual way to find the expectation in part (iii) gives an expression which is a
multiple of the pdf for the case of n 1 numbers and the variable defined as the k 1 th
smallest, and the two constant expressions cancel to give .
λ it can be shown that this is greater than λ by expressing the hyperbolic functions in
terms of exponential functions.
(9465), 2019
This document should be read in conjunction with the corresponding mark scheme in order to gain full
benefit from it. Since the complete solutions appear elsewhere, much of this Hints and Solutions document
will concentrate more on the “whys and wherefores” of the solution approach to each question and less on
the technical details.
The solutions that follow, presented either in outline or in full, are by no means the only ones, not even
necessarily the ‘best’ ones. They are simply intended to be the ones that, on the evidence of the marking
process, appear to be the ones which arose most frequently from the ideas produced by the candidates and
that worked for those who could force them through to a conclusion. If you “see” things in a different way, I
hope you can still both follow and appreciate what is given here.
Despite the fact that this differs from the usual sort of routine A-level question of this kind – in that
there are no “numbers” involved in the presentation of it – the underlying ideas are, essentially, unchanged.
The opening part contains a line with a given (trigonometric) gradient, which leads to a (right-angled) triangle
with sides of (non-numerical) lengths. The vertices and area of this triangle are thus ‘write downs’ and the
obvious approach in part (i) of using calculus to determine the minimum value of A should be clear to all.
(Now that there is no Formula Book available, it is important that candidates have learnt the various
relationships – or can find them out speedily by hand if not – between the trig. functions and their derivatives,
including the possibilities for deploying various trig. identities along the way, when necessary, in order to tidy
any answers up.)
In Part (ii), the length of the hypotenuse, XY, of this same triangle needs to be determined in some
form and, in principle at least, this is just a GCSE-level use of Pythagoras’ theorem. Finding the perimeter of
the triangle and, again, using calculus to maximise it consists of a set of well-established routines; it is only the
accompanying trig. work that requires a bit of skill, some background learning of the results, and a certain
amount of care.
Since the first curve, C, is given in parametric form, candidates should be guided along the path of
/
using the Chain rule result for parametric differentiation, , rather than reverting to finding its
/
cartesian equation, hence avoiding issues with plus/minus signs that will clutter up the problem.
𝐼𝑡 𝑖𝑠 𝑎𝑙𝑠𝑜 𝑖𝑚𝑝𝑜𝑟𝑡𝑎𝑛𝑡 𝑡𝑜 𝑡ℎ𝑖𝑛𝑘 𝑐𝑎𝑟𝑒𝑓𝑢𝑙𝑙𝑦 𝑎𝑏𝑜𝑢𝑡 𝑡ℎ𝑒 𝑑𝑖𝑓𝑓𝑒𝑟𝑒𝑛𝑐𝑒 𝑏𝑒𝑡𝑤𝑒𝑒𝑛 𝑡 and 𝑝: one is a general
descriptive parameter for the curve while the other represents a particular value of it, even though this, too, is
non-specific.
When finding the intersection, it is useful to simplify the expression using the difference of two cubes:
𝑝 𝑞 ≡ 𝑝 𝑞 𝑝 𝑝𝑞 𝑞 . This is the very useful cousin of the difference-of-two-squares
factorisation. (Students might also like to look up the sum-of-two-cubes, the difference-of-two-fourth-powers,
and similar results, for use in other problems … these are additional results often not flagged up automatically
in STEPs but occurring sufficiently frequently to warrant learning in advance.) Once the intersections are in
the required form, the constraint that the two tangents are perpendicular can now be used. Note the usefulness
of the result that 𝑝 𝑞 𝑝 𝑞 2𝑝𝑞 … another instance of a simple algebraic result that won’t
necessarily be sign-posted but which candidates should have in their ‘toolkit’ if they are to work through STEP
questions fluently, rather than having to keep stopping to do odd bits of working on one side before being able
to continue fluently with the solution to a problem.
There are various ways of finding the intersection of the two curves. One could turn everything into
cartesian form, or just compare the parameterised 𝑥 and 𝑦 coordinates. It is tempting to try to use 𝑢 𝑝 𝑞
again, but that adds an additional constraint which will not necessarily hold at the point of intersection. We
should end up with a cubic (or a disguised cubic) which needs to be factorised by first “spotting” a solution
and then using the factor theorem and polynomial division. (Don’t forget to check which of the solutions are
valid … this is another routine STEP skill)
It should then be seen that one of the solutions is a double root – and this says something very
significant about the intersection point: what? This should help with the final sketch. It is important to try to
make sure that the sketch is consistent with all the information that has been given or found – for example,
what is known about the gradient of C close to 𝑥 0? What about the symmetry of 𝐶 in the 𝑥 axis?
This question has a nice structure: firstly, an explicit “use this method” problem; followed by a “think
about what’s required for yourself, but it should be a similar kind of thing to the first bit” problem; with a
“you’re on your own now” finale.
As with Q1, there’s a lot of different ways to go about these integrations, all governed by how readily
one recognises what one has on the page at any stage of the process and on how easily it can be turned into
something that can be integrated. A mixture of methods can be applied here, from direct integration, to use of
the Chain rule in reverse (often referred to as “recognition” integration), through to the use of any one of a
number of possible substitutions, or possibly even integration-by-parts.
In the first case, working on the integrand in the suggested way leads from
1 1 sin x 1 sin x
to 2
= = sec2x – sec x tan x
1 sin x 1 sin x cos 2 x
and both of these terms in the final expression are directly integrable (being “standard” derivatives of tan x and
sec x respectively).
The obvious approach (using a similar idea to that used for the first integral) to the second integral is to
multiply top and bottom by (1 – sec x), and this leads from
1 1 sec x 1 sec x
to 2
= 2
= cot2x – cosec x cot x
1 sec x 1 sec x tan x
and this is definitely similar to the situation that arose in the first case, but now requires the extra step of
replacing cot2x with (cosec2x – 1) in order to get all terms in a directly integrable form. Alternatively, one
could start the ball rolling here by turning
1 cos x 1 cos x 1 1
into or even 1
1 sec x cos x 1 cos x 1 cos x 1
(some students favour turning all trig. functions into sine and cosine only, and this approach can work as well)
and then multiplying top and bottom by (1 – cos x). In fact, this second approach yields working which ends up
looking (essentially) exactly like the first case, only with the extra manipulation work having been done
upfront.
For the third integral, the most helpful method is not immediately obvious and one may need to be
prepared to explore various ideas before full progress can be made. In fact, the initial idea used in this question
would suggest that the best approach is to multiply top and bottom by (1 – sin x)2, and this turns out to be the
right thing to do, although one must then be prepared to split the result into (possibly) several distinct parts and
work on them separately. (As an aside, it is surprising how helpful it is to do this … almost all candidates who
try to work on “the whole thing” in such cases end up making a mess of things, even if it is only by muddling a
negative sign or missing a factor outside a bracket somewhere along the line.) I would direct the reader to the
Mark Scheme, which shows how this third integral ends up being split (using the method suggested) into a
direct integration, a substitution integration, and an integration-by-parts.
Part (i) is clearly a simple start for later reference – as it doesn’t look as if it turns up very soon from
what can immediately be seen of part (ii) – and you are helpfully directed to consider integers from the outset.
Since it is clearly a case of dealing with small numbers, it really should be possible to spot immediately that
the (positive) square-root of 3 2 2 can only be 1 2 . If this is not immediately clear, then there is
always the direct approach:
expand 𝑚 𝑛√2 to get 𝑚 2𝑛 2𝑚𝑛√2; then compare the two parts with the known
answer (hopefully avoiding a full algebraic method involving solving a quadratic-plus-linear
pair of simultaneous equations, to find that 𝑚 𝑛 1 works.
(But candidates are strongly advised to look out for every opportunity not to spend lots of time doing
unnecessary algebra.)
Part (ii) begins with a little bit of an explanation, and this sort of thing must always be addressed as
completely as possible – very many students tend to gloss over important details when it is important to be
fully convincing.
Thereafter, multiplying out 𝑥 𝑠𝑥 𝑝 𝑥 𝑠𝑥 𝑞 and equating the coefficients with those of
f 𝑥 gives three equations for 𝑝, 𝑞 and 𝑠. These can most helpfully be written in the following forms
(suggested by the given form of the “Show that …” equation in the question):
𝑝 𝑞 𝑠 10 ,
144
𝑝 𝑞 ,
𝑠
4𝑝𝑞 8,
and it is clear that p and q are to be eliminated from this “system” of equations. Noting that the two squared
terms on the left-hand-sides have a useful difference which leads to 𝑝 𝑞 𝑝 𝑞 4𝑝𝑞 and
rearranging then gives the required equation for 𝑠.
Substituting 𝑡 𝑠 produces a cubic equation and the factor theorem provides us with a method for
finding the three possible values of s2 and we can soon find one solution, 𝑡 2. Now take out the
corresponding factor, and solve a quadratic equation for the other two possible values.
If 𝑠 √2 (using the question’s directing hint towards the use of the smallest value of s2) the
usefulness of the opening result should be revealed, and we now have the simultaneous equations for p and q:
𝑝 𝑞 8, 𝑞 𝑝 6√2 … and now the two quadratic equations 𝑥 𝑠𝑥 𝑝 0 and 𝑥 𝑠𝑥 𝑞 0
can be solved separately. Each of the solutions will involve a term of the form 18 12√2, and the fact that
3 2√2 1 √2 from (i), together with a similar expression for 3 2√2, can be used to simplify the
roots.
Once again, there are two opening specific questions which require little more than a knowledge of the
properties of quadrilaterals and the meaning that accompanies the statement that two vectors are equal. As with
Q4, this is for later reference.
The start of part (i) can be approached in so many different ways, depending upon the depth of one’s
knowledge of further vector methods (where the equations of planes, independence of vectors, the vector
product, the scalar triple product, etc. could all be considered). However, for knowledge of vectors at single
maths level only, the obvious tactic is to show that the two diagonals intersect and this can be done using the
standard sorts of method for finding the vector equations of the lines PR and QS and showing that they do
indeed intersect provided the given condition holds.
Part (a) may seem unfamiliar territory, but the question gives all the necessaries, including the
definition of the centroid of a quadrilateral. Note that an “if an only if” proof consists of two parts: the if ()
direction of the argument and the only if () part. Alternatively, in simple cases, it may be clear that any steps
taken in the reasoning are entirely reversible () but this still needs to be stated clearly and not just left to be
invisibly implied.
In (b), once one has now realised that PQRS is a rhombus, one can proceed by working with the
magnitudes of the relevant vectors (i.e. the lengths of the sides of the quadrilateral), and by using the
immediately preceding result, in order to replace q, r and s with p. Then, to wrap things up (with the scalar
product not now required for Paper 1) the Cosine rule can be used in triangle PQR to obtain the displayed
result … thereafter, moving from rhombus to square by setting cosPQR = 0. At the very end, there is a simple
bit of inequality work to demonstrate the final given answer.
From its appearance, one’s initial impression of Q6 is that there’s rather a lot to it … and, in fact, this
does turn out to be the case. Nonetheless, the two parts of (i) are quite straightforward in terms of clearly stated
demands and can be approached by everyone. The process of completing the square is relatively routine, even
though one must be prepared to think of cos as a numerical part of the coefficient of x (etc.) It is possible to
factor out the 9 from the leading terms before commencing the process, but not really necessary, since the first
two terms (to all intents and purposes the constant term, 4, at the end can be dealt with separately) will not
require the use of fractions:
9x2 – 12xcos (3x – 2cos )2 – an adjusting constant.
The final, overall adjusted, constant turns out to be 4 – 4cos2 and it is helpful to replace this immediately by
4sin2 . The purpose of the completed-square form for the original expression is that the minimum value, and
the value of x which gives it, can then be written down without further ado.
The second of these given expressions is a quadratic in (x2) and one could complete the square
(effectively hinted at from the first request) or – since there is now no clear direction as to which method
should be used – find the required maximum (also 4sin2 ) using calculus. The summary result of part (i) now
follows by separating off the two bits previously re-formatted and realising that the minimum of one side can
only hit the maximum of the other – in this case – at the one instant, 4sin2 .
Setting the x’s equal and then turning the resulting equation into a quadratic in (sin x) leads to the
single solution sin x = 12 . Now, this is one of the “standard” exact trig. results which all candidates should
recognise, though it does lead to two values of x in the given interval (right back at the very start of the
question) of 0 < x < . It is then important both to consider the positive and negative square-roots of the values
for x AND to remember that, having squared along the way, there are likely to be “extraneous solutions” that
shouldn’t be there and hence need to be checked for validity at the end.
In part (ii), it soon becomes clear that the given curve has two branches, a -shaped bit and a -
shaped bit. A standard calculus approach (helped by a consideration of what happens as x 0 and as
x ) shows that there is a minimum at (2 , 4 ) and a maximum at (0, 0), as required. Next, in the given
rational expression, it can be quickly noted that the numerator is always less than or equal to 1 while the
denominator is greater than or equal to 1, all of which gives us the desired result. Finally, re-arranging the
second of the initial two established inequalities, and setting it alongside the next one gives
x2 sin 2 cos 2 x
1
4 ( x ) 1 cos 2 sin 2 x
and, with the same notion as appeared in part (i), the two can only be equal when top and bottom both take
their maximum/minimum values respectively.
This is very much a “reasoning” question, where the explanations form a major part of the deal and
there is no point skimming through these and expecting to come out with high marks. Although it is not
essential to have some grasp of the ideas behind modular arithmetic, the notations used are immensely brief
compared to the written word … so, for instance, the statement
x 2 (mod 3)
is actually saying that the number (integer) x leaves a remainder of 2 upon division by 3; as such, this is a
statement being made about an infinite set of numbers which have such a property, without worrying about the
actual whole number part when the division is undertaken (a bit like the Remainder theorem). It is also worth
noting that we can switch between positive and negative “remainders” without any need for additional
explanation; and x 2 (mod 3) is, in fact, exactly the same as saying that x –1 (mod 3) since saying a number
is 2 more than one multiple of 3 is equivalent to saying that it is 1 less than another (the next, in fact).
So, with this basic notation in mind, let’s try to avoid using it as far as possible (though it appears
within the published MS to some extent). To begin with part (i):
then have 10 1, a relationship clearly suggested by the result to which we are working, and
this can then be rearranged to the desired form.
Step 4. Writing 𝑎 3𝑘 and rearranging, 𝑏 90𝑘 𝑏 81𝑘 , which is clearly a multiple of 3. So 𝑏
must also be a multiple of 3.
Step 5. It follows that if 𝑎 is a multiple of 3 then 𝑎 and 𝑏 have 3 as a common factor, contradicting the
assumption of step 2. This is not yet enough to conclude that √2 √3 is irrational, since it is also necessary to
deal with the case when 𝑎 is not a multiple of 3. If 𝑏 is a multiple of 3, then so is 𝑎, by symmetric reasoning, so
the final case is when neither is a multiple of 3. In this case we use Step 1: 𝑎 , 𝑏 and 𝑎 𝑏 are all squares of
numbers which are not multiples of 3, so each of these is 1 more than a multiple of 3. It follows that 𝑎 𝑏 is
2 more than a multiple of 3, but 10𝑎 𝑏 is 1 more than a multiple of 3, a contradiction.
In part (ii), it should be clear that a similar type of working will be employed but there is going to be
at least one significant difference (see the comment required at the very end) so one must be careful to notice
things work slightly differently in this slightly new situation.
As expected, then, this works similarly to part (i), but is more complicated. Numbers which are not
multiples of 5 can have the form 5𝑘 1 or 5𝑘 2. Squaring each of these separately, their squares all have
the form 5𝑚 1, and the fourth powers all have the form 5𝑛 1.
Expanding powers of √6 √7 and proceeding as for Step 3 of part (i) gives the relationship
𝑎 𝑏 26𝑎 𝑏 . If either 𝑎 or 𝑏 is a multiple of 5, a similar argument to part (i) shows that 𝑎 and 𝑏 have a
common factor of 5, contradicting the assumption on 𝑎 and 𝑏. If not, we can use what we know about squares
and fourth powers to argue that the left-hand side is 2 more than a multiple of 5 but the right-hand side is either
1 more than, or 1 less than, a multiple of 5, giving a contradiction.
As a final thought, it is easy to demonstrate that a contradiction purely by considering multiples of 3 is
not possible. This is because 26 is 2 more than a multiple of 3, so if 𝑎 and 𝑏 are each 1 more than a multiple
of 3, both sides of the equation will be 2 more than a multiple of 3.
This question deals with the process of substitution integration itself and how it can be used to show
how things are related functionally.
In order to be entirely comfortable with this, one must first realise the roles being played by the
various letters. At this level, it should be clear that
2 2 2
f ( x) dx = f ( y ) dy = f (t ) dt etc.
1 1 1
since the letter being used within the integrand is irrelevant to whatever is going on … it is called a dummy
variable and is only there as an indicator. (Of course, these letters may contain within them a wider, geometric
or graphical, significance to the whole integral, such as indicating the area between the curve and the x- or y-
axes in the first two cases above; but the result itself – a number – is unchanged by the choice of letter. So,
when one sees the given definition of f(x) in integral form, the upper limit of x is what now guarantees that the
answer is indeed a function of x rather than of t and it has a rather different role.
For (i), the f( 12 x) in the given answer gives the big hint: setting t = 12 u in the original integral should
(and does) do the trick … but it is still very important to show that everything works out properly, such as
demonstrating how the limits change from those for t to those for u.
In (ii), one can either take up where (i) finished … setting v = u – 2 (say) … or go back to the original
integral and set v = 2t – 2, again being careful to show how everything works out.
In part (iii), the first real challenge is presented. What must be done can be seen intuitively, but there is
also the perfectly sensible tactic – also requiring insight – that a linear substitution must be required, but it is
not immediately clear what it is. So, try setting u = at + b and forcing it through, then comparing the outcomes
with what is needed. Doing this reveals that a = 3 and b = 2 do the trick admirably.
Having decided that a linear substitution worked in (iii), the jump now for part (iv) would seem to
require a quadratic substitution. Even something simple like y = u2 would give dy = 2u du in the substitution
working and, at first glance, though it looks as if the numerator of the final integrand is going to cause trouble,
u2 u2
we can write u2 4
du as u2 4
u du and it is now seen that the integrand is now starting to look
X
remarkably similar to the form of (ii)’s integral, namely X 4
dX , which has already been addressed. It
is only the remaining details that must be dealt with, along with the fact that the final answer is not just a single
f(--) thing.
As with many mechanics questions, a sensibly large diagram is important; if not actually essential,
then at least a considerable advantage. In this case, it is best to consider a 2-d vertical “cross-section” through
ladder and box. There are really three bits of physical insight required when setting up this diagram. The first is
that the reaction force between the box and the ladder acts at right angles to the ladder. The second is that, if
the box topples, then it will rotate around the edge diagonally opposite the contact point between the ladder
and the box. The third is that at the point of toppling the normal force from the ground on the box will act
through this tipping point.
Once the diagram has been set up suitably, the question boils down to choosing which points to take
moments about or which direction(s) to resolve in.
For part (i) we want to link the painter’s mass and the reaction force, so taking moments about the
point of contact of the ladder and the ground seems most sensible. Note also, that there is no interest in what
happens to the ladder … taking moments about any other point would require a consideration of what is going
on at the foot of the ladder.
This has not really brought in anything to do with the box toppling, so if we are looking for another
crucial equation we should take moments about one of the base corners of the box. This is what is required in
part (ii). The geometry of this can be quite tricky to deal with – the reaction force is not pointing in a
convenient direction to deal with directly. However, some angle chasing will allow you to resolve it into
vertical and horizontal components.
The final part brings in friction, so we will make use of the fact that the box topples before it slides so
the friction force must not have gone past its maximum possible value. This means that 𝐹 𝜇𝑁. Resolving
vertically and horizontally gives enough information about 𝐹 and 𝑁 to make progress with this expression, but
then it is necessary to use some of the previous parts (as so often in STEP!) and some trigonometric identities
to form the required expression.
Although not essential in this question, it is still a good idea to begin with a clear diagram; especially
since there is a slight twist to the traditional set-up here as the angle given is with the vertical (not the
horizontal), so standard results need to be modified carefully to fit.
Given this slight variation, it is best in part (i) to derive the usual trajectory equation rather than “quote
it”, though there is no harm in the latter approach: remember that t plays no part in this equation so we find a
simple equation involving t, rearrange it and eliminate t in the other equations. Then, substituting in y = h
when x = h tan , and making use of some trig. identities, leads to the required answer.
In part (a), it should be clear that one can use the results for the sum of the roots of a quadratic. This
might also suggest the idea that the product of the roots may be useful later on. We now have lots of
information involving cots so you might think that, in order to show that 1 + 2 = , one needs only to track
back to cot(1 + 2) = cot , and this indeed turns out to be the case. Although the compound angle formula
for cot is not commonly used, it is easily derived from the compound angle formula for tan. There is one
technical issue here, however, which is this: just because tan 𝐴 tan 𝐵 it does not follow that 𝐴 𝐵. For
example, tan 0 tan 180. You will have to come up with an argument about why we can equate the
arguments of the cot function in this case.
Up to this point we’ve not really made use of the fact that there are two solutions. Looking back, it is
seen that the equation (*) is a quadratic in c so there is a common method for determining how many solutions
it has.
In part (ii), an expression for the greatest height is needed, but we must be careful that 𝑢 and 𝛼 are
fixed in the question, so we are looking for the greatest height over the time of flight. This will occur when the
vertical velocity is zero. We can then get the required inequality by comparing this value to the known
achieved height h.
In the situation described in part (i), in each round, a decision is made if the coins are the same way up
(with probability 𝑝 𝑞 ) and the process continues if they are different (with probability 𝑝𝑞 𝑞𝑝 2𝑝𝑞 ).
So the probability that they continue 𝑛 1 times and then make a decision on the 𝑛th round is
2𝑝𝑞 𝑝 𝑞 .
The probability that they don’t make a decision on the first 𝑛 rounds is 2𝑝𝑞 , and so the probability
they do is 1 2𝑝𝑞 . Now 2𝑝𝑞 2𝑝 2𝑝 , and completing the square will show that this is at least ,
giving the required bound.
In part (ii), a decision is made in the first round if all coins are the same; i.e. with probability 𝑝 𝑞 .
In order to make a decision on the second round, there are two possibilities. One possibility is that the first
round has two heads and a tail, the tail is turned over to become a head, and the other two coins are tossed,
both resulting in heads. This has probability 3𝑝 𝑞 . The other possibility is the same with heads and tails
exchanged, so has probability 3𝑝𝑞 .
So we wish to find the minimum value of 𝑝 𝑞 3𝑝 𝑞 3𝑝𝑞 . One way to do this is to write it as
a function of 𝑝 only, using the fact that 𝑞 1 𝑝, and to differentiate. If done correctly, this will give three
turning points at 𝑝 0, 𝑝 and 𝑝 1. By considering the second derivative we find that 𝑝 0 and 𝑝 1
are local maxima, but 𝑝 is a local minimum; so it follows that the minimum value for 0 𝑝 1 is at
𝑝 , and evaluating the function at this point gives .
For the introductory part, first find an equation of the tangent to the curve at the point with 𝑥 𝑎.
An expression can then be found for the 𝑦‐coordinate of the point on the tangent where 𝑥 𝑝 and
this can easily be shown to be equal to 0 if and only if 𝑔 𝑎 0.
In part (i), the first result follows by identifying that 𝑔 𝑥 𝐴 𝑥 𝑞 𝑥 𝑟 allows the first result
to be applied. The gradient of the tangent can be found by differentiating 𝑓 𝑥 and then the fact
that 2𝑎 𝑞 𝑟 can be used to eliminate 𝑎 from this expression.
In part (ii) the tangent at the point where 𝑥 𝑐 is essentially another case of the tangent considered
in part (i), so the gradient of this tangent can be deduced easily. By equating the gradients of the two
tangents it can be deduced that 𝑞 𝑝 𝑟 𝑞 (although care needs to be taken to justify the choice
of square roots). The equation of the tangent at 𝑥 𝑞 can also be found and so any other points of
intersection between this tangent and the curve can be found. The result then follows easily.
From a sketch of the function, it can be seen that the first integral corresponds to an area below the
curve and the second integral corresponds to an area to the left of the curve. These two areas make
a rectangle, whose area is clearly expressed by the expression on the right of the equation.
To evaluate the integral, observe that 𝑔 𝑠 𝑠 𝑠 and apply the result shown at the start of the
question.
For the second part it must be noted that this function does not satisfy the conditions for the initial
result to be applied. However, it can be seen that ℎ 𝑡 𝑔 𝑡 2 .
It therefore follows that ℎ 𝑡 0 and the values of ℎ 0 and ℎ 8 can be deduced. By considering a
sketch of this function it can be seen how to modify the initial result to apply in this case.
The initial result can be shown by considering the four possible combinations of signs for 𝑥 and 𝑥 .
Induction can then be used to prove the more general result.
In part (i)(a) the initial result can then be applied to show that the value of 𝑓 𝑥 1 must be less
than or equal to a polynomial in |𝑥|. Furthermore, the coefficients must also be less than or equal to
𝐴 and so the value must be less than or equal to a sum that can be seen to be a geometric sequence.
In part (i)(b) the previous result can be applied in the case 𝑥 𝜔 and, since |𝜔| 1 it must be the
case that 1 |𝜔| 0. Therefore, the inequality can be multiplied by 1 |𝜔| without changing the
direction of the inequality. The required inequality then follows easily.
To show that the inequalities continue to hold if |𝜔| 1, observe that is a root of the polynomial
𝑔 𝑥 1 𝑎 𝑥 ⋯ 𝑎 𝑥 𝑥 , as 𝑔 𝑓 𝜔 0. Since 𝑔 𝑥 has the same
properties as 𝑓 𝑥 , | |
must also satisfy (*). It then only remains to consider the case |𝜔| 1.
For the final part, observe that division by 135 produces a polynomial that satisfies the conditions
specified and so the bounds on the value of 𝜔 reduces the cases to be considered to 𝜔 1 and
𝜔 2.
In the first part, if the expression to be evaluated is multiplied by sin , then three applications of
the given identity can be used. A similar process can then be used to simplify the first expression in
part (ii). For the sum, note that tan 𝑥 is the derivative of ln cos 𝑥 and so, the result can be
obtained by taking logs of the first result and then differentiating term by term.
For the final part, first change to a finite product (from 𝑘 1 to 𝑘 𝑛) and then take the limit as
𝑛 → ∞. Note however, that the product in part (ii) started at 𝑘 0, so the result in part (ii) needs to
be modified before it can be applied.
In the same way, the sum can be modified to start at 𝑘 0 (where 𝑘 𝑗 2) and then the result of
part (ii) can be applied with 𝑥 .
In part (i) the values of 𝑎 for which the sequence is constant can be found by solving the equation
𝑎 𝑓 𝑎 . The sequence will have period 2 if the equation 𝑎 𝑓 𝑓 𝑎 has a solution that is
different from those for which the sequence is constant. Although the equation 𝑎 𝑓 𝑓 𝑎 is a
quartic, it is clear that the values of 𝑎 for which the sequence is constant will be solutions of this
equation as well. This means that two factors of the quartic are known and so the remaining factor
will be a quadratic. When considering the roots of this quadratic it must also be checked to confirm
that the roots are not repeats of the values that give a constant sequence.
In part (ii), note that there cannot be a solution to the equation 𝑓 𝑎 𝑎 and so it must be the case
that either 𝑓 𝑥 𝑥 for all 𝑥 or 𝑓 𝑥 𝑥 for all 𝑥 (since 𝑓 is a continuous function). It is clear that
𝑓 𝑥 𝑥 for large values of 𝑥.
Since it must be that case that 𝑓 𝑥 𝑥 for all 𝑥 if the sequence is not constant, it must also be the
case that 𝑓 𝑓 𝑥 𝑥 for all 𝑥.
Finally, it can be seen that, in the case where 𝑞 𝑝 the sequence is of the form in part (i) and so it
should be possible to deduce a case in which there is no value of 𝑎 for which the sequence has
period 2, but there is a value of 𝑎 for which the sequence is constant.
In the first part, substituting 𝑦 𝑚𝑥 𝑐 into the differential equation will allow the values of 𝑚 and
𝑐 to be deduced. Since stationary points must satisfy 0, substituting this into the differential
equation shows that stationary points must lie on the given line. It then follows that solution curves
with 𝑘 2 cannot have stationary points as they would have to cross the straight‐line solution that
has already been found.
Given that the relationship between 𝑥 and 𝑦 for any stationary point is known, it is possible to
differentiate the differential equation and evaluate for any stationary point.
Once the substitution provided has been applied, the new differential equation can be solved by
separating the variables and have equations that can be sketched easily.
In the second part, the same approach as part (i) can be used to find the possible sets of values for
𝑚 and 𝑐. The RHS of the differential equation can be considered a function of 𝑦 𝑥 and this allows
it to be factorised. Solving 0 then shows that 𝑥 and 𝑦 must satisfy one of two linear equations
and the sign of can be deduced for points between these two lines.
The graph can then be sketched, remembering that the curve cannot cross the two straight‐line
solutions.
In part (i), taking the scalar product of 𝒂 𝒃 𝒄 with each of the vectors in turn produces a set of
three equations from which it can be deduced that 𝒂 ∙ 𝒃 𝒃 ∙ 𝒄 𝒄 ∙ 𝒂 and that any pair of them
add up to 1. Alternatively, it can be observed that 𝒂 𝒃 ∙ 𝒂 𝒃 𝒄 ∙ 𝒄 .
It can then be shown that the angle between any pair of these vectors is 120° and so a sketch shows
that the triangle must be equilateral.
In part (ii), a similar approach will lead to the given result. Alternatively, the result can be obtained
by observing that 𝒂𝟏 𝒂𝟐 ∙ 𝒂𝟏 𝒂𝟐 𝒂𝟑 𝒂𝟒 ∙ 𝒂𝟑 𝒂𝟒 . For part (a), note that it must
be the case that the angle between any pair of vectors is equal to the angle between the other two
vectors.
For part (b) use the vector 𝒂𝟏 𝒂𝟐 ) to find the length of one side of the tetrahedron. From the fact
that the tetrahedron is regular it can be deduced that 𝒂𝟏 ∙ 𝒂𝟐 𝒂𝟏 ∙ 𝒂𝟑 𝒂𝟏 ∙ 𝒂𝟒 . The side length
can then be calculated.
In part (ii), note that 𝑱 𝑰 and so the value of 𝑓 𝑱 must be either 1 or 1. The second result of
this part follows from application of the property of function 𝑓.
𝑎 𝑏
For part (iii), first show that 𝑓 0 by applying the result of part (ii) and then pre‐multiply
𝑎 𝑏
this matrix by 𝑲 to obtain one in which the second row is a multiple of the first.
For part (iv), note that 𝑷 𝑲 𝑷𝑲 in the case where 𝑘 2. This leads to the fact that 𝑓 𝑷 must
be either 0 or 1. The fact that 𝑷 exists can then be used to show that 𝑓 𝑷 cannot be 0.
In part (i), the position vector of the particle at time 𝑡 can be calculated. The distance 𝑂𝑃 is then the
modulus of this vector. It is easier to differentiate the square of the distance with respect to time
(which is sufficient as this will be increasing if and only if the distance is increasing). The resulting
√ √
expression can be shown to be positive if sin 𝛼 . Similarly, in the case where sin 𝛼 it is
possible to identify a value of 𝑡 for which the distance is certainly decreasing and show that this is
before the moment at which the particle lands.
In part (ii), the vector 𝑄𝑃 can again be calculated and then the distance 𝑃𝑄 found by taking the
modulus. As in part (i) it is simpler to deal with 𝑃𝑄 rather than 𝑃𝑄. In this case, care must be taken
with the inequality to check that both sides are positive before they are squared and used to justify
that the distance is increasing throughout the flight of 𝑃.
A diagram is very useful in this question. First, note that the triangle 𝐴𝐵𝐶 must be isosceles and then
take moments about 𝐴. In the case given in part (i) this then shows that 𝑇 𝑊 and so the string will
break.
In part (ii), resolve the forces vertically to find an expression for the reaction force and then this can
be used to find an expression for the maximum possible value for the frictional force. 𝑊 can then be
eliminated using the equation in part (i) found by taking moments about A. Rearranging then leads
to an expression that can be used to explain the required result.
For the third part, the values of 𝑘 for which breaking and slipping occur can be found from the
answers to part (i). These two values can be used to set up an inequality that must be satisfied in
order for slipping to occur before the string breaks.
In part (i), the numbers of ways of choosing the pairs can be found by checking the numbers of
possible values for 𝑛 for each choice of 𝑛 . A clear list of the possibilities for each case should then
make generalised formulae for the cases 𝑛 2𝑛 1 and 𝑛 2𝑛.
In part (ii), the possible combinations which lead to a triangle match those found in the first part of
𝑁 1
the question. There are possibilities for the shorter two rods if the length of the longest rod
2
is known, so combining this with the answers to part (i) the probability can be calculated for each of
the two cases to be considered.
In part (iii), the probability can be calculated by multiplying the probability in part (ii) for each
possible length of the longest rod by the probability that that length is the longest of the three rods.
Adding all of these together will result in the overall probability that the rods can form a triangle.
Part (i) requires a simple integration to calculate the values of 𝐸 𝑋 and 𝐸 𝑋 . The required result
then follows algebraically.
In part (ii), use integration to find the values of the quartiles and hence the interquartile range.
Square the two values to allow them to be compared with each other.
In part (iii), the binomial expansion should be easy to write down, but note that the 𝑘 1 term
is the term in 𝑥 , not 𝑥 . The lower quartile and median can be evaluated by integration of 𝑓 𝑥 .
To show the inequalities, note that 1 and that each term in the expansion is positive,
so the value must be greater than the sum of the first two terms. Similarly, the 𝑘 1 term of the
expansion can be shown to be greater than , so the result that may be assumed will lead to the
!
other inequality.
(i) Making y the subject of the first equation, it can be substituted in the second to give a second
order differential equation solely in x. Using an auxiliary quadratic equation, this can be solved for x,
and then y can be found from the first equation in terms of the same arbitrary constants, which can
be evaluated using the initial conditions to give 𝑥 𝑒 cos 𝑡 and 𝑦 𝑒 sin 𝑡 . The sketch of y
as a function of t is
Differentiating y and then x with respect to t and in each case equating to zero gives the values of t
for which y is greatest and for which x is least, giving the points , and , so
√ √ √ √
the sketch is
(ii) In this case, the first equation becomes a separable first order differential equation for x and
substituting its solution into the second equation yields a first order differential equation for y which
can be solved using an integrating factor. The arbitrary constants can be evaluated using the initial
conditions to give 𝑥 𝑒 and 𝑦 𝑡𝑒 . In order to sketch the path, x and y should both be
differentiated with respect to t which indicate that the gradient at 1,0 is ‐1 , that there is a
maximum for y at 𝑒 , 𝑒 and that as the curve approaches the origin, it approaches the y axis
tangentially, so the sketch is
(i) Substituting 𝑦 0 (and optionally 𝑥 0) in the expression for 𝑓 𝑥 𝑦 yields two possibilities,
the required 𝑓 0 1 or a second which can be dismissed because 𝑓′ 0 0 . Using (*) and the
expression for 𝑓 𝑥 𝑦 applied for 𝑦 ℎ yields a factor 𝑓 𝑥 and the limiting expression is as
required. Separating variables and applying the initial condition already found yields 𝑓 𝑥 𝑒 .
Part (ii) proceeds similarly finding 𝑔 0 0 , with a second option dismissed this time owing to the
inequality for the modulus of g. The limiting follows that of (i) with a factor this time of 1
𝑔 𝑥 giving 𝑔 𝑥 𝑘 1 𝑔 𝑥 . Separating variables and then either using partial
fractions or using the standard hyperbolic result yields the solution in one of the forms
(i) For 𝑛 1 , it is easy to show that 𝑥 𝑎 is reflexive. For 2 , Vieta’s equations yield that
𝑎 0 and 𝑎 can take any value giving 𝑥 𝑎 𝑥 . For 3 , Vieta’s equations yield 𝑎 𝑎 from
the sum of roots, and as a consequence that 𝑎 0 or 𝑎 1 from the sum of products of pairs of
roots, leading, after using the third equation to the possibilities 𝑥 𝑎 𝑥 and 𝑥 𝑥 𝑥 1.
(ii) From the sum of roots Vieta relation 𝑎 can be eliminated, and the result can then be squared
which with a little careful manipulation of the sum of product of roots two at a time Vieta relation
yields the desired result. For 𝑛 3 completion of the square yields a square on LHS and 1 minus a
sum of squares on RHS. That the coefficients are integers can only yield 𝑎 1 for 𝑎 0 and the
other coefficients for 𝑟 3, … , 𝑛 1 are zero, which establishes a contradiction when considering
the product of all roots.
(iii) As it has been deduced in (ii) that for any reflexive polynomial of degree greater than 3 that
𝑎 0 , such a polynomial can be factorised by x and then it can be carefully argued that the
resulting polynomial is reflexive and so there is an inductive argument. So essentially the solutions
are those found in (i) along with those solutions multiplied by arbitrary positive integer powers of x,
that is 𝑥 𝑎 𝑥 or 𝑥 1 𝑥 1 𝑥 with 𝑟 0, 1, 2, …
Differentiating f(x) and considering behaviour at the origin and for large x gives the following graph.
Using the substitution 𝑦 in the integral I, it can be seen to simplify to the given result if we
choose 1 . The first evaluation uses that result with 𝑐 √2, and √3 , giving a result having
used the noted standard integral of . Making the substitution, , the second evaluation can
√
be shown to be the same as the first. Returning to making the same general substitution (i.e.
𝑦 say) for part (iii),the resulting integral can be seen to be simplified by choice of b and p
(2𝑏 1 and 𝑝 1 ) to simplify to the standard integral noted in the question and hence a
result of .
The stem is achieved by using the direction given in the question, expansion and rearrangement to
obtain the desired result, with a radius r and centre a. To obtain the locus of Q, substituting for z in
terms of w, then multiplying through by 𝑤𝑤 ∗ and dividing by 𝑎𝑎∗ 𝑟 followed by some
∗ ∗
simplification yields 𝑤 ∗ ∗ which represents a circle centre ∗ and radius
∗ . The second result of (i) is obtained by equating the radii and simplifying. Equating the
centres and substituting for the denominator as each of the values derived from the result just
found gives 𝑎 𝑎 ∗ and so the two possibilities for a can be concluded. In the case that is real,
working back to the value that produced it, the radius is smaller than |𝑎| so the circle is centred on
the real axis and does not contain the origin, whereas in the other case, the opposite is true so the
origin is contained. For part (ii), the given result of part (i) still applies by a repetition of the previous
working but with the new substitution, but equating the radii now gives that any a is possible with
|𝑎| √𝑟 1 (or a is zero) so the previous statement regarding a does not apply.
(i) Treating both sides of the given equation as biquadratics and completing the squares, or
subtracting and factorising the differences of squares, yields the line pair 𝑦 𝑥 and the circle
𝑥 𝑦 𝑎 .
Following the instructions in the question for part (ii) (a), the positive discriminant condition for the
biquadratic in x yields a biquadratic inequality for y which readily factorises to give the required
result, bearing in mind that y positive ensures that two of the four linear factors are thus positive
regardless. In part (b), ‘close to the origin’ indicates that only the terms of lowest degree need
consideration, giving 𝑦 (though the negative is discounted), and ‘very far from the origin’,
√
the highest degree terms and 𝑦 𝑥 respectively. For (c) differentiating and setting equal to
zero gives a simple cubic equation in x, which gives 0, √5 , √2 ,1 , √2 ,2 but from (b) not
0, 0 as points where the tangents are parallel to the x axis. Likewise for ‘parallel to the y axis’ with
, but the working of (a) and (b) restricts the possibilities to just the single one 2 , 0 .
Using all the information gleaned, the sketch for (iii) is merely that for (ii)(c) employing symmetry in
both axes, though, in both cases, care should be taken to ensure that the relative gradients near and
from the origin reflect the results of (ii) (b).
Using the convention of labelling ABCD anticlockwise and labelling the midpoint of AB as M, it is
straightforward to find the vector MV as a scalar multiple of a unit vector and from it to deduce the
0 sin 𝛽
required vector perpendicular to AVB as sin 𝛼 . Doing the same for BVC yields 0 and
cos 𝛼 cos 𝛽
the scalar product of these generates the result for (i). Labelling the centre of the base ACD as W,
simple trigonometry enables lengths MW, BM and BW to be expressed in terms of VW, , 𝛽 and 𝜃 ,
then the first result of (ii) is using Pythagoras. Using this result and standard trigonometric
identities, it is possible to proceed to tan 𝜑 , to sec 𝜑, and so to cos 𝜑 ; doing the comparable
steps with 𝛼 and 𝛽 in the expression obtained and using the result of (i) where the product appears
yields the required expression and considering cos 𝛼 cos 𝛽 0 leads to the inequality. For the
deduction, as 𝜃 is acute, the factor 1 cos 𝜃 is positive and so can be cancelled, and also both
1 and cos 𝜃 cos 𝜃 cos 𝜃 . That cos 𝜑 cos 𝜃, given the positivity of both cosines
means the same applies to the unsquared quantities and hence the final result.
The expression for the position of the particle is 𝑎 sin 𝜃 𝑠 𝒊 𝑎 cos 𝜃 𝒋 , obtained by adding
the displacement of the particle relative to the hemisphere to that of the hemisphere, and
differentiating this with respect to time gives the first required result in (i). Conserving the
horizontal linear momentum of the system (i.e. the particle and the hemisphere together) and
rearranging for 𝑠 eliminating the masses in favour of the given variable k obtains the second result.
The deduction is achieved by substituting the second result in the first. Part (ii) is obtained by
conserving energy for the system, using the results for the speed of the hemisphere, the speed of
the particle derived from its velocity obtained in (i), the gravitational potential energy of the particle
and eliminating the masses using k. Part (iii) commences by finding 𝒓 by differentiating 𝒓 from (i),
then appreciating that the particle loses contact when 𝒓 𝑔𝒋 , equating the two expressions and
taking the scalar product of the equation with the suggested vector. Substituting the required result
in the result of (ii) leads to the equation 𝑘 1 cos 𝛼 3 cos 𝛼 2 0 , and the deduction can
be made by taking the term of degree 3 to the other side of the equation, and appreciating that in
doing so it can be seen to be positive.
A good diagram showing Q moving off along the line of centres after the collision and conserving
linear momentum perpendicular to that line obtains the first result of (i). The expression for w can
be found by conserving momentum perpendicular to the original direction of motion of P, or
alternatively in the direction of the line of centres and then substituting for u using the first result;
𝑤 𝑣 . The first result of (ii) can be found by applying Newton’s Law of Impact and then
substituting for w and u using the results of (i). Using this result, expanding using compound angle
formulae, and dividing by cos 𝜃 cos 𝛼 leads to tan 𝜃 having applied trigonometric
identities. To obtain the maximum of tan 𝜃 , it is worth simplifying the working by letting 𝑡 tan 𝛼
√
and differentiating with respect to t. The maximum value is (which occurs when )
√
which can be justified as it is the only stationary value, consideration of when 𝑡 0 , 𝑡 → ∞ and
that tan 𝜃 0 for all t.
To satisfy (i), it is necessary to find the probability that a number, say r, take sand which can be
calculated by summing the product of the Poisson probability that a number of customers arrive and
the binomial probability that r of that number take the offer; factorising out all the terms not
involved in the summation index leaves a sum that can be recognised as an exponential function,
and the result then follows. The mass taken by n customers can be expressed as a GP with first term
𝑘𝑆 and common ratio 1 𝑘 so the case 𝑘 0 needs considering separately, and then the
expectation can be found by summing the product of these masses and the probabilities using the
result of (i); each term naturally splits into two parts giving two exponential sums whose difference
leads to the given result. Using the working from (i) and (ii), if r customers take the sand, the
amount the assistant takes is 𝑆 1 𝑘 and so the probability the assistant takes the golden grain in
that case can be shown to be 𝑘 1 𝑘 . Summing over r the product of Poisson probabilities and
that just found gives the probability the assistant takes the golden grain as 𝑒 . In the case
𝑘 0 , no sand is taken by anyone at all, so the answer is zero as per the formula, and as → 1 , the
only way the assistant can get the grain is if no customer takes the sand so the probability
approaches 𝑒 . To maximise, differentiation with respect to k yields a stationary value when
𝑘 which the given condition ensures is less than 1, and justification that this is a maximum can
be shown by considering the gradient either side of this value or by using the second derivative.
The result required in the stem can be derived by considering that for each subset, each integer can
be an element of it or not leading to the number of possibilities stated. An alternative is to consider
the sum of the number of subsets there are with r elements written as binomial coefficients and
appreciating that this sum is 1 1 . 𝑃 1∈𝐴 for (i) as 1 is equally likely to be or not in the
set 𝐴 . A similar approach for part (ii) yields 𝑃 𝑡 ∈ 𝐴 ∩ 𝐴 for any particular integer t and
from that, the complement extended to all n integers gives the required result. The other two
solutions are similarly 𝑃 𝐴 ∩ 𝐴 ∩ 𝐴 ∅ and 𝑃 𝐴 ∩ 𝐴 ∩ … ∩ 𝐴 ∅ 1 .
𝑃 𝐴 ⊆𝐴 ⊆𝐴 and 𝑃 𝐴 ⊆ 𝐴 ⊆ ⋯ ⊆ 𝐴 .
©UCLES 2020
STEP 2: BRIEF SOLUTIONS
𝑑𝑢 𝑑𝑥
Must include attempt at (or )
𝑑𝑥 𝑑𝑢
1
2u 2
1
x 1 2
2 c
x
1(ii) Let 𝑥 − 2 = 𝑠
1 1
Then
dx 3 ds
3 1 1
x 2 2 x 1 2 s 2 s 3 2
3
Let s =
u 1
1 u 12 3 1
d s du 32 u 2
23 1 2 2 u 1
1 2
s s 3 2 3 u
1 1
3 2
2s x 1 2
3 32 c
s x 2
1(iii) 1 u
Let x =
u
Allow substitution leading to two algebraic factors in the denominator.
0
1 u2 1
dx 2 du
1 1 1 1
u
2 x 1 x 2 2 3 x 2 2 1 1 u 2 3 u 2
1
1
du
1
0 3 2u u 2 2
©UCLES 2020
1
1
du
1
2
0 4 1 u
2
1
1 u
arcsin
2 0
21 61 31
©UCLES 2020
2(i) 1 ky dy kx 1
y dx x
ln y ky kx ln x c
Hence, ln xy k x y c
xy 41 x y x y Ae
2 2 k x y
2 2
C1 is x y x y 2x y
2 2
C2 is x y x y 2x y 4
In both cases, the equation is invariant under (𝑥, 𝑦) ↦ (𝑦, 𝑥), so symmetrical in
𝑦 = 𝑥.
2(ii)
(4,16)
(2,4)
©UCLES 2020
2(iii) Sketches of 𝑦 = 𝑥 2 and 𝑦 = 2𝑥+4
𝑥 2 > 2𝑥+4 only when 𝑥 < −2.
v
Graph: Symmetry about 𝑦 = 𝑥
Graph: Passes through (-1,-1)
Graph: 𝑦 → 0 as 𝑥 → ∞, 𝑦 → −∞ as 𝑥 → 0
©UCLES 2020
3(i) Suppose, ∃𝑘: 2 ≤ 𝑘 ≤ 𝑛 − 1 such that 𝑢𝑘−1 ≥ 𝑢𝑘 , but 𝑢𝑘 < 𝑢𝑘+1
Since all of the terms are positive, these imply that 𝑢𝑘2 < 𝑢𝑘−1 𝑢𝑘+1 , so the sequence
does not have property L.
Therefore, if the sequence has property L, once a value 𝑘 has been reached such that
𝑢𝑘−1 ≥ 𝑢𝑘 , it must be the case that all subsequent terms also have that property (which
is the given definition of unimodality).
3(ii) ur ur 1 ur 1 u r 2 , so ur ur 1 r 2 u2 u 1
ur2 ur 1 u r 1 ur2 ur 1 2 ur 2 u r 1 2
ur ur 1 for r ⩾ 2
The first identity shows that 𝑢𝑟 > 0 for all r if 𝑢2 > 𝑎𝑢1 > 0.
Since the right hand side of the second identity is always non-negative, the sequence has
property L, and is hence unimodal.
3(iii) 𝑢1 = (2 − 1)𝛼 1−1 + 2(1 − 1)𝛼 1−2 = 1, which is correct.
𝑢2 = (2 − 2)𝛼 2−1 + 2(2 − 1)𝛼 2−2 = 2, which is correct.
Suppose that:
𝑢𝑘−2 = (4 − 𝑘)𝛼 𝑘−3 + 2(𝑘 − 3)𝛼 𝑘−4 , and
𝑢𝑘−1 = (3 − 𝑘)𝛼 𝑘−2 + 2(𝑘 − 2)𝛼 𝑘−3 .
𝑢𝑘 = 2𝛼 ((3 − 𝑘)𝛼 𝑘−2 + 2(𝑘 − 2)𝛼 𝑘−3 ) − 𝛼 2 ((4 − 𝑘)𝛼 𝑘−3 + 2(𝑘 − 3)𝛼 𝑘−4 )
= 𝛼 𝑘−1 (6 − 2𝑘 − 4 + 𝑘) + 𝛼 𝑘−2 (4𝑘 − 8 − 2𝑘 + 6)
= 𝛼 𝑘−1 (2 − 𝑘) + 2𝛼 𝑘−2 (𝑘 − 1)
which is the correct expression for 𝑢𝑘
Hence, by induction ur = (2 − r)αr−1 + 2(r − 1)αr−2
ur u r 1 2 r r 1 2 r 1 r 2 1 r r 2r r 1
r 2 2 r 1 2 3r (r 1) 2
2N r 1 2 3r N N 1 (r 1) N 12
r 2 2
2
N
r 2
N2
(r 1) rN N 2
r 2 N 1
when r = N, uN u N 1 0
N2
2 r 2
when r = N – 1, uN 1 u N 0
N2
so ur is largest when r = N
©UCLES 2020
4(i) The straight line distance between two points must be less than the length of any other
rectilinear path between the points.
4(ii)
𝑎
𝑐
Diagram showing two circles and straight line joining their centres.
Length of line and radii of circles are 𝑎, 𝑏 and 𝑐 in some order.
Either statement that the straight line is the longest of the lengths, or explanation that one
circle cannot be contained inside the other.
Explanation that the circles must meet.
4(iii) (A)
If 𝑎 + 𝑏 > 𝑐 then (𝑎 + 1) + (𝑏 + 1) > 𝑐 + 2 > 𝑐 + 1 et cycl., so 𝑎 + 1, 𝑏 + 1, 𝑐 + 1 can
always form the sides of a triangle.
(B)
If 𝑎 = 𝑏 = 𝑐 = 1 we have 1, 1, 1 which can form the sides of a triangle.
1
If 𝑎 = 1, 𝑏 = 𝑐 = 2 we have , 1, 2 which cannot form the sides of a triangle.
2
𝑎 𝑏 𝑐
Therefore, , , can sometimes, but not always form the sides of a triangle.
𝑏 𝑐 𝑎
(C)
If 𝑝 ≥ 𝑞 ≥ 𝑟 then |𝑝 − 𝑞| + |𝑞 − 𝑟| = 𝑝 − 𝑞 + 𝑞 − 𝑟 = 𝑝 − 𝑞 = |𝑝 − 𝑟|
So two of |𝑝 − 𝑞|, |𝑞 − 𝑟|, |𝑝 − 𝑟| will always sum to the third, so they never form the
sides of a triangle.
(D)
If 𝑎 + 𝑏 > 𝑐 then 𝑎2 + 𝑏𝑐 + 𝑏 2 + 𝑐𝑎 = 𝑎2 + 𝑏 2 − 2𝑎𝑏 + 𝑐(𝑎 + 𝑏) + 2𝑎𝑏
= (𝑎 − 𝑏)2 + 𝑐(𝑎 + 𝑏) + 2𝑎𝑏 > 𝑐 2 + 𝑎𝑏 et cycl.
so 𝑎2 + 𝑏𝑐, 𝑏 2 + 𝑐𝑎, 𝑐 2 + 𝑎𝑏 can always form the sides of a triangle.
4(iv) Since 𝑎 + 𝑏 > 𝑎 𝑎𝑛𝑑 𝑏,
𝑓(𝑎)
>
𝑓(𝑎+𝑏)
and
𝑓(𝑏)
>
𝑓(𝑎+𝑏)
𝑎 𝑎+𝑏 𝑏 𝑎+𝑏
©UCLES 2020
5(i) 𝑥 − 𝑞(𝑥) = ∑𝑛−1 𝑟 𝑛−1 𝑛−1 𝑟
𝑟=0 𝑎𝑟 × 10 − ∑𝑟=0 𝑎𝑟 = ∑𝑟=0 𝑎𝑟 × (10 − 1)
(𝑥 − 𝑞(𝑥)) and (𝑞(𝑥) − 𝑞(𝑞(𝑥))) are both divisible by 9 (by part (i))
and so 𝑥 is divisible by 9
𝑥 = 107𝑞(𝑞(𝑥)) and so is divisible by 107, and so is divisible by 963.
So 𝑥 = 963𝑘 for some 𝑘.
If 𝑥 has 𝑛 digits, then 𝑞(𝑥) ≤ 9𝑛. By (i), 𝑞(𝑞(𝑥)) ≤ 𝑞(𝑥) ≤ 9𝑛.
So 𝑥 ≤ 963𝑛 and 𝑥 ≥ 10𝑛−1 which implies that 𝑛 ≤ 4 and so 𝑘 ≤ 4
Checking: Only 𝑘 = 1 works.
©UCLES 2020
6(i) a 2 bc b a d
a b 2
Let M ; then M
c a d d bc
c d 2
Thus M2 = I ⇔ = 0 and = ∓1
or b = c = 0 and a2 = d2 = ±1
If 𝑏 = 𝑐 = 0 and 𝑎 = 𝑑 = ±1 , then M = I
If 𝑏 = 𝑐 = 0 and 𝑎 = −𝑑 = ±1, then = 0 and = –1
Thus M2 = +I ⇔ = 0 and = –1.
Thus M2 = –I ⇔ = 0 and = +1.
6(iii) Part (ii) implies Det(M2) = – 1, if M4 = I, but M2 ≠ I .
However, Det(M2) = Det(M)2 , so this is impossible.
Clearly M2 = I ⇒ M4 = I
Part (ii) implies that M4 = – I ⇔ Tr(M2) = 0 and Det(M2) = 1
so ⇔ Tr(M) = 2
and Det(M) = 1.
Any example, for instance a matrix satisfying the conditions for any of M2 = I, M2 = –I,
M4 = –I, which is not a rotation or reflection.
©UCLES 2020
7(i) 1 ti
2
1 ti 1 ti 1
|w – 1| =2
, which is independent of 𝑡.
1 ti 1 ti 1 ti
Points on the line 𝑅𝑒(𝑧) = 3 have the form 𝑧 = 3 + 𝑡𝑖 and
the points satisfying |𝑤 − 1| = 1 lie on a circle with centre 1.
If z = p + ti, then
2
2 p 2 c
2
2 2 p 2 c cti c 2t 2
w c
p 2 ti p 2 2 t 2
which is independent of t when 2 p 2 c c 2 p 2
2 2
1
which is when c .
p2
1 1
Thus the circle has centre at and radius .
p2 p2
2 2 p 2 2ti
w ,
p 2 ti p 2 2 t 2
so Im(w) > 0 when t < 0; that is, for those z on V with negative imaginary part.
7(ii) If z = t + qi then
2 2 2
2 2 cq t 2 ci c 2 t 2 cq 2
w ci =
t 2 qi t 2 2 q 2
2
which is independent of t when cq 2 c 2q 2
1
which is when c
q
1 1
so the circle has centre i A1 and radius c2 A1.
q q
2 2 t 2 2qi
w ,
t 2 qi t 2 2 q 2
so Re(w) > 0 when t > 2; that is, for those z on H with real part greater than 2.
©UCLES 2020
8(i)
𝑎
𝑏 𝑐
Graph: Zeroes at 𝑥 = 0, 𝑐 and one other point (ℎ: label not required) in (𝑎, 𝑏).
Graph: Turning points at 𝑥 = 0, 𝑎, 𝑏, 𝑐.
Graph: Quintic shape with curve below axis in (0, ℎ) and above axis in (ℎ, 𝑐)
The area conditions give 𝐹(0) = 𝐹(𝑐) = 0.
𝐹 ′ (𝑥) = 𝑓(𝑥), so 𝐹 ′ (0) = 𝐹 ′ (𝑎) = 𝐹 ′ (𝑏) = 𝐹 ′ (𝑐) = 0
Since 𝑓 is a quartic and the coefficient of 𝑥 4 is 1,
1
𝐹 must be a quintic and the coefficient of 𝑥 5 is .
5
𝐹(0) = 𝐹 ′ (0) = 0 and 𝐹(𝑐) = 𝐹 ′ (𝑐) = 0, so 𝐹 must have double roots at 𝑥 = 0 and 𝑐.
So 𝐹(𝑥) must have the given form.
[Explanation must be clear that the double roots are deduced from the fact that
F(x) = F ′ (x) = 0 at those points.]
1
𝐹(𝑥) + 𝐹(𝑐 − 𝑥) = 𝑥 2 (𝑥 − 𝑐)2 [(𝑥 − ℎ) + (𝑐 − 𝑥 − ℎ)]
5
1
= 𝑥 2 (𝑐 − 𝑥)2 (𝑐 − 2ℎ)
5
8(ii) Let 𝐴 be the (positive) area enclosed by the curve between 0 and 𝑎.
The maximum turning point of 𝐹(𝑥) occurs at 𝑥 = 𝑏, with 𝐹(𝑏) = 𝐴.
The minimum turning point of 𝐹(𝑥) occurs at 𝑥 = 𝑎, with 𝐹(𝑎) = −𝐴.
Therefore 𝐹(𝑥) ≥ −𝐴, with equality iff 𝑥 = 𝑎.
So 𝐹(𝑏) + 𝐹(𝑥) ≥ 0, with equality iff 𝑥 = 𝑎.
𝐹(𝑎) + 𝐹(𝑥) ≤ 0, with equality iff 𝑥 = 𝑏.
1
Since 𝐹(𝑏) + 𝐹(𝑐 − 𝑏) = 𝑏 2 (𝑐 − 𝑏)2 (𝑐 − 2ℎ),
5
either 𝑐 > 2ℎ, or 𝑐 = 2ℎ and 𝑐 − 𝑏 = 𝑎.
1
Also, 𝐹(𝑎) + 𝐹(𝑐 − 𝑎) = 𝑎2 (𝑐 − 𝑎)2 (𝑐 − 2ℎ), so
5
either 𝑐 < 2ℎ, or 𝑐 = 2ℎ and 𝑐 − 𝑎 = 𝑏.
Thus 𝑐 = 𝑎 + 𝑏 and 𝑐 = 2ℎ.
8(iii) 𝐹(𝑥) =
1
𝑥 2 (𝑥 − 𝑐)2 (2𝑥 − 𝑐)
10
1
So 𝑓(𝑥) = 𝑥(𝑥 − 𝑐)(5𝑥 2 − 5𝑥𝑐 + 𝑐 2 )
5
©UCLES 2020
The roots of the quadratic factor must be 𝑎 and 𝑏.
1
𝑓(𝑥) = (5𝑥 4 − 10𝑐𝑥 3 + 6𝑐 2 𝑥 2 − 𝑐 3 𝑥)
5
′ (𝑥) 1
𝑓 = (20𝑥 3 − 30𝑐𝑥 2 + 12𝑐 2 )
5
1 12
𝑓 ′′ (𝑥) = (60𝑥 2 − 60𝑐𝑥 + 12𝑐 2 ) = (5𝑥 2 − 5𝑐𝑥 + 𝑐 2 )
5 5
Therefore 𝑓 ′′ (𝑥) = 0 at 𝑥 = 𝑎 and 𝑥 = 𝑏 and so (𝑎, 0) and (𝑏, 0) are points of inflection.
©UCLES 2020
9 If the particles collide at time 𝑡:
𝑉𝑡 + 𝑈𝑡 cos 𝜃 = 𝑑, and
1 1
ℎ − 𝑔𝑡 2 = 𝑈𝑡 sin 𝜃 − 𝑔𝑡 2 (or ℎ = 𝑈𝑡 sin 𝜃)
2 2
The vertical velocity of the particle fired from 𝐵 at the point of collision is:
𝑔ℎ
𝑈 sin 𝜃 − 𝑔𝑡 = 𝑈 sin 𝜃 −
𝑈 sin 𝜃
𝑔ℎ
𝑈 sin 𝜃 − > 0 iff 𝑈 2 sin2 𝜃 > 𝑔ℎ
𝑈 sin 𝜃
©UCLES 2020
10(i)
H
R 𝑇
L P
2𝜃
𝑚𝑔
Resolving tangentially:
𝑇 sin 𝛼 − 𝑚𝑔 sin 2𝛼 = 0
𝜆
Therefore sin 𝛼 ( (2𝑎 cos 𝛼 − 𝑙) − 2𝑚𝑔 cos 𝛼) = 0
𝑙
10(ii) Energy:
1 𝜆 1 𝜆
𝑚𝑣 2 − 𝑚𝑔𝑎 cos 2𝜃 + (2𝑎 cos 𝜃 − 𝑙)2 = 𝑚𝑢2 − 𝑚𝑔𝑎 + (2𝑎 − 𝑙)2
2 2𝑙 2 2𝑙
©UCLES 2020
11(i) If the game has not ended after 2𝑛 turns, then the sequence has either been 𝑛 repetitions
of 𝐻𝑇 or 𝑛 repetitions of 𝑇𝐻.
So 𝑃(𝐺𝑎𝑚𝑒 ℎ𝑎𝑠 𝑛𝑜𝑡 𝑓𝑖𝑛𝑖𝑠ℎ𝑒𝑑 𝑎𝑓𝑡𝑒𝑟 2𝑛 𝑡𝑢𝑟𝑛𝑠) = 2(𝑝𝑞)𝑛 .
So the probability that the game never ends is lim𝑛→∞ 2(𝑝𝑞)𝑛 = 0.
Sequence that follows the first 𝐻 will be 𝑘 repetitions of 𝑇𝐻, followed by 𝐻, where 𝑘 ≥ 0.
𝑝
So 𝑃(𝐴 𝑤𝑖𝑛𝑠|𝑓𝑖𝑟𝑠𝑡 𝑡𝑜𝑠𝑠 𝑖𝑠 𝐻) = ∑∞ 𝑘
𝑘=0(𝑝𝑞) 𝑝 = 1−𝑝𝑞
𝑝
𝑃(𝐴 𝑤𝑖𝑛𝑠 ∩ 𝑓𝑖𝑟𝑠𝑡 𝑡𝑜𝑠𝑠 𝑖𝑠 𝐻) = 𝑝 ×
1−𝑝𝑞
𝑝2 𝑞
𝑃(𝐴 𝑤𝑖𝑛𝑠 ∩ 𝑓𝑖𝑟𝑠𝑡 𝑡𝑜𝑠𝑠 𝑖𝑠 𝑇) =
1−𝑝𝑞
𝑝2 (1+𝑞)
Therefore 𝑃(𝐴 𝑤𝑖𝑛𝑠) =
1−𝑝𝑞
©UCLES 2020
Similarly, following first toss of 𝑇:
𝐴 wins with (𝐻 followed by any sequence where A wins after first toss was H)
or (𝑇𝐻 followed by any sequence where A wins after first toss was 𝐻)
Therefore:
𝑃(A wins | the first toss is a tail) = (𝑝 + 𝑝𝑞)𝑃(A wins | the first toss is a head)
So
𝑃(𝐴 | 𝐻 𝑓𝑖𝑟𝑠𝑡) = 𝑝2 + (𝑞 + 𝑝𝑞)(𝑝 + 𝑝𝑞)𝑃(𝐴 | 𝐻 𝑓𝑖𝑟𝑠𝑡)
𝑝2
𝑃(𝐴 | 𝐻 𝑓𝑖𝑟𝑠𝑡) =
1−(𝑝+𝑝𝑞)(𝑞+𝑝𝑞)
And
𝑃(𝐴 | 𝑇 𝑓𝑖𝑟𝑠𝑡) = (𝑝 + 𝑝𝑞)(𝑝2 + (𝑞 + 𝑝𝑞)𝑃(𝐴 | 𝑇 𝑓𝑖𝑟𝑠𝑡))
𝑝2 (𝑝+𝑝𝑞)
𝑃(𝐴 | 𝑇 𝑓𝑖𝑟𝑠𝑡) =
1−(𝑝+𝑝𝑞)(𝑞+𝑝𝑞)
So
𝑝2 𝑝2 (𝑝+𝑝𝑞) 𝑝2 (1−𝑞 3 )
𝑃(𝐴 𝑤𝑖𝑛𝑠) = 𝑝 × +𝑞× =
1−(𝑝+𝑝𝑞)(𝑞+𝑝𝑞) 1−(𝑝+𝑝𝑞)(𝑞+𝑝𝑞) 1−(1−𝑝2 )(1−𝑞 2 )
Therefore:
𝑝𝑎−1 (1−𝑞 𝑏 )
𝑃(𝑊) =
1−(1−𝑝𝑎−1 )(1−𝑞 𝑏−1 )
If 𝑎 = 𝑏 = 2,
𝑝(1−𝑞 2 ) 𝑝2 (1+𝑞)
𝑃(𝑊) = = as expected.
1−(1−𝑝)(1−𝑞) 1−𝑝𝑞
©UCLES 2020
12(i) For the biased die:
1 2
𝑃(𝑅1 = 𝑅2 ) = ∑𝑛𝑖=1 ( + 𝜀𝑖 )
𝑛
1 2
𝑃(𝑅1 = 𝑅2 ) = ∑𝑛𝑖=1 1 + ∑𝑛𝑖=1 𝜀𝑖 + ∑𝑛𝑖=1 𝜀𝑖 2
𝑛2 𝑛
∑𝑛𝑖=1 𝜀𝑖 = 0 , so
1
𝑃(𝑅1 = 𝑅2 ) = + ∑𝑛𝑖=1 𝜀𝑖 2
𝑛
1
For a fair die, 𝑃(𝑅1 = 𝑅2 ) = and ∑𝑛𝑖=1 𝜀𝑖 2 > 0, so it is more likely with the biased die.
𝑛
(ii) 1
𝑃(𝑅1 > 𝑅2 ) = (1 − 𝑃(𝑅1 = 𝑅2 ))
2
Therefore, the value of 𝑃(𝑅1 > 𝑅2 ) if the die is possibly biased is ≤ 𝑃(𝑅1 > 𝑅2 ) if the die
is fair.
𝑥
Let T= ∑𝑛𝑟=1 𝑥𝑟 and, for each 𝑖, let 𝑝𝑖 = 𝑖
𝑇
Then ∑𝑛𝑖=1 𝑝𝑖 = 1, so we can construct a biased 𝑛-sided die with 𝑃(𝑋 = 𝑖) = 𝑝𝑖
𝑃(𝑅1 > 𝑅2 ) = ∑𝑛𝑖=2 ∑𝑖−1
𝑗=1 𝑝𝑖 𝑝𝑗
Therefore
𝑥𝑖 𝑥𝑗 𝑛−1
∑𝑛𝑖=2 ∑𝑖−1
𝑗=1 2 ≤ and so
𝑇 2𝑛
𝑛 𝑖−1 𝑛 2
𝑛−1
∑ ∑ 𝑥𝑖 𝑥𝑗 ≤ (∑ 𝑥𝑖 )
2𝑛
𝑖=2 𝑗=1 𝑖=1
Therefore
𝑛 𝑛 𝑛
3𝜀𝑖 3𝜀𝑖2
𝑃(𝑅1 = 𝑅2 = 𝑅3 𝑏𝑖𝑎𝑠𝑒𝑑) − 𝑃(𝑅1 = 𝑅2 = 𝑅3 𝑓𝑎𝑖𝑟) = ∑ 2 + ∑ + ∑ 𝜀𝑖 3
𝑛 𝑛
𝑖=1 𝑖=1 𝑖=1
2
3𝜀
= ∑𝑛𝑖=1 𝑖 + ∑𝑛𝑖=1 𝜀𝑖 3 (since ∑𝑛𝑖=1 𝜀𝑖 = 0)
𝑛
3𝜀𝑖2 3
= ∑𝑛𝑖=1 + 𝜀𝑖3 = ∑𝑛𝑖=1 𝜀𝑖2 ( + 𝜀𝑖 )
𝑛 𝑛
1
But 𝜀𝑖 ≥ − (since 𝑝𝑖 ≥ 0), so this sum must be positive.
𝑛
©UCLES 2020
Cambridge Assessment Admissions Testing offers a range of tests to support selection and
recruitment for higher education, professional organisations and governments around the world.
Underpinned by robust and rigorous research, our assessments include:
Cambridge Assessment
Admissions Testing
The Triangle Building
Shaftesbury Road
Cambridge
CB2 8EA
United Kingdom
www.admissionstestingservice.org/help
©UCLES 2020
STEP MATHEMATICS 3
2020
Worked Solutions
©UCLES 2020 17
STEP 3: BRIEF SOLUTIONS
𝑢 = cos 𝑎 𝑥 𝑣 ′ = cos 𝑏𝑥
1
𝑢′ = −𝑎 cos 𝑎−1 𝑥 sin 𝑥 𝑣 = sin 𝑏𝑥
𝑏
𝜋 𝜋
1 2 1
𝐼(𝑎, 𝑏) = [cos 𝑎 𝑥 sin 𝑏𝑥] − ∫02 −𝑎 cos 𝑎−1 𝑥 sin 𝑥 sin 𝑏𝑥 𝑑𝑥
𝑏 0 𝑏
𝜋 𝜋
2 2
1 𝑎
= 0 + ∫ 𝑎 cos 𝑎−1 𝑥 sin 𝑥 sin 𝑏𝑥 𝑑𝑥 = ∫ cos 𝑎−1 𝑥 sin 𝑥 sin 𝑏𝑥 𝑑𝑥
𝑏 𝑏
0 0
𝜋
𝑎
So 𝐼(𝑎, 𝑏) =
𝑏
∫02 cos 𝑎−1 𝑥 (cos(𝑏 − 1)𝑥 − cos 𝑏𝑥 cos 𝑥) 𝑑𝑥
𝑎
= [𝐼(𝑎 − 1, 𝑏 − 1) − 𝐼(𝑎, 𝑏)]
𝑏
𝑎
Thus 𝐼(𝑎, 𝑏) = 𝐼(𝑎 − 1, 𝑏 − 1) as required.
𝑎+𝑏
(ii) Suppose
2𝑘 𝑘! (2𝑚)! (𝑘 + 𝑚)!
𝐼(𝑘, 𝑘 + 2𝑚 + 1) = (−1)𝑚
𝑚! (2𝑘 + 2𝑚 + 1)!
Then by (i),
𝑘+1
𝐼(𝑘 + 1, 𝑘 + 2𝑚 + 2) = 𝐼(𝑘, 𝑘 + 2𝑚 + 1)
2𝑘 + 2𝑚 + 3
©UCLES 2020 18
𝜋
2
𝜋
1
𝐼(0, 2𝑚 + 1) = ∫ cos(2𝑚 + 1)𝑥 𝑑𝑥 = [sin(2𝑚 + 1)𝑥 ]02
2𝑚 + 1
0
1 −1 1
= if m is even or = if m is odd , or alternatively (−1)𝑚
2𝑚+1 2𝑚+1 2𝑚+1
If 𝑛 = 0 ,
2𝑛 𝑛! (2𝑚)! (𝑛 + 𝑚)! (2𝑚)! (𝑚)! 1
(−1)𝑚 = (−1)𝑚 = (−1)𝑚
𝑚! (2𝑛 + 2𝑚 + 1)! 𝑚! (2𝑚 + 1)! 2𝑚 + 1
𝑎
2𝐼(𝑎, 𝑏) =[𝐼(𝑎 − 1, 𝑏 − 1) − 𝐼(𝑎 − 1, 𝑏 + 1)]
𝑏
Eliminating 𝐼(𝑎 − 1, 𝑏 + 1) between these results gives required result.
©UCLES 2020 19
2. (i) sinh 𝑥 + sinh 𝑦 = 2𝑘
𝑑𝑦
Differentiating with respect to x, cosh 𝑥 + cosh 𝑦 =0
𝑑𝑥
𝑑𝑦
= 0 ⇒ cosh 𝑥 = 0 which is not possible as cosh 𝑥 ≥ 1 ∀𝑥 , so there are no stationary points.
𝑑𝑥
𝑑𝑦 2 𝑑2 𝑦
Differentiating again with respect to x, sinh 𝑥 + sinh 𝑦 ( ) + cosh 𝑦 =0
𝑑𝑥 𝑑𝑥 2
𝑑𝑦 −cosh 𝑥 𝑑2 𝑦 −cosh 𝑥 2
= and = 0 implies sinh 𝑥 + sinh 𝑦 ( ) =0
𝑑𝑥 cosh 𝑦 𝑑𝑥 2 cosh 𝑦
1 + sinh 𝑥 sinh 𝑦 = 0
as required.
𝑑2 𝑦
At a point of inflection, = 0 , so sinh 𝑥 + sinh 𝑦 = 2𝑘 and sinh 𝑥 sinh 𝑦 = −1 and thus,
𝑑𝑥 2
sinh 𝑥 (and sinh 𝑦 as well) is a root of 𝜆2 − 2𝑘𝜆 − 1 = 0
2𝑘 ± √4𝑘 2 + 4
𝜆=
2
−1 −1 𝑘−√𝑘 2 +1 −(𝑘−√𝑘 2 +1)
sinh 𝑥 = 𝑘 + √𝑘 2 + 1 , sinh 𝑦 = = × = = 𝑘 − √𝑘 2 + 1
𝑘+√𝑘 2 +1 𝑘+√𝑘 2 +1 𝑘−√𝑘 2 +1 𝑘 2 −(𝑘 2 +1)
and vice versa.
𝑒 𝑥 − 𝑒 −𝑥 𝑒 𝑎−𝑥 − 𝑒 𝑥−𝑎
+ = 2𝑘
2 2
Multiplying by 2𝑒 𝑥 ,
𝑒 2𝑥 − 1 + 𝑒 𝑎 − 𝑒 2𝑥 𝑒 −𝑎 = 4𝑘𝑒 𝑥
𝑒 2𝑥 (1 − 𝑒 −𝑎 ) − 4𝑘𝑒 𝑥 + (𝑒 𝑎 − 1) = 0
©UCLES 2020 20
As 𝑒 𝑥 is real, ′𝑏 2 − 4𝑎𝑐 ≥ 0′ , so 16𝑘 2 − 4(1 − 𝑒 −𝑎 )(𝑒 𝑎 − 1) ≥ 0
4𝑘 2 − 𝑒 𝑎 − 𝑒 −𝑎 + 2 ≥ 0
4𝑘 2 − 2 cosh 𝑎 + 2 ≥ 0
So cosh 𝑎 ≤ 2𝑘 2 + 1
(iii)
Alternative
𝑑𝑦
𝑑𝑦 −cosh 𝑥 𝑑2 𝑦 cosh 𝑦 sinh 𝑥−cosh 𝑥 sinh 𝑦 cosh2 𝑦 sinh 𝑥+cosh2 𝑥 sinh 𝑦
𝑑𝑥
(i) = , = −{ } = −{ }
𝑑𝑥 cosh 𝑦 𝑑𝑥 2 cosh2 𝑦 cosh3 𝑦
then as before.
©UCLES 2020 21
(ii) Substituting 𝑎 = 0 would imply 𝑒 𝑥 = 0 which is impossible.
©UCLES 2020 22
3. (i)
𝑖𝜋
𝑘 − 𝑎 = (𝑏 − 𝑎)𝑒 − 3
Therefore,
1 𝑖𝜋
𝑔𝐴𝐵 = [𝑎 + 𝑏 + (𝑎 + (𝑏 − 𝑎)𝑒 − 3 )]
3
𝑖𝜋 𝑖𝜋
2 − 𝑒− 3 1 + 𝑒− 3
= 𝑎( )+𝑏( )
3 3
𝑖𝜋 √3 + 𝑖
𝜔 =𝑒6 =
2
and so
√3 − 𝑖
𝜔∗ =
2
𝑖𝜋 1 − 𝑖√3
− 2−( )
2−𝑒 3 2 3 + 𝑖√3 1 √3 + 𝑖 1
= = = = 𝜔
3 3 6 √3 2 √3
and
𝑖𝜋 1 − 𝑖√3
1+( )
1 + 𝑒− 3 2 3 − 𝑖√3 1 ∗
= = = 𝜔
3 3 6 √3
1
Thus 𝑔𝐴𝐵 = (𝜔𝑎 + 𝜔∗ 𝑏) as required.
√3
1
(ii) 𝑔𝐴𝐵 = (𝜔𝑎 + 𝜔∗ 𝑏)
√3
1
𝑔𝐵𝐶 = (𝜔𝑏 + 𝜔∗ 𝑐)
√3
1
𝑔𝐶𝐷 = (𝜔𝑐 + 𝜔∗ 𝑑)
√3
1
𝑔𝐷𝐴 = (𝜔𝑑 + 𝜔∗ 𝑎)
√3
𝑄1 parallelogram ⇒ 𝑏 − 𝑎 = 𝑐 − 𝑑 ⇔ 𝑑 − 𝑎 = 𝑐 − 𝑏
1 1
𝑔𝐵𝐶 − 𝑔𝐴𝐵 = (𝜔(𝑏 − 𝑎) + 𝜔∗ (𝑐 − 𝑏)) =
(𝜔(𝑐 − 𝑑 ) + 𝜔∗ (𝑑 − 𝑎)) = 𝑔𝐶𝐷 − 𝑔𝐷𝐴
√3 √3
⇒ 𝑄2 parallelogram.
𝑄2 parallelogram ⇒ 𝑔𝐵𝐶 − 𝑔𝐴𝐵 = 𝑔𝐶𝐷 − 𝑔𝐷𝐴
1
{𝜔[(𝑏 − 𝑎) − (𝑐 − 𝑑 )] + 𝜔∗ [(𝑐 − 𝑏) − (𝑑 − 𝑎)]} = 0
√3
1
(𝜔∗ − 𝜔)[(𝑎 − 𝑏) − (𝑑 − 𝑐)] = 0
√3
As 𝜔∗ − 𝜔 ≠ 0 , (𝑎 − 𝑏) − (𝑑 − 𝑐) = 0 and so 𝑄1 is a parallelogram
©UCLES 2020 23
(iii)
1
𝑔𝐵𝐶 − 𝑔𝐴𝐵 = (𝜔(𝑏 − 𝑎) + 𝜔∗ (𝑐 − 𝑏))
√3
1
𝑔𝐶𝐴 − 𝑔𝐴𝐵 = (𝜔(𝑐 − 𝑎) + 𝜔∗ (𝑎 − 𝑏)) (1)
√3
1
𝜔2 (𝑔𝐵𝐶 − 𝑔𝐴𝐵 ) = (𝜔3 (𝑏 − 𝑎) + 𝜔(𝑐 − 𝑏)) (2)
√3
1
= (𝑖(𝑏 − 𝑎) + 𝜔(𝑐 − 𝑏))
√3
1 √3−𝑖 √3+𝑖
The coefficient of 𝑎 in (1) is 𝜔∗ − 𝜔 = − = −𝑖
√3 2 2
1
The coefficient of 𝑏 in (1) is −𝜔∗ = (𝑖 − 𝜔)
√3
1
The coefficient of 𝑐 in (1) is 𝜔
√3
𝜋
Thus 𝐺𝐴𝐵 𝐺𝐵𝐶 rotated through is 𝐺𝐴𝐵 𝐺𝐶𝐴 which means that 𝐺𝐴𝐵 𝐺𝐵𝐶 𝐺𝐶𝐴 is an equilateral
3
triangle.
(iii) Alternative
1
𝑥 = 𝑔𝐵𝐶 − 𝑔𝐴𝐵 = (𝜔(𝑏 − 𝑎) + 𝜔∗ (𝑐 − 𝑏))
√3
1
𝑦 = 𝑔𝐶𝐴 − 𝑔𝐴𝐵 = (𝜔(𝑐 − 𝑎) + 𝜔∗ (𝑎 − 𝑏))
√3
1 𝑖𝜋 𝑖𝜋 −𝑖𝜋 −𝑖𝜋
𝑥= (𝑒 6 𝑏 − 𝑒 6 𝑎 + 𝑒 6 𝑐 −𝑒 6 𝑏)
√3
1 𝑖𝜋 −𝑖𝜋 𝑖7𝜋
= (𝑒 2 𝑏 + 𝑒 6 𝑐 +𝑒 6 𝑎)
√3
1 𝑖𝜋 𝑖3𝜋 𝑖5𝜋
𝑦= (𝑒 6 𝑐 + 𝑒 2 𝑎 +𝑒 6 𝑏)
√3
𝑦 𝑖𝜋
=𝑒3
𝑥
𝑖𝜋
𝜋
𝑒 3 means y is x rotated through and thus ABC is an equilateral triangle.
3
[or alternatively
𝑦 𝑧
| | = 1 and similarly | | = 1 and thus all three sides are equal length]
𝑥 𝑦
©UCLES 2020 24
4. 𝜋 has equation 𝑟. 𝑛 = 0 so 𝑛 is a vector perpendicular to this plane.
𝑎
(i) The image of a point with position vector 𝑥 under T is 𝑥 − 2(𝑥. 𝑛)𝑛 , so as 𝑛 = (𝑏 ) and
𝑐
1 1 1 𝑎 𝑎 1 𝑎 1 − 2𝑎2
𝑖 = (0) , the image of 𝑖 under T is (0) − 2 (0) . (𝑏 ) (𝑏) = (0) − 2𝑎 (𝑏 ) = ( −2𝑎𝑏 )
0 0 0 𝑐 𝑐 0 𝑐 −2𝑎𝑐
𝑏 2 + 𝑐 2 − 𝑎2
Thus, the image of 𝑖 under T is ( −2𝑎𝑏 ) as required.
−2𝑎𝑐
−2𝑎𝑏 −2𝑎𝑐
Similarly, the images of 𝑗 and 𝑘 are (𝑐 2 + 𝑎2 − 𝑏 2 ) and ( −2𝑏𝑐 ) respectively.
−2𝑏𝑐 𝑎2 + 𝑏 2 − 𝑐 2
𝑏 2 + 𝑐 2 − 𝑎2 −2𝑎𝑏 −2𝑎𝑐
Thus 𝑀 = ( −2𝑎𝑏 𝑐 + 𝑎2 − 𝑏 2
2
−2𝑏𝑐 )
2 2 2
−2𝑎𝑐 −2𝑏𝑐 𝑎 +𝑏 −𝑐
(iii) Suppose the position vector of the point Q on the given line such that 𝑃𝑄 is perpendicular to
𝑎 𝑎
that line is 𝑦 , then 𝑦 = 𝜆 (𝑏 ) for some 𝜆 and (𝑦 − 𝑥). (𝑏) = 0
𝑐 𝑐
𝑎 𝑎 𝑎
So, 𝑦. (𝑏 ) − 𝑥. (𝑏 ) = 0 , i.e. 𝜆 = 𝑥. (𝑏 )
𝑐 𝑐 𝑐
𝑎 𝑎
So, the image of P under the rotation, is 𝑥 + 2(𝑦 − 𝑥) = 2𝑦 − 𝑥 = 2𝑥. (𝑏 ) (𝑏) − 𝑥
𝑐 𝑐
2𝑎2 − 1 𝑎2 − 𝑏 2 − 𝑐 2
The image of 𝑖 under the rotation is thus ( 2𝑎𝑏 ) = ( 2𝑎𝑏 ) , and of 𝑗 and 𝑘 are
2𝑎𝑐 2𝑎𝑐
2𝑎𝑏 2𝑎𝑐
(𝑏 2 − 𝑐 2 − 𝑎2 ) and ( 2𝑏𝑐 ) respectively.
2𝑏𝑐 𝑐 2 − 𝑎2 − 𝑏 2
©UCLES 2020 25
𝑎2 − 𝑏 2 − 𝑐 2 2𝑎𝑏 2𝑎𝑐
Thus 𝑁 = ( 2𝑎𝑏 𝑏 2 − 𝑐 2 − 𝑎2 2𝑏𝑐 ) , which, incidentally = −𝑀 .
2𝑎𝑐 2𝑏𝑐 𝑐 2 − 𝑎2 − 𝑏 2
Thus the single transformation is an enlargement, scale factor -1 , with centre of enlargement the
origin.
𝑥 = 𝑢 + 𝑣 where 𝑢 ∈ Π and 𝑣 ⊥ Π
𝑀𝑣 = −𝑣 𝑀𝑢 = 𝑢 𝑀𝑥 = 𝑢 − 𝑣
𝑁𝑥 = 𝑣 − 𝑢 = −𝑀𝑥
𝑁 = −𝑀
alternative for (ii) the matrix represents a reflection, an invariant point under the reflection lies on
the plane of reflection.
Therefore,
0.64 0.48 0.6 𝑥 𝑥
(0.48 0.36 −0.8) (𝑦) = (𝑦)
0.6 −0.8 0 𝑧 𝑧
This simplifies to
3𝑥 − 4𝑦 − 5𝑧 = 0
©UCLES 2020 26
5. (𝑥 − 𝑦)(𝑥 𝑛−1 + 𝑥 𝑛−2 𝑦 + ⋯ + 𝑦 𝑛−1 ) = 𝑥 𝑛 − 𝑥 𝑛−1 𝑦 + 𝑥 𝑛−1 𝑦 − 𝑥 𝑛−2 𝑦 + ⋯ + 𝑥𝑦 𝑛−1 − 𝑦 𝑛
= 𝑥𝑛 − 𝑦𝑛
as each even numbered term cancels with its subsequent term.
(i) If
1 𝐴 𝑓(𝑥)
𝐹(𝑥) = = + 𝑛
𝑥 𝑛 (𝑥− 𝑘) 𝑥 − 𝑘 𝑥
then multiplying by 𝑥 𝑛 (𝑥 − 𝑘)
1 = 𝐴𝑥 𝑛 + (𝑥 − 𝑘)𝑓(𝑥)
1
𝑥=𝑘 ⇒𝐴=
𝑘𝑛
so
𝑥𝑛
1= + (𝑥 − 𝑘)𝑓(𝑥)
𝑘𝑛
and
1 𝑥 𝑛
𝑓(𝑥) = (1 − ( ) )
𝑥−𝑘 𝑘
as required.
Thus
1 1 𝑥 𝑛
𝑛 (1 − ( ) )
𝐹(𝑥) = 𝑘 +𝑥−𝑘 𝑛 𝑘
𝑥−𝑘 𝑥
1 𝑥𝑛 − 𝑘𝑛
= −
𝑘 𝑛 (𝑥 − 𝑘) 𝑘 𝑛 𝑥 𝑛 (𝑥 − 𝑘)
and so, by the result of the stem,
𝑛
1 1
𝐹(𝑥) = 𝑛 − 𝑛 𝑛 ∑ 𝑥 𝑛−𝑟 𝑘 𝑟−1
𝑘 (𝑥 − 𝑘) 𝑘 𝑥
𝑟=1
𝑛
1 1 1
= 𝑛 − ∑ 𝑛−𝑟 𝑟
𝑘 (𝑥 − 𝑘) 𝑘 𝑘 𝑥
𝑟=1
©UCLES 2020 27
(ii)
𝑛
𝑛
1 𝑥𝑛 1 𝑥 𝑛−𝑟
𝑥 𝐹(𝑥) = = 𝑛 − ∑ 𝑛−𝑟
𝑥 − 𝑘 𝑘 (𝑥 − 𝑘) 𝑘 𝑘
𝑟=1
−1
Multiplying by
𝑥𝑛
𝑛
1 −𝑛 1 𝑛−𝑟
𝑛 2
= 𝑛 + 𝑛 2
+ ∑ 𝑛+1−𝑟 𝑟+1
𝑥 (𝑥 − 𝑘) 𝑥𝑘 (𝑥 − 𝑘) 𝑘 (𝑥 − 𝑘) 𝑘 𝑥
𝑟=1
(iii)
𝑁 𝑁 3
1 −3 1 3−𝑟
∫ 3 2
𝑑𝑥 = ∫ + 2
+ ∑ 𝑟+1 𝑑𝑥
𝑥 (𝑥 − 1) 𝑥(𝑥 − 1) (𝑥 − 1) 𝑥
2 2 𝑟=1
𝑁 3
3 3 1 3−𝑟
=∫ − + 2
+ ∑ 𝑟+1 𝑑𝑥
𝑥 (𝑥 − 1) (𝑥 − 1) 𝑥
2 𝑟=1
3 𝑁
1 3−𝑟
= [3ln 𝑥 − 3 ln(𝑥 − 1) − −∑ ]
𝑥−1 𝑟𝑥 𝑟
𝑟=1 2
3 𝑁
𝑥 1 3−𝑟
= [3 ln ( )− −∑ ]
𝑥−1 𝑥−1 𝑟𝑥 𝑟
𝑟=1 2
3 3
𝑁 1 3−𝑟 3−𝑟
= 3 ln ( )− −∑ 𝑟
− 3 ln(2) + 1 + ∑
𝑁−1 𝑁−1 𝑟𝑁 𝑟2𝑟
𝑟=1 𝑟=1
𝑁−1 𝑁−1 1 1
As 𝑁 → ∞ , ( ) → 1 , so 3 ln ( ) → 0 , and →0 , →0
𝑁 𝑁 𝑁−1 𝑁𝑟
©UCLES 2020 28
2 1 17
−3 ln 2 + 1 + + = −3 ln 2 +
2 8 8
©UCLES 2020 29
6. (i)
𝑦 = cos 𝑥 + √cos 2𝑥
𝜋 1
𝑥 = 0 ,𝑦 = 2 There is symmetry in 𝑥 = 0 𝑥=± ,𝑦 =
4 √2
𝑑𝑦 sin 2𝑥 𝑑𝑦 𝑑𝑦
= − sin 𝑥 − so 𝑥 =0, = 0 𝑥 >0, < 0 and vice versa
𝑑𝑥 √cos 2𝑥 𝑑𝑥 𝑑𝑥
𝜋 𝑑𝑦
as 𝑥 → , → −∞
4 𝑑𝑥
(ii)
©UCLES 2020 30
𝜋 1
(iii) 𝜃 = ± ,𝑟 =
4 √2
𝑟 2 − 2𝑟 cos 𝜃 + sin2 𝜃 = 0
(𝑟 − cos 𝜃)2 = cos 2 𝜃 − sin2 𝜃 = cos 2𝜃
Area required is
𝜋 𝜋
1 4 2 1 4 2
∫ (cos 𝜃 + √cos 2𝜃) 𝑑𝜃 − ∫ (cos 𝜃 − √cos 2𝜃) 𝑑𝜃
2 0 2 0
𝜋
4
= 2 ∫ cos 𝜃 √cos 2𝜃 𝑑𝜃
0
𝜋
4
= 2 ∫ cos 𝜃 √1 − 2 sin2 𝜃 𝑑𝜃
0
𝑑𝜃
Let √2 sin 𝜃 = sin 𝑢 , then √2 cos 𝜃 = cos 𝑢 ,
𝑑𝑢
©UCLES 2020 31
So the integral becomes
𝜋 𝜋 𝜋
2 cos 2 𝑢 2 cos 2𝑢 +1 sin 2𝑢 𝑢 2 𝜋
2∫ 𝑑𝑢 = √2 ∫ 𝑑𝑢 = √2 [ + ] =
0 √2 0 2 4 2 0 2√2
(iii) alternative
𝑟 ≪ 1 ⇒ −2𝑟 cos 𝜃 + sin2 𝜃 ≈ 0
sin2 𝜃 1 𝜃2
𝑟≈ = sin 𝜃 tan 𝜃 ≈
2 cos 𝜃 2 2
©UCLES 2020 32
𝑑𝑦
7. (i) 𝑢 = + 𝑔(𝑥)𝑦
𝑑𝑥
Thus
𝑑𝑢 𝑑 2 𝑦 𝑑𝑦
= 2 + 𝑔(𝑥) + 𝑔′ (𝑥)𝑦
𝑑𝑥 𝑑𝑥 𝑑𝑥
𝑑𝑢
As + 𝑓(𝑥)𝑢 = ℎ(𝑥)
𝑑𝑥
𝑑2 𝑦 𝑑𝑦 𝑑𝑦
2
+ 𝑔(𝑥) + 𝑔′ (𝑥)𝑦 + 𝑓(𝑥) ( + 𝑔(𝑥)𝑦) = ℎ(𝑥)
𝑑𝑥 𝑑𝑥 𝑑𝑥
that is
𝑑2 𝑦 𝑑𝑦
+ (𝑔(𝑥) + 𝑓(𝑥)) + (𝑔′ (𝑥) + 𝑓(𝑥)𝑔(𝑥))𝑦 = ℎ(𝑥)
𝑑𝑥 2 𝑑𝑥
as required.
(ii)
4
𝑔(𝑥) + 𝑓(𝑥) = 1 +
𝑥
4
and so 𝑓(𝑥) = 1 + − 𝑔(𝑥)
𝑥
2 2
𝑔′ (𝑥) + 𝑓(𝑥)𝑔(𝑥) = +
𝑥 𝑥2
so
4 2 2
𝑔′ (𝑥) + (1 + − 𝑔(𝑥)) 𝑔(𝑥) = + 2
𝑥 𝑥 𝑥
as requested.
©UCLES 2020 33
Or it is eliminated by the 𝑥 𝑛+1 term, in which case, 2𝑛 + 2 = 𝑛 + 1 which implies 𝑛 = −1 and
thus −𝑘 2 + 𝑘(𝑛 + 4) − 2 = 0 and considering the other two terms 𝑘 − 2 = 0
𝑑𝑢 𝑑𝑢 2
+ 𝑓(𝑥)𝑢 = ℎ(𝑥) is thus + (1 + ) 𝑢 = 4𝑥 + 12
𝑑𝑥 𝑑𝑥 𝑥
Thus
𝑑𝑢
𝑥2𝑒 𝑥 + (𝑥 2 + 2𝑥)𝑒 𝑥 𝑢 = (4𝑥 + 12)𝑥 2 𝑒 𝑥 = (4𝑥 3 + 12𝑥 2 )𝑒 𝑥
𝑑𝑥
Integrating with respect to x
𝑥 2 𝑒 𝑥 𝑢 = ∫(4𝑥 3 + 12𝑥 2 )𝑒 𝑥 𝑑𝑥 = 4𝑥 3 𝑒 𝑥 + 𝑐
𝑑𝑦 2 𝑑𝑦
As 𝑢 = + 𝑔(𝑥)𝑦 , and 𝑔(𝑥) = , when 𝑥 = 1 , 𝑦 = 5 , = −3 , we have 𝑢 = −3 + 2 × 5
𝑑𝑥 𝑥 𝑑𝑥
𝑑𝑦 2 𝑒 −𝑥
+ 𝑦 = 4𝑥 + 3𝑒 2
𝑑𝑥 𝑥 𝑥
This has integrating factor
2
𝑒 ∫𝑥 𝑑𝑥 = 𝑒 2 ln 𝑥 = 𝑥 2
So
𝑑𝑦
𝑥2 + 2𝑥𝑦 = 4𝑥 3 + 3𝑒 𝑒 −𝑥
𝑑𝑥
Integrating with respect to x
𝑥 2 𝑦 = ∫ 4𝑥 3 + 3𝑒 𝑒 −𝑥 𝑑𝑥 = 𝑥 4 − 3𝑒𝑒 −𝑥 + 𝑐′
Therefore,
7 3𝑒 −𝑥+1
𝑦 = 𝑥2 + −
𝑥2 𝑥2
©UCLES 2020 34
8. (i) All terms of the sequence are positive integers because they are all either equal to a previous
term or the sum of two previous terms which are positive integers.
Also, 𝑢2𝑘+1 − 𝑢2𝑘+2 = 𝑢𝑘 + 𝑢𝑘+1 − 𝑢𝑘+1 = 𝑢𝑘 ≥ 1 . Thus, the required result is proved for terms
from the third onwards. (The only terms not included in this proof are the first two, which are in
case both equal to 1).
(ii) Suppose that 𝑢2𝑘 = 𝑐 , and that 𝑢2𝑘+1 = 𝑑 , for 𝑘 ≥ 1 , where d and c share a common factor
greater than one, then 𝑢𝑘 = 𝑐 , as 𝑢2𝑘 = 𝑢𝑘 , and 𝑢𝑘+1 = 𝑑 − 𝑐 ≥ 1 as 𝑢2𝑘+1 = 𝑢𝑘 + 𝑢𝑘+1 and
using (i). Then as d and c share a common factor greater than one, d-c and c share a common factor
greater than one. So, two earlier terms in the sequence do share the same common factor.
Likewise, suppose that 𝑢2𝑘+2 = 𝑐 , and that 𝑢2𝑘+1 = 𝑑 , for 𝑘 ≥ 1 , where d and c share a
common factor greater than one, then 𝑢𝑘+1 = 𝑐 and 𝑢𝑘 = 𝑑 − 𝑐 giving the same result.
This is true for pairs of consecutive terms from the second term (and third) onwards. Repeating this
argument, we find that it would imply that the first two terms would share a common factor greater
than one, which is a contradiction. Hence any two consecutive terms are co-prime.
(iii) For 𝑘 ≥ 1 , and 𝑚 ≥ 1 suppose that 𝑢2𝑘 = 𝑐 and 𝑢2𝑘+1 = 𝑑 , and that 𝑢2𝑘+𝑚 = 𝑐 and
𝑢2𝑘+𝑚+1 = 𝑑 , then as 𝑑 > 𝑐 , 2𝑘 + 𝑚 is even, so 𝑚 is even, say 2𝑛 . Thus, 𝑢𝑘 = 𝑐 and
𝑢𝑘+1 = 𝑑 − 𝑐 , and 𝑢𝑘+𝑛 = 𝑐 and 𝑢𝑘+𝑛+1 = 𝑑 − 𝑐 . That is, an earlier pair of terms would appear
consecutively.
Likewise, if 𝑢2𝑘+2 = 𝑐 and 𝑢2𝑘+1 = 𝑑 , and that 𝑢2𝑘+𝑚+2 = 𝑐 and 𝑢2𝑘+𝑚+1 = 𝑑, the same
argument applies.
So the argument can be repeated down to the first two terms, which are of course equal, and it
would imply a later pair are likewise which contradicts (i).
(iv) If (𝑎, 𝑏) does not occur, where 𝑎 and 𝑏 are coprime and 𝑎 > 𝑏 , then there does not exist 𝑘
such that 𝑢2𝑘+1 = 𝑎 and 𝑢2𝑘+2 = 𝑏 . Therefore there cannot exist a k such that 𝑢𝑘+1 = 𝑏 and
𝑢𝑘 = 𝑎 − 𝑏 , the sum of which is 𝑎 , which is smaller than 𝑎 + 𝑏 .
If (𝑎, 𝑏) does not occur, where 𝑎 and 𝑏 are coprime and 𝑎 < 𝑏 , then there does not exist 𝑘 such
that 𝑢2𝑘 = 𝑎 and 𝑢2𝑘+1 = 𝑏 . Therefore there cannot exist a k such that 𝑢𝑘 = 𝑎 and 𝑢𝑘+1 = 𝑏 −
𝑎 , the sum of which is b, which is smaller than 𝑎 + 𝑏 .
(v) Suppose that there exists an ordered pair of coprime integers (a,b) which does not occur
consecutively in the sequence. Then by part (iv) the pair (a-b, b) [if a>b] or (a, b-a) [if b>a] (which
has a smaller sum) does not occur. Repeating this means that a coprime pair with sum <3 does not
occur. The only coprime pair of integers with sum <3 is (1, 1) which are the first two
terms. Contradiction and so every ordered pair of coprime integers occurs in the sequence
and by (iii) only occurs once. Therefore, there exists an 𝑛 , and that 𝑛 is unique such that
𝑢𝑛
𝑞= , for any positive rational 𝑞 (which is expressed in lowest form). So the inverse of f
𝑢𝑛+1
exists.
©UCLES 2020 35
9. (i)
𝑅 cos 𝛼 + 𝑆 cos 𝛽 = 𝑊
Resolving horizontally, (2)
𝑅 sin 𝛼 = 𝑆 sin 𝛽
Taking moments about Q, (3)
©UCLES 2020 36
(ii)
𝑊 cos 𝜙 = 2𝑅 cos(𝛼 − 𝜙)
Multiplying (5) by (sin 𝛽 + 𝜇 cos 𝛽) cos 𝜙 gives
so
1 = (cot 𝛽 − 2 tan 𝜃)(tan 𝛽 + 𝜇) + 2 tan 𝜙 (tan 𝛽 + 𝜇) + 𝜇 tan 𝛽
1 − 𝜇 tan 𝛽 − 1 − 𝜇 cot 𝛽 = 2(tan 𝜙 − tan 𝜃)(tan 𝛽 + 𝜇)
Hence,
©UCLES 2020 37
sin 𝛽 cos 𝛽 sin2 𝛽 + cos 2 𝛽 1
tan 𝛽 + cot 𝛽 = + = =
cos 𝛽 sin 𝛽 sin 𝛽 cos 𝛽 sin 𝛽 cos 𝛽
and so,
𝜇
tan 𝜃 − tan 𝜙 =
2 sin 𝛽 cos 𝛽(tan 𝛽 + 𝜇)
That is
𝜇
tan 𝜃 − tan 𝜙 =
(𝜇 + tan 𝛽) sin 2𝛽
as required.
©UCLES 2020 38
10. If the extension in the equilibrium position is , then
𝑘𝑚𝑔𝑑
𝑚𝑔 =
𝑎
𝑎
Thus, 𝑑 =
𝑘
If the extension when the particle is released is 𝑑 + 𝑥 , then the equation of motion is
𝑘𝑚𝑔(𝑑 + 𝑥) 𝑘𝑚𝑔𝑑 𝑘𝑚𝑔𝑥 𝑘𝑚𝑔𝑥
𝑚𝑥̈ = 𝑚𝑔 − = 𝑚𝑔 − − =−
𝑎 𝑎 𝑎 𝑎
𝑘𝑔𝑥
𝑥̈ = −
𝑎
2𝜋 𝑘𝑔
This is simple harmonic motion with period where Ω2 = , i.e. 𝑘𝑔 = 𝑎Ω2 as required.
Ω 𝑎
Let 𝑦 be the displacement of the platform below the centre point of its oscillation,
So,
𝑚𝑎Ω2 (ℎ − 𝑎 − 𝑥)
−𝑚𝜔2 (𝑏 − 𝑥) = 𝑚𝑔 − 𝑅 −
𝑎
That is,
𝑅 = 𝑚𝑔 + 𝑚Ω2 (𝑎 + 𝑥 − ℎ) + 𝑚𝜔2 (𝑏 − 𝑥)
as required.
Rearranging,
𝑔 + 𝜔2 𝑏 𝑎 𝜔2 𝑏 1 𝜔2 𝑏
ℎ≤ + 𝑎 = + + 𝑎 = 𝑎 (1 + ) +
Ω2 𝑘 Ω2 𝑘 Ω2
as required.
©UCLES 2020 39
Thus,
𝑚𝑔 + 𝑚Ω2 (𝑎 − ℎ) + 𝑚𝜔2 𝑏 + 2𝑚𝑏(Ω2 − 𝜔2 ) ≥ 0
and so,
𝑔 + 𝜔2 𝑏 2𝑏(Ω2 − 𝜔2 ) 1 𝜔2 𝑏 2𝜔2 𝑏 1 𝜔2 𝑏
ℎ≤ + 𝑎 + = 𝑎 (1 + ) + + 2𝑏 − = 𝑎 (1 + ) − + 2𝑏
Ω2 Ω2 𝑘 Ω2 Ω2 𝑘 Ω2
1 𝜔2 𝑏 1
Thus, if 𝜔 < Ω , ℎ ≤ 𝑎 (1 + ) + < 𝑎 (1 + ) + 𝑏 ;
𝑘 Ω2 𝑘
1 𝜔2 𝑏 1
if 𝜔 > Ω , ℎ ≤ 𝑎 (1 + ) − + 2𝑏 < 𝑎 (1 + ) + 𝑏
𝑘 Ω2 𝑘
so,
1
ℎ ≤ 𝑎 (1 + ) + 𝑏
𝑘
Alternative
©UCLES 2020 40
11. (i)
𝑃(𝑌 ≤ 𝑦) = 𝑃(𝑓(𝑋) ≤ 𝑦)
= 𝑃(𝑋 ≥ 𝑓 −1 (𝑦))
= 𝑃(𝑋 ≥ 𝑓(𝑦))
𝑏 − 𝑓(𝑦)
=
𝑏−𝑎
because X is uniformly distributed on [a,b].
𝑦 ∈ [𝑎, 𝑏]
𝑏 𝑏 𝑏
2)
−𝑓′(𝑦)
2
−𝑓(𝑦) −𝑓(𝑦)
𝐸(𝑌 =∫ 𝑦 𝑑𝑦 = [𝑦 2 ] − ∫ 2𝑦 𝑑𝑦
𝑎 𝑏−𝑎 𝑏−𝑎 𝑎 𝑎 𝑏−𝑎
by integration by parts
𝑏 𝑏 𝑏
−𝑎𝑏 2 + 𝑎2 𝑏 𝑓(𝑥) 𝑎𝑏(𝑎 − 𝑏) 𝑓(𝑥) 𝑓(𝑥)
= + ∫ 2𝑥 𝑑𝑥 = + ∫ 2𝑥 𝑑𝑥 = −𝑎𝑏 + ∫ 2𝑥 𝑑𝑥
𝑏−𝑎 𝑎 𝑏−𝑎 𝑏−𝑎 𝑎 𝑏−𝑎 𝑎 𝑏−𝑎
as required.
(ii) Considering Z as a function of X, it satisfies the three conditions for the function f in part (i), as
trivially by the definition of c the first is satisfied, considering the graph or the derivative the second
is, and by symmetry, the third is.
1 1 1
+ =
𝑍 𝑋 𝑐
so
1 1 1 𝑋−𝑐
= − =
𝑍 𝑐 𝑋 𝑐𝑋
and therefore
𝑐𝑋
𝑍=
𝑋−𝑐
Therefore,
𝑏 𝑏 𝑏
𝑐𝑥 1 𝑐 𝑥−𝑐 𝑐 𝑐 𝑐
𝐸(𝑍) = ∫ 𝑑𝑥 = ∫ + 𝑑𝑥 = ∫ 1+ 𝑑𝑥
𝑎 𝑥−𝑐 𝑏−𝑎 𝑏−𝑎 𝑎 𝑥−𝑐 𝑥−𝑐 𝑏−𝑎 𝑎 𝑥−𝑐
©UCLES 2020 41
𝑐
= [𝑥 + c ln(𝑥 − 𝑐)]𝑏𝑎
𝑏−𝑎
𝑐2 𝑏−𝑐
=𝑐+ ln ( )
𝑏−𝑎 𝑎−𝑐
From (i),
𝑏
𝑐𝑥 2𝑥
𝐸(𝑍 2 ) = −𝑎𝑏 + ∫ 𝑑𝑥
𝑎 𝑥−𝑐 𝑏−𝑎
𝑏 𝑏 2
2𝑐 𝑥2 2𝑐 𝑥 − 𝑥𝑐 𝑥𝑐 − 𝑐 2 𝑐2
= −𝑎𝑏 + ∫ 𝑑𝑥 = −𝑎𝑏 + ∫ + + 𝑑𝑥
𝑏−𝑎 𝑎 𝑥−𝑐 𝑏−𝑎 𝑎 𝑥−𝑐 𝑥−𝑐 𝑥−𝑐
𝑏
2𝑐 𝑥 2
= −𝑎𝑏 + [ + 𝑐𝑥 + 𝑐 2 ln(𝑥 − 𝑐)]
𝑏−𝑎 2 𝑎
2𝑐 3 2
𝑏−𝑐
= −𝑎𝑏 + 𝑐(𝑎 + 𝑏) + 2𝑐 + ln ( )
𝑏−𝑎 𝑎−𝑐
2𝑐 3 𝑏−𝑐 2𝑐 3 𝑏−𝑐
= −𝑎𝑏 + 𝑎𝑏 + 2𝑐 2 + ln ( ) = 2𝑐 2 + ln ( )
𝑏−𝑎 𝑎−𝑐 𝑏−𝑎 𝑎−𝑐
Thus,
2
2𝑐 3
2
𝑏−𝑐 𝑐2 𝑏−𝑐
𝑉𝑎𝑟(𝑍) = 2𝑐 + ln ( ) − (𝑐 + ln ( ))
𝑏−𝑎 𝑎−𝑐 𝑏−𝑎 𝑎−𝑐
2
2
𝑐2 𝑏−𝑐
=𝑐 −( ln ( ))
𝑏−𝑎 𝑎−𝑐
𝑐2 𝑏−𝑐
and so as 𝑐 and ln ( ) are both positive,
𝑏−𝑎 𝑎−𝑐
©UCLES 2020 42
𝑐2 𝑏−𝑐
𝑐> ln ( )
𝑏−𝑎 𝑎−𝑐
and similarly,
𝑏−𝑐 𝑏−𝑎
ln ( ) <
𝑎−𝑐 𝑐
as required.
©UCLES 2020 43
12. (i) 𝑃(𝑋 = 𝑥) = 𝑞 𝑥−1 𝑝 and 𝑃(𝑌 = 𝑦) = 𝑞 𝑦−1 𝑝 for 𝑥, 𝑦 ≥ 1
𝑠−1 𝑠−1
for 𝑠 ≥ 2 .
𝑡
𝑡−1
= 2𝑞 𝑝 ∑ 𝑞 𝑦−1 𝑝 − 𝑞 𝑡−1 𝑝𝑞 𝑡−1 𝑝
𝑦=1
1 − 𝑞𝑡
= 2𝑞 𝑡−1 𝑝2 − 𝑞 2𝑡−2 𝑝2 = 2𝑞 𝑡−1 𝑝(1 − 𝑞 𝑡 ) − 𝑞 2𝑡−2 𝑝2
1−𝑞
for 𝑡 ≥ 1 as required.
(ii)
∞ ∞
for 𝑢 ≥ 1
∞ ∞
𝑥−1 𝑥+𝑢−1
1
= 2∑𝑞 𝑝𝑞 𝑝 = 2𝑝 𝑞 ∑ 𝑞 2𝑥−2 = 2𝑝2 𝑞 𝑢
2 𝑢
1 − 𝑞2
𝑥=1 𝑥=1
1 2𝑝 𝑞 𝑢
= 2𝑝2 𝑞 𝑢 =
𝑝(1 + 𝑞) (1 + 𝑞)
and
∞ ∞ ∞
𝑥−1 𝑥−1 2 2𝑥−2
𝑝2 𝑝
𝑃(𝑈 = 0) = ∑ 𝑃(𝑋 = 𝑥, 𝑌 = 𝑥) = ∑ 𝑞 𝑝𝑞 𝑝 = 𝑝 ∑𝑞 = =
1 − 𝑞2 1 + 𝑞
𝑥=1 𝑥=1 𝑥=1
©UCLES 2020 44
𝑃(𝑊 = 𝑤) = 𝑃(𝑋 = 𝑤, 𝑌 ≥ 𝑤) + 𝑃(𝑌 = 𝑤, 𝑋 ≥ 𝑤) − 𝑃(𝑋 = 𝑤, 𝑌 = 𝑤)
for 𝑤 ≥ 1
∞ ∞
𝑤−1 𝑤+𝑦−1 𝑤−1 𝑤−1 2 2𝑤−2
=2∑𝑞 𝑝𝑞 𝑝−𝑞 𝑝𝑞 𝑝 = 2𝑝 𝑞 ∑ 𝑞 𝑦 − 𝑝2 𝑞 2𝑤−2
𝑦=0 𝑦−0
2 2 1+𝑞
= 𝑝2 𝑞 2𝑤−2 − 𝑝2 𝑞 2𝑤−2 = 𝑝2 𝑞 2𝑤−2 ( − 1) = 𝑝2 𝑞 2𝑤−2
1−𝑞 1−𝑞 1−𝑞
= 𝑝𝑞 2𝑤−2 (1 + 𝑞)
Thus,
𝑃(𝑆 = 2 , 𝑇 = 3) = 0
However,
= 𝑝𝑞 2 (𝑝(1 + 𝑞) + 𝑝(1 + 𝑞 + 𝑞 2 ))
= 𝑝2 𝑞 2 (2 + 2𝑞 + 𝑞 2 ) ≠ 0
(iv)
so
2𝑝 𝑞 𝑢
𝑃(𝑈 = 𝑢) × 𝑃(𝑊 = 𝑤) = × 𝑝𝑞 2𝑤−2 (1 + 𝑞) = 2𝑝2 𝑞 2𝑤+𝑢−2 = 𝑃(𝑈 = 𝑢 , 𝑊 = 𝑤)
(1 + 𝑞)
In the case 𝑢 = 0 ,
𝑈 = 0 and 𝑊 = 𝑤 ⇒ 𝑋 = 𝑤, 𝑌 = 𝑤
©UCLES 2020 45
so
𝑝
𝑃(𝑈 = 0) × 𝑃(𝑊 = 𝑤) = × 𝑝𝑞 2𝑤−2 (1 + 𝑞) = 𝑝2 𝑞 2𝑤−2 = 𝑃(𝑈 = 0 , 𝑊 = 𝑤)
1+𝑞
Thus, U and W are independent.
Alternative (i)
𝑡−1
𝑡−1
= 2𝑞 𝑝 ∑ 𝑞 𝑦−1 𝑝 + 𝑞 𝑡−1 𝑝𝑞 𝑡−1 𝑝
𝑦=1
1 − 𝑞 𝑡−1
= 2𝑞 𝑡−1 𝑝2 + 𝑞 2𝑡−2 𝑝2 = 2𝑞 𝑡−1 𝑝(1 − 𝑞 𝑡−1 ) + 𝑞 2𝑡−2 𝑝2
1−𝑞
©UCLES 2020 46
= 𝑞 𝑡−1 𝑝 (2 − 2𝑞 𝑡−1 + 𝑞 𝑡−1 𝑝) = 𝑝𝑞 𝑡−1 (2 − 2𝑞 𝑡−1 + (1 − 𝑞)𝑞 𝑡−1 ) = 𝑝𝑞 𝑡−1 (2 − 𝑞 𝑡−1 − 𝑞 𝑡 )
for 𝑡 ≥ 1 as required.
(ii)
for 𝑤 ≥ 1
∞ ∞
𝑤−1 𝑤+𝑦−1 𝑤−1 𝑤−1 2 2𝑤−2
=2∑𝑞 𝑝𝑞 𝑝+𝑞 𝑝𝑞 𝑝 = 2𝑝 𝑞 ∑ 𝑞 𝑦 + 𝑝2 𝑞 2𝑤−2
𝑦=1 𝑦−1
2 2𝑞 1+𝑞
= 𝑝2 𝑞 2𝑤−1 + 𝑝2 𝑞 2𝑤−2 = 𝑝2 𝑞 2𝑤−2 ( + 1) = 𝑝2 𝑞 2𝑤−2
1−𝑞 1−𝑞 1−𝑞
= 𝑝𝑞 2𝑤−2 (1 + 𝑞)
©UCLES 2020 47
Cambridge Assessment Admissions Testing offers a range of tests to support selection and
recruitment for higher education, professional organisations and governments around the world.
Underpinned by robust and rigorous research, our assessments include:
Cambridge Assessment
Admissions Testing
The Triangle Building
Shaftesbury Road
Cambridge
CB2 8EA
United Kingdom
admissionstesting.org/help
STEP MATHEMATICS 2
2021
Mark Scheme
1
cos(3𝑎𝑎 + 𝑎𝑎) ≡ cos 3𝑎𝑎 cos 𝑎𝑎 − sin 3𝑎𝑎 sin 𝑎𝑎 M1
cos(3𝑎𝑎 − 𝑎𝑎) ≡ cos 3𝑎𝑎 cos 𝑎𝑎 + sin 3𝑎𝑎 sin 𝑎𝑎
cos 4𝑎𝑎 + cos 2𝑎𝑎 ≡ 2 cos 3𝑎𝑎 cos 𝑎𝑎
1 A1
cos 𝑎𝑎 cos 3𝑎𝑎 ≡ (cos 4𝑎𝑎 + cos 2𝑎𝑎 ) 𝑨𝑨𝑨𝑨
2
sin(3𝑎𝑎 + 𝑎𝑎) ≡ sin 3𝑎𝑎 cos 𝑎𝑎 + cos 3𝑎𝑎 sin 𝑎𝑎
sin(3𝑎𝑎 − 𝑎𝑎) ≡ sin 3𝑎𝑎 cos 𝑎𝑎 − cos 3𝑎𝑎 sin 𝑎𝑎
sin 4𝑎𝑎 − sin 2𝑎𝑎 ≡ 2 cos 3𝑎𝑎 sin 𝑎𝑎
1 B1
sin 𝑎𝑎 cos 3𝑎𝑎 ≡ (sin 4𝑎𝑎 − sin 2𝑎𝑎)
2
1
cos 6𝑥𝑥 = :
2
𝜋𝜋 5𝜋𝜋 7𝜋𝜋 11𝜋𝜋 13𝜋𝜋 17𝜋𝜋
6𝑥𝑥 = , , , , ,
3 3 3 3 3 3
𝜋𝜋 5𝜋𝜋 7𝜋𝜋 11𝜋𝜋 13𝜋𝜋 17𝜋𝜋
𝑥𝑥 = , , , , , A1
18 18 18 18 18 18
sin 4𝑥𝑥 = 0:
4𝑥𝑥 = 0, 𝜋𝜋, 2𝜋𝜋, 3𝜋𝜋, 4𝜋𝜋
𝜋𝜋 𝜋𝜋 3𝜋𝜋
𝑥𝑥 = 0, , , , 𝜋𝜋
4 2 4 A1
𝜋𝜋
tan 𝑥𝑥 is undefined at 𝑥𝑥 = B1
2
𝜋𝜋 3𝜋𝜋
tan 2𝑥𝑥 is undefined at 𝑥𝑥 = ,
4 4 B1
So these are not solutions of the equation.
𝜋𝜋 5𝜋𝜋 7𝜋𝜋 11𝜋𝜋 13𝜋𝜋 17𝜋𝜋
𝑥𝑥 = 0, , , , , , , 𝜋𝜋
18 18 18 18 18 18
2
(i) 3𝑝𝑝𝑝𝑝 − 𝑝𝑝3 = 3(𝑎𝑎 + 𝑏𝑏)(𝑎𝑎2 + 𝑏𝑏 2 ) − (𝑎𝑎 + 𝑏𝑏)3 M1
= 2𝑎𝑎3 + 2𝑏𝑏 3
= 2𝑟𝑟 𝑨𝑨𝑨𝑨 A1
(iii) 𝑎𝑎 + 𝑏𝑏 = 𝑠𝑠 − 𝑐𝑐 (= 𝑝𝑝)
𝑎𝑎 + 𝑏𝑏 2 = 𝑡𝑡 − 𝑐𝑐 2 (= 𝑞𝑞)
2
𝑎𝑎3 + 𝑏𝑏 3 = 𝑢𝑢 − 𝑐𝑐 3 (= 𝑟𝑟) M1
By part (i):
3(𝑠𝑠 − 𝑐𝑐 )(𝑡𝑡 − 𝑐𝑐 2 ) − (𝑠𝑠 − 𝑐𝑐 )3 = 2(𝑢𝑢 − 𝑐𝑐 3 )
3𝑠𝑠𝑠𝑠 − 3𝑐𝑐𝑐𝑐 − 3𝑐𝑐 2 𝑠𝑠 + 3𝑐𝑐 3 − 𝑠𝑠 3 + 3𝑐𝑐𝑠𝑠 2 − 3𝑐𝑐 2 𝑠𝑠 + 𝑐𝑐 3 = 2𝑢𝑢 − 2𝑐𝑐 3 M1
6𝑐𝑐 3 − 6𝑠𝑠𝑐𝑐 2 + 3(𝑠𝑠 2 − 𝑡𝑡)𝑐𝑐 + 3𝑠𝑠𝑠𝑠 − 𝑠𝑠 3 − 2𝑢𝑢 = 0 A1
Therefore 𝑐𝑐 is a root of the equation
6𝑥𝑥 3 − 6𝑠𝑠𝑥𝑥 2 + 3(𝑠𝑠 2 − 𝑡𝑡)𝑥𝑥 + 3𝑠𝑠𝑠𝑠 − 𝑠𝑠 3 − 2𝑢𝑢 = 0 𝑨𝑨𝑨𝑨 E1
The other roots are 𝑎𝑎 and 𝑏𝑏. B1
(ii) +− M1
⇒ 𝑦𝑦 + {𝑦𝑦} − ⌊𝑦𝑦⌋ + 𝑧𝑧 + ⌊𝑧𝑧⌋ − {𝑧𝑧} = 6.4
⇒ 2{𝑦𝑦} + 2⌊𝑧𝑧⌋ = 6.4 M1
⇒ {𝑦𝑦} + ⌊𝑧𝑧⌋ = 3.2 𝑨𝑨𝑨𝑨 or {𝑥𝑥 } + ⌊𝑦𝑦⌋ = 2.1 or ⌊𝑥𝑥⌋ + {𝑧𝑧} = 1.8 A1
Similar attempts at + − ⇒ {𝑥𝑥 } + ⌊𝑦𝑦⌋ = 2.1
M1
and + − ⇒ ⌊𝑥𝑥⌋ + {𝑧𝑧} = 1.8
The remaining two 2-variable eqns. correct A1
⇒ {𝑦𝑦} = 0.2 and ⌊𝑧𝑧⌋ = 3 B1
Also (respectively) {𝑥𝑥 } = 0.1 and ⌊𝑦𝑦⌋ = 2
B1
and ⌊𝑥𝑥⌋ = 1 and {𝑧𝑧} = 0.8
Solution is 𝑥𝑥 = 1.1, 𝑦𝑦 = 2.2, 𝑧𝑧 = 3.8 A1
Since 𝑒𝑒 𝑥𝑥 > 0 for all 𝑥𝑥, the only stationary point is when 𝑥𝑥 = −1 A1
1
Coordinates of stationary point are (−1, − )
𝑒𝑒
Sketch showing:
𝑦𝑦 → ∞ as 𝑥𝑥 → ∞ and 𝑦𝑦 → 0− as 𝑥𝑥 → −∞ G1
1 G1
Curve passing through (0,0) with stationary point at (−1, − ) indicated.
𝑒𝑒
(ii) -1 B1
Sketch showing reflection of the correct portion of the graph in the line 𝑦𝑦 = 𝑥𝑥. G1
1
domain �− , ∞� and range [−1, ∞)
𝑒𝑒
(iii)
(a) 𝑒𝑒 −𝑥𝑥 = 5𝑥𝑥
1
𝑥𝑥𝑒𝑒 𝑥𝑥 =
5
M1
1
𝑓𝑓(𝑥𝑥 ) =
5
Since 𝑓𝑓(𝑥𝑥 ) > 0 there is only one solution A1
1
𝑥𝑥 = 𝑔𝑔 � �
5
(b) 2𝑥𝑥 ln 𝑥𝑥 + 1 = 0
Let 𝑢𝑢 = ln 𝑥𝑥: M1
1 M1
𝑢𝑢𝑒𝑒 𝑢𝑢 = −
2
1 1 1 E1
The minimum value of 𝑓𝑓(𝑥𝑥) is − and − < − , so there are no solutions.
𝑒𝑒 2 𝑒𝑒
(c) 3𝑥𝑥 ln 𝑥𝑥 + 1 = 0
Let 𝑢𝑢 = ln 𝑥𝑥:
1 M1
𝑢𝑢𝑒𝑒 𝑢𝑢 = −
3
1 1 E1
− < − < 0 so there are two solutions for 𝑢𝑢 and the greater of the two will be
𝑒𝑒 3
1
when 𝑢𝑢 = 𝑔𝑔 �− �.
3
1
𝑥𝑥 = 𝑒𝑒
𝑔𝑔�− �
3 is the larger value. A1
(d) 𝑥𝑥 = 3 ln 𝑥𝑥
Let 𝑢𝑢 = ln 𝑥𝑥:
1 M1
𝑢𝑢𝑒𝑒 −𝑢𝑢 =
3
1 1
(−𝑢𝑢)𝑒𝑒 −𝑢𝑢 = − , so (as in (c)) 𝑔𝑔 �− � is the greater of the two possible values for −𝑢𝑢. M1
3 3
1
−𝑔𝑔�− �
Therefore 𝑥𝑥 = 𝑒𝑒 3 is the smaller value. A1
E1
(iv) 𝑥𝑥 ln 𝑥𝑥 = ln 10
Let 𝑢𝑢 = ln 𝑥𝑥:
𝑢𝑢𝑒𝑒 𝑢𝑢 = ln 10 M1
𝑢𝑢 = 𝑔𝑔(ln 10)
𝑥𝑥 = 𝑒𝑒 𝑔𝑔(ln 10) A1
5
(i) 𝑑𝑑𝑑𝑑 𝑑𝑑𝑑𝑑 M1
= (𝑥𝑥 − 𝑎𝑎) + 𝑢𝑢
𝑑𝑑𝑑𝑑 𝑑𝑑𝑑𝑑 A1
𝑑𝑑𝑑𝑑
(𝑥𝑥 − 𝑎𝑎) �(𝑥𝑥 − 𝑎𝑎) + 𝑢𝑢� = (𝑥𝑥 − 𝑎𝑎)𝑢𝑢 − 𝑥𝑥
𝑑𝑑𝑑𝑑
𝑑𝑑𝑑𝑑
(𝑥𝑥 − 𝑎𝑎)2 = −𝑥𝑥
𝑑𝑑𝑑𝑑
−𝑥𝑥 −(𝑥𝑥 − 𝑎𝑎) − 𝑎𝑎 M1
𝑢𝑢 = � 𝑑𝑑𝑑𝑑 = � 𝑑𝑑𝑑𝑑
(𝑥𝑥 − 𝑎𝑎) 2 (𝑥𝑥 − 𝑎𝑎)2
𝑎𝑎 A1
𝑢𝑢 = − ln|𝑥𝑥 − 𝑎𝑎| + + 𝑐𝑐
𝑥𝑥 − 𝑎𝑎
𝑦𝑦 = −(𝑥𝑥 − 𝑎𝑎) ln|𝑥𝑥 − 𝑎𝑎| + 𝑎𝑎 + 𝑐𝑐(𝑥𝑥 − 𝑎𝑎) A1 (ft)
(ii)
(a) The gradient of the line through (1, 𝑡𝑡) and (𝑡𝑡, 𝑓𝑓(𝑡𝑡)) is
𝑓𝑓(𝑡𝑡)−𝑡𝑡
= 𝑓𝑓′(𝑡𝑡) M1
𝑡𝑡−1
Applying the result from (i), with a=1 or solving the d.e. directly:
𝑓𝑓(𝑥𝑥 ) = −(𝑥𝑥 − 1) ln|𝑥𝑥 − 1| + 1 + 𝑐𝑐(𝑥𝑥 − 1) B1 (ft)
𝑓𝑓(0) = 0, so 𝑐𝑐 = 1 B1 (ft)
𝑦𝑦 = −(𝑥𝑥 − 1) ln|𝑥𝑥 − 1| + 𝑥𝑥
𝑑𝑑𝑑𝑑 M1
= − ln|𝑥𝑥 − 1|
𝑑𝑑𝑑𝑑
Sketch showing:
Curve approaching (1,1) with a vertical tangent at that point. G1 (ft)
Maximum point at (2,2). G1 (ft)
The curve crossing the 𝑥𝑥-axis for some 𝑥𝑥 > 2 and 𝑦𝑦 → −∞ as 𝑥𝑥 → ∞ G1 (ft)
6
(i) The shortest distance from 𝑂𝑂 to the line 𝐴𝐴𝐴𝐴 is (𝑅𝑅 + 𝑤𝑤) cos 𝛼𝛼 B1
1 1 1 M1
Since 𝜋𝜋 ≤ 𝛼𝛼 ≤ 𝜋𝜋, 0 ≤ cos 𝛼𝛼 ≤ .
3 2 2
1
Since 𝑤𝑤 < 𝑅𝑅, (𝑅𝑅 + 𝑤𝑤) cos 𝛼𝛼 < (𝑅𝑅 + 𝑅𝑅) = 𝑅𝑅, so the midpoint of the line 𝐴𝐴𝐴𝐴 lies
2
inside the smaller circle. E1
(ii)
(a) (𝑅𝑅 + 𝑑𝑑 )2 = (𝑅𝑅 + 𝑤𝑤)2 + 𝑑𝑑 2 − 2𝑑𝑑 (𝑅𝑅 + 𝑤𝑤) cos(𝜋𝜋 − 𝛼𝛼) M1
A1
𝑅𝑅2 + 2𝑅𝑅𝑅𝑅 + 𝑑𝑑 2 = 𝑅𝑅2 + 2𝑅𝑅𝑅𝑅 + 𝑤𝑤 2 + 𝑑𝑑 2 + 2𝑑𝑑(𝑅𝑅 + 𝑤𝑤) cos 𝛼𝛼
𝑤𝑤 (2𝑅𝑅 + 𝑤𝑤) M1
𝑑𝑑 =
2(𝑅𝑅 − (𝑅𝑅 + 𝑤𝑤) cos 𝛼𝛼 ) A1
(iii) 𝑤𝑤 𝑤𝑤 M1
𝑑𝑑 � � �2 + � 1 𝑤𝑤
= 𝑅𝑅 𝑅𝑅 ≈ × A1
𝑅𝑅 2 �1 − �1 + 𝑤𝑤 � cos 𝛼𝛼� 1 − cos 𝛼𝛼 𝑅𝑅
𝑅𝑅
1 𝑤𝑤 𝑑𝑑
1 − cos 𝛼𝛼 > and is much less than 1, so is much less than 1. E1
2 𝑅𝑅 𝑅𝑅
𝑑𝑑
� � sin 𝛼𝛼 𝑑𝑑
sin(𝛼𝛼 − 𝜃𝜃) = 𝑅𝑅 < M1
𝑑𝑑 𝑅𝑅
1+� �
𝑅𝑅
sin(𝛼𝛼 − 𝜃𝜃) is much less than 1 and so (𝛼𝛼 − 𝜃𝜃) is a small angle. M1
Therefore sin(𝛼𝛼 − 𝜃𝜃) ≈ 𝛼𝛼 − 𝜃𝜃, so 𝛼𝛼 − 𝜃𝜃 is much less than 1. E1
(iv) The longer length is (𝑅𝑅 + 𝑤𝑤) × 2𝛼𝛼
The shorter length is (𝑅𝑅 + 𝑑𝑑) × 2𝜃𝜃
𝑆𝑆 = 2𝛼𝛼 (𝑅𝑅 + 𝑤𝑤) − 2𝜃𝜃 (𝑅𝑅 + 𝑑𝑑 )
𝑆𝑆 = 2(𝑅𝑅 + 𝑑𝑑 + 𝑤𝑤 − 𝑑𝑑 )𝛼𝛼 − 2(𝑅𝑅 + 𝑑𝑑 )𝜃𝜃
𝑆𝑆 = 2(𝑅𝑅 + 𝑑𝑑 )(𝛼𝛼 − 𝜃𝜃) + 2(𝑤𝑤 − 𝑑𝑑 )𝛼𝛼 B1
𝑤𝑤 sin 𝛼𝛼 M1
𝛼𝛼 − 𝜃𝜃 ≈
𝑅𝑅(1 − cos 𝛼𝛼 )
cos 𝛼𝛼 𝑤𝑤 M1
𝑑𝑑 − 𝑤𝑤 ≈ ×
(1 − cos 𝛼𝛼 ) 𝑅𝑅
𝑤𝑤 sin 𝛼𝛼 cos 𝛼𝛼 𝑤𝑤
So 𝑆𝑆 ≈ 2(𝑅𝑅 + 𝑑𝑑 ) − 2 �(1−cos × � 𝛼𝛼
𝑅𝑅(1−cos 𝛼𝛼) 𝛼𝛼) 𝑅𝑅
As a fraction of the longer path length:
𝑆𝑆 𝑅𝑅 + 𝑑𝑑 𝛼𝛼 − 𝜃𝜃 𝑤𝑤 − 𝑑𝑑 sin 𝛼𝛼 𝑤𝑤 cos 𝛼𝛼 𝑤𝑤
= × + ≈ − M1
2𝛼𝛼 (𝑅𝑅 + 𝑤𝑤) 𝑅𝑅 + 𝑤𝑤 𝛼𝛼 𝑅𝑅 + 𝑤𝑤 𝛼𝛼 (1 − cos 𝛼𝛼 ) 𝑅𝑅 (1 − cos 𝛼𝛼 ) 𝑅𝑅
sin 𝛼𝛼 − 𝛼𝛼 cos 𝛼𝛼 𝑤𝑤 A1
𝑆𝑆 ≈ � � 𝑨𝑨𝑨𝑨
𝛼𝛼 (1 − cos 𝛼𝛼 ) 𝑅𝑅
7
(i) cos 𝜙𝜙 − sin 𝜙𝜙 M1
𝑹𝑹 = � �
sin 𝜙𝜙 cos 𝜙𝜙
1 + cos 𝜙𝜙 − sin 𝜙𝜙 A1
𝑹𝑹 + 𝑰𝑰 = � �
sin 𝜙𝜙 1 + cos 𝜙𝜙
𝑐𝑐𝑐𝑐𝑐𝑐 −𝑠𝑠𝑠𝑠𝑠𝑠 M1
This must also be of the form � �, so (1 + cos 𝜙𝜙)2 + sin2 𝜙𝜙 = 1
𝑠𝑠𝑠𝑠𝑠𝑠 𝑐𝑐𝑐𝑐𝑐𝑐
1 + 2 cos 𝜙𝜙 = 0
𝜙𝜙 = 120° 𝑜𝑜𝑜𝑜 240° A1
In either case, three consecutive rotations is equivalent to a rotation through 0°, so
𝑹𝑹3 = 𝑰𝑰 𝑨𝑨𝑨𝑨 E1
(ii) det(𝑺𝑺3 ) = 1
det(𝑺𝑺3 ) = det(𝑺𝑺)3
Therefore det(𝑺𝑺) = 1 𝑨𝑨𝑨𝑨 B1
Therefore 𝑎𝑎 + 𝑑𝑑 = −1 A1
(iv) For 0 ≤ 𝑡𝑡 ≤ 1:
0 ≤ 𝑡𝑡 𝑛𝑛 (1 − 𝑡𝑡)𝑛𝑛 ≤ 1 M1
0 ≤ 𝑒𝑒 𝑡𝑡 ≤ 𝑒𝑒 M1
𝑡𝑡 𝑛𝑛 (1−𝑡𝑡)𝑛𝑛 𝑒𝑒
0≤ 𝑒𝑒 𝑡𝑡 ≤ and equality can only occur at t=0 or t=1, so 𝑇𝑇𝑛𝑛 > 0 and is less
𝑛𝑛! 𝑛𝑛!
𝑒𝑒
than the area of a rectangle with width 1 and height .
𝑛𝑛!
𝑒𝑒
0 < 𝑇𝑇𝑛𝑛 < E1
𝑛𝑛!
Therefore 𝑎𝑎𝑛𝑛 + 𝑏𝑏𝑛𝑛 𝑒𝑒 → 0 as 𝑛𝑛 → ∞
𝑎𝑎
Therefore − 𝑛𝑛 → 𝑒𝑒 as 𝑛𝑛 → ∞ E1
𝑏𝑏𝑛𝑛
9
(i)
(a) The forces acting on the particle at 𝑃𝑃 are:
𝑊𝑊 = 𝑀𝑀𝑀𝑀 (directed downwards) M1
𝑇𝑇1 = 𝑚𝑚1 𝑔𝑔 (directed towards 𝑄𝑄) A1
𝑇𝑇2 = 𝑚𝑚2 𝑔𝑔 (directed towards 𝑅𝑅)
By the triangle inequality: dM1
𝑀𝑀𝑀𝑀 < 𝑚𝑚1 𝑔𝑔 + 𝑚𝑚2 𝑔𝑔
𝑀𝑀 < 𝑚𝑚1 + 𝑚𝑚2 A1
(b) 𝑄𝑄𝑄𝑄 = 𝑃𝑃𝑃𝑃 tan 𝜃𝜃1 and 𝑆𝑆𝑆𝑆 = 𝑃𝑃𝑃𝑃 tan 𝜃𝜃2
If 𝑆𝑆 divides 𝑄𝑄𝑄𝑄 in the ratio 𝑟𝑟: 1, then 𝑄𝑄𝑄𝑄 = 𝑟𝑟𝑟𝑟𝑟𝑟
tan 𝜃𝜃1 M1
𝑟𝑟 =
tan 𝜃𝜃2
By the sine rule:
sin 𝜃𝜃2 sin 𝜃𝜃1 M1
=
𝑚𝑚1 𝑔𝑔 𝑚𝑚2 𝑔𝑔
By the cosine rule:
𝑇𝑇12 + 𝑊𝑊 2 − 𝑇𝑇22 𝑚𝑚12 + 𝑀𝑀2 − 𝑚𝑚22 M1
cos 𝜃𝜃1 = =
2𝑇𝑇1 𝑊𝑊 2𝑚𝑚1 𝑀𝑀
Similarly:
𝑚𝑚22 + 𝑀𝑀2 − 𝑚𝑚12 M1
cos 𝜃𝜃2 =
2𝑚𝑚2 𝑀𝑀
Therefore:
sin 𝜃𝜃1 cos 𝜃𝜃2
𝑟𝑟 = ×
sin 𝜃𝜃2 cos 𝜃𝜃1
𝑚𝑚22 + 𝑀𝑀2 − 𝑚𝑚12 dM1
𝑚𝑚2 2𝑚𝑚2 𝑀𝑀 𝑚𝑚22 + 𝑀𝑀2 − 𝑚𝑚12
= × 2 = 𝑨𝑨𝑨𝑨
𝑚𝑚1 𝑚𝑚1 + 𝑀𝑀2 − 𝑚𝑚22 𝑚𝑚12 + 𝑀𝑀2 − 𝑚𝑚22 A1
2𝑚𝑚1 𝑀𝑀
(ii) From the triangle of forces, the angle between 𝑇𝑇1 and 𝑇𝑇2 must be 90° (Pythagoras)
Therefore 𝜃𝜃1 + 𝜃𝜃2 = 90° B1
By (i)(b)
𝑚𝑚22 M1
𝑟𝑟 =
𝑚𝑚12
Let 𝑑𝑑 be such that 𝑄𝑄𝑄𝑄 = 𝑚𝑚22 𝑑𝑑 and 𝑆𝑆𝑆𝑆 = 𝑚𝑚12 𝑑𝑑. M1
Since triangles 𝑃𝑃𝑃𝑃𝑃𝑃 and 𝑅𝑅𝑅𝑅𝑅𝑅 are similar: M1
𝑆𝑆𝑆𝑆 𝑅𝑅𝑅𝑅 M1
=
𝑄𝑄𝑄𝑄 𝑆𝑆𝑆𝑆
𝑃𝑃𝑆𝑆 2 = 𝑚𝑚12 𝑚𝑚22 𝑑𝑑 2 A1
Therefore, 𝑆𝑆𝑆𝑆 = 𝑚𝑚1 𝑚𝑚2 𝑑𝑑 and QR= (𝑚𝑚12 + 𝑚𝑚22 )𝑑𝑑, so the ratio of 𝑄𝑄𝑄𝑄 to 𝑆𝑆𝑆𝑆 is:
𝑀𝑀2 : 𝑚𝑚1 𝑚𝑚2 A1
10
(i) To remain stationary relative to the train the bead would have to have horizontal E1
acceleration 𝑎𝑎.
There is no horizontal force on the bead at the origin, so this is impossible. E1
𝑑𝑑 1 2 M1
� (𝑥𝑥˙ + 𝑦𝑦˙ 2 ) − 𝑎𝑎𝑎𝑎 + 𝑔𝑔𝑔𝑔� = 𝑥𝑥˙ (𝑥𝑥¨ − 𝑎𝑎) + (𝑦𝑦¨ + 𝑔𝑔)𝑦𝑦˙ = 0
𝑑𝑑𝑑𝑑 2
So the expression is constant during the motion. A1
(iii) 1 M1
Initially, (𝑥𝑥˙ 2 + 𝑦𝑦˙ 2 ) − 𝑎𝑎𝑎𝑎 + 𝑔𝑔𝑔𝑔 = 0 (and throughout the motion since it is constant)
2
At the maximum vertical displacement 𝑦𝑦˙ = 0.
𝑥𝑥˙ = 0 as well would only be possible at the origin (which is not maximum vertical
displacement, therefore 𝑥𝑥˙ = 0 and 𝑥𝑥 ≠ 0 M1
Therefore, 𝑎𝑎𝑎𝑎 = 𝑔𝑔𝑔𝑔
and so 𝑔𝑔2 𝑦𝑦 2 = 𝑎𝑎2 𝑥𝑥 2 = 𝑎𝑎2 𝑘𝑘𝑘𝑘 M1
Therefore, 𝑏𝑏 satisfies
𝑔𝑔2 𝑏𝑏 2 = 𝑎𝑎2 𝑘𝑘𝑘𝑘
𝑎𝑎2 𝑘𝑘
𝑏𝑏 = 2 A1
𝑔𝑔
𝑘𝑘 A1
Maximum speed is 𝑎𝑎�
2𝑔𝑔
When 𝑥𝑥 =
𝑎𝑎𝑎𝑎 A1
2𝑔𝑔
11
(i) 1 B1
𝑃𝑃2 =
2
(ii) If passenger 𝑇𝑇1 sits in seat 𝑆𝑆𝑘𝑘 (1 < 𝑘𝑘 < 𝑛𝑛) then passengers 𝑇𝑇2 to 𝑇𝑇𝑘𝑘−1 all sit in their E1
allocated seats.
The situation just before 𝑇𝑇𝑘𝑘 arrives is then the same as for a train that did not have E1
the (𝑘𝑘 − 1) seats that have been taken and for which 𝑇𝑇𝑘𝑘 had been allocated seat 𝑆𝑆1
1
𝑇𝑇1 sits in seat 𝑆𝑆1 with probability , after which all the remaining passengers will get
𝑛𝑛
their allocated seats.
1
𝑃𝑃(𝑇𝑇1 𝑠𝑠𝑠𝑠𝑠𝑠𝑠𝑠 𝑖𝑖𝑖𝑖 𝑆𝑆1 ∩ 𝑇𝑇𝑛𝑛 𝑠𝑠𝑠𝑠𝑠𝑠𝑠𝑠 𝑖𝑖𝑖𝑖 𝑆𝑆𝑛𝑛 ) = M1
𝑛𝑛
1
For 1 < 𝑘𝑘 < 𝑛𝑛, 𝑇𝑇1 sits in seat 𝑆𝑆𝑘𝑘 with probability , so
𝑛𝑛
1
𝑃𝑃 (𝑇𝑇1 𝑠𝑠𝑠𝑠𝑠𝑠𝑠𝑠 𝑖𝑖𝑖𝑖 𝑆𝑆𝑘𝑘 ∩ 𝑇𝑇𝑛𝑛 𝑠𝑠𝑠𝑠𝑠𝑠𝑠𝑠 𝑖𝑖𝑖𝑖 𝑆𝑆𝑛𝑛 ) = 𝑃𝑃𝑛𝑛−𝑘𝑘+1 M1
𝑛𝑛
If 𝑇𝑇1 sits in 𝑆𝑆𝑛𝑛 then it will not be possible for 𝑇𝑇𝑛𝑛 to sit in 𝑆𝑆𝑛𝑛
𝑛𝑛−1 𝑛𝑛−1
1 1 1
𝑃𝑃𝑛𝑛 = + � 𝑃𝑃𝑛𝑛−𝑘𝑘+1 = �1 + � 𝑃𝑃𝑟𝑟 � 𝑨𝑨𝑨𝑨 A1
𝑛𝑛 𝑛𝑛 𝑛𝑛
𝑘𝑘=2 𝑟𝑟=2
(iii) 1 B1
𝑃𝑃𝑛𝑛 =
2
Case where 𝑛𝑛 = 1 is shown in part (i)
1
Suppose 𝑃𝑃𝑘𝑘 = for 1 ≤ 𝑘𝑘 < 𝑛𝑛:
2
1 1 1 M1
𝑃𝑃𝑛𝑛 = �1 + (𝑛𝑛 − 2) × � = A1
𝑛𝑛 2 2
1 E1
Therefore, by induction 𝑃𝑃𝑛𝑛 =
2
(iv) 1 B1
𝑄𝑄2 =
2
For 𝑛𝑛 > 2:
For 1 < 𝑘𝑘 < 𝑛𝑛 − 1:
𝑃𝑃(𝑇𝑇𝑛𝑛−1 𝑠𝑠𝑠𝑠𝑠𝑠𝑠𝑠 𝑖𝑖𝑖𝑖 𝑆𝑆𝑛𝑛−1 |𝑇𝑇1 𝑠𝑠𝑠𝑠𝑠𝑠𝑠𝑠 𝑖𝑖𝑖𝑖 𝑆𝑆𝑘𝑘 ) = 𝑄𝑄𝑛𝑛−𝑘𝑘+1 M1
(by similar reasoning as in part (ii))
𝑃𝑃(𝑇𝑇𝑛𝑛−1 𝑠𝑠𝑠𝑠𝑠𝑠𝑠𝑠 𝑖𝑖𝑖𝑖 𝑆𝑆𝑛𝑛−1 |𝑇𝑇1 𝑠𝑠𝑠𝑠𝑠𝑠𝑠𝑠 𝑖𝑖𝑖𝑖 𝑆𝑆1 𝑜𝑜𝑜𝑜 𝑆𝑆𝑛𝑛 ) = 1 M1
Therefore
𝑛𝑛−2 𝑛𝑛−1
1 1
𝑄𝑄𝑛𝑛 = �2 + � 𝑄𝑄𝑛𝑛−𝑘𝑘+1 � = �2 + � 𝑄𝑄𝑛𝑛−𝑘𝑘+1 � A1
𝑛𝑛 𝑛𝑛
𝑘𝑘=2 𝑟𝑟=3
Base case:
If 𝑛𝑛 = 3, then 𝑇𝑇2 sits in seat 𝑆𝑆2 in any case where 𝑇𝑇1 does not sit in seat 𝑆𝑆2 B1
2 M1
Suppose 𝑄𝑄𝑘𝑘 = for some 3 ≤ 𝑘𝑘 < 𝑛𝑛:
3
1 2 2 A1
𝑄𝑄𝑛𝑛 = �2 + (𝑛𝑛 − 3) × � =
𝑛𝑛 3 3
2 E1
Therefore, by induction 𝑄𝑄𝑛𝑛 = for 𝑛𝑛 ≥ 3
3
12
(i) Player A wins the match on game 𝑛𝑛 with probability 𝑝𝑝𝐴𝐴 (1 − 𝑝𝑝𝐴𝐴 − 𝑝𝑝𝐵𝐵 )𝑛𝑛−1 B1
The probability that A wins the match is the sum to infinity of a geometric series with M1
𝑎𝑎 = 𝑝𝑝𝐴𝐴 , 𝑟𝑟 = 1 − 𝑝𝑝𝐴𝐴 − 𝑝𝑝𝐵𝐵 M1
𝑝𝑝𝐴𝐴 M1
𝑨𝑨𝑨𝑨
𝑝𝑝𝐴𝐴 + 𝑝𝑝𝐵𝐵 A1
(ii) The difference between the number of games won by the two players is initially 0
and either increases or decreases by 1 after each game. E1
Therefore, it can only be an even number (and so the match can only be won) after
an even number of games. E1
Considering pairs of turns at a time M1
The game is equivalent to that in part (i), with 𝑝𝑝𝐴𝐴 = 𝑝𝑝2 and 𝑝𝑝𝐵𝐵 = 𝑞𝑞 2 , M1
and 0 < 𝑝𝑝𝐴𝐴 + 𝑝𝑝𝐵𝐵 < 1 M1
so the probability that A wins the match is
𝑝𝑝2 A1
𝑨𝑨𝑨𝑨
𝑝𝑝2 + 𝑞𝑞 2
(iii) Version 1:
The player has to win round 1 for the game to continue (with probability 𝑝𝑝). M1
Following that the game is equivalent to that in part (ii), so the probability that the M1
player wins overall is
𝑝𝑝3
𝑝𝑝2 + 𝑞𝑞 2 A1
Version 2:
The only way for the player to win is by winning two rounds in a row, so with M1
probability
𝑝𝑝2 A1
𝑑𝑑𝑑𝑑
𝑦𝑦 = 12 sin 𝑡𝑡 − 4 sin3 𝑡𝑡 so = 12 cos 𝑡𝑡 − 12 sin2 𝑡𝑡 cos 𝑡𝑡 = 12 cos 𝑡𝑡 (1 − sin2 𝑡𝑡) = 12 cos 3 𝑡𝑡
𝑑𝑑𝑑𝑑
M1
𝑑𝑑𝑑𝑑 12 cos3 𝑡𝑡
So = = cot 𝑡𝑡 A1
𝑑𝑑𝑑𝑑 12 cos2 𝑡𝑡 sin 𝑡𝑡
This simplifies to 𝑥𝑥 sin 𝜑𝜑 + 𝑦𝑦 cos 𝜑𝜑 = 12 sin 𝜑𝜑 cos 𝜑𝜑 − 4 sin3 𝜑𝜑 cos 𝜑𝜑 − 4 sin 𝜑𝜑 cos 3 𝜑𝜑
𝑑𝑑𝑑𝑑 𝑑𝑑𝑑𝑑
For 𝑥𝑥 = 8 cos 3 𝑡𝑡 , = −24 cos 2 𝑡𝑡 sin 𝑡𝑡 and for 𝑦𝑦 = 8 sin3 𝑡𝑡 , = 24 sin2 𝑡𝑡 cos 𝑡𝑡
𝑑𝑑𝑑𝑑 𝑑𝑑𝑑𝑑
This simplifies to
𝑥𝑥 sin 𝜑𝜑 + 𝑦𝑦 cos 𝜑𝜑 = 8 sin3 𝜑𝜑 cos 𝜑𝜑 + 8 sin 𝜑𝜑 cos 3 𝜑𝜑 = 8 sin 𝜑𝜑 cos 𝜑𝜑 (sin2 𝜑𝜑 + cos 2 𝜑𝜑)
That is 𝑥𝑥 sin 𝜑𝜑 + 𝑦𝑦 cos 𝜑𝜑 = 8 sin 𝜑𝜑 cos 𝜑𝜑 as required. A1 (4)
Alternative 1
the normal is a tangent to the second curve if it has the same gradient and the point
(8 cos 3 𝜑𝜑 , 8 sin3 𝜑𝜑 ) lies on the normal. M1
Substitution 𝑥𝑥 sin 𝜑𝜑 + 𝑦𝑦 cos 𝜑𝜑 = 8 sin 𝜑𝜑 cos 3 𝜑𝜑 + 8 sin3 𝜑𝜑 cos 𝜑𝜑 = 8 sin 𝜑𝜑 cos 𝜑𝜑 (sin2 𝜑𝜑 +
cos 2 𝜑𝜑) = 8 sin 𝜑𝜑 cos 𝜑𝜑 as required or 𝑥𝑥 tan 𝜑𝜑 + 𝑦𝑦 = 8 sin 𝜑𝜑 cos 𝜑𝜑 (sin2 𝜑𝜑 + cos 2 𝜑𝜑) A1
Alternative 2
2 −1 2 −1 𝑑𝑑𝑑𝑑
𝑥𝑥 3 + 𝑦𝑦 3 =0
3 3 𝑑𝑑𝑑𝑑
M1
𝑑𝑑𝑑𝑑
(ii) 𝑥𝑥 = cos 𝑡𝑡 + 𝑡𝑡 sin 𝑡𝑡 so = − sin 𝑡𝑡 + 𝑡𝑡 cos 𝑡𝑡 + sin 𝑡𝑡 = 𝑡𝑡 cos 𝑡𝑡
𝑑𝑑𝑑𝑑
𝑑𝑑𝑑𝑑
𝑦𝑦 = sin 𝑡𝑡 − 𝑡𝑡 cos 𝑡𝑡 so = cos 𝑡𝑡 − cos 𝑡𝑡 + 𝑡𝑡 sin 𝑡𝑡 = 𝑡𝑡 sin 𝑡𝑡 M1
𝑑𝑑𝑑𝑑
𝑑𝑑𝑑𝑑
So = tan 𝑡𝑡 A1
𝑑𝑑𝑑𝑑
M1 A1ft
Alternatives which can be followed through to perpendicular distance step, or alternative method #
are
M1 A1ft A1
M1 A1ft
So the curve to which this normal is a tangent is a circle centre (0,0) , radius 1 which is thus
𝑥𝑥 2 + 𝑦𝑦 2 = 1 M1 A1 (5)
1 −𝑥𝑥 𝑥𝑥 𝑎𝑎 𝑎𝑎 − (𝑏𝑏 − 𝑐𝑐 )𝑥𝑥 𝑎𝑎 − 𝑎𝑎 0
2. (i) � 𝑦𝑦 1 −𝑦𝑦� �𝑏𝑏 � = �𝑏𝑏 − (𝑐𝑐 − 𝑎𝑎)𝑦𝑦� = �𝑏𝑏 − 𝑏𝑏 � = �0� as required. M1 A1*
−𝑧𝑧 𝑧𝑧 1 𝑐𝑐 𝑐𝑐 − (𝑎𝑎 − 𝑏𝑏)𝑧𝑧 𝑐𝑐 − 𝑐𝑐 0
𝑎𝑎 0 0
−1
As a, b and c are distinct, they cannot all be zero. If 𝑀𝑀 exists �𝑏𝑏 � = 𝑀𝑀−1 �0� = �0� which is a
𝑐𝑐 0 0
contradiction.
1 −𝑥𝑥 𝑥𝑥
So, 𝑀𝑀−1 does not exist and thus 𝑑𝑑𝑑𝑑𝑑𝑑 � 𝑦𝑦 1 −𝑦𝑦� = 0 , M1
−𝑧𝑧 𝑧𝑧 1
i.e. 1 − 𝑥𝑥𝑥𝑥𝑥𝑥 + 𝑥𝑥𝑥𝑥𝑥𝑥 + 𝑦𝑦𝑦𝑦 + 𝑧𝑧𝑧𝑧 + 𝑥𝑥𝑥𝑥 = 0 , (Sarus)
which simplifies to
(𝑥𝑥 + 𝑦𝑦 + 𝑧𝑧)2 ≥ 0
B1 M1 A1
2 −𝑥𝑥 −𝑥𝑥 𝑎𝑎 0
As a, b and c are positive, they cannot all be zero. Thus as �−𝑦𝑦 2 −𝑦𝑦� �𝑏𝑏 � = �0� ,
−𝑧𝑧 −𝑧𝑧 2 𝑐𝑐 0
2 −𝑥𝑥 −𝑥𝑥
as in part (i), 𝑑𝑑𝑑𝑑𝑑𝑑 �−𝑦𝑦 2 −𝑦𝑦� = 0 ,
−𝑧𝑧 −𝑧𝑧 2
i.e. 8 − 𝑥𝑥𝑥𝑥𝑥𝑥 − 𝑥𝑥𝑥𝑥𝑥𝑥 − 2𝑦𝑦𝑦𝑦 − 2𝑧𝑧𝑧𝑧 − 2𝑥𝑥𝑥𝑥 = 0 , that is M1 A1
M1 A1
Multiplying by (𝑏𝑏 + 𝑐𝑐 )(𝑐𝑐 + 𝑎𝑎)(𝑎𝑎 + 𝑏𝑏) , all three factors of which are positive, gives
(2𝑎𝑎 + 𝑏𝑏 + 𝑐𝑐 )(𝑎𝑎 + 2𝑏𝑏 + 𝑐𝑐 )(𝑎𝑎 + 𝑏𝑏 + 𝑐𝑐 ) > 5(𝑏𝑏 + 𝑐𝑐 )(𝑐𝑐 + 𝑎𝑎)(𝑎𝑎 + 𝑏𝑏) as required. A1* (4)
2𝑎𝑎 2𝑎𝑎 2𝑏𝑏 2𝑐𝑐
𝑥𝑥 = > as a, b, and c are positive, and similarly both, 𝑦𝑦 > and 𝑧𝑧 >
𝑏𝑏+𝑐𝑐 𝑎𝑎+𝑏𝑏+𝑐𝑐 𝑎𝑎+𝑏𝑏+𝑐𝑐 𝑎𝑎+𝑏𝑏+𝑐𝑐
M1
2𝑎𝑎 2𝑏𝑏 2𝑐𝑐 2(𝑎𝑎+𝑏𝑏+𝑐𝑐)
Thus 4 + 𝑥𝑥 + 𝑦𝑦 + 𝑧𝑧 + 1 > 4 + + + +1=4+ +1=7
𝑎𝑎+𝑏𝑏+𝑐𝑐 𝑎𝑎+𝑏𝑏+𝑐𝑐 𝑎𝑎+𝑏𝑏+𝑐𝑐 𝑎𝑎+𝑏𝑏+𝑐𝑐
dM1
and thus following the argument used to obtain the previous result
(2𝑎𝑎 + 𝑏𝑏 + 𝑐𝑐 )(𝑎𝑎 + 2𝑏𝑏 + 𝑐𝑐 )(𝑎𝑎 + 𝑏𝑏 + 𝑐𝑐 ) > 7(𝑏𝑏 + 𝑐𝑐 )(𝑐𝑐 + 𝑎𝑎)(𝑎𝑎 + 𝑏𝑏) as required.
A1* (3)
3. (i)
𝛽𝛽
1 1
(𝐼𝐼𝑛𝑛+1 + 𝐼𝐼𝑛𝑛−1 ) = �(sec 𝑥𝑥 + tan 𝑥𝑥 )𝑛𝑛+1 + (sec 𝑥𝑥 + tan 𝑥𝑥 )𝑛𝑛−1 𝑑𝑑𝑑𝑑
2 2
0
𝛽𝛽
1
= �(sec 𝑥𝑥 + tan 𝑥𝑥 )𝑛𝑛−1 ((sec 𝑥𝑥 + tan 𝑥𝑥 )2 + 1) 𝑑𝑑𝑑𝑑
2
0
M1
𝛽𝛽
1
= �(sec 𝑥𝑥 + tan 𝑥𝑥 )𝑛𝑛−1 (sec 2 𝑥𝑥 + 2 sec 𝑥𝑥 tan 𝑥𝑥 + tan2 𝑥𝑥 + 1) 𝑑𝑑𝑑𝑑
2
0
𝛽𝛽
M1
𝛽𝛽
1 1
= � sec 𝑥𝑥 + tan 𝑥𝑥 � = ((sec 𝛽𝛽 + tan 𝛽𝛽 )𝑛𝑛 − 1)
( ) 𝑛𝑛
𝑛𝑛 0 𝑛𝑛
M1 A1 *A1 (5)
as required.
1 1
(𝐼𝐼𝑛𝑛+1 + 𝐼𝐼𝑛𝑛−1 ) − 𝐼𝐼𝑛𝑛 = (𝐼𝐼𝑛𝑛+1 − 2𝐼𝐼𝑛𝑛 + 𝐼𝐼𝑛𝑛−1 )
2 2
𝛽𝛽
1
= �(sec 𝑥𝑥 + tan 𝑥𝑥 )𝑛𝑛+1 − 2(sec 𝑥𝑥 + tan 𝑥𝑥 )𝑛𝑛 + (sec 𝑥𝑥 + tan 𝑥𝑥 )𝑛𝑛−1 𝑑𝑑𝑑𝑑
2
0
M1
𝛽𝛽
1 2
= �(sec 𝑥𝑥 + tan 𝑥𝑥 )𝑛𝑛−1 �(sec 𝑥𝑥 + tan 𝑥𝑥 ) − 1� 𝑑𝑑𝑑𝑑
2
0
M1 A1
2
�(sec 𝑥𝑥 + tan 𝑥𝑥 ) − 1� > 0 for all x>0
𝜋𝜋
sec 𝑥𝑥 ≥ 1 for 0 ≤ 𝑥𝑥 < and hence for 0 ≤ x < 𝛽𝛽 and similarly tan 𝑥𝑥 ≥ 0 , and thus also
2
(sec 𝑥𝑥 + tan 𝑥𝑥 )𝑛𝑛−1 > 0 . E1
1
Therefore, (𝐼𝐼𝑛𝑛+1 + 𝐼𝐼𝑛𝑛−1 ) − 𝐼𝐼𝑛𝑛 > 0 , A1
2
1 1
and so 𝐼𝐼𝑛𝑛 < (𝐼𝐼𝑛𝑛+1 + 𝐼𝐼𝑛𝑛−1 ) = ((sec 𝛽𝛽 + tan 𝛽𝛽 )𝑛𝑛 − 1) as required. M1 *A1 (7)
2 𝑛𝑛
Alternative 1: it has already been shown that
1 𝛽𝛽
(𝐼𝐼𝑛𝑛+1 + 𝐼𝐼𝑛𝑛−1 ) = ∫0 (sec 𝑥𝑥 + tan 𝑥𝑥 )𝑛𝑛−1 (sec 2 𝑥𝑥 + sec 𝑥𝑥 tan 𝑥𝑥 ) 𝑑𝑑𝑑𝑑
2
𝛽𝛽
which is greater than 𝐼𝐼𝑛𝑛 as the expression being integrated is greater than (sec 𝑥𝑥 + tan 𝑥𝑥 )𝑛𝑛 because
sec 𝑥𝑥 > 0 over this domain.
Alternative 2:-
𝛽𝛽
𝛽𝛽
M1 A1 A1
For 0 < x < 𝛽𝛽 , sec 𝑥𝑥 > 1 , tan 𝑥𝑥 > 0 so sec 𝑥𝑥 + tan 𝑥𝑥 > 1 E1 and thus 𝐼𝐼𝑛𝑛+1 − 𝐼𝐼𝑛𝑛 > 𝐼𝐼𝑛𝑛 − 𝐼𝐼𝑛𝑛−1 A1
1 1
and so 𝐼𝐼𝑛𝑛 ≤ (𝐼𝐼𝑛𝑛+1 + 𝐼𝐼𝑛𝑛−1 ) = ((sec 𝛽𝛽 + tan 𝛽𝛽 )𝑛𝑛 − 1) M1 *A1 (7)
2 𝑛𝑛
1 1 𝛽𝛽
(ii)
2
(𝐽𝐽𝑛𝑛+1 + 𝐽𝐽𝑛𝑛−1 ) =
2
∫0 (sec 𝑥𝑥 cos 𝛽𝛽 + tan 𝑥𝑥 )𝑛𝑛+1 + (sec 𝑥𝑥 cos 𝛽𝛽 + tan 𝑥𝑥 )𝑛𝑛−1 𝑑𝑑𝑑𝑑
𝛽𝛽
1
= �(sec 𝑥𝑥 cos 𝛽𝛽 + tan 𝑥𝑥 )𝑛𝑛−1 ((sec 𝑥𝑥 cos 𝛽𝛽 + tan 𝑥𝑥 )2 + 1) 𝑑𝑑𝑑𝑑
2
0
M1
𝛽𝛽
1
= �(sec 𝑥𝑥 cos 𝛽𝛽 + tan 𝑥𝑥 )𝑛𝑛−1 (sec 2 𝑥𝑥 cos 2 𝛽𝛽 + 2 sec 𝑥𝑥 cos 𝛽𝛽 tan 𝑥𝑥 + tan2 𝑥𝑥 + 1) 𝑑𝑑𝑑𝑑
2
0
𝛽𝛽
1
= �(sec 𝑥𝑥 cos 𝛽𝛽 + tan 𝑥𝑥 )𝑛𝑛−1 (sec 2 𝑥𝑥 (1 − sin2 𝛽𝛽 ) + 2 sec 𝑥𝑥 cos 𝛽𝛽 tan 𝑥𝑥 + tan2 𝑥𝑥 + 1) 𝑑𝑑𝑑𝑑
2
0
𝛽𝛽
= �(sec 𝑥𝑥 cos 𝛽𝛽 + tan 𝑥𝑥 )𝑛𝑛−1 �(sec 2 𝑥𝑥 + sec 𝑥𝑥 cos 𝛽𝛽 tan 𝑥𝑥 ) − sec 2 𝑥𝑥 sin2 𝛽𝛽�𝑑𝑑𝑑𝑑
0
M1
𝛽𝛽
𝛽𝛽
1
�(sec 𝑥𝑥 cos 𝛽𝛽 + tan 𝑥𝑥 )𝑛𝑛−1 (sec 2 𝑥𝑥 + sec 𝑥𝑥 cos 𝛽𝛽 tan 𝑥𝑥 )𝑑𝑑𝑑𝑑 = � (sec 𝑥𝑥 cos 𝛽𝛽 + tan 𝑥𝑥 )𝑛𝑛 �
𝑛𝑛 0
0
M1
1
= ((1 + tan 𝛽𝛽 )𝑛𝑛 − cos 𝑛𝑛 𝛽𝛽 )
𝑛𝑛
A1
𝛽𝛽
by a similar argument to part (i), namely sec 2 𝑥𝑥 sin2 𝛽𝛽 > 0 for any x, and sec 𝑥𝑥 cos 𝛽𝛽 + tan 𝑥𝑥 > 0
𝜋𝜋
as sec 𝑥𝑥 > 0 and tan 𝑥𝑥 ≥ 0 for 0 ≤ x < 𝛽𝛽 < E1
2
1 1
Hence (𝐽𝐽𝑛𝑛+1 + 𝐽𝐽𝑛𝑛−1 ) < ((1 + tan 𝛽𝛽 )𝑛𝑛 − cos 𝑛𝑛 𝛽𝛽 ) A1
2 𝑛𝑛
But
𝛽𝛽
1 1 2
(𝐽𝐽𝑛𝑛+1 + 𝐽𝐽𝑛𝑛−1 ) − 𝐽𝐽𝑛𝑛 == �(sec 𝑥𝑥 cos 𝛽𝛽 + tan 𝑥𝑥 )𝑛𝑛−1 �(sec 𝑥𝑥 cos 𝛽𝛽 + tan 𝑥𝑥 ) − 1� 𝑑𝑑𝑑𝑑 > 0
2 2
0
M1
1 1
as before, and thus 𝐽𝐽𝑛𝑛 < (𝐽𝐽𝑛𝑛+1 + 𝐽𝐽𝑛𝑛−1 ) < ((1 + tan 𝛽𝛽 )𝑛𝑛 − cos 𝑛𝑛 𝛽𝛽 ) as required. *A1 (8)
2 𝑛𝑛
4. (i)
1 1
𝒎𝒎. 𝒂𝒂 = (𝒂𝒂 + 𝒃𝒃). 𝒂𝒂 = (1 + 𝒂𝒂. 𝒃𝒃) = 𝑚𝑚 cos 𝛼𝛼 where 𝛼𝛼 is the non-reflex angle between a and m
2 2
1 1
𝒎𝒎. 𝒃𝒃 = (𝒂𝒂 + 𝒃𝒃). 𝒃𝒃 = (1 + 𝒂𝒂. 𝒃𝒃) = 𝑚𝑚 cos 𝛽𝛽 where 𝛼𝛼 is the non-reflex angle between 𝒃𝒃 and m
2 2
M1 A1
Thus cos 𝛼𝛼 = cos 𝛽𝛽 and so 𝛼𝛼 = 𝛽𝛽 as for 0 ≤ 𝜏𝜏 ≤ 𝜋𝜋 , there is only one value of 𝜏𝜏 for any given
value of cos 𝜏𝜏 . E1 (3)
(ii) 𝒂𝒂𝟏𝟏 . 𝒄𝒄 = (𝒂𝒂 − (𝒂𝒂. 𝒄𝒄)𝒄𝒄). 𝒄𝒄 = 𝒂𝒂. 𝒄𝒄 − 𝒂𝒂. 𝒄𝒄 𝒄𝒄. 𝒄𝒄 = 0 as required. *B1
|𝒂𝒂𝟏𝟏 |𝟐𝟐 = 𝒂𝒂𝟏𝟏 . 𝒂𝒂𝟏𝟏 = (𝒂𝒂 − (𝒂𝒂. 𝒄𝒄)𝒄𝒄). (𝒂𝒂 − (𝒂𝒂. 𝒄𝒄)𝒄𝒄) = 𝒂𝒂. 𝒂𝒂 − 2𝒂𝒂. 𝒄𝒄 𝒂𝒂. 𝒄𝒄 + 𝒂𝒂. 𝒄𝒄 𝒂𝒂. 𝒄𝒄 𝒄𝒄. 𝒄𝒄
M1
𝒂𝒂𝟏𝟏 . 𝒃𝒃𝟏𝟏 = (𝒂𝒂 − (𝒂𝒂. 𝒄𝒄)𝒄𝒄). (𝒃𝒃 − (𝒃𝒃. 𝒄𝒄)𝒄𝒄) = 𝒂𝒂. 𝒃𝒃 − 2(𝒂𝒂. 𝒄𝒄)(𝒃𝒃. 𝒄𝒄) + (𝒂𝒂. 𝒄𝒄)(𝒃𝒃. 𝒄𝒄)(𝒄𝒄. 𝒄𝒄)
= cos 𝜃𝜃 − cos 𝛼𝛼 cos 𝛽𝛽
M1 A1
and hence,
cos 𝜃𝜃 − cos 𝛼𝛼 cos 𝛽𝛽
cos 𝜑𝜑 =
sin 𝛼𝛼 sin 𝛽𝛽
as required. *A1 (8)
1 1 1
(iii) 𝒎𝒎𝟏𝟏 = 𝒎𝒎 − (𝒎𝒎. 𝒄𝒄)𝒄𝒄 = (𝒂𝒂 + 𝒃𝒃) − � (𝒂𝒂 + 𝒃𝒃). 𝒄𝒄� 𝒄𝒄 = (𝒂𝒂𝟏𝟏 + 𝒃𝒃𝟏𝟏 ) B1
2 2 2
𝒎𝒎𝟏𝟏 bisects the angle between 𝒂𝒂𝟏𝟏 and 𝒃𝒃𝟏𝟏 if and only if
𝒎𝒎𝟏𝟏 . 𝒂𝒂𝟏𝟏 𝒎𝒎𝟏𝟏 . 𝒃𝒃𝟏𝟏
=
sin 𝛼𝛼 sin 𝛽𝛽
M1
A1
So
(𝒂𝒂𝟏𝟏 . 𝒃𝒃𝟏𝟏 − sin 𝛼𝛼 sin 𝛽𝛽 )(sin 𝛼𝛼 − sin 𝛽𝛽 ) = 0
A1
and thus, sin 𝛼𝛼 = sin 𝛽𝛽 in which case 𝛼𝛼 = 𝛽𝛽 as both angles are acute, *A1
or cos 𝜃𝜃 − cos 𝛼𝛼 cos 𝛽𝛽 = sin 𝛼𝛼 sin 𝛽𝛽 , meaning that cos 𝜃𝜃 = cos 𝛼𝛼 cos 𝛽𝛽 + sin 𝛼𝛼 sin 𝛽𝛽 = cos(𝛼𝛼 − 𝛽𝛽 )
M1 *A1 (9)
5. (i) The curves meet when 𝑎𝑎 + 2 cos 𝜃𝜃 = 2 + cos 2𝜃𝜃
Alternatively, for the curves to touch, they must have the same gradient, so differentiating,
1 1 𝜋𝜋 3 𝜋𝜋
(ii) If 𝑎𝑎 = then at points where they touch, cos 𝜃𝜃 = so 𝜃𝜃 = ± and thus � , ± �. M1A1
2 2 3 2 3
5
𝑟𝑟 = 𝑎𝑎 + 2 cos 𝜃𝜃 is symmetrical about the initial line which it intercepts at � , 0 � and has a cusp at
2
1 1 𝜋𝜋
�0 , ± cos −1 �− � � . It passes through � , ± � and only exists for
4 2 2
1 1
− cos −1 �− � < 𝜃𝜃 < cos −1 �− � .
4 4
𝑟𝑟 = 2 + cos 2𝜃𝜃 is symmetrical about both the initial line, and its perpendicular. It passes through
𝜋𝜋
(3,0) , (3, 𝜋𝜋) , and �1 , ± �
2
Sketch G6 (8)
(iii) If 𝑎𝑎 = 1 , then the curves meet where 2 cos 2 𝜃𝜃 − 2 cos 𝜃𝜃 = 0 , i.e. cos 𝜃𝜃 = 1 at (3,0) where
𝜋𝜋
they touch, and cos 𝜃𝜃 = 0 at �1, ± �
2
𝑟𝑟 = 𝑎𝑎 + 2 cos 𝜃𝜃 is symmetrical about the initial line which it intercepts at (3 , 0 ) and has a cusp at
1 2𝜋𝜋 𝜋𝜋
�0 , ± cos −1 �− � � = �0 , ± � . It passes through �1 , ± � and only exists for
2 3 2
2𝜋𝜋 2𝜋𝜋
− < 𝜃𝜃 < .
3 3
Sketch G3
If 𝑎𝑎 = 5 , then the curves meet where 2 cos 2 𝜃𝜃 − 2 cos 𝜃𝜃 − 4 = 0 , i.e. only cos 𝜃𝜃 = −1 at (3, 𝜋𝜋)
where they touch, as cos 𝜃𝜃 ≠ 2 .
Sketch G3 (6)
6. (i)
𝑥𝑥 tan 𝛼𝛼 + 1
𝑓𝑓𝛼𝛼 (𝑥𝑥 ) = tan−1 � �
tan 𝛼𝛼 − 𝑥𝑥
1 (tan 𝛼𝛼 − 𝑥𝑥 ) tan 𝛼𝛼 + (𝑥𝑥 tan 𝛼𝛼 + 1)
𝑓𝑓′𝛼𝛼 (𝑥𝑥 ) = 2
𝑥𝑥 tan 𝛼𝛼 + 1 (tan 𝛼𝛼 − 𝑥𝑥 )2
1+� �
tan 𝛼𝛼 − 𝑥𝑥
M1 A1
tan2 𝛼𝛼 + 1
=
(tan 𝛼𝛼 − 𝑥𝑥 )2 + (𝑥𝑥 tan 𝛼𝛼 + 1)2
sec 2 𝛼𝛼 sec 2 𝛼𝛼 1
= = =
tan 𝛼𝛼 + 𝑥𝑥 + 𝑥𝑥 tan 𝛼𝛼 + 1 sec 𝛼𝛼 (1 + 𝑥𝑥 ) 1 + 𝑥𝑥 2
2 2 2 2 2 2
M1 M1 *A1 (5)
as required.
Alternative
𝑥𝑥 tan 𝛼𝛼 + 1
𝑓𝑓𝛼𝛼 (𝑥𝑥 ) = tan−1 � �
tan 𝛼𝛼 − 𝑥𝑥
𝑥𝑥 + cot 𝛼𝛼
= tan−1 � �
1 − 𝑥𝑥 cot 𝛼𝛼
𝜋𝜋
tan(tan−1 𝑥𝑥 ) + tan � − 𝛼𝛼�
= tan −1 � 2 �
𝜋𝜋
1 − tan(tan 𝑥𝑥 ) tan � − 𝛼𝛼�
−1
2
M1 A1
𝜋𝜋
= tan−1 �tan �tan−1 𝑥𝑥 + − 𝛼𝛼��
2
M1
𝜋𝜋 𝜋𝜋
= tan−1 𝑥𝑥 + − 𝛼𝛼 if this is less than , i.e. if 𝑥𝑥 < tan 𝛼𝛼
2 2
𝜋𝜋
or = tan−1 𝑥𝑥 − − 𝛼𝛼 if 𝑥𝑥 > tan 𝛼𝛼 M1
2
𝑑𝑑 1
So 𝑓𝑓′𝛼𝛼 (𝑥𝑥 ) = (tan−1 𝑥𝑥 ) = *A1 (5)
𝑑𝑑𝑑𝑑 1+𝑥𝑥 2
Sketch G1 G1 G1 (3)
𝜋𝜋 𝜋𝜋
�− − 𝛼𝛼� − � − 𝛽𝛽� = β − α − π for tan 𝛼𝛼 < 𝑥𝑥 < tan 𝛽𝛽
2 2
𝜋𝜋 𝜋𝜋
and �− − 𝛼𝛼� − �− − 𝛽𝛽� = β − α for 𝑥𝑥 > tan 𝛽𝛽
2 2
Sketch G1 G1 G1 (3)
Sketch G1 G1 G1 G1 (4)
7.
𝑒𝑒 𝑖𝑖𝑖𝑖 + 𝑒𝑒 𝑖𝑖𝑖𝑖
𝑧𝑧 =
𝑒𝑒 𝑖𝑖𝑖𝑖 − 𝑒𝑒 𝑖𝑖𝑖𝑖
cos 𝜃𝜃 + 𝑖𝑖 sin 𝜃𝜃 + cos 𝜑𝜑 + 𝑖𝑖 sin 𝜑𝜑
=
cos 𝜃𝜃 + 𝑖𝑖 sin 𝜃𝜃 − cos 𝜑𝜑 − 𝑖𝑖 sin 𝜑𝜑
M1
𝜃𝜃 + 𝜑𝜑 𝜃𝜃 − 𝜑𝜑 𝜃𝜃 + 𝜑𝜑 𝜃𝜃 − 𝜑𝜑
2 cos cos + 2𝑖𝑖 sin cos
= 2 2 2 2
𝜃𝜃 + 𝜑𝜑 𝜃𝜃 − 𝜑𝜑 𝜃𝜃 + 𝜑𝜑 𝜃𝜃 − 𝜑𝜑
−2 sin sin + 2𝑖𝑖 cos sin
2 2 2 2
M1 A1 A1
𝜃𝜃 − 𝜑𝜑 𝜃𝜃 + 𝜑𝜑 𝜃𝜃 + 𝜑𝜑
2 cos �cos + 𝑖𝑖 sin �
= 2 2 2
𝜃𝜃 − 𝜑𝜑 𝜃𝜃 + 𝜑𝜑 𝜃𝜃 + 𝜑𝜑
2 sin �𝑖𝑖 cos − sin �
2 2 2
𝜃𝜃 − 𝜑𝜑
= −𝑖𝑖 cot
2
𝜑𝜑 − 𝜃𝜃
= 𝑖𝑖 cot
2
*A1 (5)
as required.
Alternatively,
𝜃𝜃−𝜑𝜑 𝜃𝜃−𝜑𝜑 𝜃𝜃 − 𝜑𝜑
𝑖𝑖� � −𝑖𝑖� �
𝑒𝑒 𝑖𝑖𝑖𝑖 + 𝑒𝑒 𝑖𝑖𝑖𝑖 𝑒𝑒 2 + 𝑒𝑒 2 2 cos
𝑧𝑧 = 𝑖𝑖𝑖𝑖 = 𝜃𝜃−𝜑𝜑 = 2 = −𝑖𝑖 cot 𝜃𝜃 − 𝜑𝜑 = 𝑖𝑖 cot 𝜑𝜑 − 𝜃𝜃
𝑒𝑒 − 𝑒𝑒 𝑖𝑖𝑖𝑖
𝑖𝑖� � −𝑖𝑖�
𝜃𝜃−𝜑𝜑
� 𝜃𝜃 − 𝜑𝜑 2 2
𝑒𝑒 2 − 𝑒𝑒 2 2𝑖𝑖 sin
2
M1 M1 A1 A1 *A1 (5)
𝜃𝜃 − 𝜑𝜑
|𝑧𝑧| = �cot �
2
M1 A1
𝜋𝜋
|arg 𝑧𝑧| =
2
𝜋𝜋 3𝜋𝜋
[or arg 𝑧𝑧 = 𝑜𝑜𝑜𝑜 ]
2 2
M1 A1 (4)
(ii) Let 𝑎𝑎 = 𝑒𝑒 𝑖𝑖𝑖𝑖 and 𝑏𝑏 = 𝑒𝑒 𝑖𝑖𝑖𝑖 M1 then 𝑥𝑥 = 𝑎𝑎 + 𝑏𝑏 = 𝑒𝑒 𝑖𝑖𝑖𝑖 + 𝑒𝑒 𝑖𝑖𝑖𝑖 and 𝐴𝐴𝐴𝐴 = 𝑏𝑏 − 𝑎𝑎 = 𝑒𝑒 𝑖𝑖𝑖𝑖 − 𝑒𝑒 𝑖𝑖𝑖𝑖
𝑥𝑥 𝑒𝑒 𝑖𝑖𝑖𝑖 + 𝑒𝑒 𝑖𝑖𝑖𝑖
arg 𝑥𝑥 − arg 𝐴𝐴𝐴𝐴 = arg = arg 𝑖𝑖𝑖𝑖
𝐴𝐴𝐴𝐴 𝑒𝑒 − 𝑒𝑒 𝑖𝑖𝑖𝑖
𝜋𝜋
so using (i), |arg 𝑥𝑥 − arg 𝐴𝐴𝐴𝐴| = A1 and thus OX and AB are perpendicular, since 𝑥𝑥 = 𝑎𝑎 + 𝑏𝑏 ≠ 0
2
and 𝑎𝑎 ≠ 𝑏𝑏 as A and B are distinct. E1 (3)
Alternative:- 0, 𝑎𝑎, 𝑎𝑎 + 𝑏𝑏, 𝑏𝑏 define a rhombus OAXB as |𝑎𝑎| = |𝑏𝑏| = 1. Diagonals of a rhombus are
perpendicular (and bisect one another).
𝐴𝐴𝐴𝐴 𝑏𝑏 + 𝑐𝑐
=
𝐵𝐵𝐵𝐵 𝑐𝑐 − 𝑏𝑏
B1
as 𝑐𝑐 − 𝑏𝑏 ≠ 0
From (ii),
𝐴𝐴𝐴𝐴 𝜋𝜋
�arg �=
𝐵𝐵𝐵𝐵 2
so BC is perpendicular to AH E1
(iv) 𝑝𝑝 = 𝑎𝑎 + 𝑏𝑏 + 𝑐𝑐 𝑞𝑞 = 𝑏𝑏 + 𝑐𝑐 + 𝑑𝑑 𝑟𝑟 = 𝑐𝑐 + 𝑑𝑑 + 𝑎𝑎 𝑠𝑠 = 𝑑𝑑 + 𝑎𝑎 + 𝑏𝑏
𝑎𝑎+𝑞𝑞 𝑎𝑎+𝑏𝑏+𝑐𝑐+𝑑𝑑
The midpoint of AQ is = and so by its symmetry it is also the midpoint of BR, CS, and
2 2
DP, B1 E1
and thus ABCD is transformed to PQRS by a rotation of 𝜋𝜋 radians about midpoint of AQ. E1 B1 (4)
E1
Then 𝑥𝑥𝑘𝑘+1 = 𝑥𝑥𝑘𝑘 2 − 2 ≥ [2 + 4𝑘𝑘−1 (𝑎𝑎 − 2)]2 − 2 = 4 + 4𝑘𝑘 (𝑎𝑎 − 2) + 42𝑘𝑘−2 (𝑎𝑎 − 2)2 − 2
For 𝑛𝑛 = 1 , 2 + 4𝑛𝑛−1 (𝑎𝑎 − 2) = 2 + 𝑎𝑎 − 2 = 𝑎𝑎 so in this case, 𝑥𝑥𝑛𝑛 = 2 + 4𝑛𝑛−1 (𝑎𝑎 − 2) B1 and thus
by induction 𝑥𝑥𝑛𝑛 ≥ 2 + 4𝑛𝑛−1 (𝑎𝑎 − 2) for positive integer n. E1 (5)
So to consider |𝑎𝑎| ≥ 2 , we only need consider 𝑎𝑎 > 2 to discuss the behaviour of all terms after the
first. Therefore, from part (i), we know 𝑥𝑥𝑛𝑛 ≥ 2 + 4𝑛𝑛−1 (|𝑎𝑎| − 2) for 𝑛𝑛 ≥ 2 , and thus 𝑥𝑥𝑛𝑛 → ∞ as
𝑛𝑛 → ∞ ; B1 hence we have shown 𝑥𝑥𝑛𝑛 → ∞ as 𝑛𝑛 → ∞ if and only if |𝑎𝑎| ≥ 2 . (5)
(iii)
𝐴𝐴𝑥𝑥1 𝑥𝑥2 ⋯ 𝑥𝑥𝑘𝑘
𝑦𝑦𝑘𝑘 =
𝑥𝑥𝑘𝑘+1
𝐴𝐴𝑥𝑥1 𝑥𝑥2 ⋯ 𝑥𝑥𝑘𝑘+1 𝑥𝑥𝑘𝑘+1 2
𝑦𝑦𝑘𝑘+1 = = 𝑦𝑦
𝑥𝑥𝑘𝑘+2 𝑥𝑥𝑘𝑘+2 𝑘𝑘
M1
Suppose that
�𝑥𝑥𝑘𝑘+1 2 − 4
𝑦𝑦𝑘𝑘 =
𝑥𝑥𝑘𝑘+1
for some positive integer k, E1 then
and thus,
�𝑥𝑥𝑘𝑘+2 + 2�𝑥𝑥𝑘𝑘+2 − 2 �𝑥𝑥𝑘𝑘+2 2 − 4
𝑦𝑦𝑘𝑘+1 = =
𝑥𝑥𝑘𝑘+2 𝑥𝑥𝑘𝑘+2
M1 A1
then 𝐴𝐴𝑥𝑥1 = �𝑥𝑥2 2 − 4 , that is 𝐴𝐴2 𝑥𝑥1 2 = 𝑥𝑥2 2 − 4 , and as 𝑥𝑥1 = 𝑎𝑎 , 𝑥𝑥2 = 𝑥𝑥1 2 − 2 = 𝑎𝑎2 − 2
so
𝐴𝐴2 𝑎𝑎2 = (𝑎𝑎2 − 2)2 − 4 = 𝑎𝑎4 − 4𝑎𝑎2 , 𝐴𝐴2 = 𝑎𝑎2 − 4 , and thus 𝑎𝑎 = √𝐴𝐴2 + 4 , as 𝑎𝑎 ≠ 0 nor
−√𝐴𝐴2 + 4 because 𝑎𝑎 > 2. A1 E1
So as the result is true for 𝑦𝑦1 , and we have shown it to be true for 𝑦𝑦𝑘𝑘+1 if it is true for 𝑦𝑦𝑘𝑘 , it is true
by induction for all positive integer 𝑛𝑛 that
�𝑥𝑥𝑛𝑛+1 2 − 4
𝑦𝑦𝑛𝑛 =
𝑥𝑥𝑛𝑛+1
E1 (8)
As 𝑎𝑎 > 2 from (ii) 𝑥𝑥𝑛𝑛 → ∞ as 𝑛𝑛 → ∞ M1 and thus using result just proved, 𝑦𝑦𝑛𝑛 → 1 as 𝑛𝑛 → ∞ ,
i.e. the sequence converges. *A1 (2)
9.
A1
Hence
√3 1 √3 1
𝑥𝑥 � cos 𝜑𝜑 + sin 𝜑𝜑� � cos 𝜃𝜃 + sin 𝜃𝜃� = (𝑎𝑎 − 𝑥𝑥 ) sin 𝜑𝜑 sin 𝜃𝜃
2 2 2 2
giving
�√3 cot 𝜑𝜑 + 1��√3 cot 𝜃𝜃 + 1�𝑥𝑥 = 4(𝑎𝑎 − 𝑥𝑥 )
If the ball has speed 𝑣𝑣1 moving from P to Q, speed 𝑣𝑣2 moving from Q to R, and speed 𝑣𝑣3 moving
from R to P,
2𝜋𝜋 𝜋𝜋
then CLM at Q parallel to CA gives 𝑣𝑣1 cos � − 𝜃𝜃� = 𝑣𝑣2 cos and NELI perpendicular to CA gives
3 3
2𝜋𝜋 𝜋𝜋 2𝜋𝜋 𝜋𝜋
𝑒𝑒𝑣𝑣1 sin � − 𝜃𝜃� = 𝑣𝑣2 sin , and dividing these gives 𝑒𝑒 tan � − 𝜃𝜃� = tan
3 3 3 3
M1 A1
and similarly,
𝜋𝜋
CLM at R parallel to AB gives 𝑣𝑣2 cos = 𝑣𝑣3 cos 𝜑𝜑 and NELI perpendicular to AB gives
3
𝜋𝜋 𝜋𝜋
𝑒𝑒𝑣𝑣2 sin = 𝑣𝑣3 sin 𝜑𝜑 , and dividing these gives 𝑒𝑒 tan = tan 𝜑𝜑 . A1
3 3
2𝜋𝜋 𝜋𝜋 −√3−tan 𝜃𝜃
𝑒𝑒 tan � − 𝜃𝜃� = tan yields 𝑒𝑒 = √3 M1 which simplifies to
3 3 1−√3 tan 𝜃𝜃
𝑒𝑒�√3 + tan 𝜃𝜃� = √3 �√3 tan 𝜃𝜃 − 1� , or in turn, (3 − 𝑒𝑒) tan 𝜃𝜃 = √3(1 + 𝑒𝑒) and so
(3−𝑒𝑒)
cot 𝜃𝜃 = A1
√3(1+𝑒𝑒)
𝜋𝜋 1
𝑒𝑒 tan = tan 𝜑𝜑 yields cot 𝜑𝜑 = A1
3 𝑒𝑒√3
Substituting these two expressions into the first result of the question,
1 (3 − 𝑒𝑒)
� + 1� � + 1� 𝑥𝑥 = 4(𝑎𝑎 − 𝑥𝑥 )
𝑒𝑒 (1 + 𝑒𝑒)
M1
This simplifies to
1 + 𝑒𝑒 4
𝑥𝑥 = 4(𝑎𝑎 − 𝑥𝑥 )
𝑒𝑒 1 + 𝑒𝑒
that is
𝑥𝑥 = 𝑒𝑒(𝑎𝑎 − 𝑥𝑥 )
so
𝑎𝑎𝑎𝑎
𝑥𝑥 =
1 + 𝑒𝑒
as required. *A1 (8)
2𝜋𝜋
To continue the motion at P, then similarly to before, the third impact gives 𝑒𝑒 tan � − 𝜑𝜑� = tan 𝜃𝜃
3
M1
So
−√3 − tan 𝜑𝜑 √3(𝑒𝑒 + 1)
tan 𝜃𝜃 = 𝑒𝑒 = 𝑒𝑒
1 − √3 tan 𝜑𝜑 3𝑒𝑒 − 1
That is 𝑒𝑒(3 − 𝑒𝑒) = 3𝑒𝑒 − 1 , that is 𝑒𝑒 2 = 1 and as 𝑒𝑒 ≥ 0 , 𝑒𝑒 = 1 (and not -1) *B1 (4)
10. (i) At time t, the point where the string is tangential to the cylinder, M1 say T is at
(𝑎𝑎 cos 𝜃𝜃, 𝑎𝑎 sin 𝜃𝜃 ) , A1 the piece of string that remains straight is of length 𝑏𝑏 − 𝑎𝑎θ, M1 , the vector
−sin 𝜃𝜃
representing the string is thus (𝑏𝑏 − 𝑎𝑎θ) � � dM1 A1 so the particle is at the point
cos 𝜃𝜃
(𝑎𝑎 cos 𝜃𝜃 − (𝑏𝑏 − 𝑎𝑎θ) sin 𝜃𝜃 , 𝑎𝑎 sin 𝜃𝜃 + (𝑏𝑏 − 𝑎𝑎θ) cos 𝜃𝜃 ) . M1 A1 (7)
𝑥𝑥̇ = −𝑎𝑎𝜃𝜃̇ sin 𝜃𝜃 − (𝑏𝑏 − 𝑎𝑎θ)𝜃𝜃̇ cos 𝜃𝜃 + 𝑎𝑎𝜃𝜃̇ sin 𝜃𝜃 = −(𝑏𝑏 − 𝑎𝑎θ)𝜃𝜃̇ cos 𝜃𝜃
𝑦𝑦̇ = 𝑎𝑎𝜃𝜃̇ cos 𝜃𝜃 − (𝑏𝑏 − 𝑎𝑎θ)𝜃𝜃̇ sin 𝜃𝜃 − 𝑎𝑎𝜃𝜃̇ cos 𝜃𝜃 = −(𝑏𝑏 − 𝑎𝑎θ)𝜃𝜃̇ sin 𝜃𝜃
M1 A1
2 2
Thus the speed is ��(𝑏𝑏 − 𝑎𝑎θ)𝜃𝜃̇ cos 𝜃𝜃� + �(𝑏𝑏 − 𝑎𝑎θ)𝜃𝜃̇ sin 𝜃𝜃� = (𝑏𝑏 − 𝑎𝑎θ)𝜃𝜃̇ as required. M1 A1 (4)
(ii) The only horizontal force on the particle is the tension in the string, which is perpendicular to the
velocity at any time, so kinetic energy is conserved. E1 Therefore,
1 2 1
𝑚𝑚 �(𝑏𝑏 − 𝑎𝑎θ)𝜃𝜃̇� = 𝑚𝑚𝑢𝑢2
2 2
M1
and so, as (𝑏𝑏 − 𝑎𝑎θ)𝜃𝜃̇ and 𝑢𝑢 are both positive (𝑏𝑏 − 𝑎𝑎θ)𝜃𝜃̇ = 𝑢𝑢 *A1 (3)
(iii) The tension in the string, using instantaneous circular motion, at time t is
𝑚𝑚𝑢𝑢2
(𝑏𝑏 − 𝑎𝑎θ)
M1 A1
but when 𝑡𝑡 = 0 , 𝜃𝜃 = 0 so 𝑐𝑐 = 0 . M1 A1
𝑎𝑎𝜃𝜃2
Thus, 𝑏𝑏𝑏𝑏 − = 𝑢𝑢𝑢𝑢
2
i.e.
(𝑏𝑏 − 𝑎𝑎θ)2
− = 𝑢𝑢𝑢𝑢 + 𝑘𝑘
2𝑎𝑎
M1
𝑏𝑏 2
When 𝑡𝑡 = 0 , 𝜃𝜃 = 0 so 𝑘𝑘 = − M1 A1
2𝑎𝑎
𝑚𝑚𝑢𝑢2
√𝑏𝑏 2 − 2𝑎𝑎𝑎𝑎𝑎𝑎
*A1 (6)
11. (i)
𝑛𝑛+1
𝑛𝑛+1
𝑃𝑃(𝑌𝑌 = 𝑛𝑛) = 𝑃𝑃 (𝑛𝑛 ≤ 𝑋𝑋 < 𝑛𝑛 + 1) = � 𝜆𝜆 𝑒𝑒 −𝜆𝜆𝜆𝜆 𝑑𝑑𝑑𝑑 = �− 𝑒𝑒 −𝜆𝜆𝜆𝜆 �𝑛𝑛 = − 𝑒𝑒 −𝜆𝜆(𝑛𝑛+1) + 𝑒𝑒 −𝜆𝜆𝜆𝜆
𝑛𝑛
M1 M1
(ii)
∞ ∞ 𝑟𝑟+𝑧𝑧 ∞
−𝜆𝜆𝜆𝜆 𝑟𝑟+𝑧𝑧
𝑃𝑃(𝑍𝑍 < 𝑧𝑧) = � 𝑃𝑃(𝑟𝑟 ≤ 𝑋𝑋 < 𝑟𝑟 + 𝑧𝑧) = � � 𝜆𝜆 𝑒𝑒 𝑑𝑑𝑑𝑑 = ��− 𝑒𝑒 −𝜆𝜆𝜆𝜆 �𝑟𝑟
𝑟𝑟=0 𝑟𝑟=0 𝑟𝑟 𝑟𝑟=0
M1 M1
∞ ∞
−𝜆𝜆(𝑟𝑟+𝑥𝑥) −𝜆𝜆𝜆𝜆
= ��− 𝑒𝑒 + 𝑒𝑒 � = ��1 − 𝑒𝑒 −𝜆𝜆𝜆𝜆 �𝑒𝑒 −𝜆𝜆𝜆𝜆
𝑟𝑟=0 𝑟𝑟=0
M1 A1
1 1 − 𝑒𝑒 −𝜆𝜆𝜆𝜆
= �1 − 𝑒𝑒 −𝜆𝜆𝜆𝜆 � =
1 − 𝑒𝑒 −𝜆𝜆 1 − 𝑒𝑒 −𝜆𝜆
using sum of an infinite GP with magnitude of common ratio less than one.
M1 *A1 (6)
1−𝑒𝑒 −𝜆𝜆𝜆𝜆 𝑑𝑑 1−𝑒𝑒 −𝜆𝜆𝜆𝜆 𝜆𝜆𝜆𝜆 −𝜆𝜆𝜆𝜆
(iii) As 𝑃𝑃(𝑍𝑍 < 𝑧𝑧) = , 𝑓𝑓𝑍𝑍 (𝑧𝑧) = � �= M1
1−𝑒𝑒 −𝜆𝜆 𝑑𝑑𝑑𝑑 1−𝑒𝑒 −𝜆𝜆 1−𝑒𝑒 −𝜆𝜆
so
1 1
𝜆𝜆𝜆𝜆 −𝜆𝜆𝜆𝜆 1 1
𝐸𝐸 (𝑍𝑍) = � 𝑧𝑧 −𝜆𝜆
𝑑𝑑𝑑𝑑 = −𝜆𝜆
� �−𝑧𝑧𝑒𝑒 −𝜆𝜆𝜆𝜆 �0 + � 𝑒𝑒 −𝜆𝜆𝜆𝜆 𝑑𝑑𝑑𝑑�
1 − 𝑒𝑒 1 − 𝑒𝑒
0 0
M1 M1
1
1 −𝜆𝜆
𝑒𝑒 −𝜆𝜆𝜆𝜆 1 −𝜆𝜆
𝑒𝑒 −𝜆𝜆 1
= �−𝑒𝑒 − � � � = �−𝑒𝑒 − + �
1 − 𝑒𝑒 −𝜆𝜆 𝜆𝜆 0 1 − 𝑒𝑒 −𝜆𝜆 𝜆𝜆 𝜆𝜆
A1
1 𝑒𝑒 −𝜆𝜆
= −
𝜆𝜆 1 − 𝑒𝑒 −𝜆𝜆
or alternatively
1 �1 − (𝜆𝜆 + 1)𝑒𝑒 −𝜆𝜆 �
𝜆𝜆 1 − 𝑒𝑒 −𝜆𝜆
A1 (5)
(iv)
𝑃𝑃 (𝑌𝑌 = 𝑛𝑛 𝑎𝑎𝑎𝑎𝑎𝑎 𝑧𝑧1 < 𝑍𝑍 < 𝑧𝑧2 ) = 𝑃𝑃(𝑛𝑛 + 𝑧𝑧1 < 𝑋𝑋 < 𝑛𝑛 + 𝑧𝑧2 )
𝑛𝑛+𝑧𝑧2
𝑛𝑛+𝑧𝑧2
= � 𝜆𝜆 𝑒𝑒 −𝜆𝜆𝜆𝜆 𝑑𝑑𝑑𝑑 = �− 𝑒𝑒 −𝜆𝜆𝜆𝜆 �𝑛𝑛+𝑧𝑧 = − 𝑒𝑒 −𝜆𝜆(𝑛𝑛+𝑧𝑧2 ) + 𝑒𝑒 −𝜆𝜆(𝑛𝑛+𝑧𝑧1 ) = 𝑒𝑒 −𝜆𝜆𝜆𝜆 �𝑒𝑒 −𝜆𝜆𝑧𝑧1 − 𝑒𝑒 −𝜆𝜆𝑧𝑧2 �
1
𝑛𝑛+𝑧𝑧1
M1 A1
𝑃𝑃(𝑌𝑌 = 𝑛𝑛 𝑎𝑎𝑎𝑎𝑎𝑎 𝑧𝑧1 < 𝑍𝑍 < 𝑧𝑧2 ) = 𝑒𝑒 −𝜆𝜆𝜆𝜆 �𝑒𝑒 −𝜆𝜆𝑧𝑧1 − 𝑒𝑒 −𝜆𝜆𝑧𝑧2 �
1 − 𝑒𝑒 −𝜆𝜆𝑧𝑧2 1 − 𝑒𝑒 −𝜆𝜆𝑧𝑧1
= �1 − 𝑒𝑒 −𝜆𝜆 �𝑒𝑒 −𝜆𝜆𝜆𝜆 � − �
1 − 𝑒𝑒 −𝜆𝜆 1 − 𝑒𝑒 −𝜆𝜆
M1 A1
If 𝑋𝑋23 = 1 , then players 2 and 3 score the same as one another. In that case, 𝑋𝑋12 = 1 would mean
1
that player 1 also obtained that same score so 𝑃𝑃(𝑋𝑋12 = 1|𝑋𝑋23 = 1) = = 𝑃𝑃 (𝑋𝑋12 = 1).
6
If 𝑋𝑋23 = 0 , 𝑋𝑋12 = 0 would mean that player 1 obtained a different score to player 2 so
5
𝑃𝑃 (𝑋𝑋12 = 0|𝑋𝑋23 = 0) = = 𝑃𝑃(𝑋𝑋12 = 0)
6
Hence 𝑋𝑋12 is independent of 𝑋𝑋23 . M1 A1 (2)
Alternatively,
𝑋𝑋12 𝑋𝑋23
1 1 requires players 2 and 3 to both score same as player 1 so
1 1 1
𝑃𝑃(𝑋𝑋12 = 1 𝑎𝑎𝑎𝑎𝑎𝑎 𝑋𝑋23 = 1 ) = = × = 𝑃𝑃 (𝑋𝑋12 = 1) × 𝑃𝑃 (𝑋𝑋23 = 1)
36 6 6
1 0 requires player 2 to score the same as player as player 1, and player 3 score differently so
5 1 5
𝑃𝑃(𝑋𝑋12 = 1 𝑎𝑎𝑎𝑎𝑎𝑎 𝑋𝑋23 = 0 ) = = × = 𝑃𝑃 (𝑋𝑋12 = 1) × 𝑃𝑃 (𝑋𝑋23 = 0)
36 6 6
0 1 requires players 2 and 3 to score the same as one another, and player 1 score differently so
5 5 1
𝑃𝑃(𝑋𝑋12 = 0 𝑎𝑎𝑎𝑎𝑎𝑎 𝑋𝑋23 = 1 ) = = × = 𝑃𝑃 (𝑋𝑋12 = 0) × 𝑃𝑃 (𝑋𝑋23 = 1)
36 6 6
0 0 requires both player 1 and 3 to score differently to player 2 so
25 5 5
𝑃𝑃(𝑋𝑋12 = 0 𝑎𝑎𝑛𝑛𝑑𝑑 𝑋𝑋23 = 0 ) = = × = 𝑃𝑃 (𝑋𝑋12 = 0) × 𝑃𝑃 (𝑋𝑋23 = 0)
36 6 6
Hence 𝑋𝑋12 is independent of 𝑋𝑋23 . M1 A1 (2)
M1
so
1 5 𝑛𝑛(𝑛𝑛 − 1)
𝐸𝐸 (𝑇𝑇) = 𝐸𝐸 �� 𝑋𝑋𝑖𝑖𝑖𝑖 � = � 𝐸𝐸�𝑋𝑋𝑖𝑖𝑖𝑖 � = 𝑛𝑛𝐶𝐶2 𝐸𝐸 (𝑋𝑋12 ) = 𝑛𝑛𝐶𝐶2 �1 × +0× �=
6 6 12
𝑖𝑖<𝑗𝑗 𝑖𝑖<𝑗𝑗
M1 A1
1 5 12
𝑉𝑉𝑉𝑉𝑉𝑉(𝑇𝑇) = 𝑉𝑉𝑉𝑉𝑉𝑉 �� 𝑋𝑋𝑖𝑖𝑖𝑖 � = � 𝑉𝑉𝑉𝑉𝑉𝑉�𝑋𝑋𝑖𝑖𝑖𝑖 � = 𝑛𝑛𝐶𝐶2 𝑉𝑉𝑉𝑉𝑉𝑉(𝑋𝑋12 ) = 𝑛𝑛𝐶𝐶2 �12 × + 02 × − �
6 6 6
𝑖𝑖<𝑗𝑗 𝑖𝑖<𝑗𝑗
M1
5𝑛𝑛(𝑛𝑛 − 1)
=
72
A1 (5)
(ii)
𝑉𝑉𝑉𝑉𝑉𝑉(𝑌𝑌1 + 𝑌𝑌2 + … + 𝑌𝑌𝑚𝑚 ) = 𝐸𝐸 ((𝑌𝑌1 + 𝑌𝑌2 + … + 𝑌𝑌𝑚𝑚 )2 ) − [𝐸𝐸 (𝑌𝑌1 + 𝑌𝑌2 + … + 𝑌𝑌𝑚𝑚 )]2
= 𝐸𝐸�𝑌𝑌1 2 + 𝑌𝑌2 2 + … + 𝑌𝑌𝑚𝑚 2 + 2𝑌𝑌1 𝑌𝑌2 + 2𝑌𝑌1 𝑌𝑌3 + … + 2𝑌𝑌𝑛𝑛−1 𝑌𝑌𝑛𝑛 � − [𝐸𝐸 (𝑌𝑌1 ) + 𝐸𝐸 (𝑌𝑌2 ) + … + 𝐸𝐸 (𝑌𝑌𝑚𝑚 )]2
𝑚𝑚 𝑚𝑚−1 𝑚𝑚
2
= 𝐸𝐸 �� 𝑌𝑌𝑖𝑖 � + 2𝐸𝐸 � � � 𝑌𝑌𝑖𝑖 𝑌𝑌𝑗𝑗 � − (0 + 0 + ⋯ + 0)2
𝑖𝑖=1 𝑖𝑖=1 𝑗𝑗=𝑖𝑖+1
𝑚𝑚 𝑚𝑚−1 𝑚𝑚
2
= � 𝐸𝐸�𝑌𝑌𝑖𝑖 � + 2 � � 𝐸𝐸�𝑌𝑌𝑖𝑖 𝑌𝑌𝑗𝑗 �
𝑖𝑖=1 𝑖𝑖=1 𝑗𝑗=𝑖𝑖+1
M1 *A1 (2)
(iii)
1 1 1
𝑃𝑃(𝑍𝑍12 = 1) = × =
2 6 12
If 𝑍𝑍23 = 1 then player 2 has rolled an even score and player 3 has scored the same so, in this case,
1
for 𝑍𝑍12 = 1 , require player 1 to roll the score that player has so 𝑃𝑃(𝑍𝑍12 = 1|𝑍𝑍23 = 1) = .
6
Therefore, 𝑃𝑃(𝑍𝑍12 = 1) ≠ 𝑃𝑃 (𝑍𝑍12 = 1|𝑍𝑍23 = 1) and thus 𝑍𝑍12 and 𝑍𝑍23 are not independent.
Alternatively,
1 1
𝑃𝑃(𝑍𝑍12 = 1) = , 𝑃𝑃(𝑍𝑍23 = 1) =
12 12
For 𝑍𝑍12 = 1 and 𝑍𝑍23 = 1 we require all three players to score the same even number so
3 1 1 1 1 1
𝑃𝑃(𝑍𝑍12 = 1 𝑎𝑎𝑎𝑎𝑎𝑎 𝑍𝑍23 = 1 ) = × × = ≠ × = 𝑃𝑃(𝑍𝑍12 = 1) × 𝑃𝑃(𝑍𝑍23 = 1)
6 6 6 72 12 12
and thus they are not independent. M1 A1 (2)
Using part (ii), let 𝑌𝑌1 = 𝑍𝑍12 , let 𝑌𝑌2 = 𝑍𝑍13 , … let 𝑌𝑌𝑚𝑚 = 𝑍𝑍(𝑛𝑛−1)𝑛𝑛
𝑛𝑛(𝑛𝑛−1)
(and with 𝑚𝑚 = 𝑛𝑛𝐶𝐶2 = ).
2
1 1 5 1
𝑃𝑃(𝑍𝑍12 = 1) = , 𝑃𝑃(𝑍𝑍12 = −1) = , 𝑃𝑃(𝑍𝑍12 = 0) = so 𝐸𝐸 (𝑍𝑍12 ) = 0 and 𝐸𝐸�𝑍𝑍12 2 � = and
12 12 6 6
likewise for all other Z (Y!). B1 B1
so
B1
Otherwise 𝑍𝑍12 = 0
1 1 1
So 𝐸𝐸 (𝑍𝑍12 𝑍𝑍13 ) = 1 × 1 × + −1 × −1 × = M1 A1
72 72 36
So
𝑛𝑛(𝑛𝑛 − 1) 1 𝑛𝑛−1 1 𝑛𝑛(𝑛𝑛 − 1) 𝑛𝑛(𝑛𝑛 − 1)(𝑛𝑛 − 2)
𝑉𝑉𝑉𝑉𝑉𝑉(𝑈𝑈) = × + 2 × 𝑛𝑛 × 𝐶𝐶2 × = +
2 6 36 12 36
M1 M1 A1